-
1
- 1. A 25 year old male presented to OPD with complains of fatigue for 1 month. He also complains of decreased sleep and decreased appetite. He mentioned that he could not concentrate well in his lecture classes. He feels emptiness in his life and tried ending his life twice in the past week. On asking further, he told that he was diagnosed as a case of Schizophrenia 5 years back and he is currently under regular medications. Lab investigations show; TSH 4.5, ESR 10, Hb 9.7, MCV 78, normal electrolytes, SGOT 52, SGPT 43 and ALP 50 (standard units are mentioned). His father also has been diagnosed as a case of Schizophrenia. What is the most likely diagnosis at this point of time in the patient?
- a. Schizophrenia
- b. Hypothyroidism
- c. Schizoaffective disorder
- d. Mood disorder with psychotic features
d. Mood disorder with psychotic features
(The patient already has Schizophrenia and still taking medications, so we can assume he may still have its symptoms as there is no mention in the question regarding its complete cure. So,this patient seems to have both depressive/mood symptoms and schizophrenic symptoms together (and not at different times). Hence we can tick “d. Mood disorder with psychotic features” as the best possible answer.)
- A. Schizoaffective disorder
- • Shares symptoms with both schizophrenia and mood disorders (major depressive or bipolar disorder).
- • To differentiate from a mood disorder with psychotic features, patient must have > 2 weeks of psychotic symptoms without a manic or depressive episode.
- B. Schizophrenia
- • Chronic illness causing profound functional impairment. Symptom categories include:
- • Positive—hallucinations, delusions, unusual thought processes, disorganized speech, bizarre behavior
- • Negative—flat or blunted affect, apathy, anhedonia, alogia, social withdrawal
- • Cognitive—reduced ability to understand or make plans, diminished working memory, inattention
- • Diagnosis requires ≥ 2 of the following active symptoms, including ≥ 1 from symptoms #1–3:
- 1. Delusions
- 2. Hallucinations, often auditory
- 3. Disorganized speech
- 4. Disorganized or catatonic behavior
- 5. Negative symptoms
- • Requires ≥ 1 month of active symptoms over the past 6 months; onset ≥ 6 months prior to diagnosis.
- C. MDD with psychotic features
- • MDD + hallucinations or delusions. Psychotic features are typically mood congruent (eg, depressive themes of inadequacy, guilt, punishment, nihilism, disease, or death) and occur only in the context of major depressive episode (vs schizoaffective disorder).
- • Treatment: antidepressant with atypical antipsychotic, ECT.
- 2. Drugs cause urticaria by direct degranulation of mast cells, interfering with arachidonic acid metabolism, IgE-mediated degranulation of mast cells or Complement-mediated degranulation of mast cells. Which of the following drugs cause urticarial by IgE-mediated degranulation of mast cells?
- a. Aspirin
- b. Penicillin
- c. Radioactive contrast
- d. Sulphonamide
b. Penicillin
- Transmitted Diseases, Neena Khanna, 4th edition, page no. 211
- Drugs causing Urticaria and Angioedema
- Pathogenesis
- • Drugs cause urticaria by:
- • Direct degranulation of mast cells.
- • Interfering with arachidonic acid metabolism.
- • IgE-mediated degranulation of mast cells.
- • Complement-mediated degranulation of mast cells.
- Clinical Features
- Urticaria and angioedema can occur independently or may also be part of a severe and generalized reaction (anaphylaxis), which includes bronchospasm and circulatory collapse.
- Drugs Implicated
- A. Drugs which directly direct degranulate mast cells:
- • Aspirin.
- • Indomethacin.
- B. Drugs which interfere with arachidonic acid metabolism:
- • Opiates: morphine and codeine.
- • Sulfonamides.
- • Curare.
- • Radioactive contrast.
- C. Drugs which mediate urticaria through IgE:
- • Penicillin.
- D. Drugs which mediate urticaria through complement system:
- • Blood products.
- 3. Secretin and CCK are important hormones which regulate secretion of Pancreatic juice. Which one of the following statements is true regarding this regulation?
- a. Secretin causes release of juice rich in enzymes but low in volume
- b. CCK causes release of juice rich in HCO3− and poor in enzymes
- c. Secretin acts on duct cells by increasing cAMP
- d. CCK acts on duct cells by increasing cGMP
c. Secretin acts on duct cells by increasing cAMP
- REGULATION OF THE SECRETION OF PANCREATIC JUICE
- • Secretion of pancreatic juice is primarily under hormonal control.
- • Secretin acts on the pancreatic ducts to cause copious secretion of a very alkaline pancreatic juice that is rich in HCO3− and poor in enzymes. The effect on duct cell is due to an increase in intracellular cAMP. Secretin also stimulates bile secretion.
- • CCK acts on the acinar cells to cause the release of zymogen granules and production of pancreatic juice rich in enzymes but low in volume. Its effect is mediated by phospho-lipase C. Thus, by acting together, CCK and secretin add enzymes to the pancreatic juice and ensure that they are washed into the intestine.
-
2. A patient tries to make a fist, but digits 2 and 3 remain partially extended. What nerve is injured
Ans : Median nerve
- • When the median nerve is severed in the elbow region, flexion of the proximal interphalangeal joints of the 1st–3rd digits is lost and flexion of the 4th and 5th digits is weakened. Flexion of the distal interphalangeal joints of the 2nd and 3rd digits is also lost. Flexion of the distal interphalangeal joints of the 4th and 5th digits is not affected because the medial part of the FDP, which produces these movements, is supplied by the ulnar nerve. The ability to flex the metacarpophalangeal joints of the 2nd and 3rd digits is affected because the digital branches of the median nerve supply the 1st and 2nd lumbricals. Thus, when the person attempts to make a fist, the 2nd and 3rd fingers remain partially extended (“hand of benediction”).
- • Thenar muscle function (function of the muscles at the base of the thumb) is also lost, as in carpal tunnel syndrome. When the anterior interosseous nerve is injured, the thenar muscles are unaffected, but paresis (partial paralysis) of the flexor digitorum profundus and flexor pollicis longus occurs. When the person attempts to make the “okay” sign, opposing the tip of the thumb and index finger in a circle, a “pinch” posture of the hand results instead owing to the absence of flexion of the interphalangeal joint of the thumb and distal interphalangeal joint of the index finger (anterior interosseous syndrome).
-
3. Coarse facies, hepatosplenomegaly and cloudy cornea are characteristic feature of:
a. Morquio syndrome A
b. Hurler’s disease
c. Hunters disease
d. Natowicz syndrome
- Ans: B
- Ref: Harper Biochemistry 30th Edition, Page No: 639
- • Hurler disease: Mental retardation, coarse facial features, hepatosplenomegaly, cloudy cornea
- • Hunter disease: Mental retardation
- • Morquio syndrome A : Skeletal dysplasia, short stature
- • Natowicz syndrome: Joint pain, short stature
-
4
- 1. You are called urgently to see an 80-year-old man who is 6 days following open anterior resection for rectal carcinoma with defunctioning stoma. The patient reported seeing a gush of pink fluid from the central laparotomy wound. You notice that the small bowel is eviscerating from the wound. Following initial resuscitation, the next best step is to
- a. Cover the small bowel with a sterile saline-soaked gauze
- b. Administer intravenous cefuroxime 1.5 g
- c. Return the patient to theatre for deep-tension abdominal wall closure
- d. Apply vacuum-assisted closure therapy
- Ans: a
- The most appropriate next step following resuscitation is to protect the bowel from the atmosphere with a saline-soaked gauze. This should precede the subsequent management, which would include calling a senior colleague, administering intravenous antibiotics and returning the patient to theatre for immediate closure of the abdomen using deep tension sutures.
- Vacuum-assisted closure therapy can be used in cases of wound dehiscence, but not where the bowel is eviscerated.
- 2. Frost bite is treated by:
- a. Rapid rewarming
- b. Gentle rewarming
- c. IV pentoxyphiline
- d. Amputation
- Ans: b
- Ref: Bailey and Love 27th Edition, Page No: 422
- Frost bite and immersion injuries (trench foot)
- • Frost bite occurs when a part of the body freezes.
- • The cells are disrupted and the tissue dies. It is in effect a ‘cold’ burn and can be categorized according to the depth that it affects in the same way as a conventional burn. Other mechanisms at play include vasoconstriction caused by cold, capillary sludging and reperfusion injury with the release of free radicals, which occurs on rewarming the part. It commonly involves the fingers, toes, cheeks, the tip of the nose and the ears.
- • When frozen the tissue feels hard and cannot be indented. Immersion injury is a cold injury, which does not involve actual freezing of the tissue and is commonly caused by prolonged immersion in cold water (hence trench foot).
- • The patient may also be hypothermic. Warming should be gentle as the heat used may actually cause a burn! Rehydration with warm fluids and use of non-steroidal anti-inflammatory drugs like ibuprofen are beneficial.
- • Demarcation will occur between dead and viable tissue and at this stage no surgery should be undertaken as there is often considerable deep recovery. The injured area should be kept clean and dry and efforts made to prevent further injury, as well as to prevent infection. Definitive surgery to excise dead tissue can be left for many months.
- • Recent developments, such as the use of tissue plasminogen activator (TPA) and nerve blocks, show promising results in reducing amputations, but have to be started within 24 hours and are seldom possible in the field.
- 3. A 65-year old man presented with an episode of syncope. He said he felt dizzy during defecation and noticed gross bleeding in the pan. Fecal occult blood test done 3 months ago as a part of routine screening for colon cancer was negative. There is no history of recent weight loss. What is the likely colonoscopic finding?
- a. Early stage carcinoma colon
- b. Sigmoid diverticulitis
- c. Microscopic colitis
- d. Dilated mucosal and submucosal veins in the colon
- Ans: d
- Ref: Harrison 19th Edition, Page No: 731 ; Bailey 26th Edition, Page No:1174, 25th Edition, Page No:1157
- Diagnosis in a 65 year old patient of bleeding per rectum with syncope and negative fecal occult blood test and no history of recent weight loss is HEYDE’S Syndrome.
- The colonoscopic finding in Hyde’s syndrome is dilated mucosal and submucosal veins in the colon.
- HEYDE’S Syndrome: Triad of aortic stenosis, acquired coagulopathy and anaemia due to bleeding from intestinal angiodysplasia.
- 4. A 28 year old male presents with weight loss. On Barium study, pulled up cecum is seen. Likely diagnosis is?
- a. Ca colon
- b. Carcinoid
- c. Ileocecal tuberculosis
- d. Crohn’s disease
- Ans: c
- Ref: Bailey & Love 25th Edition, Page No: 1174
- TB of small intestine
- Clinical features:
- • Attacks of acute abdominal pain with intermittent diarrhea
- • Mass in right iliac fossa in a patient with vague ill health
- • Features of blind loop syndrome may develop due to stasis, distension and chronic infection in the segment of ileum proximal to obstruction.
- Barium meal radiography reveals:
- • Persistent narrowing of the affected segment in the terminal ileum and cecum
- • Caecum is pulled up and may become subhepatic
-
5
- 1. World Health Day 2020 marked acknowledgement of the role of:
- a. Doctors
- b. Midwives
- c. Pharmacist
- d. Research scientist
- Ans: b
- Happy World Health Day 2020
- 7 April 2020 is the day to celebrate the work of nurses and midwives and remind world leaders of the critical role they play in keeping the world healthy. Nurses and other health workers are at the forefront of COVID-19 response - providing high quality, respectful treatment and care, leading community dialogue to address fears and questions and, in some instances, collecting data for clinical studies. Quite simply, without nurses, there would be no response.
- 2. PFK -1 inhibitor:
- a. AMP
- b. Citrate
- c. ATP
- d. Both b and c
- Ans: D
- Ref: Harper biochemistry; 30th edition; Page No: 189
- Phosphofructokinase (phosphofructokinase-1) occupies a key position in regulating glycolysis and is also subject to feedback control. It is inhibited by citrate and by normal intracellular concentrations of ATP and is activated by 5′ AMP. At the normal intracellular [ATP] the enzyme is about 90% inhibited; this inhibition is reversed by 5′AMP.
- 3. Which of the following promotes high levels of HbF synthesis?
- a. Hydroxyurea
- b. Butyrates
- c. Cytarabine
- d. All of the above
- Ans: D
- Ref: Harrison 19th Edition, Page No: 639
- Reestablishing high levels of HbF synthesis can ameliorate symptoms of thalassemia.
- Hydroxyurea & cytarabine promote high levels of HbF synthesis by stimulating proliferation of F cell progenitors. Butyrates also stimulate HbF production.
- 4. A couple is using OCP for 3 years. They are hesitant about the daily use and missed pills. They want alternative long lasting contraception plan. You are explaining them about the other methods available. IUCD fascinates them. Characteristics of an ideal candidate for copper-T insertion include all of the following except:
- a. Has born at least one child
- b. Is willing to check IUD tail
- c. Has a history of ectopic pregnancy
- d. Has normal menstrual periods
- Ans: c
- Ref: Shaw Gynecology 16th Edition, Page No: 269
- IUCDs are a good contraceptive choice for the following groups of women:
- ��� Low risk of STD
- • Multiparous woman
- • Monogamous relationship
- • Desirous of long-term reversible method of contraception, but not yet desirous of permanent sterilization
- • Unhappy or unreliable users of oral contraception or barrier contraception
- Contraindications
- • Suspected pregnancy
- • Pelvic inflammatory disease (PID), lower genital tract infection
- • Presence of fibroids—because of misfit
- • Menorrhagia and dysmenorrhoea, if Copper T is used
- • Severe anaemia
- • Diabetic women who are not well controlled—because of slight increase in pelvic infection
- • Heart disease—risk of infection
- • Previous ectopic pregnancy
- • Scarred uterus
- • Preferably avoid its use in unmarried and nulliparous patients because of the risk of PID and subsequent tubal infertility
- • LNG IUCD in breast cancer
- • Abnormally shaped uterus, septate uterus
- 5. A one-week-old, full-term infant born by uncomplicated vaginal delivery is brought to a pediatrician by his mother, who notes that her daughter's skin appears yellow. She reports that the child cries several times per day and sleeps 7-8 hours at night, uninterrupted. She has been breastfeeding exclusively and feels that he has been feeding well. Which of the following is the best treatment for this condition?
- a. Phenobarbital administration
- b. Plasma exchange
- c. Switch from breast milk to formula
- d. Continue normal breast feeding
- Ans: d
- • The infant in this case most likely has breast milk jaundice. No intervention is needed, and the mother does not need to stop breast feeding.
- • Breast milk jaundice occurs because the infant liver is not mature enough to process lipids. It presents between the 4th and 7th day of life. If breast feeding stops, bilirubin levels fall rapidly; however, if breast feeding continues, bilirubin levels will fall slowly.
- • Distinct from breast milk jaundice, breastfeeding jaundice is the most common cause of neonatal jaundice in the first week of life and occurs in 10% of births. It is characterized by insufficient feeding, which leads to fewer bowel movements and, therefore, decreased bilirubin clearance. The infant in this vignette has been breastfeeding normally.
- • While switching to formula may improve the jaundice, mothers should be encouraged to continue breast feeding because of the improved health benefits of breast milk over formula.
-
6
- 1. Hotline number for ambulance services in Nepal is:
- a. 100
- b. 101
- c. 102
- d. 103
- Ans: c
- • In the event of an emergency, dial 100 to contact the police in Nepal.
- • Dial 101 for fire.
- • Dial 102 for ambulance service.
- • Dial 103 for traffic control.
- • Dial 197 for telephone inquiry.
- 2. Catecholamines are increased in:
- a. Neuroblastoma
- b. Retinoblastoma
- c. Medulloblastoma
- d. Nephroblastoma
- Ans: a
- Ref: Robbins Basic Pathology 10th Edition, Page No: 286, 289
- Neuroblastoma
- • The term neuroblastic includes tumors of the sympathetic ganglia and adrenal medulla that are derived from primordial neural crest cells populating these sites; neuroblastoma is the most important member of this family.
- • It is the second most common solid malignancy of childhood after brain tumors, accounting for 7% to 10% of all pediatric neoplasms, and as many as 50% of malignancies diagnosed in infancy.
- • About 90% of neuroblastomas, regardless of location, produce catecholamines (similar to the catecholamines associated with pheochromocytomas), which constitutes an important diagnostic feature (i.e., elevated blood levels of catecholamines and elevated urine levels of catecholamine metabolites such as vanillylmandelic acid [VMA] and homovanillic acid [HVA]). Despite the elaboration of catecholamines, hypertension is much less frequent with these neoplasms than with pheochromocytomas.
- Retinoblastoma
- • Most common primary intraocular malignancy of children.
- • Approximately 40% of the tumors are associated with a germline mutation in the RB gene and are therefore heritable.
- • The remaining 60% of the tumors develop sporadically, and these have somatic RB gene mutations. Familial cases typically are associated with development of multiple tumors that are bilateral, although they may be unifocal and unilateral.
- • All of the sporadic, nonheritable tumors are unilateral and unifocal.
- • Patients with familial retinoblastoma also are at increased risk for the development of osteosarcoma and other soft tissue tumors.
- 3. Rave drug is:
- a. Cannabis
- b. Cocaine
- c. Heroin
- d. Amphetamine
- Ans: d
- MDMA is also called Rave drug and ecstasy.
- Street names:
- Gamma Hydroxy butyrate (GHB): Liquid ecstasy, Grievous bodily harm
- Phencyclidine and Ketamin: Angel dust, Hog, Special K
- Cocaine: Crack (vapour to be smoked), Rush, Coke, Snow, Blow, Peruvian marching Powder
- Methylene dioxymethamphetamine (MDMA): Ecstasy, Rave drug
- Lysergic acid diethylamide (LSD): Windowpane, Twenty-five
- 4. A child is below the third percentile for height.His growth velocity is normal,but chronological age is more than skeletal age. The most likely diagnosis is:
- a. Constitutional growth delay
- b. Hypopituitarism
- c. Familial short stature
- d. Cretinism
- Ans: a
- Ref: OP Ghai 9th edition page 33
- Constitutional growth delay:
- • These children are born with a normal length and weight and grow normally for the first 6-12 months of life. Their growth then shows a deceleration so that the height and weight fall below the 3rd centile.
- • By 3 Years of age, normal height velocity is resumed and they continue to grow just below and parallel to the 3rd centile with a normal height velocity.
- • The onset of puberty and adolescent growth spurt is also delayed in these children but final height is within normal limits.
- • Bone age is lower than chronological age and corresponds to the height age. History of delayed puberty and delayed height spurt is usually present in one or both parents.
- 5. The greatest risk factor for Necrotizing enterocolitis is
- a. Low birth weight
- b. Intestinal ischemia
- c. Enteral nutrition
- d. Prematurity
- Ans: d
- Ref: Nelson 20th edition page 869
- Although NEC is a multifactorial disease primarily associated with intestinal immaturity, the concept of “risk factors” for NEC is controversial. The triad of intestinal ischemia (injury), enteral nutrition (metabolic substrate), and bacterial translocation has classically been linked to NEC.
- The greatest risk factor for NEC is prematurity. The disorder probably results from an interaction between loss of mucosal integrity due to a variety of factors (ischemia, infection, inflammation) and the host’s response to that injury (circulatory, immunologic, inflammatory), leading to necrosis of the affected area. Coagulation necrosis is the characteristic histologic finding in intestinal specimens.
-
7
- 1. A 19-year-old woman in her first pregnancy presents to you with an outbreak of primary herpes simplex infection on her labia. She is 33 weeks pregnant. What is the best advice regarding her herpes?
- a. Acyclovir from 36 weeks until delivery
- b. Caesarean section should be performed if she labors within the next 8 weeks
- c. Reassure as the infection will pass and pose no further concern
- d. If she labors within 6 weeks, a caesarean should be recommended
- Ans: d
- If primary herpes develops in pregnancy it is imperative to consider risk of vertical transmission. If the herpes presents at the time of delivery or within 6 weeks of the due date a caesarean is the safest mode of delivery.
- If she labours within 6 weeks (not 8 weeks) of developing primary herpes she should consider a caesarean. If she refuses to have a caesarean then IV acyclovir during labour and close liaison with the neonatologist is recommended.
- You can reassure the woman but only if she delivers after 6 weeks when the herpes should have cleared. It would be wrong to tell her that all will be well regardless of the timing of delivery.
- 2. A 12-year-old boy presents to his GP with left-sided unilateral breast development stage III. He is very upset as he is being bullied at school. His mother is worried as her friend’s sister has just been diagnosed with breast cancer and wants to know if he could have breast cancer? What is the management?
- a. Refer for a breast ultrasound
- b. Test alpha fetoprotein
- c. Reassure and explain this is a normal part of puberty; it will resolve but the other breast may enlarge transiently as well
- d. Do a fine needle aspirate on his left breast
- Ans: c
- This is a common presentation for adolescent males and the correct answer is to reassure. Normal puberty for boys starts between 9 and 13 years old with increasing testicular volume above 4 mL. This is followed by penis enlargement, pubic hair growth and lastly the growth spurt. It is not uncommon for boys to develop transient gynaecomastia during puberty. He does not need investigation or intervention, unless the problem does not resolve.
- 3. Which of the following drug is not used in TOF?
- a. Isoprenaline
- b. Sodium bicarbonate
- c. Morphine
- d. Propranolol
- Ans: a
- Ref: OP Ghai Pediatrics 9th Edition, Page No: 408
- Cyanotic spells/ Tet spells are acute hypoxemic attacks in tetralogy of Fallot.
- Treatment of Tet spells is done by following methods
- Knee chest positioning or squatting- increases systemic vascular resistance at femoral veins & decreases systemic venous return.
- Morphine: suppress respiratory center & abolishes hyperpnea
- Oxygen : Acidosis if any should be treated with NaHCO3 intravenously.
- Propranolol: Propranolol by decreasing cardiac contractility may decrease infundibular obstruction of right ventricular outflow.
- Prevention
- Counsel parents regarding the possibility of recurrence of spells and precipitating factors (dehydration, fever, pain)
- Encourage early surgical repair
- 4. Expectant management in ectopic pregnancy is done if:
- a. Initial beta hCG level <1000 IU/ml & falling
- b. Initial beta hCG level <2000 IU/ml & falling
- c. Diameter of sac <5 cm
- d. All of the above
- Ans: a
- Ref: Dutta Obstetrics 8th Edition, Page No: 216
- Expectant management: where only observation is done hoping spontaneous resolution.
- Indications are:
- Initial beta–hCG level < 1000 IU/ml & falling
- Gestational sac size < 4 cm
- No fetal heart beat on TVS
- No evidence of bleeding or rupture
- 5. Pseudoparalysis In infant is suggestive of:
- a. Vitamin K deficiency
- b. Vitamin A deficiency
- c. Vitamin C deficiency
- d. Vitamin B deficiency
- Ans: c
- Ref: OP Ghai Pediatrics 8th Edition, Page No: 120
- Vitamin C (ascorbic acid), structurally related to glucose, has a striking capacity for reversible oxidation-reduction. Humans and other primates do not synthesize vitamin C.
- Sources
- • Dietary sources include vegetables (cauliflower, broccoli, cabbage) and fruits (berries, citrus). Much of vitamin C may be lost in cooking, but is stable in canned and frozen foods. Vitamin C in human and cow milk ranges from 5 to 15 mg/100 kcal and 0.2-2.0 mg/100 kcal respectively. Daily requirements are 30-40 mg for infants and 40-70 mg for children.
- Absorption and metabolism
- • Ascorbic acid is absorbed by an active, sodium-dependent process in the upper small intestine. The vitamin circulates in plasma in its free, anionic form, reaching high concentrations in adrenal and pituitary glands and in leukocytes. Vitamin C appears unchanged in the urine when renal threshold is exceeded.
- Biologic action
- • Vitamin C functions as a strong reducing agent or in electron transport within biological systems. Ascorbic acid is essential for normal function of leukocytes, fibroblasts, osteoblasts and microsomes and participates in metabolism of carnitine, serotonin and folate. Ascorbic acid affects the immune response, detoxification, collagen synthesis and wound healing.
- Deficiency
- • Prolonged vitamin C deficiency results in scurvy. It usually occurs in those who are deprived of citrus fruits, fresh vegetables or vitamins for some cultural or geographic reasons. In infancy, features of scurvy are anorexia, diarrhea, pallor, irritability and increased susceptibility to infections. Subperiosteal hemorrhages and long bone tenderness (pseudoparalysis of lower extremities) can occur; radiologic abnormalities are frequent. In older children, hemorrhagic signs predominate, with bleeding from gums, conjunctiva and intestinal tract.
- 6. What is the earliest most common presenting feature of anterior cervical fibroid?
- a. Frequency of urine
- b. Bleeding
- c. Acute abdomen
- d. Constipation
- Ans: a
- Ref: Dutta Gynecology 6th Edition, Page No: 284
- Cervical fibroid
- Symptoms
- In nonpregnant state, the symptoms are predominantly due to pressure effect on the surrounding structures.
- • Anterior cervical: Bladder symptoms like frequency or even retention of urine are conspicuous. The retention is more due to pressure than elongation of urethra.
- • Posterior cervical: Rectal symptom is in the form of constipation.
- • Lateral cervical: Vascular obstruction may lead to hemorrhoids and edema legs (rare). The ureter is pushed laterally and below the tumor.
-
8
- 1. What is the defect in Prader-Willi syndrome?
- a. Maternally derived chromosome 12
- b. Paternally derived chromosome 12
- c. Maternally derived chromosome 15
- d. Paternally derived chromosome 15
- Ans: d
- Ref: Nelson Pediatrics, 20th edition, page 602 – 603
- • The 2 copies of most autosomal genes are functionally equivalent. However, in some cases only 1 copy of a gene is transcribed and the other copy is silenced.
- • In imprinting, gene expression depends on the parent of origin of the chromosome. Imprinting disorders result from an imbalance of active copies of a given gene, which can occur for several reasons. Prader-Willi and Angelman syndromes, two distinct disorders associated with developmental impairment, are illustrative. Both can be caused by microdeletions of chromosome 15q11-12.
- • The microdeletion in Prader-Willi syndrome is always on the paternally derived chromosome 15, whereas in Angelman syndrome it is on the maternal copy.
- 2. Gartner's duct cyst is seen in:
- a. Vagina
- b. Cervix
- c. Hymen
- d. Labia minora
- Ans: a
- Ref: Shaw Gynecology 16th Edition, Page No: 388, 389
- Vaginal Cysts
- The vaginal cyst is rare, and most commonly located in the anterior vaginal wall. They are usually small, but may attain a size of 7.5 cm in diameter.
- • Gartner’s duct cyst arises from the remnants of the meso¬nephric duct and lies in the anterolateral aspect of the vaginal wall. A small cyst remains asymptomatic. A large cyst if causing dyspareunia requires excision.
- • Inclusion cyst is mainly seen at the lower end of the vagina on its posterior surface and is caused by tags of mucosa embedding inside the scar those later forms a cyst.
- • Bartholin cyst at times extends into the vagina and causes dyspareunia.
- • Endometriotic cyst appears as a bluish bulge in the posterior fornix. It behaves similar to endometriotic cyst of the vulva. It is treated with either danazol or surgical excision.
- 3. According to WHO, what does X2 denote in xerophthalmia in Vitamin A deficiency
- a. Conjunctival xerosis
- b. Corneal xerosis
- c. Bitot’s spot
- d. Corneal ulcer
- Ans: b
- Ref: AK khurana, 6thedition,page 468
- First clinical sign of Vitamin A deficiency is conjunctival xerosis
- First clinical symptom of Vitamin A deficiency is night blindness.
- WHO classification of xerophthalmia
- Class Signs
- X1A Conjunctival xerosis
- X1B Bitot’s spot
- X2 Corneal xerosis
- X3A Corneal xerosis with ulceration
- X3B Keratomalacia
- 4. If you were told by a colleague that their assessment of a patient’s Glasgow Coma Scale was E2, V3, M5, how would you interpret this?
- a. The patient’s eyes open to sound, they are orientated are able to obey commands
- b. The patient’s eyes open to pain, they can utter some words but do not form sentences, and they are able to localise to trapezius pinch.
- c. The patient’s eyes open spontaneously; they are orientated and able to obey commands
- d. The patient’s eyes open spontaneously; they are orientated and they are able to localise to trapezius pinch.
- Ans: b
- Ref: Bailey and Love Surgery, 27th Edition, Page 325
- If the colleague has used the numbers correctly they mean that the patient’s eyes open to pain, they can utter some words but do not form sentences, and they are able to localise to trapezius pinch. But you should ask them to confirm this is the case by using the words, which should be used in preference to numbers for this purpose.
- • Eye (E): 1-4: Does not open eyes ---Opens eyes in response to pain---Opens eyes in response to voice---Opens eyes spontaneously
- • Verbal ( V ) : 1-5: Makes no sounds---Makes sounds---Words---Confused, disoriented---Oriented, converses normally
- • Motor (M): 1-6: Makes no movements---Extension to painful stimuli (decerebrate response)---Abnormal flexion to painful stimuli (decorticate response)---Flexion / Withdrawal to painful stimuli---Localizes to painful stimuli---Obeys commands
- 5. Carvallo’s sign is related to which of the following:
- a. Aortic regurgitation
- b. Mitral regurgitation
- c. Tricuspid regurgitation
- d. Pericarditis
- Ans: c
- Ref: Harrison Medicine 19th Edition, Page No: 1548
- A prominent RV pulsation along the left parasternal region and a blowing holosystolic murmur along the lower left sternal margin, which may be intensified during inspiration & reduced during expiration or the strain of the Valsalva maneuver (Carvallo’s sign), are characteristic findings of TR.
☆ [@ ViT C ; Vitum sign and Carvello's sign seen in TR]
-
9
- 1. In systemic inflammatory response syndrome:
- a. Respiratory rate < 10/min
- b. Heart rate > 90/min
- c. White blood count < 6 × 109/L
- d. PaCO2 normal
- Ans: b
- Ref: Davidson’s Medicine 22ndd Edition, page No: 184
- Systemic inflammatory response syndrome (SIRS)
- • Defined by the presence of two or more of:
- o Respiratory rate > 20/min
- o Heart rate > 90/min
- o White blood count > 12 × 109/L or < 4 × 109/L
- o Temperature > 38.0°C or < 36.0°C
- o PaCO2 < 4.3 kPa (< 32 mmHg) or ventilated
- • A wide pulse pressure, e.g. 115/42 mmHg, may be an early pointer to systemic sepsis
- • Cause may be infection or a non-infective condition, e.g. pancreatitis, trauma, cardiopulmonary bypass, vasculitis etc.
- • Hypothermia and septic neutropenia indicate more severe infection
- 2. All of the following statements concerning the ureters are correct except:
- a. They are retroperitoneal.
- b. They cross the pelvic brim anterior to the bifurcation of the common iliac arteries.
- c. They are fibrous tubes connecting the kidneys to the urinary bladder.
- d. Their superior halves lie in the abdomen and the inferior halves lie in the pelvis.
- Ans: c
- Ref: Moore's Anatomy 7th Edition, Page No: 363
- The ureters are muscular tubes, 25 to 30 cm long, that connect the kidneys to the urinary bladder. They are retroperitoneal, with their superior halves in the abdomen and their inferior halves in the pelvis. The pelvic part of the ureters begins where it crosses the bifurcation of the common iliac artery.
- 3. The area is declared free of epidemic:
- a. Till last secondary case recovers
- b. No new case reported for the incubation period of disease since the last case
- c. No new case reported for twice the incubation period of disease since the last case
- d. No new case reported for six months since the last case
- Ans: c
- Ref: Park PSM 23rd Edition, Page No: 132
- The search for new cases (secondary cases) should be carried out every day, till the area is declared free of epidemic. This period is usually taken as twice the incubation period of the disease since the occurrence of last case.
- 4. Satellite lesion in keratitis occurs due to:
- a. Pneumococcus
- b. Aspergillus
- c. Herpes zoster
- d. Herpes simplex virus
- Ans: B
- Ref: AK Khurana Ophthalmology 6th Edition, Page No: 106
- A typical fungal corneal ulcer has following salient features :
- • Corneal ulcer is dry-looking, greyish white, with elevated rolled out margins.
- • Pigmented ulcer (brownish) may be caused by some species of fungi, e.g., dermatiaceous fungi.
- • Delicate feathery finger-like extensions are present into the surrounding stroma under the intact epithelium.
- • A sterile immune ring (yellow line of demarcation) may be present where fungal antigen and host antibodies meet.
- • Multiple, small satellite lesions may be present around the ulcer.
- • Usually a big hypopyon is present even if the ulcer is very small. Unlike bacterial ulcer, the hypopyon may not be sterile as the fungi can penetrate into the anterior chamber without perforation.
- • Endothelial plaque, composed of fibrin and leucocytes, may be located under the stromal lesion. It may be present in the absence of hypopyon.
- • Perforation in mycotic ulcer is rare but can occur.
- • Corneal vascularization is conspicuously absent in pure mycotic ulcer.
- 5. ATPase activity is present in
- a. Myosin head
- b. Myosin tail
- c. Actin filaments
- d. Desmin filaments
- Ans: a
- Ref: Ganong physiology 25th Edition, Page No: 104
- Before contraction begins, the heads of the cross bridges binds with ATP. The ATPase activity of the myosin head immediately cleaves the ATP but leaves the cleavage products, ADP plus phosphate ion, bound to the head. The sliding during muscle contractions occurs when the myosin heads bind firmly to actin, bend at the junction of the head with the neck, and then detach. The power stroke depends on the simultaneous hydrolysis of ATP.
- 6. Twitch of a single motor unit is called
- a. Myoclonic jerk
- b. Fasciculation
- c. Tremor
- d. Chorea
- Ans: b
- Ref: Ganong physiology 25th Edition, Page No: 110
- • Fasciculations are jerky, visible contractions of group of muscle fibres that occur as a result of pathologic discharge of spinal motor neurons.
- • Fibrillations are fine, irregular contractions of individual muscle fibres. They may occur due to denervation hypersensitivity resulting from LMN lesion. Usually the contractions are not visible grossly.
- • Myoclonic jerk or clonus is the regular, rhythmic contractions of a muscle subjected to sudde maintained stretch. It is due to increased Gamma efferent discharge.
-
10
- 1. Liver biopsy of biliary atresia shows:
- a. Proliferation of small bile ducts
- b. Atrophy of small bile ducts
- c. Fibrosis and calcifications of small bile ducts
- d. All of the above
Ans: a
- 2. DOG-1 is marker of tumour of which organ?
- a. Prostate
- b. Lung
- c. Liver
- d. Intestine
Ans: d
- 3. Vaginal pH after menopause is:
- a. 3-4
- b. 5
- c. 5-6
- d. 7
Ans: d
- 4. Lamellar cataract is seen in:
- a. Hypoglycemia
- b. Hyperglycemia
- c. Hypocalcemia
- d. Hypercalcemia
Ans: a
- 5. Rocher’s sign is helpful in differentiating:
- a. Breast abscess from mastitis
- b. Torsion of testis and epididymitis
- c. Appendicitis from Diverticulitis
- d. Intussusception from Hirschsprung disease
Ans: b
-
11
- 1. IV loading dose of MgSO4 Prophylaxis in Pre eclampsia is?
- a. 8 ml MgSO4 + 10 ml of NS
- b. 10 ml MgSO4 + 10 ml of NS
- c. 8 ml MgSO4 + 12 ml of NS
- d. 12 ml of MgSO4 + 8 ml of NS
- Ans: c
- Ref: Dutta Obstetrics 8th Edition, Page No: 273
- Pritchard regimen:
- Loading dose: IV: 4 gm (20 % solution)/IM 10 gm (50%), 5 gm in each buttock
- 1 vial of MgSO4 contains = 2 ml solution
- = 1 gm MgSO4 (w/v 50%)
- Question is asking about= IV loading dose, means: 4gm in 20 %
- 4 vials needed for 4 gm
- 4 vials = 8 ml of MgSO4 in 50 %
- = Add 12 ml NS to make it 20 % (simple calculation)
- 2. A 23-year-old male basketball player injured his foot during training and is admitted to the emergency department. An MRI examination reveals a hematoma in the medial malleolus. Upon physical examination the patient shows overeversion of his foot. Which of the following ligaments most likely has a tear?
- a. Plantar calcaneonavicular (spring)
- b. Calcaneofibular
- c. Long plantar
- d. Deltoid
- Ans: d
- Ref: Gray’s Anatomy for Students, 605, 606
- • One important function of the deltoid ligament is the prevention of overeversion of the ankle. The ligament is so strong that excessive eversion can cause the medial malleolus to be pulled off (an avulsion fracture) rather than tearing the deltoid ligament.
- • Deltoid ligament of the ankle: The complex of the medial collateral ligaments of the ankle joint is collectively called deltoid ligament. It attaches the medial malleolus to multiple tarsal bones.
- • The ligament is composed of two layers. The superficial layer has variable attachments and crosses two joints while the deep layer has talar attachments and crosses one joint: It is a strong triangular band attached to the apex and the anterior and posterior borders of the medial malleolus. The plantar calcaneonavicular ligament can be considered as part of the medial complex.
- 3. A patient tries to make a fist, but digits 2 and 3 remain partially extended. What nerve is injured?
- a. Ulnar nerve
- b. Radial nerve
- c. Median nerve
- d. Musculocutaneous nerve
- Ans: c
- Ref: Moore's Anatomy 7th Edition, Page No: 768
- • When the median nerve is severed in the elbow region, flexion of the proximal interphalangeal joints of the 1st–3rd digits is lost and flexion of the 4th and 5th digits is weakened. Flexion of the distal interphalangeal joints of the 2nd and 3rd digits is also lost. Flexion of the distal interphalangeal joints of the 4th and 5th digits is not affected because the medial part of the FDP, which produces these movements, is supplied by the ulnar nerve. The ability to flex the metacarpophalangeal joints of the 2nd and 3rd digits is affected because the digital branches of the median nerve supply the 1st and 2nd lumbricals. Thus, when the person attempts to make a fist, the 2nd and 3rd fingers remain partially extended (“hand of benediction”).
- • Thenar muscle function (function of the muscles at the base of the thumb) is also lost, as in carpal tunnel syndrome. When the anterior interosseous nerve is injured, the thenar muscles are unaffected, but paresis (partial paralysis) of the flexor digitorum profundus and flexor pollicis longus occurs. When the person attempts to make the “okay” sign, opposing the tip of the thumb and index finger in a circle, a “pinch” posture of the hand results instead owing to the absence of flexion of the interphalangeal joint of the thumb and distal interphalangeal joint of the index finger (anterior interosseous syndrome).
- 4. Microglia is derived from:
- a. Ectoderm
- b. Mesenchymal cells
- c. Neural crest cells
- d. Endoderm
- Ans: b
- Ref: Ginhoux F, Prinz M (July 2015). "Origin of microglia: current concepts and past controversies". Cold Spring Harbor Perspectives in Biology. 7
- For a long time it was thought that microglial cells differentiate in the bone marrow from hematopoietic stem cells, the progenitors of all blood cells. However, recent studies show that microglia originate in the yolk sac during a remarkably restricted embryonal period and populate the brain mesenchyme. Additionally, they continuously renew themselves and persist throughout life without replenishment from peripheral monocytic precursors.
- 5. Glucose transport by GLUT is an example of:
- a. Primary active transport
- b. Secondary active transport
- c. Facilitated diffusion
- d. Active transport
- Ans: c
- Ref: Ganong physiology 25th Edition, Page No: 48
- • Carriers are transport proteins that bind ions and other molecules and then change their configuration, moving the bound molecule from one side of the cell membrane to the other.
- • Molecules move from areas of high concentration to areas of low concentration (down their chemical gradient), and cations move to negatively charged areas whereas anions move to positively charged areas (down their electrical gradient).
- • When carrier proteins move substances in the direction of their chemical or electrical gradients, no energy input is required and the process is called facilitated diffusion. A typical example is glucose transport by the glucose transporter, which moves glucose down its concentration gradient from the ECF to the cytoplasm of the cell.
- • Other carriers transport substances against their electrical and chemical gradients. This form of transport requires energy and is called active transport. In animal cells, the energy is provided almost exclusively by hydrolysis of ATP. Not surprisingly, therefore, many carrier molecules are ATPases, enzymes that catalyze the hydrolysis of ATP.
- • One of these ATPases is sodium–potassium adenosine triphosphatase (Na, K ATPase), which is also known as the Na, K pump. There are also H, K ATPases in the gastric mucosa and the renal tubules. Ca2+ ATPase pumps Ca2+ out of cells.
-
12
- 1. A 22-year-old male is admitted to the hospital with groin pain and blood in the semen. An MRI examination and biopsy reveal testicular cancer. Which of the following lymph nodes will be first involved in case of metastasis?
- a. Internal iliac
- b. Superficial inguinal
- c. Deep inguinal
- d. Paraaortic and lumbar
- Ans: d
- Ref: Gray’s Anatomy for Students, 477, 496
- The paraaortic and lumbar nodes at the level of the kidneys will most likely be infiltrated by metastasis of testicular cancer because testicular lymphatics run in close association with the testicular vessels and drain the testicles and epididymis.
- Testicular cancer is said to be a disease that is especially dangerous for young men. The internal iliac nodes drain the inferior pelvis and deep perineal structures. The external iliac nodes drain all anterosuperior pelvic structures. The superficial inguinal nodes drain all of the superficial structures below the umbilicus. Finally, the deep inguinal nodes receive more superficial vessels and drain the glans penis in males.
- 2. The head of the pancreas is embraced by which of the following structures?
- a. Stomach
- b. Spleen
- c. Cecum
- d. C-shaped curve of the duodenum
- Ans: d
- Ref: Moore's Anatomy 7th Edition, Page No: 257
- The head of the pancreas is embraced by the C-shaped curve of the duodenum to the right of the superior mesenteric vessels.
- 3. The superior end of the vagina surrounds which of the following structures?
- a. Round ligament
- b. Urogenital hiatus
- c. Urethra
- d. Cervix
- Ans: d
- Ref: Moore's Anatomy 7th Edition, Page No: 371
- The superior end of the vagina surrounds the cervix; the lower end passes anteroinferiorly through the pelvic floor to open in the vestibule.
- 4. Pacemaker regulating the rate of respiration is:
- a. Pneumotaxic center
- b. Dorsal group of nucleus
- c. Apneustic center
- d. Pre-Bötzinger
- Ans: d
- Ref: Ganong Physiology 24th Edition, Page No: 658
- Pacemaker of respiration is pre-Bötzinger complex of neurons, which is a part of VRG in medulla
- • The main components of the respiratory control pattern generator responsible for automatic respiration are located in the medulla.
- • Rhythmic respiration is initiated by a small group of synaptically coupled pacemaker cells in the pre-Bötzinger complex (pre-BÖTC) on either side of the medulla between the nucleus ambiguus and the lateral reticular nucleus.
- • These neurons discharge rhythmically, and they produce rhythmic discharges in phrenic motor neurons that are abolished by sections between the pre-Bötzinger complex and these motor neurons. They also contact the hypoglossal nuclei, and the tongue is involved in the regulation of airway resistance.
- 5. Prime driving force for counter current multiplier system is:
- a. Medullary hyperosmolarity
- b. Reabsorption of Na+ in thick ascending limb
- c. Action of ADH via Aquaporin channels
- d. Urea recycling
- Ans: b
- Ref: Guyton and Hall, 13th Edition, Page No: 374
- The major factors that contribute to the buildup of solute concentration into the renal medulla are as follows:
- • Active transport of sodium ions and co-transport of potassium, chloride, and other ions out of the thick portion of the ascending limb of the loop of Henle into the medullary interstitium
- • Active transport of ions from the collecting ducts into the medullary interstitium
- • Facilitated diffusion of urea from the inner medullary collecting ducts into the medullary interstitium
- • Diffusion of only small amounts of water from the medullary tubules into the medullary interstitium— far less than the reabsorption of solutes into the medullary interstitium
-
13
- 1. Posterior herniation of the synovial membrane in the popliteal fossa is:
- a. Semi-membranous bursa
- b. Clergyman’s knee
- c. Housemaid’s knee
- d. Morrant-Baker's cyst
Ans: D
- Different bursae in relation to the knee, which may present as swelling are:
- • Semi-membranous bursa causes painless oval swelling at the postero-medial aspect of the knee;
- • Infra-patellar bursa (Clergyman’s knee) lying deep to the ligamentum patellae;
- • Pre-patellar bursa (Housemaid’s knee) lying in front of the patella;
- • Morrant-Baker's cyst – a posterior herniation of the synovial membrane in the popliteal fossa.
- 2. How should chest compressions be performed on an infant?
- a. With the heel of one hand and the other hand on top of the first
- b. With the heel of one hand only
- c. With 4 fingers of one hand
- d. With 2 fingers of one hand
- Ans: d
- Compressions: Infants (<1 year, excluding newborns)
- • Check pulse at brachial artery
- • Compression landmarks: lower third of sternum between the nipples
- • Compression method: two fingers if alone or thumb-encircling if multiple providers
- • Depth: at least one-third AP chest diameter
- • Depth: about 1.5 inches (4 cm)
- • Allow complete chest recoil after each compression.
- • Compression rate: 100-120 per minute
- • Compressions-to-ventilations ratio: 30:2 if single rescuer, 15:2 if multiple rescuers
- • Continuous compressions if advanced airway present and timed ventilation
- • Rotate compressor every 2 minutes or if fatigued.
- • Minimize interruptions in compressions to less than 10 seconds.
- • Avoid excessive ventilation.
- Compressions: Children aged 1 year to puberty
- • Check pulse at carotid artery.
- • Compression landmarks: lower half of sternum between the nipples
- • Compression method: heel of one hand, other hand on top if needed
- • Depth: at least one-third anteroposterior (AP) chest diameter
- • Depth: about 2 inches (5 cm)
- • Allow complete chest recoil after each compression
- • Compression rate: 100-120 per minute
- • Compressions-to-ventilations ratio: 30:2 if single rescuer, 15:2 if multiple rescuers
- • Continuous compressions if advanced airway present and asynchronous ventilation
- • Rotate compressor every 2 minutes or if fatigued.
- • Minimize interruptions in compressions to less than 10 seconds.
- • Avoid excessive ventilation.
- 3. Imperforate hymen is usually noticed at what age?
- a. Prenatally
- b. At birth
- c. 5 – 6 years
- d. 14 – 16 years
- Ans: d
- Ref: Dutta’s Gynecology, 6th Revised Edition, Page 41
- • Gross hymenal abnormality of significance is imperforate hymen. It is due to failure of disintegration of the central cells of the Müllerian eminence that projects into the urogenital sinus
- • The existence is almost always unnoticed until the girl attains the age of 14–16 years. As the uterus is functioning normally, the menstrual blood is pent up inside the vagina behind the hymen (cryptomenorrhea).
- • Depending upon the amount of blood so accumulated, it first distends the vagina (hematocolpos). The uterus is next involved and the cavity is dilated (hematometra). In the late and neglected cases, the tubes may also be distended after the fimbrial ends are closed by adhesions (Hematosalpinx)
- 4. What is the total conduction delay in A-V node &bundle system of heart?
- a. 0.04 second
- b. 0.09 second
- c. 0.13 second
- d. 0.16 second
- Ans: c
- Nodal delay: 0.09 s
- Nodal and bundle delay: 0.13 s
- Ref: Guyton’s Physiology 13th Edition, Page No: 125
- • The impulse generated in SA node after traveling through the intermodal pathways, reaches the A-V node in about 0.03 second.
- • Then there is a delay of another 0.09 second in the A-V node itself before the impulse enters the penetrating portion of the A-V bundle, where it passes into the ventricles.
- • A final delay of another 0.04 second occurs mainly in this penetrating A-V bundle, which is composed of multiple small fascicles passing through the fibrous tissue separating the atria from the ventricles.
- • Thus, the total delay in the A-V nodal and A-V bundle system is about 0.13 second.
- • This delay, in addition to the initial conduction delay of 0.03 second from the sinus node to the A-V node, makes a total delay of 0.16 second before the excitatory signal finally reaches the contracting muscles of the ventricle.
- 5. A 20-year-old man complains of difficulty in reading newspaper in the right eye 4 weeks after gunshot injury in the left eye. The likely diagnosis is:
- a. Macular edema
- b. Sympathetic ophthalmitis
- c. Optic nerve avulsion
- d. Delayed vitreous hemorrhage
- Ans: b
- Ref: Parson Ophthalmology 22nd Edition, Page No: 398
- Sympathetic ophthalmitis
- • This is a condition in which serious inflammation attacks the sound eye after injury (including intraocular surgeries) to the other.
- • Sympathetic ophthalmitis or ophthalmia almost always results from a penetrating wound, occurring in 0.2–0.5% of such cases.
- • Wounds involving the ciliary body and leading to its incarceration in the scar, have always been considered particularly dangerous.
- • Incarceration of the iris or lens capsule are also more likely to set up sympathetic ophthalmitis than others.
-
14
- 1. Duramater is supplied by all cranial nerves except:
- a. 12
- b. 10
- c. 5
- d. 4
- Ans: d
- Ref: Moore's Anatomy 7th Edition, Page No: 865, 872
- The cranial dura mater (dura), a thick, dense, bilaminar membrane, is also called the pachymeninx (G. pachy, thick + G. menix, membrane).
- It is adherent to the internal table of the calvaria. The two layers of the cranial dura are an external periosteal layer, formed by the periosteum covering the internal surface of the calvaria, and an internal meningeal layer, a strong fibrous membrane that is continuous at the foramen magnum with the spinal dura covering the spinal cord.
- The external periosteal layer of dura adheres to the internal surface of the cranium; its attachment is tenacious along the suture lines and in the cranial base. The external periosteal layer is continuous at the cranial foramina with the periosteum on the external surface of the calvaria.
- This outer layer is not continuous with the dura mater of the spinal cord, which consists of only a meningeal layer.
- Innervation of dura mater is derived mainly from three sources:
- 1) Three divisions of trigeminal nerve
- 2) C2 & 3 spinal nerves and
- 3) Cervical sympathetic trunk.
- Vagus and hypoglossal nerves, and possibly the facial and glossopharyngeal nerves may also supply duramater (less established).
- 2. Phlebotomus argentipes is killed by:
- a. Malathion
- b. Pyrethrum
- c. DDT
- d. Parathion
- Ans: c
- Ref: Park PSM 23rd Edition, Page No: 308
- Sandfly control
- • The application of residual insecticides has proved effective in the control of sand flies. DDT is the first choice since the vector of kala-azar, P. argentipes is susceptible to DDT. Insecticide spraying should be undertaken in human dwellings, animal shelters and all other resting places upto a height of 6 feet (2 metres) from floor level. DDT (two rounds per year) at the rate of 1-2 g per sq. metre is considered sufficient to control transmission. Spraying should be preceded and followed by an assessment of susceptibility. Any sign of resistance in vector should lead to an immediate change in insecticide. BHC should be kept as a second line of defence.
- • Spraying should be repeated at regular intervals to keep down the density of sandflies. For long-lasting results, insecticidal spraying should be combined with sanitation measures, viz elimination of breeding places (e.g., cracks in mud or stone walls, rodent burrows, removal of firewood, bricks or rubbish around houses), location of cattle sheds and poultry at a fair distance from human dwellings, and improvement of housing and general sanitation.
- 3. Coarse facies, hepatosplenomegaly and cloudy cornea are characteristic feature of:
- a. Morquio syndrome A
- b. Hurler’s disease
- c. Hunters disease
- d. Natowicz syndrome
- Ans: B
- Ref: Harper Biochemistry 30th Edition, Page No: 639
- • Hurler disease: Mental retardation, coarse facial features, hepatosplenomegaly, cloudy cornea
- • Hunter disease: Mental retardation
- • Morquio syndrome A : Skeletal dysplasia, short stature
- • Natowicz syndrome: Joint pain, short stature
- 4. A female neonate on 10th day presented with vaginal bleeding. What would be the next step of management?
- a. Estrogen
- b. Progesterone
- c. OCP
- d. Reassurance and do nothing
- Ans: d
- Ref: Dutta’s Textbook of Gynaecology including Contraception, 6th edition, page 543
- • Bleeding per vaginum (mostly blood stained to frank bleeding at times) usually occurs in newborns within 10 days following birth.
- • It occurs due to decline in level of estrogen, which is unable to support the endometrium, resulting in withdrawal bleeding.
- • There is no cause for concern and patient party can be simply reassured.
- 5. Pilon fracture is:
- a. Avulsion fracture of lateral tibial condyle; associated with ACL injury
- b. Intraarticular fracture of tibial condyle
- c. Intraarticular comminuted distal tibia fracture
- d. Salter-Harris III of lateral distal tibia (because of lateral epiphyseal fusion)
- Ans: c
- Ref: Apley’s Orthopedics, 9th Edition; Page No: 916
- PILON FRACTURES
- Unlike the twisting injuries that cause the common ankle fractures, this injury to the ankle joint occurs when a large force drives the talus upwards against the tibial plafond, like a pestle (pilon) being struck into a mortar. There is considerable damage to the articular cartilage and the subchondral bone may be broken into several pieces; in severe cases, the comminution extends some way up the shaft of the tibia.
- • Segond: Avulsion fracture of lateral tibial condyle; associated with ACL injury
- • Bumper: Intraarticular fracture of tibial condyle
- • Pilon: Intraarticular comminuted distal tibia fracture
- • Tillaux: Salter-Harris III of lateral distal tibia (because of lateral epiphyseal fusion)
- • Triplane: Salter III/IV fracture of distal tibia
-
15
- 1. A young patient presents with lethargy and malaise, bone pain, bleeding gums and impaired wound healing. Definitive sign of the disease in X-ray will be:
- a. Ringed epiphysis
- b. Ground glass appearance
- c. White line in metaphysis
- d. Thin cortex
- Ans: a
- Ref: OP Ghai 9th Edition, Page No: 121
- Diagnosis of scurvy: The diagnosis is made by presence of characteristic physical findings and history of inadequate dietary intake of vitamin C. X-rays of long bones show a ground glass appearance with thinning of cortex (pencil thin cortex). An irregular thickened white line appears at the metaphysis (white line of Frankel), representing the zone of well-calcified cartilage. There is a zone of rarefaction proximal to this line, which represents poorly formed trabeculae (Triimmerfeld zone). The lateral part of the rarefaction appears as a triangular defect called Pelken spurs. The epiphyses are surrounded by a thin white line (Wimberger ring sign)
- • Ringed epiphysis (Wimberger sign): It is white ring surrounding the epiphyseal centers of ossification. It is relatively specific for scurvy.
- • White line in metaphysis (Fraenkel) can be seen in scurvy, healing rickets, Plumbism, severe PEM, congenital syphilis and acute leukemia.
- 2. For a screening test, 90 percent specificity means that 90 % of non diseased person will give:
- a. True positive
- b. True negative
- c. False positive
- d. False negative
- Ans: b
- Ref: Park PSM; 23rd edition; Pg 139
- Sensitivity
- • The term sensitivity was introduced by Yerushalmyin 1940s as a statistical index of diagnostic accuracy.
- • It has been defined as the ability of a test to identify correctly all those who have the disease, that is "true-positive". A 90 percent sensitivity means that 90 per cent of the diseased people screened by the test will give a "true-positive" result and the remaining 10 per cent a "false-negative" result.
- Specificity
- • It is defined as the ability of a test to identify correctly those who do not have the disease, that is, "true-negatives".
- • A 90 per cent specificity means that 90 per cent of the non-diseased persons will give "true-negative" result, 10 percent of non-diseased people screened by the test will be wrongly classified as "diseased" when they are not.
- 3. Typical deformity in anterior hip dislocation is:
- a. Flexion, adduction, internal rotation
- b. Abduction, external rotation
- c. Flexion, abduction, internal rotation
- d. Abduction, internal rotation
- Ans: b
- Ref: Maheshwari Orthopedics 5th Edition Page No: 56
- Typical deformities in dislocations
- • Shoulder (anterior): Abduction
- • Elbow (posterior): Flexion
- • Hip:
- Posterior: Flexion, adduction, internal rotation
- Anterior: Abduction, external rotation
- • Knee: Flexion, external rotation
- • Ankle: Varus
- 4. Rose Waaler test is done in:
- a. SLE
- b. Rheumatic fever
- c. Rheumatoid arthritis
- d. Polymyositis
- Ans: C
- Ref: Surinder singh microbiology; 1st edition; Pg155
- Rose-Waaler Test
- A special type of passive agglutination test is the Rose-Waaler test. In rheumatoid arthriris, RA factor (an anti-gammaglobulin autoantibody) appears in the serum. It acts as antibody to human IgG. The RA factor is able to agglutinate red cells coated with globulins. The antigen used for the test is a suspension of sheep erythrocytes sensitized with a sub agglutinating dose of rabbit anti-sheep erythrocyte antibody (amboceptor).
- 5. Tuohy needle is used for:
- a. Spinal anesthesia
- b. CSF tapping
- c. Biopsy
- d. Epidural anesthesia
- Ans: D
- Ref: Morgan 5th edition Page No: 959
- Epidural Needles
- The standard epidural needle is typically 17–18 gauge, 3 or 3.5 inches long, and has a blunt bevel with a gentle curve of 15–30° at the tip. The Tuohy needle is most commonly used .The blunt, curved tip theoretically helps to push away the dura after passing through the ligamentum flavum instead of penetrating it. Straight needles without a curved tip (Crawford needles) may have a greater incidence of dural puncture, but facilitate passage of an epidural catheter. Needle modifications include winged tips and introducer devices set into the hub designed for guiding catheter placement.
-
16
- 1. Ankle sprain due to forced inversion of a plantar flexed foot is due to injury to:
- a. Posterior talofibular ligament
- b. Calcaneofibular ligament
- c. Posterior fibres of deltoid
- d. Anterior talofibular ligament
- Ans. d. Anterior talofibular ligament
- Ref: Apley 8/e p733-736
- Over 90% of ankle ligament injuries (twisted ankle or ankle sprain) involve the lateral ligament complex, usually the anterior tibiofibular ligament.
- A. Medial Collateral Ligament:
- I. Also known as deltoid ligament.
- II. It is a strong ligament and major stabilizer of the ankle joint
- III. Two components: Superficial: Resist eversion of hind foot (Talo-tibial, naviculo- tibial, calcaneo-tibial) and Deep (Intra-articular): Prevents external rotation of talus (Anterior and posterior talotibial)
- B. Lateral Collateral Ligament:
- I. It is a weak ligament, so involved in over 90% of ankle injuries.
- II. It has three parts:
- i. Anterior talofibular: Most commonly injued.
- ii. Middle calcaneofibular: 2nd most commonly injured.
- iii. Posterior talofibular: Torn in most severe injuries.
- 2. A 25-year-old man presents to his primary care physician with crampy abdominal pain for 2 days. He reports having loose stools and losing 15 lbs (6.8 kg) over a 3-month duration. He also reports increased fatigue. Further evaluation revealed Crohn’s disease. The earliest manifestation in Crohn’s disease is :
- a. Aphthous Ulcer
- b. Anal fistula
- c. Rose thorn appearance on Barium swallow
- d. Skip lesions
- Ans, a. Aphthous ulcer
- Ref: Harrison 19th edition/pg. 1952
- i. The earliest lesions are Aphthous ulcerations & focal crypt abscesses with loose aggregations of macrophages, which form non-caseating granulomas in all layers of the bowel wall.
- ii. In early mild Crohn’s disease, the colonic mucosa appears endoscopically normal or small.
- iii. Punched-out aphthous ulcers are often seen.
- iiii. Aphthous ulcers occur as a result of submucosal lymphoid follicle expansion.
- v. In moderate Crohn's Disease, aphthous ulcers coalesce into larger ulcers, which may take on the appearance of a star (stellate ulcers).
- vi. As Crohn's disease severity increases, submucosal edema and injury can result in cobblestoning of the mucosa, which is seen more often in Crohn’s disease than in ulcerative colitis.
- 3. Resistant hypertension is defined as:
- a. Resistance to aldosterone
- b. Resistance to Angiotension II receptors
- c. Resistance to 2 or more anti hypertensive drugs including thiazides
- d. Resistance to 3 or more anti-hypertensive drugs including thiazides
- Ans,d.Resistance to 3 or more anti-hypertensive drugs including thiazides
- Ref: Harrison 18th edition, chapter 247
- I. Resistant hypertension refers to patients with blood pressures persistently >140/90 mmHg despite taking three or more antihypertensive agents, including a diuretic, in a reasonable combination and at full doses.
- II. Resistant or difficult-to-control hypertension is more common in patients >60 years than in younger patients.
- III. Resistant hypertension may be related to:
- i. “Pseudoresistance” (high office blood pressures and lower home blood pressures), nonadherence to therapy.
- ii. Identifiable causes of hypertension (including obesity and excessive alcohol intake.
- iii. Use of any of a number of nonprescription and prescription drugs.
-
17
- Q1.Aqueous humour is produced by:
- a.Iris
- b.Lens
- c.Ciliary body
- d.Choroid
Ans: c ; Ciliary body
- Ref:AK Khurana 6th e/p 148
- • Ciliary body is forward continuation of the choroid at ora serrata. In cut-section, it is triangular in shape. The anterior side of the triangle forms the part of the angle of anterior and posterior chambers. In its middle the iris is attached.
- • The outer side of the triangle lies against the sclera with a suprachoroidal space in between. The inner side of the triangle is divided into two parts. The anterior part (about 2 mm) having finger-like ciliary processes is called pars plicata and the posterior smooth part (about 4 mm) is called pars plana.
- • Functions of ciliary body
- • Formation of aqueous humour.
- • Ciliary muscles help in accommodation.
- Q2.A 43-year-old man with diabetes has a 4-cm nonhealing foot ulcer. Culture of the ulcer yields Staphylococcus aureus, Bacteroides fragilis, and a gram-negative bacillus that swarms across the blood agar plate covering the entire surface of the agar after 36 hours. The gram-negative bacillus is a member of the genus:
- a.Escherichia
- b. Enterobacter
- c.Serratia
- d. Proteus
Ans:d,Proteus
- Ref: Jawetz Melnick and Adelberg’s medical microbiology 27th/e p 233
- • Proteus–Morganella–Providencia group—The members of this group deaminate phenylalanine, are motile, grow on potassium cyanide medium (KCN), and ferment xylose.
- • Proteus species move very actively by means of peritrichous flagella, resulting in “swarming” on solid media unless the swarming is inhibited by chemicals, such as phenylethyl alcohol or CLED (cystine-lactose-electrolytedeficient) medium.
- • Whereas Proteus species and Morganella morganii are urease positive, Providencia species usually are urease negative. The Proteus–Providencia group ferments lactose very slowly or not at all.
- Q3. A 3-month-old child presents with intermittent stridor aggravated on keeping the child supine. Most likely cause is:
- a. Laryngotracheobronchitis
- b. Laryngomalacia
- c. Respiratory obstruction
- d. Foreign body aspiration
Ans:b,Laryngomalacia
- Ref: Ghai 8th ed pg 369
- • Laryngomalacia is the most common congenital laryngeal anomaly, accounting for up to 60% and the most common cause of infant stridor.
- • Inspiratory stridor is the hallmark of the condition.
- • Symptoms are typically aggravated when the child is supine or crying.
- • Flexible endoscopy reveals partial collapse of a flaccid supraglottic airway with inspiration.
- • If present, gastroesophageal reflux, should be managed.
- • Laryngomalacia is generally benign and selflimited, as most cases resolve by 18 months of age.
- • Surgical intervention is advised for either respiratory distress or failure to thrive
-
18
- 1. Which radiological procedure is used for studying vesco-ureteric reflux?
- a. Ascending pyelogram
- b. Cystogram
- c. Intravenous urogram
- d. Micturition cystourethrogram
- Ans, D, Micturation cystourethrogram
- Ref, Smith’s General Urology, 17e, Chapter12
- Explanation
- The diagnosis of vesico-ureteric reflux is made using micturation cystourethrogram. Other techniques used in diagnosing vesico-ureteric reflux are simple or delayed cystography or voiding cine fluoroscopy.
- In a case of vesico-ureteric reflux, cystogram may show one of the following findings:
- Persistently dilated lower ureter, areas of dilatation in the ureter, ureter visualized throughout its entire length shows, presence of hydroureteronephrosis with a narrow
- juxtavesical ureteral segment or changes of healed pyelonephritis.
- 2. Light microscopic appearance of the PAS-stained specimen from a renal biopsy shows wire loop glomerular capillary lesions. Which of the following is specific marker of the condition?
- a. Scl-70
- b. Anti-Sm
- c. Jo-1
- d. Anti-HLA-B27
- Ans, B, Anti-Sm
- Ref, Robbins & Cotran, Pathologic basis of disease, 7e, p231-233
- Explanation
- Wire loop glomerular capillary lesions seen in lupus nephritis.
- Anti-Sm and anti–double stranded DNA are specific for systemic lupus erythematosus. Anti-Sm is present in only 25% of cases, however.
- Scl-70 isa marker for diffuse systemic sclerosis. Jo-1 is most specific for polymyositis.
- HLA-B27 is seen in ankylosing spondylitis.
- Anticentromere antibody is seen most often with limited scleroderma.
- 3. Central value of a set of 180 values can be obtained by
- a. 90th percentile
- b. 2nd tertile
- c. 9th decile
- d. 2nd quartile
- Ans, D, 2nd quartile
- Ref, Methods in Biostatistics by Mahajan, 7/e p45
- Explanation
- • Quartile: Divides a distribution into 4 equal parts, so the number of intercepts required will be 3, i.e. Q1, Q2, Q3
- – So, Zero – Q1 covers 25% values
- – Similarly, Q2 – Q1, Q3 – Q2 and 100 – Q3 all cover 25% values each
- – Thus, Q2 – Zero, 100 – Q2 and Q3 – Q1 all cover 50% values each
- – Q1 divides a distribution in a ratio of 25 : 75 OR 1 : 3
- – Q2 divides a distribution in a ratio of 50 : 50 OR 1 : 1, Thus second quartile is equivalent to median
- – Q3 divides a distribution in a ratio of 75 : 25 OR 3 : 1.
- In the given question, n = 180
- Thus Q2 which is equivalent to median, divides a distribution in a ratio of 50 : 50 OR 1 : 1.
-
19 (Repeat)
Answers to yesterday's questions.
Ans, DCB
- 1. A 25 year old male presented to OPD with complains of fatigue for 1 month. He also complains of decreased sleep and decreased appetite. He mentioned that he could not concentrate well in his lecture classes. He feels emptiness in his life and tried ending his life twice in the past week. On asking further, he told that he was diagnosed as a case of Schizophrenia 5 years back and he is currently under regular medications. Lab investigations show; TSH 4.5, ESR 10, Hb 9.7, MCV 78, normal electrolytes, SGOT 52, SGPT 43 and ALP 50 (standard units are mentioned). His father also has been diagnosed as a case of Schizophrenia. What is the most likely diagnosis at this point of time in the patient?
- a. Schizophrenia
- b. Hypothyroidism
- c. Schizoaffective disorder
- d. Mood disorder with psychotic features
- Ans, D
- Ref. FA, USMLE step 1, 2020, Page no. 560,561
(The patient already has Schizophrenia and still taking medications, so we can assume he may still have its symptoms as there is no mention in the question regarding its complete cure. So,this patient seems to have both depressive/mood symptoms and schizophrenic symptoms together (and not at different times). Hence we can tick “d. Mood disorder with psychotic features” as the best possible answer.)
- A. Schizoaffective disorder
- • Shares symptoms with both schizophrenia and mood disorders (major depressive or bipolar disorder).
- • To differentiate from a mood disorder with psychotic features, patient must have > 2 weeks of psychotic symptoms without a manic or depressive episode.
- B. Schizophrenia
- • Chronic illness causing profound functional impairment. Symptom categories include:
- • Positive—hallucinations, delusions, unusual thought processes, disorganized speech, bizarre behavior
- • Negative—flat or blunted affect, apathy, anhedonia, alogia, social withdrawal
- • Cognitive—reduced ability to understand or make plans, diminished working memory, inattention
- • Diagnosis requires ≥ 2 of the following active symptoms, including ≥ 1 from symptoms #1–3:
- 1. Delusions
- 2. Hallucinations, often auditory
- 3. Disorganized speech
- 4. Disorganized or catatonic behavior
- 5. Negative symptoms
- • Requires ≥ 1 month of active symptoms over the past 6 months; onset ≥ 6 months prior to diagnosis.
- C. MDD with psychotic features
- • MDD + hallucinations or delusions. Psychotic features are typically mood congruent (eg, depressive themes of inadequacy, guilt, punishment, nihilism, disease, or death) and occur only in the context of major depressive episode (vs schizoaffective disorder).
- • Treatment: antidepressant with atypical antipsychotic, ECT.
- 2. Secretin and CCK are important hormones which regulate secretion of Pancreatic juice. Which one of the following statements is true regarding this regulation?
- a. Secretin causes release of juice rich in enzymes but low in volume
- b. CCK causes release of juice rich in HCO3− and poor in enzymes
- c. Secretin acts on duct cells by increasing cAMP
- d. CCK acts on duct cells by increasing cGMP
- Ans, C
- Ref, Textbook of Medical Physiology, Guyton and Hall, 13th edition, page no. 454
- REGULATION OF THE SECRETION OF PANCREATIC JUICE
- • Secretion of pancreatic juice is primarily under hormonal control.
- • Secretin acts on the pancreatic ducts to cause copi¬ous secretion of a very alkaline pancreatic juice that is rich in HCO3− and poor in enzymes. The effect on duct cells is due to an increase in intracellular cAMP. Secretin also stimulates bile secretion.
- • CCK acts on the acinar cells to cause the release of zymogen granules and production of pancreatic juice rich in enzymes but low in volume. Its effect is mediated by phospho-lipase C. Thus, by acting together, CCK and secretin add enzymes to the pancreatic juice and ensure that they are washed into the intestine.
- 3. Drugs cause urticaria by direct degranulation of mast cells, interfering with arachidonic acid metabolism, IgE-mediated degranulation of mast cells or Complement-mediated degranulation of mast cells. Which of the following drugs cause urticarial by IgE-mediated degranulation of mast cells?
- a. Aspirin
- b. Penicillin
- c. Radioactive contrast
- d. Sulphonamide
- Ans, B
- Ref, Illustrated Synopsis of Dermatology and Sexually
- Transmitted Diseases, Neena Khanna, 4th edition, page no. 211
- Drugs causing Urticaria and Angioedema
- Pathogenesis
- • Drugs cause urticaria by:
- • Direct degranulation of mast cells.
- • Interfering with arachidonic acid metabolism.
- • IgE-mediated degranulation of mast cells.
- • Complement-mediated degranulation of mast cells.
- Clinical Features
- Urticaria and angioedema can occur independently or may also be part of a severe and generalized reaction (anaphylaxis), which includes bronchospasm and circulatory collapse.
- Drugs Implicated
- A. Drugs which directly direct degranulate mast cells:
- • Aspirin.
- • Indomethacin.
- B. Drugs which interfere with arachidonic acid metabolism:
- • Opiates: morphine and codeine.
- • Sulfonamides.
- • Curare.
- • Radioactive contrast.
- C. Drugs which mediate urticaria through IgE:
- • Penicillin.
- D. Drugs which mediate urticaria through complement system:
- • Blood products.
-
20
- Q1. Fenestration operation falls into which type of tympanoplasty as classified by Wullstein?
- a. Type II
- b. Type III
- c. Type IV
- d. Type V
- Ans: d, Type V
- Ref: PL Dhingra, 6/e, p30
- Wullstein classified tympanoplasty into five types.
- ·Type I :
- ▪Defect is perforation of tympanic membrane which is repaired with a graft. It is also called myringoplasty.
- ·Type II :
- ▪Defect is perforation of tympanic membrane with erosion of malleus. Graft is placed on the incus or remnant of malleus.
- ·Type III :
- ▪Malleus and incus are absent. Graft is placed directly on the stapes head. It is also called myringostapediopexy or columella tympanoplasty.
- ·Type IV :
- ▪Only the footplate of stapes is present. It is exposed to the external ear, and graft is placed between the oval and round windows.
- ▪A narrow middle ear (cavum minor) is thus created to have an air pocket around the round window.
- ▪A mucosa-lined space extends from the Eustachian tube to the round window.
- ▪Sound waves in this case act directly on the footplate while the round window has been shielded.
- ·Type V :
- ▪Stapes footplate is fixed but round window is functioning. In such cases, another window is created on horizontal semicircular canal and covered with a graft. Also called fenestration operation.
- Q2. All of the following statements regarding inflammatory bowel disease are correct except?
- a. Ulcerative colitis affects colon; Crohn’s disease can affect any ;part of the GI tract
- b. Ulcerative colitis is associated with Perianal disease, whereas this is unusual in Crohn’s disease
- c. Crohn’s disease more commonly causes stricturing and fistulation
- d. Granulomas may be found on histology in Crohn’s disease, but not in Ulcerative colitis
- Ans: b, Ulcerative colitis is associated with Perianal disease, whereas this is unusual in Chron’s disease
- Ref: Bailey and Love 27/e, p1243
- Differences between ulcerative colitis (UC) and Crohn’s disease (CD)
- UC affects the colon; CD can affect any part of the gastrointestinal tract, but particularly the small and large bowel.
- UC is a mucosal disease, whereas CD affects the full thickness of the bowel.
- UC produces confluent disease in the colon and rectum, whereas CD is characterised by skip lesions.
- CD more commonly causes stricturing and fistulation.
- Granulomas may be found in histology in CD, but not in UC.
- CD is often associated with perianal disease, whereas this is unusual in UC.
- CD affecting the terminal ileum may produce symptoms mimicking appendicitis, but this does not occur in UC.
- Resection of colon and rectum cures the patients with UC, whereas recurrence is common after resection in CD.
- Q3. Commonest birth mark occurring most commonly on the head and neck and 70% of which gets resolved by 7 years of age is?
- a. Capillary haemangioma
- b. Port wine stain
- c. Salmon Patch
- d. Campbell de Morgan Spots
- Ans: a, Capillary haemangioma
- Ref: Bailey and Love, 27/e, p613
- Capillary haemangioma (strawberry naevus)
- ▪This is the commonest ‘birth mark’, occurring most commonly on the head and neck (Figure 40.43). 90% appear at birth, and, as a consequence of intravascular thrombosis, fibrosis and mast cell infiltration, 10% resolve each subsequent year, with 70% resolved by 7 years old.
- ▪White skin is affected most commonly and girls are affected three times more than boys.
- Campbell de Morgan spots
- ▪These are arteriovenous fistulae at the dermal capillary level in sun-exposed skin of older patients.
- Salmon patch
- ▪A salmon patch is a haemangioma that presents as a pinkish macule, usually at the nape of neck.
- ▪It is caused by an area of persistent fetal dermal circulation, which usually disappears at 1 year.
- Capillary vascular malformations ‘port-wine’ stains
- ▪Capillary vascular malformations [‘port-wine stains’ (PWS)] are 20 times less common than capillary haemangiomata and result from defective maturation of cutaneous sympathetic innervation during embryogenesis, leading to localised intradermal capillary vasodilatation.
- ▪They appear at birth as flat, smooth, intensely purple-stained areas, most frequently on the head and neck, often within the maxillary and mandibular dermatomes of the trigeminal nerve.
- ▪Treatment with intense pulsed light and pulse dye laser are successful. PWS may be associated with various syndromes.
-
21
- Q1.A simple febrile seizure is characterized by all of the following features except:
- a.occur between the ages of 6 and 60 mo with a temperature of 38°C (100.4°F) or higher.
- b.Last for a maximum of 15 min.
- c.Is not recurrent within a 24-hr period
- d.Focal in nature
- Ans:d,Focal in nature
- Ref:Nelson 21st e/p 12071
- •Febrile seizures are seizures that occur between the ages of 6 and 60 mo (peak 12-18 mo) with a temperature of 38°C (100.4°F) or higher, that are not the result of CNS infection or any metabolic imbalance, and that occur in the absence of a history of prior afebrile seizures.
- •A simple febrile seizure is a primary generalized, usually tonic-clonic, attack associated with fever, lasting for a maximum of 15 min, and not recurrent within a 24-hr period.
- •A complex febrile seizure is more prolonged (>15 min), and/or is focal, and/or recurs within 24 hr. Febrile status epilepticus is a febrile seizure lasting longer than 30 min.
- • Most patients with simple febrile seizures have a very short postictal state and usually return to their baseline normal behavior and consciousness within minutes of the seizure.
- •Febrile infection–related (or refractory ) epilepsy (FIRES) is a very different disorder seen predominantly in older (>5 yr) usually male children and associated with an encephalitis-like illness but without an identifiable infectious agent. Children with FIRES were previously normal but subsequently develop difficult-to-treat epilepsy.
- Q2. Arden index is related to:
- a.ERG(Electroretinogram)
- b.EOG(Electro-oculogram)
- c.VER(Visual evoked response)
- d.Perimetry
- Ans:b,EOG
- Ref: AK Khurana 6th e/p519
- Electro-oculography (EOG)
- •Electro-oculography is based on the measurement of resting potential of the eye which exists between the cornea (+ve) and back of the eye (-ve).
- •Interpretation of results. Results of EOG are interpreted by finding out the Arden ratio as follows:
- Arden ratio = Maximum height of light peak/ Minimum height of dark trough × 100
- •Normal curve values are 185 or above.
- •Subnormal curve values are less than 150.
- •Flat curve values are less than 125.
- •Uses. Since the EOG reflects the presynaptic function of the retina, any disease that interferes with the functional interplay between the retinal pigment epithelium (RPE) and the photoreceptors will produce an abnormal or absent light rise in the EOG. Thus, EOG is affected in diseases such as retinitis pigmentosa, vitamin A deficiency, retinal detachment and toxic retinopathies. Hence, EOG serves as a test that is supplementary and complementary to ERG and in certain states is more sensitive than the ERG, e.g. in diagnosis of Best’s disease.
- Q3. A teenager who plays with dogs developed skin rash, eosinophilia, and an enlarged liver and spleen for 1 year. The most likely cause of this infection is
- a. Trichinosis
- b. Schistosomiasis
- c. Toxoplasmosis
- d. Visceral larva migrans
Ans: d, visceral larva migrans
- Ref:Paniker 8th ed pg 167
- Visceral Larva Migrans
- This condition is caused by the migration of larvae of nonhuman species of nematodes that infect by the oral route.
- The most common cause is the dog ascarid, Toxocara canis and less often the cat ascarid, T. cati. Visceral larva migrans may also be caused by Anisakis, which are large ascarid parasites of marine animals and also by Gnathostoma spinigerum, Angiostrongylus cantonensis. Human nematodes like A. lumbricoides and S. stercoralis may produce visceral larva migrans, when they get lost in ectopic sites
- Clinical manifestations depend on the sites affected and the degree and duration of infection.
- • As children are more likely to swallow dirt, this condition is much more frequent in them.
- • Fever, hepatomegaly, pneumonitis, hyperglobulinemia and pica are th e common findings.
- • Patients may develop neurological disturbances (neural larva migrans) and endophthalmitis (ophthalmic larva migrans).
- • Marked leukocytosis occurs with persistently high eosinophilia.
-
22
- Points to Remember
- • Adult onset papilloma – seen in adults in the third decade
- • Ca which presents with neck nodes = supraglottis Ca
- • Highest lymphnode involvement occurs in – supraglottic Ca
- • Hoarseness is the presenting symptom – Glottic Ca
- • Stridor is the presenting symptom in Subglottic Ca.
- • Laryngeal cancer with worst prognosis = subglottic Ca
- • Ca with best prognosis = Glottic Ca
-
23
- 1. C3 level is decreased in:
- a. PSGN
- b. IgA nephropathy
- c. HSP
- d. Goodpasture syndrome
Ans, A, PSGN
- I. Decreased C3 may be seen in:
- A. Systemic diseases:
- ▪Lupus nephritis
- ▪SABE
- ▪Shunt nephritis,
- ▪Essential mixed cryoglobulinemia
- ▪Visceral abscesses
- B. Renal diseases:
- ▪PSGN
- ▪MPGN type
- II. Normal C3 is seen in:
- A. Systemic diseases:
- ▪PAN
- ▪Hypersensitivity vasculitis
- ▪HSP
- ▪Granulomatosis with polyangitis
- ▪Goodpasteur syndrome
- B. Renal diseases
- ▪IgA nephropathy
- ▪RPGN
- ▪Post infectious GN
- 2. 22-year-old Rama is being counseled after a first-trimester miscarriage. Her physical examination is unremarkable, no significant medical problems and in counseling her you explain in lay terms that the most likely cause of her miscarriage is chromosomal abnormality. The most common chromosomal abnormality causing miscarriage is?
- a. Trisomy 16
- b. 45,XO
- c. Triploidy
- d. Unbalanced translocation
Ans, B, 45,XO
- {This is a bit confusing. Over all the most common cause for miscarriage is aneuploidy (abnormal numbers of chromosome).
- Among them, the most common is trisomy i.e. 47 chromosomes, about 50%. Among which trisomy 16 on third of all trisomies i.e. around 16% of abortuses.
- Then comes monosomy but, 45,XO appears to be cause in 20% alone so, its the best answer in this question.}
- SPONTANEOUS ABORTION (MISCARRIAGE)
- DEFINITION:
- •Abortion is the expulsion or extraction from its mother of an embryo or fetus weighing 500 g or less when it is not capable of independent survival (WHO).
- •This 500 g of fetal development is attained approximately at 22 weeks (154 days) of gestation. The expelled embryo or fetus is called abortus.
- The word miscarriage is the recommended terminology for spontaneous abortion.
- GENETIC FACTORS RELATED TO MISCARRIAGE:
- •Majority (50%) of early miscarriages are due to chromosomal abnormality in the conceptus.
- •Autosomal trisomy is the commonest (50%) cytogenetic abnormality.
- •Trisomy for every chromosome has been reported.
- •The most common trisomy is trisomy 16 (30%). Polyploidy has been observed in about 22% of abortuses. (Polyploidy refers to the presence of three or more multiples of a haploid number of chromosome, e.g. 3n = 69, 4n= 92. Triploidy is more common than tetraploidy).
- •Monosomy X (45, X) is the single most common chromosomal abnormality in miscarriages (20%). Structural chromosomal rearrangements are observed in 2–4% of abortuses.
- •These include translocation, deletion, inversion and ring formation.
- •Other chromosomal abnormalities like mosaic, double trisomy, etc. are found in about 4% of abortuses.
- 3. A 30-year-old man has excessive sleep at work attributed to sleep discomfort at night. He also has recent history of falling while partying with friends. What are the other features that can be seen associated with his condition:
- a. Paralysis during sleep-wake transition with hallucinations
- b. Snoring with witnessed sleep apnea
- c. Pain in the legs before going to sleep
- d. Generalized seizures in the wake state
Ans, A, Paralysis during sleep-wake transition with hallucinations
- History of fall is highly suggestive of atonia/paralysis and may be due to Narcolepsy. In obstructive sleep apnea,patient usually falls asleep during daytime with no loss of postural tone. Narcolepsy is both a disorder of the ability to sustain wakefulness voluntarily and a disorder of REM sleep regulation. Paralysis during sleep-wake transition with hallucinations is seen in narcolepsy.
- Narcolepsy
- Narcolepsy is a common sleep disorder characterized by excessive day time sleepiness, cataplexy (episodes of muscle weakness triggered by emotions), hypnagogic hallucinations and sleep paralysis.
- Narcolepsy (Classic tetrad)
- • Excessive day time somnolence.
- • Sudden weakness or loss of muscle tone without loss of consciousness often elicited by emotions (cataplexy).
- • Hallucination at sleep onset (Hypnagogic hallucination).
- • Muscle paralysis upon awakening.
- 4. Which of the following structure in the spine is pain insensitive?
- a. Nucleus pulposus
- b. Dura
- c. Annulus fibrosus
- d. Longitudinal ligaments
Ans, A, Nucleus pulposus
- Pain sensitive structures of the spine are:
- ▪Facet joints
- ▪Periosteum of vertebrae
- ▪Annulus fibrosus of intervertebral disc
- ▪Dura
- ▪Epidural veins and arteries
- ▪Longitudinal ligaments
- Nucleus pulposus of intervertebral disk is not pain sensitive.
- 5. A previously diagnosed case (2 years old baby) of tubercular meningitis comes with delayed milestones and mental retardation. After examination and investigations, he was found to have increased ICP. This is because of decreased absorption of CSF with normal production of it from ventricles and choroid plexus. Production of CSF from walls of ventricles and the choroid plexus is about?
- a. 20ml/hr
- b. 40ml/hr
- c. 60ml/hr
- d. 80ml/hr
- Ans, A, 20ml/hr
- Explanation:
- The total CSF volume is normally about 150 mL. Production from the walls of the ventricles and the choroid plexus is about 20 mL per hour. Hydrocephalus refers to an increase in CSF volume with ventricular enlargement, often presenting symptoms of raised ICP.
-
24
- 1.Diffusion into gas-containing spaces is a problem with:
- a. Halothane
- b. Nitrous oxide
- c. Isoflurane
- d. Ether
- Ans: B,Nitrous Oxide
- Ref: Morgan 5/e p166
- Contraindications Of nitrous oxide
- •Although nitrous oxide is insoluble in comparision with other inhalation agents, it is 35 times more soluble than nitrogen in blood.
- •Thus, it tends to diffuse into air-containing cavities more rapidly than nitrogen is absorbed by the bloodstream.
- •For instance, if a patient with a 100-mL pneumothorax inhales 50% nitrous oxide, the gas content of the pneumothorax will tend to approach that of the bloodstream.
- •Because nitrous oxide will diffuse into the cavity more rapidly than the air (principally nitrogen) diffuses out, the pneumothorax expands until it contains 100 mL of air and 100 mL of nitrous oxide.
- •If the walls surrounding the cavity are rigid, pressure rises instead of volume.
- •Examples of conditions in which nitrous oxide might be hazardous include venous or arterial air embolism, pneumothorax, acute intestinal obstruction with bowel distention, intracranial air (pneumocephalus following dural closure or pneumoencephalography), pulmonary air cysts, intraocular air bubbles, and tympanic membrane grafting .
- •Nitrous oxide will even diffuse into tracheal tube cuffs, increasing the pressure against the tracheal mucosa.
- •Obviously, nitrous oxide is of limited value in patients requiring high inspired oxygen concentrations.
- 2. A lady presents after 2 days following complete perineal tear, best time for perineal repair is
- A.Immediately
- B.After 1 week
- C.No need to repair
- d.After 3 months
- Ans,d; after 3 months
- Ref dc dutta 8/e P490
- Timing of repair of perineal tear
- •Recent tear should be repaired immediately following the delivery of the placenta.
- •This reduces the chance of infection and minimizes the blood loss.
- •In cases of delay beyond 24 hours, the repair is to be withheld.
- •Antibiotics should be started to prevent infection.
- •The complete tear should be repaired after 3 months if delayed beyond 24 hours.
- •In case of any doubt to grade of 3rd degree tear, it is advisable to classify to the higher degree rather than lower degree.
- 3. You are a medical officer of Sukraraj tropical hospital, you are providing direct care to COVID-19 patients, you are not involved in Aerosol-generating procedures. According to WHO which of the following PPE is NOT necessary for you?
- A. Respirator N95 or FFP2 standard
- B. Gown
- C. Eye protection (goggles or face shield)
- D. All are recommended
- Ans A Respirator N95 or FFP2 standard
- Ref, WHO rational use of personal protective equipment for coronavirus disease 2019 (COVID-19)
-
25
- Q1. The action of which of the following muscles can displace bilateral fractures of the mandible in the canine region posteriorly?
- a. Thyrohyoid, genioglossus and geniohyoid muscles
- b. Anterior belly of digastric, geniohyoid and genioglossus muscles
- c. Mylohyoid, genioglossus and styloglossus muscles
- d. Mylohyoid, masseter and geniohyoid muscles
- Ans: b, Anterior belly of digastric, geniohyoid and genioglossus muscles
- Ref: Bailey and Love 25/e, p331
- Fractures of the mandible
- The condylar neck is the weakest part of the mandible and is the most frequent site of fracture, while other fractures tend to occur through unerupted teeth (the impacted wisdom tooth) or where the roots are long (the canine tooth). The mandible may fracture directly at the point of the blow, or indirectly where the force from the blow is transmitted and the mandible fractures at a point of weakness distant from the original blow. The latter is characteristically seen in the so called ‘guardsman’ fracture, where a blow to the chin point may cause a fracture of the symphysis or parasymphysis of the lower jaw, and indirect transmission of the kinetic energy causes a unilateral or bilateral fracture of the mandibular condyles. Individual sharp blows with a blunt instrument may fracture a segment away from the mandible. Blows from below may cause the mandible to be thrust upwards, fracturing the alveolus and teeth as they strike the maxillary dentition.
- Much has been made in the past of the ‘butterfly’ fracture of the mandible. Here, a segment of mandible is detached from the rest of the mandible in the canine regions. The segment of bone may include the anterior insertion of the tongue (genioglossus and geniohyoid). Theoretically, the tongue may fall back and occlude the airway.
- The muscles attached to the anterior mandible which can cause posterior displacement in a bilateral parasymphyseal mandibular fracture are the anterior belly of digastric, geniohyoid and genioglossus.
- Q2. Which of the following statements regarding the vertigo associated with Ménière’s disease are true?
- a. It lasts for days to weeks.
- b. It lasts for minutes to hours.
- c. It lasts for seconds to minutes.
- d. It is caused by recurrent infection.
- Ans: b, It lasts for minutes to hours
- Ref: Bailey and Love, 25/e, p699.
- Ménière’s disease describes an intermittent vertigo which lasts for minutes to hours and is associated with tinnitus, hearing loss and a feeling of aural fullness. The cause is unclear although it may be related to pressure changes within the inner ear. Investigation will include MRI to exclude mass lesions at the cerebellopontine angle, which may mimic the condition. Treatment includes diuretics which may help lower the inner ear pressure, injection of gentamicin to the middle ear to perform a pharmaceutical larbyrinthectomy or more destructive surgical interventions to destroy the labyrinth or the nerves that supply it.
- Q3. Which of the following is not true in relation to strangulated hernias?
- a. They present with local and then generalised abdominal pain and vomiting.
- b. A normal hernia can strangulate at any time.
- c. This is more common in femoral hernia.
- d. They can be reliably excluded in irreducible hernias on clinical examination.
- Ans: d, They can be reliably excluded in irreducible hernias on clinical examination
- Ref: Bailey and Love 25/e, p969-70
- Strangulation is a genuine surgical emergency and requires prompt surgery. The clinical features may mimic gastroenteritis in a strangulated Richter’s hernia, which may delay diagnosis. The initial symptoms in a strangulated omentocele are similar to a strangulated bowel, but vomiting and constipation may be absent.
-
26
- 1. A couple has been experiencing discharge from their sex organs. The male partner also gives history of dysuria while the girl doesn’t have that. They have been indulged in sexual activities since their third date 1 month back. They deny use of condoms but has been strictly following calendar method of contraception. Both of them had previous relationship. On gram’s stain of discharge Gram –ve intracellular diplococci is revealed. What is the incubation period of the likely venereal disease?
- a. 1-2 days
- b. 1-5 days
- c. 1-30 days
- d. 5-14 days
- Ans, B, 1-5 days
- Ref, Illustrated Synopsis of Dermatology and Sexually Transmitted Diseases, Neena Khanna, 4th edition, page no. 315
- Gonorrhea
- Etiology: Neisseria gonorrhoeae, transmitted sexually; sometimes vertically (from mother to child) causing ocular infection in neonates.
- Incubation period: 1–5 days.
- Males: Urethritis manifesting as profuse urethral discharge and dysuria.
- Females: Usually asymptomatic carriers; may have vaginal discharge.
- Complications:
- Infection of adjoining structures and glands.
- Late complications like urethral stricture (males) and pelvic inflammatory disease and infertility (females).
- Diagnosis: Clinical suspicion confirmed in males by demonstration of Gram-negative intracellular diplococci on Gram stain. And in females by culturing the organism.
- Treatment:
- Uncomplicated gonococcal infection: Single oral dose of cefixime (400 mg) or ciprofloxacin (500 mg) or intramuscular ceftriaxone (125 mg);
- Complicated infections: Need longer treatment.
- 2. Having very hard time raising a son, parents come to a pediatrician. They complain that their son turned 4 years of age recently and since that time around, has been developing undesirable symptoms. The boy has difficulty sitting still at a place for long time, doesn’t seem to listen to anybody, gets easily distracted, talks excessively and appears fidgety all the time. Attention deficit disorder with hyperactivity is the commonest type of ADD. Which of the following symptoms suggest poor attention of the child?
- a. Difficulty sitting still at a place for long time
- b. Appears fidgety all the time
- c. Doesn’t seem to listen
- d. Talks excessively
- Ans, C, Doesn’t seem to listen
- Ref, A Short Textbook of Psychiatry, Niraj Ahuja, 7th edition, page no. 166
- Attention deficit disorder (ADD)
- A relatively common disorder, it occurs in about 3% of school age children. Males are 6-8 times more often affected. The onset occurs before the age of 7 years and a large majority of patients exhibit symptoms by the 4th year of age.
- Attention deficit disorder (ADD) is of four clinical types:
- 1. Attention deficit disorder with hyperactivity (Hyperkinetic disorder)
- This is the commonest type. The characteristic clinical features are:
- a) Poor attention span with distractibility
- • Fails to finish the things started.
- • Shifts from one uncompleted activity to another.
- • Doesn’t seem to listen.
- • Easily distracted by external stimuli.
- • Often loses things.
- b) Hyperactivity
- • Fidgety.
- • Difficulty in sitting still at one place for long.
- • Moving about here and there.
- • Talks excessively.
- • Interference in other people’s activities.
- c) Impulsivity
- • Acts before thinking, on the spur of the moment.
- • Difficulty in waiting for turn at work or play.
- 2. Attention deficit disorder without hyperactivity
- It is a rare disorder with similar clinical features, except hyperactivity.
- 3. Residual type
- It is usually diagnosed in a patient in adulthood, with a past history of ADD and presence of a few residual features in adult life.
4. Hyperkinetic disorder with conduct disorder (Hyperkinetic conduct disorder).
- 3. A researcher observes pattern of sleep in a normal human being. One of the volunteers is asked to sleep in a dim-light room. He then observes for any movements or other findings he might find notable. Suddenly, after a while, the researcher notices rapid movement of the sleeper’s eyes which is clearly appreciated as wavy movement of eyelids. In a normal sleep, what is the average duration of a bout of Rapid Eye Movement sleep?
- a. 5-30 mins
- b. 30-60 mins
- c. Around 90 mins
- d. 2 hours
- Ans, A, 5-30 mins
- Ref, Textbook of Medical Physiology, Guyton and Hall, 13th edition, page no. 763
- Types of sleep
- Each night, a person goes through stages of two major types of sleep that alternate with each other. These types are called
- 1. Rapid eye movement sleep (REM sleep), in which the eyes undergo rapid movements even though the person is still asleep.
- • REM sleep occurs in episodes that occupy about 25 percent of the sleep time in young adults; each episode normally recurs about every 90 minutes.
- • In a normal night of sleep, bouts of REM sleep lasting 5 to 30 minutes usually appear on average every 90 minutes in young adults.
- • When a person is extremely sleepy, each bout of REM sleep is short and may even be absent. As the person becomes more rested through the night, the durations of the REM bouts increase.
- • This type of sleep is not so restful, and it is often associated with vivid dreaming.
- 2. Slow-wave sleep or non-REM (NREM) sleep, in which the brain waves are strong and of low frequency.
- • Most sleep during each night is of the slow-wave (NREM) variety, which is the deep, restful sleep that the person experiences during the first hour of sleep after having been awake for many hours.
- • This sleep is exceedingly restful.
-
27
- Q. Posterior inferior cerebellar artery is the branch of
- a. Vertebral artery
- b. Basilar artery
- c. Subclavian artery
- d. Internal carotid artery
- Ans: a. Vertebral artery
- Ref: Ref: Ref: Human anatomy BD Chaurasia 4th Vol 3 P202
Branches of vertebral artery
- a. Cervical branches
- 1. Spinal branches
- 2. Muscular branches
- b. Cranial branches
- 1. Meningeal branches
- 2. Anterior spinal artery
- 3. Posterior spinal artery ( usually a branch of posterior interior cerebellar artery )
- 4. Posterior inferior cerebellar artery ( largest branch )
- 5. Medullary arteries
- Q. Burning foot syndrome is caused by deficiency of:
- a. Riboflavin
- b. Niacin
- c. Pantothenic acid
- d. Folic acid
- Ans: c. Pantothenic acid
- Ref: Biochemistry U. Satyanarayana 4th e P 150
- Deficiency symptoms of pantothenic acid
- It is a surprise to biochemists that despite the involvement of pantothenic acid (as coenzyme A) in a great number of metabolic reactions, its deficiency manifestations have not been reported in humans. This may be due to the widespread distribution of this vitamin or the symptoms of pantothenic acid may be similar to other vitamin deficiencies. Dr. Gopalan, a world renowned nutritionist from India, linked the burning feet syndrome (pain and numbness in the toes, sleeplessness, fatigue etc.) with pantothenic acid deficiency.
- Q. Which of the following drug can cause thyroid dysfunction?
- a. Amiodarone
- b. Ampicillin
- c. Ibutilide
- d. Acyclovir
- Ans: a. Amiodarone
- Essentials of medical pharmacology KD Tripathee 7th e P 534
- Adverse effects of Amiodarone
- These are dose-related and increase with duration of therapy.
- • Fall in BP, bradycardia and myocardial depression occurs on i.v. injection and after drug cumulation.
- • Nausea, gastrointestinal upset may attend oral medication, especially during the loading phase.
- • Photosensitization and sun burn like skin pigmentation occurs in about 10% patients.
- • Corneal microdeposits are common with longterm use, may cause headlight dazzle, but are reversible on discontinuation.
- • Pulmonary alveolitis and fibrosis is the most serious toxicity of prolonged use, but is rare if daily dose is kept below 200 mg.
- • Peripheral neuropathy generally manifests as weakness of shoulder and pelvic muscles. Liver damage is rare.
- • Amiodarone interferes with thyroid function in many ways including inhibition of peripheral conversion of T4 to T3 and interaction with thyroid hormone receptor. Goiter, hypothyroidism and rarely hyperthyroidism may develop on chronic use.
-
28
- 1.All are true regarding Caroli’s disease except?
- A. Intrahepatic stone formation
- B. Biliary stasis
- C. Commonly presents with ascites
- D. Malignant change can occur
Correct answer : C. Commonly presents with ascites
CT scan of cholangiocarcinoma. Malignant change can occur in Caroli’s disease.
- Caroli’s disease is congenital dilatation of the intrahepatic biliary tree.
- This disease is complicated by presence of intrahepatic stone formation.
- Patients can present with abdominal pain or sepsis.
- Biliary stasis with stone formation can lead to biliary sepsis.
- Malignant change in the ductal system can lead to cholangiocarcinoma
- 2.Which of the following is not a component of Total Parenteral Nutrition?
- A. Amino acids
- B. Fats
- C. Vitamins
- D. Fiber
- Correct answer : D. Fiber
- Ref. Bailey and Love 27/e, p286-87
- COMPONENTS OF TOTAL PARENTERAL NUTRITION
- Total parenteral nutrition should provide both energy and other essential nutrients.
- Carbohydrates, lipids and amino acids provide energy.
- The other essential nutrients to be included are: minerals, vitamins and water.
- Dietary fiber is required only in case of enteral nutrition. It is not given as a part of parenteral nutrition.
- 3.Carcinoma of buccal mucosa commonly drain to the following lymph nodes sites:
- a. Submental
- b. Submandibular
- c. Supraclavicular
- d. Cervical
- Ans. is b i.e. Submandibular
- Ref. Dhingra 5/e, p 240; 6/e, p 227
- • M/C lymph node involved in any oral malignancy is Submandibular LN
- • Maximum LN metastases is seen in cancer tongue followed by floor of mouth.
- • Lymphatic metastasis is least in lip cancer followed by hard palate.
- Lymphatic drainage of tongue, Floor of mouth and Buccal mucosa:
- • Level I: Submandibular LN
- • Level II: Upper deep cervical LN
- • Level III: Middle deep cervical LN
- • Level IV: Lower deep cervical LN
- Lymphatic drainage of oropharyngeal tumors:
- • Level I: Jugulodigastric LN
- • Level II: Upper deep cervical LN
- • Level III: Jugulomohyoid or middle deep cervical LN
- • Level IV: Lower deep cervical LN
-
29
- 1. A young girl presents with abdominal pain and a recent change in bowel habit, with passage of mucus in stool. There is no associated blood in stool and symptoms are increased with stress. The most likely diagnosis is:
- a. Irritable bowel syndrome
- b. Ulcerative Colitis
- c. Crohn’s disease
- d. Amebiasis
- Ans, a. Irritable bowel syndrome
- Ref: Harrison’s 18/e p2496-97, 17/e p1899, 1900
- Irritable bowel syndrome (IBS) is a functional bowel disorder characterized by abdominal pain or discomfort and altered bowel habits in the absence of detectable structural abnormalities. No clear diagnostic markers exist for IBS, thus the diagnosis of the disorder is based on clinical presentation.
- IBS is a disorder that affects all ages, although most patients have their first symptoms before age 45. Older individuals have a lower reporting frequency. Women are diagnosed with IBS two to three times as often as men and make up 80% of the population with severe IBS. Pain or abdominal discomfort is a key symptom for the diagnosis of IBS. These symptoms should be improved with defecation and/or have their onset associated with a change in frequency or form of stool. Painless diarrhea or constipation does not fulfill the diagnostic criteria to be classified as IBS. Supportive symptoms that are not part of the diagnostic criteria include defecation straining, urgency or a feeling of incomplete bowel movement, passing mucus and bloating.
- 2. Most common cause of death in primary amyloidosis is
- a. Respiratory failure
- b. Renal failure
- c. Cardiac failure
- d. Septicemia
- Ans, c. Cardiac failure
- Ref: Harrison’s 18/e p948
- Amyloidosis is the term for diseases caused by the extracellular deposition of insoluble polymeric protein fibrils in tissues and organs. These diseases are a subset of a growing group of disorders attributed to misfolding of proteins. Among these are Alzheimer’s disease and other neurodegenerative diseases, transmissible prion diseases, and genetic diseases caused by mutations that lead to misfolding, aggregation, and protein loss of function, such as certain of the cystic fibrosis mutations. Amyloid fibrils share a common beta pleated sheet structural conformation that confers unique staining properties.
- For patients with impaired cardiac function or arrhythmias due to amyloid involvement of the myocardium, median survival is only about 6 months without treatment, and stem cell mobilization and high-dose chemotherapy are dangerous. In these patients, cardiac transplantation can be performed, followed by treatment with HDM/SCT to prevent amyloid deposition in the transplanted heart or other organs.
- 3. A 55 year old female came with hip flexor contracture. What is the most likely test to be done in this case:
- a. Allis test
- b. Thomas test
- c. Ober test
- d. Trendelenburg test
- Ans, b.Thomas test
- Ref: Apley’s system of orthopaedics and fracture 9th ed., pg. 495
- Full name of the test is Hugh Owen well leg hip flexion Thomas test. This test is used to find out fixed flexion deformity of hip joint.
- Ober test is to determine Iliotibial band contracture
- Trendelenberg test is to find out abductor weakness of hip joint.
- Allis test or Galeazzi sign is used to diagnose DDH in children.
-
30
- 1) Bullet which is left in the body for very long time is known as?
- a. Tandem bullet
- b. War bullet
- c. Souvenir bullet
- d. Tracer bullet
Ans, C, Souvenir bullet
Ref, SK Singhal’s Forensics, 4th edition, page no. 188
- Types of bullet:
- i. Tandem bullet/piggy tail bullet: Bullet not released on first fire comes out with second bullet on second shot.
ii. Souvenir bullet : Bullet which is left in the body for long.
iii. Tracer bullet : Bullet that leaves a visible mark in path that a person can see.
iv. Ricocheting bullet : Bullet deviated from its path after coming in contact with a surface.
- 2) What is the maximum diameter of Tardieu spots that is seen in case of asphyxia?
- a. 0.5mm
- b. 1mm
- c. 2mm
- d. 5mm
Ans, C, 2mm
Ref, The Essentials of Forensic Medicine and Toxicology, Reddy, 33rd edition, page no.141
Tardieu spots are numerous where the capillaries are least firmly supported, as in sub-conjunctival tissues and under the pleural and pericardial membranes but they can appear almost anywhere if the degree of congestion and cyanosis is sufficient. Tardieu spots are usually rounded, dark and well-defined, varying in size from a pin's head to 2 mm. They may occur as isolated minute haemorrhages or present in large numbers, and at times fuse to form patches of red colour, especially at the back of the heart.
- 3) Chocolate agar is used for isolation of;
- a. N. gonorrhoea
- b. H. influenzae
- c. B. pertusis
- d. E.coli
Ans, B, H. influenza
Ref: First aid for USMLE Step 1 2020 pg170
- •H influenzae = Chocolate agar
- •N gonorrhoeae, N meningitidis=Thayer-Martin agar.
- •B pertussis= Bordet-Gengou agar (Bordet for Bordetella),Regan-Lowe medium
- •C diphtheriae =Tellurite agar, Löffler medium
- •M tuberculosis =Löwenstein-Jensen medium, Middlebrook medium
- •M pneumoniae= Eaton agar
- •E coli = Eosin–methylene blue (EMB)
- •Legionella=Charcoal yeast extract agar buffered with cysteine and iron
- •Fungi =Sabouraud agar
-
31
- 1) Which of the following formula can be used for calculation for mental age (MA) if chronological age (CA) is known?
- a. CA= (CA- MA)/IQ x 100
- b. IQ=MA/CA x 100
- c. CA=IQ/MA x 100
- d. IQ=CAx100/MA
Ans, B
Ref, Niraj Ahuja - A Short Textbook of Psychiatry, 20th Edition, Page no. 156
- A classification of mental retardation on the basis of IQ ( Intelligence Quotient, which is equal to mental age, i.e. MA, divided by chronological age, i.e. CA, multiplied by 100;
- i.e. IQ = MA/CA × 100), is provided in Table.
- 2) A 10 year old boy presented with a mild tibial swelling.On X-ray,lytic lesion with sclerotic margins was seen.What is the most likely diagnosis?
- a. Osteoid osteoma
- b. Fibrous cortical defect
- c. Osteosarcoma
- d. Fibrous dysplasia
Ans, A, Osteoid osteoma
(Ref: Apley 9th ed, pg 194-204)
-The most liekly diagnosis in a 10 year old boy who presents with a mild tibial swelling and lytic lesion with sclerotic margins on X-ray is Osteoid osteoma.
- 3) All of the followings are true about hernias except;
- a. All hernias require surgical repair
- b. Small hernias can be more dangerous than large
- c. Pain, tenderness and skin color changes imply high risk of strangulation
- d. Femoral hernia should always be repaired
Ans, A, All hernias require surgical repair
Ref: Bailey and love 27th edition, pg no.1026
- Explanation:
- Management principles of hernia
- Not all hernias require surgical repair.
- Small hernias can be more dangerous than large.
- Pain, tenderness and skin colour changes imply high risk of strangulation.
- Femoral hernia should always be repaired.
-
32
- 1) Which of the following about atherosclerosis is true?
- a. Intake of unsaturated fatty acid is associated with decreased risk
- b. Thoracic aorta involvement is more severe than abdominal aorta involvement
- c. Extent of lesion in veins is same as that in arteries
- d. Hypercholesterolemia does not always increase the risk of atherosclerosis
Ans, A, Intake of unsaturated fatty acid associated with decreased risk
Ref- Harrison 17th edition page no.2429
- · Atherosclerosis is a condition in which an artery wall thickens as a result of the accumulation of fatty materials such as cholesterol.
- · It is a syndrome affecting arterial blood vessels, a chronic inflammatory response in the walls of arteries, caused largely by the accumulation of macrophage white blood cells and promoted by low-density lipoproteins without adequate removal of fats and cholesterol from the macrophages by functional high density lipoproteins (HDL),
- · It is commonly referred to as a hardening or furring of the arteries.
- · It is caused by the formation of multiple plaques within the arteries.
- The athermatous plaque is divided into three distinct components:
- ☆ The atheroma which is the nodular accumulation of a soft, flaky, yellowish material at the center of large plaques, composed of macrophages nearest the lumen of the artery
- ☆ Underlying areas of cholesterol crystals
- ☆ Calcification at the outer base of older/more advanced lesions.
- Role of PUFA
- · N-3 polyunsaturated fatty acids (n-3 PUFAs) are present in high concentration in fish and in flax seeds.
- · The most widely used n-3 PUFAs for the treatment of hyperlipidemias are the two active molecules in fish oil: eicosapentanoic acid (EPA) and decohexanoic acid (DHA).
- · N-3 PUFAs have been concentrated into tablets and decrease fasting triglycerides in doses of 3–4 g/d.
- · Fish oils can result in an increase in plasma LDL-C levels in some patients.
- · Fish oil supplements can be used in combination with fibrates, niacin, or statins to treat hypertriglyceridemia. In general, fish oils are well tolerated and appear to be safe, at least at doses up to 3–4 g.
- · Although fish oil administration is associated with a prolongation in bleeding time, no increase in bleeding has been seen in clinical trials.
- · A lower dose of omega 3 (about 1 g) has been associated with reduction in cardiovascular events in CHD patients and is used by some clinicians for this purpose.
- 2) SIADH is characterised by all except;
- a. Urine osmolarity > 100mOsm/kg
- b. Serum osmolarity >280mOsm/kg
- c. Serum sodium < 135mEq/l
- d. Urinary sodium > 30mEq/l
Ans, B, Serum osmolarity >280mOsm/kg
Ref: Nelson textbook of pediatrics 21st edition pg 2070
- Diagnostic Criteria for Syndrome of Inappropriate Antidiuretic Hormone Secretion
- • Absence of: Renal, adrenal, or thyroid insufficiency, Heart failure, nephrotic syndrome, or cirrhosis, Diuretic ingestion, Dehydration
- • Urine osmolality >100 mOsm/kg (usually > plasma)
- • Serum osmolality <280 mOsm/kg and serum sodium <135 mEq/L
- • Urine sodium >30 mEq/L
- • Reversal of “sodium wasting” and correction of hyponatremia with water restriction
- 3) Leucopenia is NOT seen in?
- a. New born
- b. Starvation
- c. Enteric Fever
- d. Viral or protozoal infection
Ans, D, Viral or protozoal infection
Ref: Robbins 10th Edition p582-83
- Leukocytosis is seen with Viral infections on account of increase in number of natural killer cells
- Leucopenia : MI, Sepsis, CLL, Lithium intoxication, Down's syndrome, Polyarteritis nodosa, Trauma, Cushing's disease
-
33
- 1) What is the correct sequence of stages of overcoming addiction?
- a. Contemplation, pre-contemplation, action, preparation, maintenance, relapse
- b. Preparation, pre-contemplation, contemplation, action, maintenance, relapse
- c. Pre-contemplation, contemplation, preparation, action, maintenance, relapse
- d. Pre-contemplation, contemplation, action, preparation, maintenance, relapse
Ans, C, Pre-contemplation, contemplation, preparation, action, maintenance, relapse
Ref. FA, USMLE step 1, 2020, Page no.568
- Stages of change in overcoming addiction
- Pre-contemplation—denying problem
- Contemplation—acknowledging problem, but unwilling to change
- Preparation/determination—preparing for behavioral changes
- Action/willpower—changing behaviors
- Maintenance—maintaining changes
- Relapse—(if applicable) returning to old behaviors and abandoning changes
- 2) Freiberg's disease is:
- a. Avascular necrosis of navicular
- b. Avascular necrosis of lunate
- c. Osteochondritis of the calcaneal apophysis
- d. Osteochondritis of the second metatarsals
Ans, D, Osteochondritis of the second metatarsals
(Ref: Apley 8th ed, pg 101-102)
-Frieberg's disease is a crushing type of osteochondritis of the second metatarsal head (rarely the third ) affects mostly young women.
- 3) Which artery is not the branch of internal carotid artery?
- a. Ophthalmic artery
- b. Caroticotympanic artery
- c. Middle cerebral artery
- d. Posterior cerebral artery
Ans, D, Posterior cerebral artery
- Branches of Internal carotid artery (10 branches)
- 1. Cervical part
- • No branches
- 2. Petrous part
- a) Caroticotympanic artery
- b) Pterygoid artery
- 3. Cavernous part
- a) Cavernous artery
- b) Hypophyseal artery
- c) Meningeal artery
- 4. Cerebral part
- a) Posterior communicating artery
- b) Ophthalmic artery
- c) Middle cerebral artery
- d) Anterior cerebral artery
- e) Anterior choroid artery
-
34
- 1) Which of the following radiological sign will be present in a young lady with symptoms suggestive of pulmonary embolism?
- a. Hampton's hump
- b. Westermark sign
- c. Fleischner sign
- d. All of the above
Ans, D, All of the above
- Pulmonary infarction may occur if the pulmonary venous pressure is elevated or the
- bronchial arterial supply to a region is deficient. The cone-shaped area of pulmonary
- infarction has been called a Hampton's hump. An area of radiolucency, corresponding to diminished pulmonary vascularity distal to a pulmonary embolism, is occasionally seen and is called the Westermark sign. There may also be an increase in the size of the pulmonary artery proximal to a large central pulmonary embolus (Fleischner sign).
- 2) What are pathognomic lesions of acne vulgaris?
- a. Tiny pustule
- b. Nodule
- c. Comedone
- d. Keratin accumulation
Ans, C, Comedone
Ref, Illustrated Synopsis of Dermatology and Sexually Transmitted Diseases, Neena Khanna, 4th edition, page no.112
- &Comedones: Are pathognomonic lesions of acne vulgaris
- Two main types of comedones recognized:
- Open comedones (black heads):are dueto plugging of the pilosebaceous orificeby keratin and sebum on the skin surface.
- Closed comedones (white heads):are due to keratin and sebum accretions plugging the pilosebaceous ducts below the skin surface. Some closed comedones are deep seated (submarine comedones) and are best seen by stretching the skin.Submarine comedones respond poorly to medical treatment.
- 3) Congenital deficiency of which factor result in parahemophilia?
- a. Factor V
- b. Factor VIII
- c. Factor IX
- d. Factor XI
Ans: A, Factor V
Ref: Nelson textbook of pediatrics 21st edition pg 10194
- ☆ V Labile factor, proaccelerin Congenital deficiency (parahemophilia)
- ☆ VIII Antihemophilic factor Congenital deficiency is hemophilia A (classic hemophilia)
- ☆ IX Christmas factor Congenital deficiency is hemophilia B (sometimes referred to asChristmas disease)
- ☆ XI Plasma thromboplastinantecedentCongenital deficiency (sometimes referred to as hemophilia C)
-
35
- 1) Normal PT is found in patients with deficiency of factor(s)
- a. VIII
- b. VII
- c. IX
- d. Both a and c
Ans, D, Both a and c
Ref: Nelson textbook of pediatrics 21st edition pg 10203
In The laboratory, prothrombin time (PT) measures the activation of factor X by factor VII and is therefore normal in patients with factor VIII or factor IXdeficiency
- 2) With the patient lying in lithotomy position, the classical location of hemorrhoids are?
- a. 3, 9 and 12 o’clock position
- b. 3, 7 and 12 o’clock position
- c. 3, 7 and 11 o’clock position
- d. 3, 9 and 11 o’clock position
Ans: C, 3, 7 and 11 o’clock position
Ref: Bailey and Love 27th edition, Pg no. 1355
- Explanation:
- Haemorrhoids: clinical features
- Haemorrhoids or piles are symptomatic anal cushions. They are more common when intra-abdominal pressure is raised, e.g. in obesity, constipation and pregnancy. Classically, they occur in the 3, 7 and 11 o’clock positions with the patient in the lithotomy position.
- Symptoms of haemorrhoids: Bright-red, painless bleeding, mucus discharge, prolapse, pain only on prolapsed.
- 3) Upper part of the faucial pillars and most of the uvula are visible ,what is the malampatti grading of this patient;
- a. I
- b. II
- c. III
- d. IV
- Ans, B, II
- Ref: Text book of clinical anesthesiology, Morgan and Mikhail, 5th edition, page 313
- Mallampati classifi cation: a frequently performed test that examines the size of the tongue in relation to the oral cavity. The greater the tongue obstructs the view of the pharyngeal structures, the more difficult intubation may be.
- ■ Class I: the entire palatal arch, including the bilateral faucial pillars, are visible down to their bases.
- ■ Class II: the upper part of the faucial pillars and most of the uvula are visible.
- ■ Class III: only the soft and hard palates are visible.
- ■ Class IV: only the hard palate is visible.
- • Th yromental distance: the distance between the mentum and the superior thyroid notch. A distance greater than 3 finger breadths is desirable.
- • Neck circumference: a neck circumference of greater than 27 in is suggestive of difficulties in visualization of the glottic opening. Although the presence of these findings may not be particularly sensitive for detecting a difficult intubation, the absence of these findings is predictive for relative ease of intubation.
-
36
- Q 7) A transplant from one member to another in a common species is called?
- a. Autograft
- b. Isograft
- c. Xenograft
- d. Allograft
Ans, D, Allograft
Ref, Textbook of Medical Physiology, Guyton and Hall, 13th edition, page no.481
- Autografts, Isografts, Allografts, and Xenografts.
- A transplant of a tissue or whole organ from one part of the same animal to another part is called anautograft;
- from one identical twin to another, an isograft;
- from one human being to another or from any animal to another animal of the same species, an allograft;
- from a non-human animal to a human being or from an animal of one species to one of another species, a xenograft.
- Q 8 Marker specific for Myocardial infarction :
- a. Troponin I
- b. Troponin C
- c. Troponin T
- d. LDH
Ans, A, Troponin I
( Ref: Harrison 19th ed/ pg. 1600)
-The best test for diagnosis of MI is troponin I which can be quantified as value more than 0.04 ng/dl after 3 hours of onset of myocardia infarction.
- Q 9 All of the following antimicrobial drugs acts by binding to 50s subunit of ribosome EXCEPT
- a. Macrolides
- b. Chloramphenicol
- c. Tetracycline
- d. Clindamycin
Ans, C, Tetracycline
- Drugs Acting on 50S subunit
- • Macrolides (Azithromycin, Clarithromycin, Erythromycin)
- • Chloramphenicol
- • Clindamycin
- • Linezolid
- • Streptogramins
- - Quinuristin
- - Dalfopristin
-
37
- Q 4) UMN lesion causes facial nerve palsy:
- a. Contralateral upper face
- b. Contralateral lower face
- c. Ipsilateral upper face
- d. Ipsilateral lower face
Ans, B, Contralateral lower face
( Ref : Harrison 17th ed/ pg. 2487)
- Weakness of the lower two thirds of the face with preservation of upper third suggests UMN lesion, whereas weakness of an entire side suggests a LMN lesion.
- Q5) The little’s area is supplied by :
- a. Superior labial, anterior ethmoidal, greater palatine and spheno-ethmoidal arteries.
- b. Superior labial, posterior ethmoidal, greater palatine and spheno-ethmoidal arteries.
- c. Superior labial, anterior ethmoidal, lesser palatine and spheno-ethmoidal arteries.
- d. None of the above.
Ans: D, None of the above
Ref: PL Dhingra
- Explanation:
- Kiesselbach's plexus, which lies in Kiesselbach's area, Kiesselbach's triangle, or Little's area, is a region in the anteroinferior part of the nasal septum where four arteries anastomose to form a vascular plexus of that name. The arteries are:
- ● Anterior ethmoidal artery (from the ophthalmic artery)
- ● Sphenopalatine artery (terminal branch of the maxillary artery)
- ● Greater palatine artery (from the maxillary artery)
- ● Septal branch of the superior labial artery (from the facial artery)
- Q6) Which of the following condition gives the characteristic radiological finding "bone within a bone" appearance?
- a. Osteogenesisimperfecta
- b. Osteopetrosis
- c. Scurvy
- d. Rickets
Ans, B, Osteoporosis
- Reference: radiopedia.org
- Osteopetrosis also known as Albers-Schonberg disease or marble bone disease is an uncommon hereditary disorder that results from defective osteoclasts.
-
38
- Q 1) Limiting amino acids in Cereals & Pulses respectively:
- a. Lysine& Threonine
- b. Lysine& Cysteine
- c. Glycine& Methionine
- d. Lysine& Methionine
Ans, D, Lysine& Methionine
Cereal proteinsare deficient in lysine and threonine, and pulse proteins in methionine. These are known as "limiting" amino acids. Whentwo or more of vegetarian foods are eaten together (as for example, rice-dhal combination in India) their proteinssupplement one another and provide a protein comparable to animal protein in respect of EAA.
- Q 2) Which of the following is less likely correct regarding indications and use of ECT in a schizophrenic patient?
- a. Catatonia is one of the indications
- b. Used in acute exacerbation not controlled with drugs
- c. Preferred if severe side-effect with drugs occur
- d. Usually one to two ECT appointments is enough to treat the patient
Ans, D, Usually one to two ECT appointments is enough to treat the patient
- Electroconvulsive Therapy (ECT)
- Schizophrenia is not a primary indication for ECT. The indications for ECT in schizophrenia include:
- 1. Catatonic stupor.
- 2. Uncontrolled catatonic excitement.
- 3. Acute exacerbation not controlled with drugs.
- 4. Severe side-effect with drugs, in presence of untreatedschizophrenia.
- Usually 8-12 ECTs are needed (although up to 18have been given in poor responders), administered two or three times a week.
- Q 3) A complication of common cold:
- a. Otitis externa.
- b. Quinsy.
- c. Facial palsy.
- d. Maxillary sinusitis.
Ans, D, Maxillary sinusitis.
- ☆ Otitis media is a recognized complication of common cold.
- ☆ Quinsy is a recognized complication of acute tonsillitis.
- ☆ Facial palsy may be a complication of otitis media.
- ☆ Common cold is the most common cause of maxillary sinusitis.
-
39
- Q. Lateral boundary of cubital fossa is formed by?
- a. Brachioradialis
- b. Pronator teres
- c. Bicipital aponeurosis
- d. Supinator
Ans: a. Brachioradialis
- ☆ Boundaries of cubital fossa
- Laterally — medial border of the brachioradialis
- Medially — lateral border of the pronator teres. — is directed upwards, and is represented by an imaginary line joining the front of two epicondyles of the humerus.
- Apex — is directed downwards, and is formed by the meeting point of the lateral and medial boundaries.
- Roof:
- The roof of the cubital fossa is formed by: (a) Skin (b) superficial fascia containing the median cubital vein, the lateral cutaneous nerve of the forearm and the medial cutaneous of the forearm (c)deep fascia. (d) bicipital aponeurosis.
- Floor:
- It is formed by: (i) the brachialis and (ii) the supinator muscles
- Q. Injury to which part of the brachial plexus cause Erb's Paralysis?
- a. Root
- b. Upper trunk
- c. Lower trunk
- d. Posterior cord
Ans: b. Upper trunk
- Erb's Paralysis
- Site of injury :The region of the upper trunk of the brachial plexus is called Erb's point .Six nerves meet here. Injury to the upper trunk causes Erb's paralysis
- Causes of injury
- Undue separation of the head from the shoulder, which is commonly encountered in: (i) birth injury, (ii) fall on the shoulder, and (iii) during anaesthesia.
- Nerve roots involved
- Mainly C5 and partly C6. Muscles paralysed. Mainly biceps, deltoid, brachialis and brachioradialis. Partly supraspinatus, infraspinatus and supinator.
- Deformity (position of the limb) Arm:
- Hangs by the side; it is adducted and medially rotated. Forearm: Extended and pronated. The deformity is known as 'policeman's tip hand' or 'porter's tip hand'.
- Disability.
- The following movements are lost:
- 1 . Abduction and lateral rotation of the arm (shoulder)
- 2.Flexion and supination of the forearm.
- 3.Biceps and supinator jerks are lost.
- 4.Sensations are lost over a small area over the lower part of the deltoid.
- Q. An agent which prevents the action of an agonist on a receptor or the subsequent response, but does not have any effect of its own is called:
- a. Agonist
- b. Inverse agonist
- c. Antagonist
- d. Partial antagonist
Ans: c. Antagonist
- The following terms are used in describing drug-receptor interaction:
- Agonist : An agent which activates a receptor to produce an effect similar to that of the physiological signal molecule.
- Inverse agonist : An agent which activates a receptor to produce an effect in the opposite direction to that of the agonist.
- Antagonist : An agent which prevents the action of an agonist on a receptor or the subsequent response, but does not have any effect of its own.
- Partial agonist : An agent which activates a receptor to produce submaximal effect but antagonizes the action of a full agonist.
- Ligand : Any molecule which attaches selectively to particular receptors or sites.
-
40
- Q1. Depth perception and colour vision develops by age of:
- a.2 weeks
- b.1-2months
- c.3-4 months
- d.5-6 months
Ans: d, 5-6 months
- Q2.cANCAis associated with:
- a.Microscopic polyangitis
- b.Wegners granulomatosis
- c.SLE
- d.Polyarteritis nodusa
Ans: b, Wegners granulomatosis
- Q3. Pseudomembranous colitis is caused by
- a.Metronidazole
- b.Oral vancomycin
- c.Clindamycin
- d.All of the above
Ans: c, Clindamycin
- Clostridium difficile
- • Produces toxins A and B, which damage enterocytes. Both toxins lead to watery diarrhea= pseudomembranous colitis.Often 2° to antibiotic use, especially clindamycin or ampicillin; associated with PPIs.
- • Diagnosed by PCR or antigen detection of one or both toxins in stool.
- • Complications: toxic megacolon.
- • Treatment: oral vancomycin, metronidazole, or fidaxomicin. For recurrent cases, consider repeating prior regimen or fecal microbiota transplant.
-
41
- 1.Which of the following is not a risk factor for Carcinoma endometrium?
- a. Family history
- b. Early menopause
- c. Obesity
- d. Use of HRT
Ans.B Early menopause
- The following are found to be related to carcinoma endometrium
- • Estrogen—Persistent stimulation of endometrium with unopposed estrogen is the single most important factor for the development of endometrial cancer.
- • Age—About 75 percent are postmenopausal with a median age of 60 (c.f. Carcinoma cervix is more common in perimenopausal period). About 10 percent of women with postmenopausal bleeding have endometrial cancer.
- • Parity—It is quite common in unmarried and in married, nulliparity is associated in about 30 percent. (c.f. – Carcinoma cervix is associate more with multiparae).
- • Late menopause—The chance of carcinom increases, if menopause fails to occur beyond 52 years.
- • Corpus cancer syndrome — encompasses –obesity, hypertension and diabetes.
- • Obesity leads to high level of free estradiol as the sex hormone binding globulin level is low.
- • Unopposed estrogen stimulation in conditions such as functioning ovarian tumors (granulosa cell) or polycystic ovarian syndrome (PCOS) is associated with increased risk of endometrial cancer. Unopposed estrogen replacement therapy in postmenopausal women is associated with increased risk of endometrial cancer. Use of cyclic progestin reduces the risk. Prior use of combined oral contraceptives reduces the risk significantly (50%). Tamoxifen is antiestrogenic as well as weakly estrogenic. It is used for the treatment of breast cancer. Increased risk of endometrial cancer is noted when it is used for a long time due to its weak estrogenic effect. Family history or personal history of colon, ovarian or breast cancer increases the risk of endometrial cancer. Genetic inheritance (Lynch 11 syndrome family) is currently thought of. Fibroid is associated in about 30 percent cases. Endometrial hyperplasia precedes carcinoma in about 25 percent cases.
- 2.An 80-year-old female who has never taken estrogen,develops pink vaginal discharge. An endometrial biopsy shows an adenocarcinoma of the endometrium. Papanicolaou smear is negative. Of the following what is the most important indicator of prognosis?
- a. Body habitus
- b. Level of CA – 125
- c. Nutritional status
- d. Histologic type of tumour
Ans D.Histologic type of tumor
- Prognosis of endometrial carcinoma:
- The poorly differentiated tumor, the greater degree of myometrial penetration, lymphovascular space invasion and the advanced stages are prognostically poor. Aneuploid tumors are prognostically worst. Histologically non-endometrioid tumors are aggressive and carry increased risk of recurrence. The prognostic factors to be considered are
- • Age at diagnosis (older the patient poorer the prognosis).
- • Stage of the disease.
- • Histologic type (typical adenocarcinomas — better prognosis, papillary serous, clear cell carcinoma — poor prognosis).
- • Histologic differentiation.
- • Histologic grade ,grade 3 tumors have 5 times more risk of recurrence and low 5 year survival rate.
- • Myometrial penetration.
- • lymphovascular space invasion.
- • lymph node metastasis.
- • extension to cervix.
- • Peritoneal cytology.
- • Tumor size (>2 cm → more lymph node metastasis).
- • Hormone receptor status (receptor positive tumors have got better prognosis).
- • Ploidy status — aneuploid tumors have got poor prognosis compared to diploid tumors.
- • oncogene expression — HeR – 2/neu, poor prognosis .
- 3.what is the further investigation required for a follicular cyst of 3cm seen in USG?
- A.CA-125
- B.Laparoscopy
- C.Repeated ultrasound in 3 to 6 months
- D.No further investigation required
Ans.D ,No further investigation
- Follicular Cysts
- Follicular cysts are the commonest functional cyst.They are usually multiple and small as seen in cases of cystic glandular hyperplasia of the endometrium or in association of fibroid. Hyperestrinism is implicated as its cause. However, an isolated cyst may be formed in unruptured Graafian follicle, which may be enlarged but usually not exceeding 5 cm. The cyst is lined by typical granulosa cells without lutein cells or the cells may be flattened due to pressure In majority of cases, the detection is made accidentally on bimanual examination, sonography, laparoscopy or laparotomy. The cyst may remain asymptomatic or may produce vague pain.Management:
- A follicular cyst < 3 cm requires no further investigations. A simple cyst, < 8 cm,unilocular, echo free without solid areas or papillary projections, with normal serum CA 125 ,should be followed up with repeat ultrasound in 3 to 6 months time.Whenever a cyst persists or grow, it should be removed by laparoscopy/laparotomy.
-
42
- Q.1 Immediate life threatening injuries of the chest include all of the following except ?
- a. Airway obstruction
- b. Pericardial tamponade
- c. Flail chest
- d. Tracheo-bronchial injuries
Ans: d, Tracheo-bronchial
- Explanation:
- The ‘deadly dozen’ threats to life from chest injury;
- Immediately life threatening
- Airway obstruction
- Tension pneumothorax
- Pericardial tamponade
- Open pneumothorax
- Massive haemothorax
- Flail chest
- Potentially life threatening
- Aortic injuries
- Tracheobronchial injuries
- Myocardial contusion
- Rupture of diaphragm
- Oesophageal injuries
- Pulmonary contusion
- Q.2 A mass like lesion on the lateral side of the neck (upper one third of sternocleidomastoid muscle) is suspected as a case of Branchial cyst? From where does this cyst develop from?
- a. First pharyngeal cleft
- b. Second pharyngeal cleft
- c. First pharyngeal arch
- d. Second pharyngeal arch
Ans:b, second pharyngeal cleft
- Explanation:
- A branchial cyst develops from the vestigial remnants of the second branchial cleft, is lined by squamous epithelium and contains thick, turbid fluid. The cyst usually presents in the upper neck in early or middle adulthood and is found at the junction of the upper third and middle third of the sternomastoid muscle at its anterior border. It is a fluctuant swelling that may transilluminate and is often soft in its early stages so that it may be difficult to palpate.
- Q.3 All are the purpose of metabolic changes following injury except?
- a. Provide essential substrates for survival
- b. Postpone anabolism
- c. Optimize host defense
- d. Postpone catabolism
Ans: d, postpone catabolism
- Explanation:
- Purpose of neuro-endocrine changes following injury
- The constellation of neuro-endocrine changes following injury acts to:
- · Provide essential substrates for survival
- · Postpone anabolism
- · Optimize host defense
- These changes may be helpful in the short term, but may be harmful in the long term, especially to the severely injured patient who would otherwise not have survived without medical intervention.
-
43
- 1. Shilling’s test is used to determine deficiency of:
- a. Vitamin B12
- b. Vitamin B6
- c. Folic Acid deficiency
- d. Vitamin D deficiency
Ans.a, Vitamin B12
- The Schilling test is a medical investigation used for patients with vitamin B12 (cobalamin) deficiency.The purpose of the test is to determine how well the patient is able to absorb B12 from their intestinal tract.
- In this test, the patient will ingest tiny amount of radioactive Vit B12.Then the blood is checked whether the patient’s body had absorbed Vit B12 or not. After that, the patient will ingest the combination of Vit B12 and intrinsic factor. If the radioactive Vit B12 is absorbed only when taken with intrinsic factor, it shows that the patient lacks intrinsic factor.
- 2.Most common nephrotic syndrome in elderly patients:
- a. Minimal change GN
- b. Membranous GN
- c. IgA nephropathy
- d. None of the above
Ans.b, Membranous GN
Membranous glomerulonephritis (MGN) , or membranous nephropathy accounts for approximately 30% of cases of nephrotic syndrome in adults, with a peak incidence between the ages of 30 and 50 years and a male to female ratio of 2:1. It is rare in childhood and the most common cause of nephritic syndrome in the elderly. In 25–30% of cases, MGN is associated with a malignancy (solid tumors of the breast, lung, colon), infection (hepatitis B, malaria, schistosomiasis), or rheumatologic disorders like lupus or rarely rheumatoid arthritis.
- 3.The most common site of intestinal obstruction in gallstone ileus is:
- a. Duodenum
- b. Jejunum
- c. Ileum
- d. Sigmoid colon
Ans.c, Ileum
Gallstone ileus refers to mechanical intestinal obstruction resulting from the passage of a large gallstone into the bowel lumen. The stone customarily enters the duodenum through a cholecystoenteric fistula at that level. The site of obstruction by the impacted gallstone is usually at the ileocecal valve, provided that the more proximal small bowel is of normal caliber. The majority of patients do not give a history of either prior biliary tract symptoms or complaints suggestive of acute cholecystitis or fistulization. Large stones, >2.5 cm in diameter, are thought to predispose to fistula formation by gradual erosion through the gallbladder fundus. Diagnostic confirmation may occasionally be found on the plain abdominal film (e.g., small-intestinal obstruction with gas in the biliary tree and a calcified, ectopic gallstone) or following an upper gastrointestinal series (cholecystoduodenal fistula with small-bowel obstruction at the ileocecal valve).
-
44
- You are a first year resident in psychiatry department of TUTH and a patient came and you assessed him as Type A personality. All are cluster A personality disorders except which one of the following?
- a. Borderline
- b. Schizoid
- c. Schizotypal
- d. Paranoid
Ans, A, Borderline
- There are three types of personality disorders which are as follows.
- Cluster A: Paranoid, Schizoid, Schizotypal
- Cluster B: Antisocial, Histrionic, Borderline, Narcissistic
- Cluster C: Dependent, Anxious/Avoidant, Obsessive compulsive
-
45
- Romana's sign occurs in
- a. Babesiosis
- b. Leishmaniasis
- c. Trypanosomiasis
- d. Schisotosomiasis
- Ans: c
- Acute chagas disease:
- .Acute phase occurs soon after infection and may last for 1- 4 months
- . lt is seen often in children under 2 years of age.
- • First sign appears within a week after invasion of parasite.
- • "Chagoma" is the typical subcutaneous lesion occurring at the site of inoculation. Inoculation of the parasite in conjunctiva causes unilateral, painless edema of periocular tissues in the eye called as Romana's sign,. This is a classical finding of the acute Chagas disease.
- • In few patients, there may be generalized infection with fever, lymphadenopathy and hepatosplenomegaly.
- • The patient may die of acute myocarditis and meningoencephalitis.
- • Usually within 4-8 weeks, acute signs and symptoms resolve spontaneously and patients, then enter the asymptomatic or indeterminate phase of chronic T. cruzi infection.
-
46
- Larval forms of which parasite can be found in muscle biopsy
- a. Ascaris lumbricoides
- b. Taenia solium
- c. Trichuris trichiura
- d. Ancylostoma duodenale
- Ans: b
- MUSCLE BIOPSY
- Spiral larval form of Trichinella spiralis, larval form of T. solium (cyslicercus cellulosae) and
- amastigote ofTrypanosoma cruzi can be demonstrated in skeletal muscle biopsy. In trichinosis, muscle biopsy (gascrocnemius, deltoid and biceps) specimen must be examined by compressing thetissue be tween two slides and checking the preparation under low-power (lOX) objective. this method doesn' t become positive until 2-3 weeks after the illness.
- Ref: Panikers textbook of medical parasitology 8th edition pg243
-
47
- Q.3The current treatment of choice for a large antrochoanal polyp in a 10-year-old is:
- a. Intranasal polypectomy
- b.Caldwell Luc operation
- c. FESS
- d. Lateral rhinotomy and excision
Ans. is c i.e. FESS
- An antrochoanal polyp is easily removed by avulsion either through the nasal or oral route.
- Recurrence is uncommon after complete removal. In cases which do recur, Caldwell–Luc operation may be required to remove the polyp completely from the site of its origin and to deal with coexistent maxillary sinusitis.
- These days, endoscopic sinus surgery has superseded other modes of polyp removal. Caldwell–Luc operation is avoided.
-
48
- Q. 2 Most common cause of upper GI bleeding is?
- a. Oesophageal varices
- b. Erosions
- c. Vascular lesions
- d. Ulcers
Ans: d
- Explanation:
- Causes of upper gastrointestinal bleeding.
- Condition %
- Ulcers 60
- Oesophageal 6
- Gastric 21
- Duodenal 33
- Erosions 26
- Oesophageal 13
- Gastric 9
- Duodenal 4
- Mallory–Weiss tear 4
- Esophageal varices 4
- Tumour 0.5
- Vascular lesions 0.5
- Others 5
-
49
- Q.1 Treatment of high grade mucoepidermoid tumor of salivary gland is?
- a. Wide local excision.
- b. Wide local excision + Regional node dissection.
- c. Wide local excision + Regional node dissection + Radiotherapy.
- d. Wide local excision + Regional node dissection + Radiotherapy + Chemotherapy.
Ans: (c) Wide local excision + Regional node dissection + Radiotherapy.
- Explanation:
- - For Low grade malignant tumors excision of the tumor with complete parotid (with clear margins) is thenaim.
- - No adjuvant chemotherapy is required.
- - For high grade cancers after generous primary excision, for clinically palpable nodes radical or modified radical neck dissection is performed followed by radiotherapy in selected group.
-
50
- What is the maximum diameter of Tardieu spots that is seen in case of asphyxia?
- a. 0.5mm
- b. 1mm
- c. 2mm
- d. 5mm
Ans, C, 2mm
Ref, The Essentials of Forensic Medicine and Toxicology, Reddy, 33rd edition, page no.141
Tardieu spots are numerous where the capillaries are least firmly supported, as in sub- conjunctival tissues and under the pleural and pericardia membranes but they can appear almost anywhere if the degree of congestion and cyanosis is sufficient. Tardieu spots are usually rounded, dark and well-defined, varying in size from a pin's head to 2 mm. They may occur as isolated minute haemorrhages or present in large numbers, and at times fuse to form patches of red colour, especially at the back of the heart.
-
51
What is the correct sequence of stages of overcoming addiction?
- a. Contemplation, pre-contemplation, action, preparation, maintenance, relapse
- b. Preparation, pre-contemplation, contemplation, action, maintenance, relapse
- c. Pre-contemplation, contemplation, preparation, action, maintenance, relapse
- d. Pre-contemplation, contemplation, action, preparation, maintenance, relapse
Ans, C, Pre-contemplation, contemplation, preparation, action, maintenance, relapse
Ref. FA, USMLE step 1, 2020, Page no.568
- Stages of change in overcoming addiction
- i. Pre-contemplation—denying problem
- ii. Contemplation—acknowledging problem, but unwilling to change
- iii. Preparation/determination—preparing for behavioral changes
- iv. Action/willpower—changing behaviors
- v. Maintenance—maintaining changes
- vi. Relapse—(if applicable) returning to old behaviors and abandoning changes
-
52
- Elfin facies are seen in:
- a.Down syndrome
- b.William syndrome
- c.Mucopolysaccharidosis
- d.Cushing syndrome
- Ans : b
- .Mongoloid/Down facies
- Upward slant of eyes, epicanthic folds,
- hypertelorism, flattened nasal bridge,
- protuberant tongue.
- • Elfin facies
- Upromed nose, broad upper lip, small mandible, prominent maxilla, short statute.
- • Moon/steroid/cushingoid facies
- Rounded face, chubby and puffy look, double chin, hitsutism.
- • Hemolytic facies
- Pallor, malar prominence, frontal bossing, depressed nasal bridge.
- • Risus sa;dooicus
- Due to spasm of the platysma muscle, the
- cornets of the mouth move downwards and outwards, the lip is stretched across the incisor teeth.
- • Coarse facies
- Dry skin, thick tongue, thickened facial outlines
- • Potter facies
- Suborbital creases, depressed nasal tip, low set ears, retrognathia, associated with positional defonmities of limbs, like clubfeet.
- Ref:Clinical Methods in pediatrics 4th ed pg 186
-
53
- Q.1 Heineke–Mikulicz principle to ‘lengthen’ the frenulum is used in?
- a. Frenulum breve
- b. Paraphimosis
- c. Phimosis
- d. Peyronie´s disease
- Ans: a
- Explanation:
- Frenulum breve
- Phimosis should not be confused with this condition, where the frenulum is short, such that it causes pain when the fore-skin is retracted. Another possible presentation is tearing of the frenulum during sexual activity. Treatment is by frenu-loplasty, which utilises the Heineke–Mikulicz principle to ‘lengthen’ the frenulum.
- Ref: Bailey and love 27th edition , Pg no.1489
- Q.2 The adult prevalence of visible varicose veins is?
- a. 10-20%
- b. 20-30%
- c. 30-50%
- d. 50-60%
- Ans: c
- Explanation:
- The adult prevalence of visible varicose veins is between 30% and 50%. Factors affecting prevalence include:
- ● Gender: the vast majority of studies report a higher preva-lence in women than men, though community prevalence may differ.
- ● Age: the prevalence of varicose veins increases with age.
- In the Edinburgh Vein study, the prevalence of trunk vari-cosities in the age groups 18–24 years, 25–34 years, 35–44 years, 45–57 years and 55–64 years was 11.5%, 14.6%, 28.8%, 41.9% and 55.7%, respectively.
- ● Ethnicity: does seem to inf l uence the prevalence of vari-cose veins.
- ● Body mass and height: increasing body mass index and height may be associated with a higher prevalence of vari-cose veins.
- ● Pregnancy: increases the risk of varicose veins.
- ● Family history: evidence supports familial susceptibility to varicose veins.
- ● Occupation and lifestyle factors: there is inconclusive evi-dence regarding increased prevalence of varicose veins in smokers, patients who suffer constipation and occupations that involve prolonged standing.
- Ref: Bailey and love 27th edition, Pg no. 975
-
54
Secretory proteins are synthesized in
- a) First in cytoplasm and then in endoplasmic reticulum
- b) First in endoplasmic reticulum and then in cytoplasm
- c) Endoplasmic reticulum
- d) Cytoplasm
Ans, C, Endoplasmic reticulum
Ref, Harper’s Biochemistry, 26th edition, page no. 498-502
- The proteins which are meant to be transported or secreted are synthesized in ribosomes attached to the endoplasmic reticulum. But, proteins which have to be delivered in the cytosol are synthesized in free polyribosomes in cytosol.
- The secretory proteins and transport proteins contain signal peptides, which helps them to be transported into Golgi apparatus via endoplasmic reticulum.
- Golgi body is the organelle where sorting of proteins for their respective destination occurs. The proteins which enter the Golgi apparatus are segregated into two types when they reach the exit side of the Golgi apparatus. These two types are-
- i. Transport proteins- they are destined for plasma membrane. They are stored in transport vesicles and are secreted in constitutive manner.
- ii. Secretory proteins- They are destined for secretion and are stored in secretory vesicles.
-
55
- Bullet which is left in the body for very long time is known as?
- a) Tandem bullet
- b) War bullet
- c) Souvenir bullet
- d) Tracer bullet
Ans, C, Souvenir bullet
Ref, SK Singhal’s Forensics, 4th edition, page no. 188
- Types of bullet:
- i. Tandem bullet/piggy tail bullet: Bullet not released on first fire comes out with second bullet on second shot.
- ii. Souvenir bullet: Bullet which is left in the body for long.
- iii. Tracer bullet: Bullet that leaves a visible mark in path that a person can see
- iv. Ricocheting bullet:Bullet deviated from its path after coming in contact with a surface.
-
56
- Q.1 During repair of hernia using mesh, position of mesh can be?
- a. Just outside the muscle in the subcutaneous space.
- b. Between fascial layers in the abdominal wall.
- c. Immediately extraperitoneally, against muscle or fascia (also sublay).
- d. Any of the above.
- Ans: d
- Explanation:
- Positioning the mesh
- The strength of a mesh repair depends on host–tissue in-growth. Meshes should be placed on a firm, well vascularised tissue bed with generous overlap of the defect.
- The mesh can be placed:
- • Just outside the muscle in the subcutaneous space (onlay);
- • Within the defect (inlay) – only applies to mesh plugs in small defects;
- • Between fascial layers in the abdominal wall (intraparietal or sublay);
- • Immediately extraperitoneally, against muscle or fascia (also sublay);
- • Intraperitoneally.
- At open surgery all of these planes are used but laparoscopic surgeons currently use only intraperitoneal or extraperitoneal planes.
- Ref: Bailey and love 27th edition
-
57
- Q.3 To see the tympanic membrane, pinna has to be pulled?
- a. Upward, backward and laterally
- b. Upward, backward and medially
- c. Downward, backward and laterally
- d. Downward, front and laterally
- Ans: a
- Explanation: EAC is not a straight tube; its outer part is directed upwards, backwards and medially while its inner part is directed downwards, forwards and medially. Therefore, to see the tympanic membrane, the pinna has to be pulled upwards, backwards and laterally so as to bring the two parts in alignment.
- Ref: Diseases of ear, nose and throat and head and neck surgery, PL Dhingra,6th edition, Pg,no.2
-
58
- 1. The presence of nitrates in water indicates:
- A. Temporary hardness of water
- B. Permanent hardness of water
- C. Fresh faecal pollution
- D. Past faecal pollution
Ans: D. Past faecal pollution
Faecal and animal wastes products containing inorganic nitrogen and organic nitrogen are first decomposed to give ammonia, which is then oxidized to nitrite and nitrate later on.
- 2. Threshold radiation dose for haematological syndrome is?
- A. 1 Gy
- B. 5 Gy
- C. 25 Gy
- D. 50 Gy
Ans: A. 1 Gy
Threshold radiation of acute radiation syndromes:
- Hematological syndrome - 1Gy
- Gastrointestinal syndrome - 5Gy
- CNS syndrome - 20Gy
- 3. Which ONE of the following is not an endogenous mediator of increased vascular permeability?
- A. Angiotensin
- B. C3a and C5a
- C. 5-hydroxytryptamine
- D. Kallikrein9.
Ans: A. Angiotensin
☆ Angiotensin is produced by the action of renin on angiotensinogen and is involved in the secretion of aldosterone and in pressor effects.
- ☆ C3a and C5a (B) are part of the complement cascade and are activated C3 and C5;
- they act by liberating histamine from mast cells.
☆ 5-hydroxytryptamine(C)or serotonin causes increased vascular permeability in rodents but not in man.
☆ Kallikrein (D) is produced by the activation of Hageman factor (factor XII) producing prekallikrein activator which converts prekallikrein to kallikrein.
-
59
- 1. A patient is treated for a skin disease. Grattage test is seen when examined. Which is the disease?
- a) Psoriasis
- b) Erythema multiforme
- c) Lichen planus
- d) Tinea corporis
Ans, A
- Various bed side tests and signs are associated with psoriasis. They are as follows:
- i. Grattage test: scraping of psoriasis lesion causes accentuation of silvery scales, due to trapping of air.
- ii. Auspitz’s sign: scraping of psoriasis lesion causes accentuation of silvery scales but on continuing further, a glistening white membrane, punctate (pin-point) bleeding becomes visible, referred to as auspitz’s sign.
- iii. Woronoff’s ring: Occasionally a hypopigmented ring is seen surrounding the lesion, known as woronoff’s ring. Mostly associated with topical steroids.
- 2. Filigree burn is seen in
- a) Electrocution
- b) Lightning
- c) Fire
- d) Heat stroke
Ans, B, Lightning
Lightning strike causes arborescent or filigree burns or lichtenberg’s flowers which are superficial, thin, irregular and tortuous markings on the skin resembling the branches of a tree.
- 3. A patient comes to office with visual hallucinations. Which of the following is more likely cause?
- a) Depression
- b) Epilepsy
- c) Schizophrenia
- d) Drug intoxication
Ans, D, Drug intoxication
Ref. FA, USMLE step 1, 2018, Page no. 543
- Hallucinations Perceptions in the absence of external stimuli (eg, seeing a light that is not actually present). Contrast with illusions, misperceptions of real external stimuli. Types include:
- Visual – It is more commonly a feature of medical illness (e.g., drug intoxication) than psychiatric illness.
- Auditory—more commonly a feature of psychiatric illness (eg, schizophrenia) than medical illness.
- Olfactory—often occur as an aura of temporal lobe epilepsy (e.g., burning rubber) and in brain tumors.
- Gustatory—rare, but seen in epilepsy.
- Tactile—common in alcohol withdrawal and stimulant use (e.g., cocaine, amphetamines), delusional parasitosis, “cocaine crawlies.”
- Hypnagogic—occurs while going to sleep. Sometimes seen in narcolepsy.
- Hypnopompic—occurs while waking from sleep (“pompous upon awakening”). Sometimes seen in narcolepsy.
-
60
- Which of the following is the most common drug induced skin reaction?
- a) Papular rash
- b) Fixed drug eruption
- c) Morbilliform rash
- d) Photosentivity reaction
Ans, C, Morbiliform rash
Ref, Harrison’s Internal medicine, 18th edition, Chapter 55
- Morbilliform or maculopapular eruptions are the most common of all drug induced reactions, often start on the trunk or intertriginous areas, and consist of erythematous macules and papules that are frequently symmetric and may become confluent.
- Delayed hypersensitivity directed by drug specific T cells is probably the most important mechanism in the etiology of the most common drug eruptions, “morbilliform” (measles like) eruptions, and also of rare and severe forms such as hypersensitivity syndrome, acute generalized exanthematous pustulosis, Stevens Johnson syndrome and toxic epidermal necrolysis.
Eg. Penicillins, sulfonamides, phenytoin or gold will develop a maculopapular ruption.
-
61
Stapes footplate covers:
A. Round window.
B. Oval window.
C. Sinus tympani.
D. Aditus ad antrum.
Ans:B
The footplate of the stapes helps with hearing. The stapes bone notifies the ovalwindow when movement occurs. The stapes footplate proceeds into the oval window, sending the round window membrane out, allowing for the fluid in the cochlea to move, which leads to the cochlear inner hair cells’ movement, and finally, hearing.
-
62
- Retention of urine in Fowler´s syndrome is best treated by?
- a. Intermittent self catheterization
- b. Urethral dilatation
- c. Urethroplasty
- d. None
- Ans: a
- Explanation:
- Fowler’s syndrome
- This idiopathic condition, which was described by Fowler and Kirby, is associated with an abnormal myotonic discharge in the striated urethral sphincter that can be detected by sphincter electromyography. It is often associated with polycystic ovaries and causes urinary retention in women and should not be confused with a urethral stricture. Urethral dilatation is ineffective and the retention is best treated by intermittent self-catheterisation. There is some evidence that sacral neuro-modulation can be effective.
- Ref: Bailey and love 27th edition, Pg no.1485
-
63
- 1. A 13-year-old boy has been drinking large quantities of fluids and has an insatiable appetite. He is losing weight and has become more tired and listless for the past month. Laboratory findings include normal CBC and fasting serum glucose of 175 mg/dL. His parents, two brothers, and one sister are healthy. A maternal uncle is also affected. Which of the following is the probable inheritance pattern of his disease?
- a. Autosomal dominant
- b. Autosomal recessive
- c. Mitochondrial DNA
- d. Multifactorial
Ans: d
Type 1 diabetes mellitus has an increased frequency in some families, but the exact mechanism of inheritance is unknown. The risk is increased for offspring when first-order relatives are affected. HLA-linked genes and other genetic loci and environmental factors are considered important. This pat¬tern of inheritance is multifactorial. The other listed inheritance patterns are not seen with most cases of diabetes mellitus.
- 2. A 5-year-old child is exposed to Mycobacterium tubercu¬losis. A month later the child’s tuberculin skin test is positive. The child then develops fever, inspiratory stridor, and non¬productive cough. Which of the following findings is most likely to be present on the chest radiograph of this child?
- a. Hilar lymphadenopathy
- b. Miliary pulmonary nodules
- c. Pneumonic consolidation
- d. Upper lobe cavitation
Ans: a
- • The child has primary tuberculosis. Most healthy per¬sons have subclinical disease, and a minority develop clini¬cal manifestations; of those, most have limited pulmonary involvement without dissemination.
- • Primary tuberculosis is marked by the Ghon complex, which is a small subpleural granuloma at mid-lung along with prominent enlarged hilar lymph nodes. These nodes may impinge upon central airways. When the cell-mediated immune response is poor, then there can be numerous small granulomas scattered throughout the lungs, or disseminated to other organs, as a miliary pattern (granulomas that are the size of millet seeds).
- • Progressive pri¬mary tuberculosis can lead to more extensive lung involve¬ment with pneumonic infiltrates.
- • Upper lobe cavitary disease is characteristic for secondary tuberculosis (reactivation or reinfection) in persons who have previously mounted an im¬mune response.
- 3. Platelet aggregation is inhibited by all except:
- a. Aspirin
- b. Clopidogrel
- c. Thromboxane A2
- d. Eptifibatide
Ans: c
- Antiplatelet Drugs
- • Aspirin
- • Dipyridamole
- • Ticlopidine
- • Abciximab
- • Clopidogrel
- • Eptifibatide
- • Prasugrel
- • Tirofiban
- 4. Which drug is given in delayed vomiting after chemotherapy:
- a. Metoclopramide
- b. Hyoscine
- c. Domperidone
- d. Aprepitant
Ans: d
- Aprepitant
- • Selective, high affinity NK1 receptor antagonist that blocks the emetic action of substance P, with little effect on 5 HT3 and D2 or other receptors.
- • Gastrointestinal motility is not affected.
- • Oral aprepitant (125 mg + 80 mg + 80 mg over 3 days) combined with standard i.v. ondansetron + dexamethasone regimen significantly enhanced the antiemetic efficacy against high emetogenic cisplatin based chemotherapy.
- • Greater additional protection was afforded against delayed vomiting than against acute vomiting.
- • It was particularly useful in patients undergoing multiple cycles of chemotherapy.
- • Adjuvant benefit of aprepitant has also been demonstrated in cyclophosphamide based moderately emetogenic chemotherapy.
-
64
- The active form of Remdesivir, drug under clinical trial for treatment of COVID 19, is:
- a. GC-376
- b. GS-441524
- c. B-5701
- d. GS-5734
- Ans: b
- Ref: Agostini ML, Andres EL, Sims AC, Graham RL, Sheahan TP, Lu X, et al. (March 2018). "Coronavirus Susceptibility to the Antiviral Remdesivir (GS-5734) Is Mediated by the Viral Polymerase and the Proofreading Exoribonuclease
- • Remdesivir is a prodrug that metabolizes into its active form GS-441524.
- • An adenosine nucleotide analog, GS-441524 interferes with the action of viral RNA-dependent RNA polymerase and evades proofreading by viral exoribonuclease (ExoN), causing a decrease in viral RNA production.
- • Though in some viruses, such as the respiratory syncytial virus but not Ebola virus, it causes the RNA-dependent RNA polymerases to pause, its predominant effect is to induce an irreversible chain termination.
- • Unlike with many other chain terminators, this was not mediated by preventing addition of the immediately subsequent nucleotide, but is instead delayed, occurring after five additional bases have been added to growing RNA chain.
- • During 2020, several clinical trials were underway. The first placebo controlled trial of Remdesivir in China, according to a WHO report that was published early in error, showed that Remdesivir had no clinical or virological benefits, and the drug was stopped early in 11.6% of patients due to adverse effects. In the large industry sponsored study for Remdesivir, for severe COVID-19, the trial was modified, mid trial, increasing in size from 400 subjects to 2,400 participants. The primary end point of the trial was also changed during the trial.
-
65
- Deep Brain Stimulation done for Parkinson’s disease primarily targets
- a. Globus Pallidus internus
- b. Globus Pallidus externus
- c. Substantia Nigra compacta
- d. Substantia nigra reticulata
Ans: a
- • DBS for PD primarily targets the STN or the GPi. It provides dramatic results, particularly with respect to tremor and reducing both “off” time and dyskinesias, but does not provide superior clinical benefits or improve features that do not respond to levodopa such as freezing, falling, and dementia.
- • The procedure is thus primarily indicated for patients who suffer disability resulting from severe tremor, or levodopa-induced motor complications that cannot be satisfactorily controlled with drug manipulation. In such patients, DBS has been shown to provide benefits in comparison to best medical therapy.
- • Recent studies indicate that benefits following DBS of the STN and GPi are comparable, but that GPi stimulation may be associated with a reduced frequency of depression. Although not all PD patients are candidates, the procedure can be profoundly beneficial for many.
-
66
- Mantle cell lymphoma are positive for all except:
- a. CD 23
- b. CD 5
- c. CD 43
- d. CD 20
Ans: a
- • Almost all tumors have an (11;14) translocation that fuses the cyclin D1 gene to the IgH locus. This translocation leads to over expression of cyclin D1, which you will recall stimulates growth by promoting the progression of cells from the G1 phase to the S phase of the cell cycle.
- • The tumor cells express surface IgM and IgD, the B cell antigen CD20, and CD5, and they contain high levels of cyclin D1 protein.
-
67
- Achondroplasia is caused by point mutation of:
- a. TCIRG1
- b. Mutation in the genes that encode the α1 and α2 chains
- c. carbonic anhydrase 2
- d. FGFR3
Ans: d
- • Mutation in the genes that encode the α1 and α2 chains : Osteogenesis Imperfecta
- • TCIRG1 and carbonic anhydrase 2 mutation : Osteopetrosis
- • FGFR3 mutation : Achondroplasia and Thanatophoric Dysplasia
-
68
- Meniere’s disease is characterized by:
- a. Conductive hearing loss and tinnitus
- b. Vertigo ear discharge tinnitus and headache
- c. Vertigo, tinnitus hearing loss and headache
- d. Vertigo, tinnitus and hearing loss
Ans: c > d
- Cardinal symptoms of Ménière’s disease are
- (i) episodic vertigo
- (ii) fluctuating hearing loss
- (iii) tinnitus and
- (iv) sense of fullness or pressure in the involved ear.
- It is concluded that Ménière’s disease is associated with headache that can be handicapping, and tricyclic antidepressants with pain alleviating medication is often needed to treat the headache in MD whereas sumatriptan did not alleviate the headache of the non-migraine patients.
-
69
- • The corpus luteum continues to grow and forms the corpus luteum of pregnancy (corpus luteum graviditatis).
- • By the end of the third month, this structure may be one third to one half of the total size of the ovary.
- • Yellowish luteal cells continue to secrete progesterone until the end of the fourth month; thereafter, they regress slowly as secretion of progesterone by the trophoblastic component of the placenta becomes adequate for maintenance of pregnancy.
- • Removal of the corpus luteum of pregnancy before the fourth month usually leads to abortion.
-
70
- 1. Which of the following is important in patient-centered communication?
- a. Empathy
- b. Sympathy
- c. Kindness
- d. Attitude
- Ans: a
- Patient-centered communication is critical to good patient care. Patient-centered communication drives treatment planning through the transmission of information and provides a therapeutic and supportive environment for the patient. Empathy is of particular importance in effective patient-centered communication.
- A nurse’s ability to recognize patients’ empathic opportunities and respond to a patient empathically, communicating a desire to understand, can help patients understand and cope effectively with their illnesses.
- 2. Inferior thyroid vein drains into:
- a. External jugular vein
- b. Internal jugular vein
- c. Inferior venacava
- d. Brachiocephalic vein
Ans: d
- • The venous drainage of the thyroid gland is usually via superior, middle and inferior thyroid veins.The superior thyroid vein emerges from the upper part of the gland and runs with the superior thyroid artery towards the carotid sheath; it drains into the internal jugular vein.
- • The middle thyroid vein collects blood from the lower part of the gland; it emerges from the lateral surface of the gland and drains into the internal jugular vein. The inferior thyroid veins arise in a glandular venous plexus, which also connects with the middle and superior thyroid veins. These veins form a pretracheal plexus, from which the left inferior vein descends to join the left brachiocephalic vein, and the right descends obliquely across the brachiocephalic artery to join the right brachiocephalic vein at its junction with the superior vena cava.
- • The inferior thyroid veins often open via a common trunk into the superior vena cava or left brachiocephalic vein. They drain the oesophageal, tracheal and inferior laryngeal veins and have valves at their terminations.
- 3. Transient tachypnea of the newborn develops in:
- a. Term baby by spontaneous vaginal delivery
- b. Term baby by forceps
- c. Term baby by cesarean section
- d. Preterm with ventouse
Ans: c
- • Transient tachypnea is most common after term cesarean delivery.
- • It is characterized by the early onset of tachypnea, sometimes with retractions, or expiratory grunting and, occasionally, cyanosis that is relieved by minimal oxygen supplementation (<40%).
- • Most infants recover rapidly, usually within 3 days.
- • The chest generally sounds clear without crackles or wheeze, and the chest radiograph shows prominent pulmonary vascular markings, fluid in the intralobar fissures, overaeration, flat diaphragms, and, rarely, small pleural effusions.
- TTN (wet lung syndrome) is a clinical diagnosis. It is caused by a delayed resorption of intrauterine pulmonary liquids.
- Normally, pulmonary fluids are cleared by:
- • Bronchial squeezing during delivery, 30%
- • Absorption, 30%: lymphatics, capillaries
- • Suction, 30%
- 4. A 6-year-old boy is brought to the emergency department by his mother due to swelling around his eyes and legs. The mother reports that the patient recently recovered from an upper respiratory tract infection. Physical exam is significant for periorbital and lower extremity edema. Laboratory testing is significant for hypoalbuminemia and normal complement levels. Urinalysis demonstrates 4+ protein. Which of the following is false about the disease the children might have been suffering?
- a. Known as nil lesion
- b. Commonest cause of nephrotic syndrome in adults
- c. Acellular urinary sediment
- d. Selective proteinuria
Ans: b
Minimal change disease (MCD), sometimes known as nil lesion, causes 70–90% of nephrotic syndrome in childhood but only 10–15% of nephrotic syndrome in adults. Minimal change disease usually presents as a primary renal disease but can be associated with several other conditions, including Hodgkin’s disease, allergies, or use of nonsteroidal anti-inflammatory agents; significant interstitial nephritis often accompanies cases associated with nonsteroidal drug use.
-
71
- Coarse facies, hepatosplenomegaly and cloudy cornea are characteristic feature of:
- a. Morquio syndrome A
- b. Hurler’s disease
- c. Hunters disease
- d. Natowicz syndrome
Ans: B
- • Hurler disease: Mental retardation, coarse facial features, hepatosplenomegaly, cloudy cornea
- • Hunter disease: Mental retardation
- • Morquio syndrome A : Skeletal dysplasia, short stature
- • Natowicz syndrome: Joint pain, short stature
-
72
- A female neonate on 10th day presented with vaginal bleeding. What would be the next step of management?
- a. Estrogen
- b. Progesterone
- c. OCP
- d. Reassurance and do nothing
- Ans: d
- Ref: Dutta’s Textbook of Gynaecology including Contraception, 6th edition, page 543
- • Bleeding per vaginum (mostly blood stained to frank bleeding at times) usually occurs in newborns within 10 days following birth.
- • It occurs due to decline in level of estrogen, which is unable to support the endometrium, resulting in withdrawal bleeding.
- • There is no cause for concern and patient party can be simply reassured.
-
73
- Pilon fracture is:
- a. Avulsion fracture of lateral tibial condyle; associated with ACL injury
- b. Intraarticular fracture of tibial condyle
- c. Intraarticular comminuted distal tibia fracture
- d. Salter-Harris III of lateral distal tibia (because of later epiphyseal fusion)
- Ans: c
- Ref: Apley’s Orthopedics, 9th Edition; Page No: 916
- PILON FRACTURES
- Unlike the twisting injuries that cause the common ankle fractures, this injury to the ankle joint occurs when a large force drives the talus upwards against the tibial plafond, like a pestle (pilon) being struck into a mortar. There is considerable damage to the articular cartilage and the subchondral bone may be broken into several pieces; in severe cases, the comminution extends some way up the shaft of the tibia.
- • Segond: Avulsion fracture of lateral tibial condyle; associated with ACL injury
- • Bumper: Intraarticular fracture of tibial condyle
- • Pilon: Intraarticular comminuted distal tibia fracture
- • Tillaux: Salter-Harris III of lateral distal tibia (because of later epiphyseal fusion)
- • Triplane: Salter III/IV fracture of distal tibia
-
74
- 1. Which of the following is not correct about treatment of sunburn?
- a. Calamine lotion
- b. Topical steroid
- c. Allylamines
- d. NSAIDs
- Ans, C, Allylamines
- Ref, Illustrated Synopsis of Dermatology and Sexually Transmitted Diseases, Neena Khanna, 4th edition, page no.192, 193
- Allylamines are antifungals, eg, Terbinafine.
- Sunburn
- Cause:
- Action spectrum: UVB which induces release of cytokines in skin, resulting in pain, redness, erythema edema and even blistering.
- Skin type: Most frequent and intense in individuals who are skin type I and II.
- Clinical features:
- • Seen in light skinned.
- • Areas overexposed to UVR become painful and deeply erythematous after several hours.
- • Redness peaks at 24 h and subsides over next 48–72 h, followed by sheet-like peeling of skin and then hyperpigmentation
- Treatment
- Prevention
- • Avoiding overexposure to sun (e.g., sunbathing), especially by light-skinned individuals.
- • Using protective clothing and sun shades.
- • Using UVB protective sunscreens.
- Symptomatic treatment
- • Calamine lotion provides comfort.
- • Topical steroids help, if used early.
- Nonsteroidal anti-inflammatory drugs like aspirin not only relieve pain but also the inflammation.
- 2. A 45 years old male comes with complains of very salty-tasting skin, persistent coughing, at times with phlegm, frequent past lung infections including pneumonia and bronchitis, frequent greasy, bulky stools and difficulty with bowel movements and infertility. This is typical of cystic fibrosis. What is the position of amino acid in the protein determined by CFTR gene mutated that caused this condition in the patient?
- a. 308
- b. 408
- c. 508
- d. 608
- Ans: C, 508
- Ref, Ganong’s Review of Medical Physiology, 26th edition, page no. 612
- Cystic Fibrosis
- • Among whites, cystic fibrosis is one of the most common genetic disorders; greater than 3% of the United States popula¬tion are carriers for this autosomal recessive disease.
- • The gene that is abnormal in cystic fibrosis is located on the long arm of chromosome 7 and encodes the cystic fibro¬sis transmembrane conductance regulator (CFTR), a regu¬lated Cl– channel located on the apical membrane of various secretory and absorptive epithelia.
- • The number of reported mutations in the CFTR gene that cause cystic fibrosis is large (> 1000) and the mutations are now grouped into five classes (I–V) based on their effects on cellular function.
- 1. Class I muta¬tions do not allow for synthesis of the protein.
- 2. Class II muta¬tions have protein processing defects.
- 3. Class III mutations have a block in their channel regulation.
- 4. Class IV mutations display altered conductance of the ion channel.
- 5. Class V mutations display reduced synthesis of the protein.
- The most common mutation causing cystic fibrosis is loss of the phenylalanine residue at amino acid position 508 of the pro¬tein (ΔF508), a Class II mutation that limits the amount of CFTR protein that gets to the plasma membrane.
- 3. You come across with a patient who says he wants to wear pink t-shirts, tight pants and female inner garments. He feels sexually aroused on doing so. What type of paraphilia does this patient seems to show?
- a. Exhibitionism
- b. Fetishistic transvestism
- c. Voyeurism
- d. Frotteurism
- Ans: B, Fetishistic transvestism
- Ref, A Short Textbook of Psychiatry, Niraj Ahuja, 7th edition, page no. 124-126
- Paraphilias (sexual deviations; perversions) are disorders of sexual preference in which sexual arousal occurs persistently and significantly in response to objects which are not a part of normal sexual arousal (e.g. nonhuman objects; suffering or humiliation of self and/or sexual partner; children or nonconsenting person). They are as follows:
- 1. Fetishism: In fetishism, the sexual arousal occurs either solely or predominantly with a nonliving object, which is usually intimately associated with the human body.
- 2. Fetishistic Transvestism: This disorder occurs exclusively in heterosexual males. The person actually or in fantasy wears clothes of the opposite sex (cross-dressing) for sexual arousal.
- 3. Sexual Sadism: In this disorder, the person (the ‘sadist’) is sexually aroused by physical and/or psycho logical humiliation, suffering or injury of the sexual partner (the ‘victim’).
- 4. Sexual Masochism: This is just the reverse of sexual sadism. Here the person (the ‘masochist’) is sexually aroused by physical and/or psychological humiliation, suffering or injury inflicted on self by others (usually ‘sadists’).
- 5. Exhibitionism: Exhibitionism is a persistent (or recurrent) and significant method of sexual arousal by the exposure of one’s genitalia to an unsuspecting stranger.
- 6. Voyeurism: This is a persistent or recurrent tendency to observe unsuspecting persons (usually of the other sex) naked, disrobing or engaged in sexual activity.
- 7. Frotteurism: This is a persistent or recurrent involvement in the act of touching and rubbing against an unsuspecting, nonconsenting person (usually of the other sex).
- 8. Paedophilia: Paedophilia is a persistent or recurrent involvement of an adult (age >16 years and at least 5 years older than the child) in sexual activity with prepubertal children, either heterosexual or homosexual.
- 9. Zoophilia (Bestiality): Zoophilia as a persistent and signifi cant involvement in sexual activity with animals is rare.
- Other Paraphilias: These include sexual arousal with urine (urophilia); faeces (coprophilia); enemas (klismaphilia); corpses (necrophilia), among many others.
-
75
- 1. The arteries supplying the pelvic part of ureter in a female includes all except
- a. Renal artery
- b. Vesical artery
- c. Middle rectal artery
- d. Uterine artery
Ans, A
- The ureter is supplied by three sets of long arteries:
- 1. The upper part receives branches from the renal artery. It may also receive branches from the gonadal, or colic vessels.
- 2. The middle part receives branches from the aorta. It may also receive branches from the gonadal, or iliac vessels.
- 3. The pelvic part is supplied by branches from the vesical, middle rectal, or uterine vessels
- 2. The resistance to isoniazid is associated with
- a. inhA gene
- b. katG gene
- c. ahpC gene
- d. All of the above
Ans, D
Resistance to isoniazid is associated with mutations resulting in overexpression of inhA, which encodes an NADH-dependent acyl carrier protein reductase; mutation or deletion of the katG gene; promoter mutations resulting in overexpression of ahpC, a gene involved in protection of the cell from oxidative stress; and mutations in kasA.
- 3. Which of the following is the protein complex II of the respiratory chain?
- a. NADH-Q oxidoreductase
- b. Cytochrome c oxidase
- c. Q-cytochrome c oxidoreductase
- d. Succinate-Q reductase
Ans, D
Components of the Respiratory Chain Are Contained in Four Large Protein Complexes Embedded in the Inner Mitochondrial Membrane
Electrons flow through the respiratory chain through a redox span of 1.1 V from NAD+/NADH to O2/2H2O, passing through three large protein complexes:
- ❖ NADH-Q oxidoreductase (Complex I), where electrons are transferred from NADH to coenzyme Q (Q) (also called ubiquinone)
- ❖ Q-cytochrome c oxidoreductase (Complex III), which passes the electrons on to cytochrome c; and
❖ Cytochrome c oxidase (Complex IV), which completes the chain, passing the electrons to O2 and causing it to be reduced to H2O.
- ❖ Some substrates with more positive redox potentials than NAD+/NADH (eg, succinate) pass electrons to Q via a fourth complex, succinate-Q reductase (Complex II), rather than Complex I.
- ❖ The four complexes are embedded in the inner mitochondrial membrane, but Q and cytochrome c are mobile.
- ❖ Q diffuses rapidly within the membrane, while cytochrome c is a soluble protein.
-
76
- Q.1.Detection of pulsations of the vaginal and uterine arteries in the vaginal fornices in early pregnancy is:
- A. Jacquemier’s sign
- B. Goodell’s sign
- C. Osiander’s sign
- D. Piskacek’s sign
Ans C ; Osiander’s sign
• Jacquemier’s or Chadwick’s sign : It is the dusky hue of the vestibule and anterior vaginal wall visible at about 8th week of pregnancy. The discoloration is due to local vascular congestion.
- • Vaginal sign:
- ☆ Apart from the bluish discoloration of the anterior vaginal wall
- ☆ The walls become softened and Copious non-irritating mucoid discharge appears at 6th week
- ☆ There is increased pulsation, felt through the lateral fornices at 8th week called Osiander’s sign.
- • Cervical signs:
- ☆ Cervix becomes soft as early as 6th week (Goodell’s sign), a little earlier in multiparae. The pregnant cervix feels like the lips of the mouth, while in the non-pregnant state, like that of tip of the nose.
- ☆ On speculum examination, the bluish discoloration of the cervix is visible. It is due to increased vascularity.
- • Uterine signs:
- ☆ Size, shape and consistency— The uterus is enlarged to the size of hen’s egg at 6th week, size of a cricket ball at 8th week and size of a fetal head by 12th week.
- ☆ The pyriform shape of the non-pregnant uterus becomes globular by 12 weeks.
- ☆ There may be asymmetrical enlargement of the uterus if there is lateral implantation. This is called Piskacek’s sign where one half is more firm than the other half.
- ☆ As pregnancy advances, symmetry is restored. The pregnant uterus feels soft and elastic.
- Q.2. Mapleson A circuit is?
- a. Ayre tube
- b. Jackson-Reese circuit
- c. McGill circuit
- d. Bain circuit
- Ans: C, McGill circuit
- • Mapleson A, also called McGill circuit, is the circuit of choice for spontaneous ventilation.
- • Mapleson B: is not common in clinical use.
- • Mapleson C: is the circuit of choice in post-operative recovery room.
- • Mapleson D: (Bain circuit) is most commonly used for controlled ventilation.
- • Mapleson E: (Ayre tube) is used for infants and young children.
- • Mapleson F: (Jackson & Reese circuit) is used for children <6 years of age and <20 kg weight
- Q.3 Adipocere formation starts under the influence of:
- a. Intrinsic lipase
- b. Lipase from bacteria
- c. Lecithinase
- d. Dehydrogenase
- Ans, A, Intrinsic lipase
- ADIPOCERE(SAPONIFICATION)
- Adipocere (cire=wax)
- • Is a modification of putrefaction.
- • In this, the fatty tissues of the body change into a substance known as adipocere.
- • It is seen most commonly in bodies immersed in water or in damp, warm environment.
- Mechanism:
- • The change is due to the gradual hydrolysis and hydrogenation of pre-existing fats, such as olein, into higher fatty acids, which combine with calcium and ammonium ions to form insoluble soaps, which being acidic, inhibit putrefactive bacteria.
- • Ultimately, the whole of the fat is converted into palmitic, oleic, stearic and hydroxystearic acid, together with some glycerol, and a mixture of these substances forms adipocere.
- • These form a matrix for remnants of tissue fibres, nerves and muscles.
- • At the time of death, body fat contains, about half percent of fatty acids, but in adipocere they rise to 20% within a month and over 70% in three months.
- • The process starts under the influence of intrinsic lipases, and is continued by the bacterial enzymes of the clostridia group, mainly Cl. perfringens, as the bacteria produce lecithinase, which facilitates hydrolysis and hydrogenation.
- • Water is essential for the bactetial and enzymatic processes involved in adipocere formation.
-
77
- 1. Ankle sprain due to forced inversion of a plantar flexed foot is due to injury to:
- a. Posterior talofibular ligament
- b. Calcaneofibular ligament
- c. Posterior fibres of deltoid
- d. Anterior talofibular ligament
- Ans. d. Anterior talofibular ligament
- Ref: Apley 8/e p733-736
- Over 90% of ankle ligament injuries (twisted ankle or ankle sprain) involve the lateral ligament complex, usually the anterior tibiofibular ligament.
- A. Medial Collateral Ligament:
- I. Also known as deltoid ligament.
- II. It is a strong ligament and major stabilizer of the ankle joint
- III. Two components: Superficial: Resist eversion of hind foot (Talo-tibial, naviculo- tibial, calcaneo-tibial) and Deep (Intra-articular): Prevents external rotation of talus (Anterior and posterior talotibial)
- B. Lateral Collateral Ligament:
- I. It is a weak ligament, so involved in over 90% of ankle injuries.
- II. It has three parts:
- i. Anterior talofibular: Most commonly injued.
- ii. Middle calcaneofibular: 2nd most commonly injured.
- iii. Posterior talofibular: Torn in most severe injuries.
- 2. A 25-year-old man presents to his primary care physician with crampy abdominal pain for 2 days. He reports having loose stools and losing 15 lbs (6.8 kg) over a 3-month duration. He also reports increased fatigue. Further evaluation revealed Crohn’s disease. The earliest manifestation in Crohn’s disease is :
- a. Aphthous Ulcer
- b. Anal fistula
- c. Rose thorn appearance on Barium swallow
- d. Skip lesions
- Ans, a. Aphthous ulcer
- Ref: Harrison 19th edition/pg. 1952
- i. The earliest lesions are Aphthous ulcerations & focal crypt abscesses with loose aggregations of macrophages, which form non-caseating granulomas in all layers of the bowel wall.
- ii. In early mild Crohn’s disease, the colonic mucosa appears endoscopically normal or small.
- iii. Punched-out aphthous ulcers are often seen.
- iiii. Aphthous ulcers occur as a result of submucosal lymphoid follicle expansion.
- v. In moderate Crohn's Disease, aphthous ulcers coalesce into larger ulcers, which may take on the appearance of a star (stellate ulcers).
- vi. As Crohn's disease severity increases, submucosal edema and injury can result in cobblestoning of the mucosa, which is seen more often in Crohn’s disease than in ulcerative colitis.
- 3. Resistant hypertension is defined as:
- a. Resistance to aldosterone
- b. Resistance to Angiotension II receptors
- c. Resistance to 2 or more anti hypertensive drugs including thiazides
- d. Resistance to 3 or more anti-hypertensive drugs including thiazides
- Ans,d.Resistance to 3 or more anti-hypertensive drugs including thiazides
- Ref: Harrison 18th edition, chapter 247
- I. Resistant hypertension refers to patients with blood pressures persistently >140/90 mmHg despite taking three or more antihypertensive agents, including a diuretic, in a reasonable combination and at full doses.
- II. Resistant or difficult-to-control hypertension is more common in patients >60 years than in younger patients.
- III. Resistant hypertension may be related to:
- i. “Pseudoresistance” (high office blood pressures and lower home blood pressures), nonadherence to therapy.
- ii. Identifiable causes of hypertension (including obesity and excessive alcohol intake.
- iii. Use of any of a number of nonprescription and prescription drugs.
-
78
- Q1.Aqueous humour is produced by:
- a.Iris
- b.Lens
- c.Ciliary body
- d.Choroid
- Ans:c,Ciliary body
- Ref:AK Khurana 6th e/p 148
- • Ciliary body is forward continuation of the choroid at ora serrata. In cut-section, it is triangular in shape. The anterior side of the triangle forms the part of the angle of anterior and posterior chambers. In its middle the iris is attached.
- • The outer side of the triangle lies against the sclera with a suprachoroidal space in between. The inner side of the triangle is divided into two parts. The anterior part (about 2 mm) having finger-like ciliary processes is called pars plicata and the posterior smooth part (about 4 mm) is called pars plana.
- • Functions of ciliary body
- • Formation of aqueous humour.
- • Ciliary muscles help in accommodation.
- Q2.A 43-year-old man with diabetes has a 4-cm nonhealing foot ulcer. Culture of the ulcer yields Staphylococcus aureus, Bacteroides fragilis, and a gram-negative bacillus that swarms across the blood agar plate covering the entire surface of the agar after 36 hours. The gram-negative bacillus is a member of the genus:
- a.Escherichia
- b. Enterobacter
- c.Serratia
- d. Proteus
- Ans:d,Proteus
- Ref: Jawetz Melnick and Adelberg’s medical microbiology 27th/e p 233
- • Proteus–Morganella–Providencia group—The members of this group deaminate phenylalanine, are motile, grow on potassium cyanide medium (KCN), and ferment xylose.
- • Proteus species move very actively by means of peritrichous flagella, resulting in “swarming” on solid media unless the swarming is inhibited by chemicals, such as phenylethyl alcohol or CLED (cystine-lactose-electrolytedeficient) medium.
- • Whereas Proteus species and Morganella morganii are urease positive, Providencia species usually are urease negative. The Proteus–Providencia group ferments lactose very slowly or not at all.
- Q3. A 3-month-old child presents with intermittent stridor aggravated on keeping the child supine. Most likely cause is:
- a. Laryngotracheobronchitis
- b. Laryngomalacia
- c. Respiratory obstruction
- d. Foreign body aspiration
- Ans:b,Laryngomalacia
- Ref: Ghai 8th ed pg 369
- • Laryngomalacia is the most common congenital laryngeal anomaly, accounting for up to 60% and the most common cause of infant stridor.
- • Inspiratory stridor is the hallmark of the condition.
- • Symptoms are typically aggravated when the child is supine or crying.
- • Flexible endoscopy reveals partial collapse of a flaccid supraglottic airway with inspiration.
- • If present, gastroesophageal reflux, should be managed.
- • Laryngomalacia is generally benign and selflimited, as most cases resolve by 18 months of age.
- • Surgical intervention is advised for either respiratory distress or failure to thrive
-
79
- 1. Radiological appearance of thimble' bladder is seen in the following condition:
- a. Cystitis cystica
- b. Chronic tuberculous cystitis
- c. Neurogenic bladder
- d. Acute tuberculous cystitis
Ans: B
- Explanation
- Early tuberculosis of the bladder commences around the ureteric orifice or trigone, the earliest evidence being pallor of the mucosa due to submucosal oedema. Subsequently, tubercles may be seen and, in long standing cases, there is marked fibrosis and the capacity of the bladder is greatly reduced giving the radiological appearance of thimble bladder'.
- 2. Leucopenia is NOT seen in?
- a. New born
- b. Starvation
- C. Enteric Fever
- d. Viral or protozoal infection
- Ans : D
- Ref: Robbins 10th Edition p582-83
- Leukocytosis is seen with Viral infections on account of increase in number of natural killer cells
- Leucopenia: MI, Sepsis, CLL, Lithium intoxication, Down's syndrome, polyarteritis nodosa, trauma, Cushing's disease
- 3. Limiting amino acids in Cereals & Pulses respectively:
- a. Lysine & Threonine
- b.Lysine & Cysteine
- c.Glycine & Methionine
- d.Lysine & Methionine
- Ans: D, Lysine & Methionine
- Ref: Park PSM 23rd Edition, Page No: 610
- Cereal proteins are deficient in lysine and threonine, and pulse proteins in methionine. These are known as "limiting" amino acids. When two or more of vegetarian foods are eaten together (as for example, rice-dhal combination in India) their proteins supplement one another and provide a protein comparable to animal protein in respect of EAA.
-
80
- Most powerful sensitizer of helper T- cell is:
- a. Interleukins
- b. Mature dendritic cells
- c. Macrophages
- d. Interferons
- Ans: b
- Ref: Robbins Basic Pathology 10th Edition, Page No: 128
- Dendritic cells act as antigen-presenting cells: they activate helper T-cells and killer T-cells as well as B-cells by presenting them with antigens derived from the pathogen, alongside non-antigen specific costimulatory signals.
- Dendritic cells can also induce T-cell tolerance (unresponsiveness). Certain C-type lectin receptors (CLRs) on the surface of dendritic cells, some functioning as PRRs, help instruct dendritic cells as to when it is appropriate to induce immune tolerance rather than lymphocyte activation.
- Dendritic cells (DCs) are the most important antigen presenting cells for initiating T-cell responses against protein antigens.
- These cells have numerous fine cytoplasmic processes that resemble dendrites, from which they derive their name. Several features of DCs account for their key role in antigen capture and presentation.
-
81
- Mechanism of action of Stiripentol, though not a new molecule, was approved in Europe in 2007 for a very specific type of epilepsy, is:
- a. Enhance GABAergic transmission in the brain, partly through a barbiturate-like effect
- b. Prolongs inactivation of VG Na+ channels
- c. Enhances K+ channel opening
- d. Action on synaptic protein SV2A
- Ans: a
- Ref: Katzung Pharmacology 12th Edition, Page No: 415
- Stiripentol
- • The drug is used with clobazam and valproate in the adjunctive therapy of refractory generalized tonic-clonic seizures in patients with severe myoclonic epilepsy of infancy (SMEI, Dravet’s syndrome) whose seizures are not adequately controlled with clobazam and valproate.
- • The mechanism of action of stiripentol is not well understood but it has been shown to enhance GABAergic transmission in the brain, partly through a barbiturate-like effect, ie, prolonged opening of the Cl – channels in GABA A receptors.
- • It also increases GABA levels in the brain. It can increase the effect of other AEDs by slowing their inactivation by cytochrome P450.
- Drugs and Mechanism of Action:
- • Lamotrigine: Prolongs inactivation of VG Na+ channels; acts presynaptically on VG Ca2+ channels, decreasing glutamate release
- • Rufinamide: Prolongs inactivation of VG Na+ channels
- • Tiagabine: Blocks GABA reuptake in forebrain by selective blockade of GAT-1
- • Retigabine: Enhances K+ channel opening
- • Topiramate: Multiple actions on synaptic function, probably via actions on Phosphorylation
- • Levetiracetam: Action on synaptic protein SV2A
-
82
- α1 = α2; β1 >> β2 is seen with:
- a. Epinephrine
- b. Norepinephrine
- c. Phenylephrine
- d. Isoproterenol
- Ans: b
- Ref: Katzung Pharmacology 12th Edition, Page No: 133
- Alpha agonists
- • Phenylephrine, methoxamine α1 > α2 >>>>> β
- • Clonidine, methylnorepinephrine α2 > α1 >>>>> β
- Mixed alpha and beta agonists
- • Norepinephrine α1 = α2; β1 >> β2
- • Epinephrine α1 = α2; β1 = β2
- Beta agonists
- • Dobutamine1 β1 > β2 >>>> α
- • Isoproterenol β1 = β2 >>>> α
- • Albuterol, terbutaline, metaproterenol, ritodrine β2 >> β1 >>>> α
- Dopamine agonists
- • Dopamine D1 = D2 >> β >> α
- • Fenoldopam D1 >> D2
-
83
- A 66-year-old man with chronic cough has an episode of hemoptysis. On physical examination, there are no abnormal findings. A chest radiograph shows a 6-cm mass in the right lung. A sputum cytologic analysis shows neoplastic squamous cells. Metastases from his lung lesion are most likely to be found at which of the following sites?
- a. Cerebral hemisphere
- b. Chest wall muscle
- c. Hilar lymph nodes
- d. Splenic red pulp
- Ans: c
- Ref: Robbins Basic Pathology 9th Edition, Page No: 168–169
- Carcinomas metastasize through lymphatics most of¬ten, usually to regional nodes first. Hematogenous metastases are possible, however, to sites such as bone marrow, liver, or the opposite lung. About half of all cerebral metastases arise from lung primary carcinomas. Soft-tissue metastases to muscle, fat, and connective tissues are rare, as are splenic metastases.
-
84
- Atropine is most sensitive to:
- a. Mucous and pharyngeal secretions
- b. Heart
- c. Pupil
- d. GI tract motility
- Ans: a
- Ref: KDT Pharmacology, 7th Edition; Page No: 115
- The sensitivity of different organs and tissues to atropine varies and can be graded as:
- Saliva, sweat, bronchial secretion > eye, bronchial muscle, heart > smooth muscle of intestine, bladder > gastric glands and smooth muscle.
- • Atropine markedly decreases sweat, salivary, tracheobronchial and lacrimal secretion (M3 blockade). Skin and eyes become dry, talking and swallowing may be difficult.
- • Atropine decreases secretion of acid, pepsin and mucus in the stomach, but the primary action is on volume of secretion so that pH of gastric contents may not be elevated unless diluted by food.
- • Since bicarbonate secretion is also reduced, rise in pH of fasting gastric juice is only modest.
-
85
- Thiopentone is used for induction of anaesthesia. It shows marked redistribution which is a characteristic of:
- a. Highly lipid soluble drugs
- b. Highly water soluble drugs
- c. Weak electrolytes
- d. Highly plasma protein bound drugs
- Ans: a
- Ref: KDT Pharmacology, 7th Edition; Page No: 18
- Redistribution
- • Highly lipid-soluble drugs get initially distributed to organs with high blood flow, i.e. brain, heart, kidney, etc. Later, less vascular but more bulky tissues (muscle, fat) take up the drug—plasma concentration falls and the drug is withdrawn from the highly perfused sites.
- • If the site of action of the drug was in one of the highly perfused organs, redistribution results in termination of drug action.
- • Greater the lipid solubility of the drug, faster is its redistribution.
- • Anaesthetic action of thiopentone sod. Injected i.v. is terminated in few minutes due to redistribution.
-
86
- 1. 9th, 10th and 11th cranial nerves form nucleus ambiguus which is classified as
- a. Special visceral efferent
- b. General somatic efferent
- c. General visceral efferent
- d. General somatic afferent
- Ans: a
- Ref: Inderbir Singh embryology 12th edition; Pg 297-299
- Afferent columns in alar lamina: The four functional columns in alar lamina are:
- • General visceral afferent (GVA): concerned with visceral sensations conveyed through 9th and 10th cranial nerves.
- • Special visceral afferent (SVA): concerned with taste sensation conveyed through 7th, 9th and 10th cranial nerves from the tongue, pharynx and soft palate.
- • General somatic afferent (GSA): contains mesencephalic, chief and spinal nuclei of trigeminal nerve concerned with proprioception, touch, and pain and temperature sensations respectively from the head and neck region.
- • Special somatic afferent (SSA): contains vestibular and cochlear nuclei concerned with equilibrium and hearing.
- Efferent columns of basal lamina: The three functional columns in basal lamina are:
- • General somatic efferent (GSE): contains nuclei of cranial nerves (3rd, 4th, 6th and 12th) that supply the extraocular muscles of eyeball and musculature of tongue.
- • Special visceral efferent/Branchial efferent (SVE): contains the nuclei of cranial nerves (5th, 7th, 9th, 10th and 11th) that supply to musculature derived from pharyngeal arches.
- • General visceral efferent (GVE): contains the cranial nerve nuclei of origin for preganglionic neurons of peripheral parasympathetic ganglia.
- • Edinger-Westphal nucleus (3rd cranial nerve): Ciliary ganglion
- • Superior salivatory nucleus (7th cranial nerve): Pterygopalatine and submandibular ganglia
- • Inferior salivatory nucleus (9th cranial nerve): Otic ganglion
- • Dorsal nucleus of vagus (10th cranial nerve): for supply of thoracic and abdominal viscera.
- 2. In triage the second priority is given to which of the color coding
- a. Black
- b. Blue
- c. Yellow
- d. Green
- Ans: C
- Ref: Park PSM 23rd Edition, Page No: 796-797
- Triage
- When the quantity and severity of injuries overwhelm the operative capacity of health facilities, a different approach to medical treatment must be adopted. The principle of "first come, first treated", is not followed in mass emergencies. Triage consists of rapidly classifying the injured on the basis of the severity of their injuries and the likelihood of their survival with prompt medical intervention. It must be adopted to locally available skills. Higher priority is granted to victims whose immediate or long-term prognosis can be dramatically affected by simple intensive care. Moribund patients who require a great deal of attention, with questionable benefit, have the lowest priority. Triage is the only approach that can provide maximum benefit to the greatest number of injured in a major disaster situation.
- Although different triage systems have been adopted and are still in use in some countries, the most common classification uses the internationally accepted four colour code system.
- • Red indicates high priority treatment or transfer,
- • Yellow signals medium priority,
- • Green indicates ambulatory patients and
- • Black for dead or moribund patients.
Triage should be carried out at the site of disaster, in order to determine transportation priority, and admission to the hospital or treatment centre, where the patient's needs and priority of medical care will be reassessed. Ideally, local health workers should be taught the principles of triage as part of disaster training. Persons with minor or moderate injuries should be treated at their own homes to avoid social dislocation and the added drain on resources of transporting them to central facilities. The seriously injured should be transported to hospitals with specialized treatment facilities.
- 3. Infliximab is:
- a. IgG1 chimeric monoclonal antibody against TNF alpha
- b. IgG1 fully human monoclonal antibody against TNF alpha
- c. IgG4 chimeric monoclonal antibody against TNF alpha
- d. P75 TNF receptor fusion protein
- Ans: a
- Ref: Katzung Pharmacology 12th Edition, Page No: 993
- Infliximab
- Human-mouse chimeric IgG 1 monoclonal antibody possessing human constant (Fc) regions and murine variable regions.
- It is administered intravenously but has the same anti- TNF-α activity as adalimumab and etanercept. Infliximab is currently approved for use in Crohn’s disease, ulcerative colitis, rheumatoid arthritis, ankylosing spondylitis, plaque psoriasis, and psoriatic arthritis.
- 4. Rugger Jersey spine is seen in
- a. Fluorosis
- b. Achondroplasia
- c. Renal osteodystrophy
- d. Marfan’s syndrome
- Ans: c
- Rugger jersey spine describes the prominent endplate densities at multiple contiguous vertebral levels to produce an alternating sclerotic-lucent-sclerotic appearance.
- This mimics the horizontal stripes of a rugby jersey. This term and pattern are distinctive for hyperparathyroidism.
- Seen in osteopetrosis and renal osteodystrophy. In patient with renal osteodystrophy, this appearance is due to hyperparathyroidism and osteosclerosis.
- 5. Ormond’s disease is:
- a. Idiopathic lymphadenopathy
- b. Retractile testis
- c. Idiopathic retroperitoneal fibrosis
- d. Idiopathic mediastinitis
- Ans: c
- Ref: Bailey and Love 27th Edition, Page No: 1064-1065
- Retroperitoneal fibrosis
- • This is a relatively rare diagnosis characterized by the development of a flat grey/white plaque of tissue, which is found first in the low lumbar region but then spreads laterally and upwards to encase the common iliac vessels, ureters and aorta. Histological appearances vary from active inflammation with a high cellular content interspersed with bundles of collagen through to one of acellularity and mature fibrosis/calcification.
- • Its etiology is obscure in most cases (idiopathic-) being allied to other fibromatoses (others being Dupuytren’s contracture and Peyronie’s disease). In other patients the cause is known.
- • The clinical presentation may be one of ill-defined chronic backache or occur as a result of compromise to involved structures, e.g. lower limb or scrotal edema secondary to venous occlusion, or chronic renal failure secondary to ureteric obstruction. Treatment will be directed to the cause, the modification of disease activity when appropriate, e.g. immunomodulation with steroids, tamoxifen and restoration of flow in affected structures, e.g. ureteric stenting.
Causes of retroperitoneal fibrosis: Idiopathic (Ormond’s disease)
-
87
- Q13. The recommended treatment for Q-fever endocarditis is:
- a. Emergent surgery; antibiotics are not effective
- b. Levofloxacin monotherapy for 6 weeks
- c. 18 months of combination therapy with doxycycline and hydroxychloroquine
- d. Penicillin and gentamicin combination therapy using IgG titers to determine duration
- Ans: c, 18 months of combination therapy with doxycycline and hydroxychloroquine.
- Ref: Jawetz Melnick and Adelberg’s medical microbiology 27th/e p 347
- • C. burnetii is a small obligate organism that has a membrane similar to gram-negative bacteria, does not stain with Gram stain, multiplies within vacuoles, and causes the disease Q fever.
- • C burnetii exists in two antigenic forms called phase I and phase II. Phase I is the virulent form that is found in humans with Q fever and infected vertebrate animals, and it is the infectious form. Phase II is the avirulent form.
- • C burnetii is found in sheep, goats, cattle, and a variety of other animals, which are usually asymptomatic. Transmission to humans is by inhalation of contaminated dirt from animal feces, products of conception, or dust from animal products such as contaminated hides.
- • Q fever is characterized by acute and chronic infections. Acute pneumonia and hepatitis are associated with antibodies to phase II antigens; endocarditis is the most common form of chronic infection and is associated with antibodies to phase I antigens.
- • Diagnosis is clinically suspected and confirmed largely by serology or PCR performed in reference laboratories that have developed and verified their own assays.
- • Doxycycline is the drug of choice for the treatment of acute Q fever. The newer macrolides have also been shown to be effective in the treatment of acute pneumonia. Chronic Q fever requires prolonged treatment for 18 months or longer with a combination of doxycycline and hydroxychloroquine. Duration of treatment is long as mentioned earlier and should be determined by decrease in phase I antibody titers. In endocarditis, combination therapy is necessary to prevent relapse; occasionally, valve replacement is required and can prolong survival.
- Q14. Photodisruption is the basic mechanismof action of:
- a. Argon LASER
- b. Nd:YAG LASER
- c. Excimer LASER
- d. All of the above
Ans: b, Nd:YAG LASER
- Ref:AK Khurana 6th e/p 462
- MECHANISMS OF LASER EFFECT
- • Photocoagulation: Argon, diode, krypton and diode pump frequency doubled Nd:YAG lasers are based on this mechanism.
- • Photovaporization: Vaporization of tissue to CO2 and water occurs when temperature rises to 60–100 degree Centigrade or greater, e.g., CO2 absorbed by water of cells and heat generated causes cauterization or causes disintegration thereby incising the tissue.
- • Photodisruption: Nd:YAG laser and femtosecond laser are based on this mechanism.
- • Photoablation: Lasers based on this mechanism produce UV light of very short wavelength which breaks chemical bonds of biologic materials, converting them into small molecules that diffuse away. These lasers are collectively called excimer (excited dimer) lasers. These act by tissue modelling.
- • Photoradiation (Photochemical effect): Photochemical effect following visible/infrared particularly after administration of exogenous chromophore, e.g., hematoporphyrin or benzaporphyrin and is used for photodynamic therapy, treatment of ocular tumours and choroidal neovascularization.
- Q15. Dobutamine acts primarily on:
- a. D1 receptors
- b. D2 receptors
- c. Beta 1 receptors
- d. Beta 2 receptors
Ans:c, Beta 1 receptors
Ref:Ghai 8th e/p 718
-
88
- 1. Which of the following is not correct about treatment of sunburn?
- a. Calamine lotion
- b. Topical steroid
- c. Allylamines
- d. NSAIDs
Ans, C, Allylamines
Allylamines are antifungals, eg, Terbinafine.
- Sunburn
- Cause:
- Action spectrum: UVB which induces release of cytokines in skin, resulting in pain, redness, erythema edema and even blistering.
- Skin type: Most frequent and intense in individuals who are skin type I and II.
- Clinical features:
- • Seen in light skinned.
- • Areas overexposed to UVR become painful and deeply erythematous after several hours.
- • Redness peaks at 24 h and subsides over next 48–72 h, followed by sheet-like peeling of skin and then hyperpigmentation
- Treatment
- Prevention
- • Avoiding overexposure to sun (e.g., sunbathing), especially by light-skinned individuals.
- • Using protective clothing and sun shades.
- • Using UVB protective sunscreens.
- Symptomatic treatment
- • Calamine lotion provides comfort.
- • Topical steroids help, if used early.
- Nonsteroidal anti-inflammatory drugs like aspirin not only relieve pain but also the inflammation.
- 2. A 45 years old male comes with complains of very salty-tasting skin, persistent coughing, at times with phlegm, frequent past lung infections including pneumonia and bronchitis, frequent greasy, bulky stools and difficulty with bowel movements and infertility. This is typical of cystic fibrosis. What is the position of amino acid in the protein determined by CFTR gene mutated that caused this condition in the patient?
- a. 308
- b. 408
- c. 508
- d. 608
Ans: C, 508
- Cystic Fibrosis
- • Among whites, cystic fibrosis is one of the most common genetic disorders; greater than 3% of the United States popula¬tion are carriers for this autosomal recessive disease.
- • The gene that is abnormal in cystic fibrosis is located on the long arm of chromosome 7 and encodes the cystic fibro¬sis transmembrane conductance regulator (CFTR), a regu¬lated Cl– channel located on the apical membrane of various secretory and absorptive epithelia.
- • The number of reported mutations in the CFTR gene that cause cystic fibrosis is large (> 1000) and the mutations are now grouped into five classes (I–V) based on their effects on cellular function.
- 1. Class I muta¬tions do not allow for synthesis of the protein.
- 2. Class II muta¬tions have protein processing defects.
- 3. Class III mutations have a block in their channel regulation.
- 4. Class IV mutations display altered conductance of the ion channel.
- 5. Class V mutations display reduced synthesis of the protein.
- The most common mutation causing cystic fibrosis is loss of the phenylalanine residue at amino acid position 508 of the pro¬tein (ΔF508), a Class II mutation that limits the amount of CFTR protein that gets to the plasma membrane.
- 3. You come across with a patient who says he wants to wear pink t-shirts, tight pants and female inner garments. He feels sexually aroused on doing so. What type of paraphilia does this patient seems to show?
- a. Exhibitionism
- b. Fetishistic transvestism
- c. Voyeurism
- d. Frotteurism
Ans: B, Fetishistic transvestism
- Paraphilias (sexual deviations; perversions) are disorders of sexual preference in which sexual arousal occurs persistently and significantly in response to objects which are not a part of normal sexual arousal (e.g. nonhuman objects; suffering or humiliation of self and/or sexual partner; children or nonconsenting person). They are as follows:
- 1. Fetishism: In fetishism, the sexual arousal occurs either solely or predominantly with a nonliving object, which is usually intimately associated with the human body.
- 2. Fetishistic Transvestism: This disorder occurs exclusively in heterosexual males. The person actually or in fantasy wears clothes of the opposite sex (cross-dressing) for sexual arousal.
- 3. Sexual Sadism: In this disorder, the person (the ‘sadist’) is sexually aroused by physical and/or psycho logical humiliation, suffering or injury of the sexual partner (the ‘victim’).
- 4. Sexual Masochism: This is just the reverse of sexual sadism. Here the person (the ‘masochist’) is sexually aroused by physical and/or psychological humiliation, suffering or injury inflicted on self by others (usually ‘sadists’).
- 5. Exhibitionism: Exhibitionism is a persistent (or recurrent) and significant method of sexual arousal by the exposure of one’s genitalia to an unsuspecting stranger.
- 6. Voyeurism: This is a persistent or recurrent tendency to observe unsuspecting persons (usually of the other sex) naked, disrobing or engaged in sexual activity.
- 7. Frotteurism: This is a persistent or recurrent involvement in the act of touching and rubbing against an unsuspecting, nonconsenting person (usually of the other sex).
- 8. Paedophilia: Paedophilia is a persistent or recurrent involvement of an adult (age >16 years and at least 5 years older than the child) in sexual activity with prepubertal children, either heterosexual or homosexual.
- 9. Zoophilia (Bestiality): Zoophilia as a persistent and signifi cant involvement in sexual activity with animals is rare.
- Other Paraphilias: These include sexual arousal with urine (urophilia); faeces (coprophilia); enemas (klismaphilia); corpses (necrophilia), among many others.
-
89 (Repeat)
- Q1. The recommended treatment for Q-fever endocarditis is:
- a. Emergent surgery; antibiotics are not effective
- b. Levofloxacin monotherapy for 6 weeks
- c. 18 months of combination therapy with doxycycline and hydroxychloroquine
- d. Penicillin and gentamicin combination therapy using IgG titers to determine duration
Ans: c, 18 months of combination therapy with doxycycline and hydroxychloroquine.
- • C. burnetii is a small obligate organism that has a membrane similar to gram-negative bacteria, does not stain with Gram stain, multiplies within vacuoles, and causes the disease Q fever.
- • C burnetii exists in two antigenic forms called phase I and phase II. Phase I is the virulent form that is found in humans with Q fever and infected vertebrate animals, and it is the infectious form. Phase II is the avirulent form.
- • C burnetii is found in sheep, goats, cattle, and a variety of other animals, which are usually asymptomatic. Transmission to humans is by inhalation of contaminated dirt from animal feces, products of conception, or dust from animal products such as contaminated hides.
- • Q fever is characterized by acute and chronic infections. Acute pneumonia and hepatitis are associated with antibodies to phase II antigens; endocarditis is the most common form of chronic infection and is associated with antibodies to phase I antigens.
- • Diagnosis is clinically suspected and confirmed largely by serology or PCR performed in reference laboratories that have developed and verified their own assays.
- • Doxycycline is the drug of choice for the treatment of acute Q fever. The newer macrolides have also been shown to be effective in the treatment of acute pneumonia. Chronic Q fever requires prolonged treatment for 18 months or longer with a combination of doxycycline and hydroxychloroquine. Duration of treatment is long as mentioned earlier and should be determined by decrease in phase I antibody titers. In endocarditis, combination therapy is necessary to prevent relapse; occasionally, valve replacement is required and can prolong survival.
- Q2. Photodisruption is the basic mechanismof action of:
- a. Argon LASER
- b. Nd:YAG LASER
- c. Excimer LASER
- d. All of the above
Ans: b, Nd:YAG LASER
- Ref:AK Khurana 6th e/p 462
- MECHANISMS OF LASER EFFECT
- • Photocoagulation: Argon, diode, krypton and diode pump frequency doubled Nd:YAG lasers are based on this mechanism.
- • Photovaporization: Vaporization of tissue to CO2 and water occurs when temperature rises to 60–100 degree Centigrade or greater, e.g., CO2 absorbed by water of cells and heat generated causes cauterization or causes disintegration thereby incising the tissue.
- • Photodisruption: Nd:YAG laser and femtosecond laser are based on this mechanism.
- • Photoablation: Lasers based on this mechanism produce UV light of very short wavelength which breaks chemical bonds of biologic materials, converting them into small molecules that diffuse away. These lasers are collectively called excimer (excited dimer) lasers. These act by tissue modelling.
- • Photoradiation (Photochemical effect): Photochemical effect following visible/infrared particularly after administration of exogenous chromophore, e.g., hematoporphyrin or benzaporphyrin and is used for photodynamic therapy, treatment of ocular tumours and choroidal neovascularization.
- Q3. Dobutamine acts primarily on:
- a. D1 receptors
- b. D2 receptors
- c. Beta 1 receptors
- d. Beta 2 receptors
Ans:c, Beta 1 receptors
Ref:Ghai 8th e/p 718
-
90
- 1. All are true regarding Zollinger Ellison syndrome, except:
- a. Diarrhoea
- b. Recurrence after operation
- c. Hypergastrinemia
- d. Decreased ratio of BAO to MAO
Ans,d. Decreased ratio of BAO to MAO
- Basic pathology in patients with Zollinger Ellison syndrome is increased secretion of gastrin from non Beta cell endocrine tumour. The increased level of gastrin causes increased basal acid output secretion which is 60% of the total acid output. Then the ratio of basal acid out to maximal acid output is increased in Zollinger Ellison syndrome.
- This increased level of gastrin and acid produces peptic ulcers and diarrhoea which are common manifestations of Zollinger Ellison Syndrome.
- Diagnosis of Zollinger Ellison Syndrome:
- • First step in the diagnosis is to obtain fasting Gastrin level which is elevated.
- • It is > 150 pg/ml in almost all cases.
- • A level of >1000 pg/ml almost confirms the diagnostic of Zollinger Ellison syndrome.
- • Second step is to assess acid secretion. Basal acid output is increased.
- • Gastrin provocative test are also helpful in the diagnosis of Zollinger Ellison Syndrome. Especially in patients with indeterminate acid secretion. These tests are :
- • Secretion stimulation test (Most sensitive & Specific)
- • Calcium infusion test
- • Standard meal test
- Treatment:
- • DOC → Proton pump inhibitors (Omeprazole and Lansoprazole)
- • Surgery → Recurrence rate is high as it is difficult to exactly localize the tumour.
- 2. A patient comes with thrombocytopenia, eczema and recurrent infections. What is the most probable diagnosis:
- a. Wiskott Aldrich syndrome
- b. Beta gammaglobulinemia
- c. Chediak Hegashi syndrome
- d. Lazy leukocyte syndrome
Ans,a. Wiskott Aldrich syndrome
- • This X-linked disease characterized by eczema, thrombocytopenia, and repeated infections is caused by mutations in WAS gene.
- • Affected male infants often presents with bleeding, and most do not survive childhood, dying of complications of bleeding, infection, or lymphoreticular malignancy.
- • The immunologic defects include low serum concentrations of IgM, while IgA and IgG are normal and IgE is frequently increased.
- • Although the number and class distribution of B lymphocytes are usually normal, these patients fail to make antibodies to polysaccharide antigens normally, and responses to protein antigens may be impaired late in the course of disease.
- • Most patients eventually acquire severe T cell deficiency, rendering them vulnerable to overwhelming infections with herpes simplex virus and other infectious agents.
- 3. Bennet fracture dislocation involves base of:
- a. 1st metacarpal
- b. 5th metacarpal
- c. 1st metatarsal
- d. 5th metatarsal
Ans,a.1st metacarpal
- • Benett’s fracture is an intra-articular fracture dislocation of the palmar base of first metacarpal bone of the thumb with either subluxation or dislocation of first carpometacarpal joint, i.e. trapezometacarpal joint.
- • The common mechanism of injury is an axial blow directed against the partially flexed metacarpal, in most cases during “Fist fights. Patient complains of pain, swelling and tenderness over the base of the thumb. Movements of thumb are restricted.
-
91
- Q1. Surgical landmark for endolymphatic sac during surgery is:
- a. Solid angle
- b. Trautman triangle
- c. Utelli's angle
- d. Donaldson line
Ans: d, Donaldson line
- Donaldson's line: This line is a surgical landmark for endolymphatic sac. It passes through horizontal bisecting the posterior semicircular canal. The endolymphatic sac that appears as thickening of the posterior cranial fossa dura is situated inferior to Donaldson's line.
- Also Know:
- • Citelli's angle (sinodural angle): It lies between the sigmoid sinus and middle fossa dura mater.
- • Bill's island: This thin plate of bone left on sigmoid sinus during mastoidectomy helps in retracting the sigmoid sinus. It should not be confused with Bill's bar, which lies in the fundus of internal auditory canal.
- • Solid angle: This area of bony labyrinth lies between the three semicircular canals.
- • Trautmann's triangle: This area is bounded by the bony labyrinth anteriorly, sigmoid sinus posteriorly and the superior petrosal sinus superiorly. Any infection in the posterior canal fossa can spread through this triangle and can be approached by removing the bone in between the triangle.
- Q2. Peritoneal loose bodies also known as peritoneal mice are mostly asymptomatic. They are composed of;
- a. cartilage surrounded by saponified fat
- b. fibrin and cartilage
- c. saponified fat surrounded by fibrin
- d. fat balls only
Ans: c, saponified fat surrounded by fibrin
- Peritoneal loose bodies (peritoneal mice)
- Peritoneal loose bodies (peritoneal mice) may be confused with a small tumour but almost never cause symptoms. One or more may be found in a hernial sac or in the pouch of Douglas. The loose body may come from an appendix epiploica that has undergone axial rotation followed by necrosis of its pedicle and detachment, but they are also found in those who have subacute attacks of pancreatitis. These hyaline bodies attain the size of a pea or bean and contain saponified fat surrounded by fibrin.
- Q3. Vicryl, an absorbable suture material used commonly in surgical practice is a:
- a. Homopolymer of polydiaxone
- b. Co-polymer of glycolide and lactide
- c. Homopolymer of glycolide
- d. Homopolymer of lactide
Ans: b. Co-polymer of glycolide and lactide
- • Vicryl, a polyglactin is a Synthetic, Absorbable, Multifilament, lubricant coated suture.
- • It is a copolymer of lactide and glycolide in a 90:10 ratio, coated with polyglactin and calcium stearate.
- • Complete absorption by 60-90 days; (increases in alkaline environment and high temps; don’t use to close bladder).
- Q4. For infection prevention and control, WHO calls to action on 5 may 2020, which action is meant for nurses?
- a. Clean and safe care starts with you
- b. Your hands make all the difference for mothers and babies
- c. Empower nurses and midwives in providing clean care
- d. Safer care for you, with you
Ans: a. Clean and safe care starts with you
SAVE LIVES: Clean Your Hands 5 May 2020
- Nurses and Midwives, clean care is in your hands!
- • Clean health care is recognized as one of WHO’s urgent challenges to be tackled in the next 10 years by the global community, in our race to meeting SDG deadline.
- • Therefore, clean care, including hand hygiene best practices, and the central role played by nurses and midwives in achieving this, is the focus of this year’s 5 May campaign.
- • The idea has been to partner with the Year of the Nurse and the Midwife that WHO has declared for 2020, and to recognize their crucial contribution to strengthening quality health systems.
- • We are calling all health care workers and other target audiences to join hands in celebrating and empowering nurses and midwives in the monumental work that they do to keep our patients, families, and their colleagues safe by preventing health care-associated infections.
- 5 MAY 2020 CALLS TO ACTION
- •Nurses: “Clean and safe care starts with you."
- •Midwives: “Your hands make all the difference for mothers and babies.”
- •Policy Makers: "Increase nurse staffing levels to prevent infections and improve quality of care. Create the means to empower nurses and midwives.”
- •IPC Leaders: "Empower nurses and midwives in providing clean care."
- •Patients and Families: "Safer care for you, with you."
- Q5. Adipocere formation starts under the influence of:
- a. Intrinsic lipase
- b. Lipase from bacteria
- c. Lecithinase
- d. Dehydrogenation
Ans, A, Intrinsic lipase
- ADIPOCERE(SAPONIFICATION)
- Adipocere (cire=wax)
- • Is a modification of putrefaction.
- • In this, the fatty tissues of the body change into a substance known as adipocere.
- • It is seen most commonly in bodies immersed in water or in damp, warm environment.
- Mechanism:
- • The change is due to the gradual hydrolysis and hydrogenation of pre-existing fats,such as olein, into higher fatty acids, which combine with calcium and ammonium ions to form insoluble soaps, which being acidic, inhibit putrefactive bacteria.
- • Ultimately, the whole of the fat is converted into palmitic, oleic, stearic and hydroxystearic acid, together with some glycerol, and a mixture of these substances forms adipocere.
- • These form a matrix for remnants of tissue fibers, nerves and muscles.
- • At the time of death, body fat contains, about half percent of fatty acids, but in adipocere they rise to 20% within a month and over 70% in three months.
- • The process starts under the influence of intrinsic lipases, and is continued by the bacterial enzymes of the clostridia group, mainly Cl. perfringes, as the bacteria produce lecithinase, which facilitates hydrolysis and hydrogenation.
- • Water is essential for the bacterial and enzymatic processes involved in adipocere formation.
-
92
- Q1. Effect of progestogens on lipid profile is
- a.Increase LDL
- b.Decrease HDL
- c.Both
- d.No effect on lipid profile
- .
- Ans.c.both
- SIDE EFFECTS OF PROGESTOGENS:
- nausea—subsides gradually
- • leg cramps
- • mastalgia
- • Weight gain due to salt and water retention
- • acne
- • scanty periods due to androgenic progestins
- • loss of libido
- • Virilism
- • Headaches—migraines should be excluded
- • Depression and mood changes—may be due to low level of pyridoxin as progestins alter the tryptophan metabolism.
- • lipid profile—increase in LDL and decrease inHDL level.
- Q2. A 40 years old man is accused of anal intercourse to a seven years old boy. An adult who repeatedly engages in sexual activities with children is called:
- a.Pedophile
- b. Catamite
- c. Paederasty
- d.Gerontophilia
- .
- Ans , a.Pedophile
- SODOMY
- • Sodomy is the anal intercourse between two males, or between a male and female.
- • This used to be practiced in a town called Sodom. It is also called buggery.
- • It is called
- o gerontophilia- when the passive agent is an adult
- o paederasty- when the passive agent is a child, who is known as catamite.
- •A pedophile-is an adult who repeatedly engages in sexual activities with children.
- •The Greeks of the "Golden Age" also practised it, and it is sometimes referred to as Greek love. It can be heterosexual or homosexual.
- •When practised between two men, they may alternately act as active and passive agents.
- Q3. You are a medical officer in Shukraraj Tropical & Infectious Disease Hospital,you are treating patients infected with COVID-19,which precaution is needed for you
- a.Standard precautions
- b.Transmission-Based precautions
- c.Basic precautions
- d.Both a and b
Ans d ,both a and b
- Infection Control Basics
- There are 2 tiers of recommended precautions to prevent the spread of infections in healthcare settings:
- Standard Precautions and Transmission-Based Precautions.
- Standard Precautions for All Patient Care
- Standard precautions are used for all patient care. They’re based on a risk assessment and make use of common sense practices and personal protective equipment use that protect healthcare providers from infection and prevent the spread of infection from patient to patient.
- Transmission-Based Precautions
- Transmission-based precautions are used in addition to Standard Precautions for patients with known or suspected infections.
-
93
- 1. The neck- shaft angle of femur is:
- a. 15°
- b. 75°
- c. 125°
- d. 160°
- Ans: c.125°
- Ref: Human anatomy BD Chaurasia 4th edition Vol2 P16
- The neck-shaft angle is about 125° in adults. It is less in females due to their wider pelvis.
- The angle facilitates movements of the hip joint. It is strengthened by a thickening of bone called the calcar femorale present along its concavity.
- The angle of femoral torsion or angle of anteversion is formed between the transverse axes of the upper and lower ends of the femur. It is about 15°.
- 2. Which amino acid contains sulfur atoms in the side chain?
- a. Alanine
- b. Methionine
- c. Tyrosine
- d. Tryptophan
- Ans: b. Methionine
- Harper’s illustrated Biochemistry 31th edition P 51
- A. Amino acids with side chain containing Hydroxylic (OH) groups
- Serine
- Threonine
- Tyrosine
- B. Amino acids with side chain containing sulfur atoms
- Cysteine
- Methionine
- C. Amino acids with side chain containing acidic groups or their amides
- Aspartic acid
- Asparagine
- Glutamic acid
- Glutamine
- D. Amino acids with side chain containing basic groups
- Arginine
- Lysine
- Histidine
- 3. Which of the following drug is delivered through skin patch for the treatment of parkinsonism?
- a. Levadopa
- b. Rotigotine
- c. Apomorphine
- d. Arantil
- Ans: b. Rotigotine
- Ref: Basic and clinical Pharmacology, Katzung 14th P 499
- The dopamine agonist rotigotine, delivered daily through a skin patch, is approved for treatment of early Parkinson’s disease. It supposedly provides more continuous dopaminergic stimulation than oral medication in early parkinsonism; its efficacy in moreadvanced disease is less clear. Benefits and side effects are similar to those of other dopamine agonists but reactions may also occur at the application site and are sometimes serious.
-
94
- 1. What is involved in alopecia universalis?
- a. Whole scalp
- b. Whole body
- c. Whole body except armpits and pubis
- d. Whole scalp plus armpits and pubis
Ans, b, Whole body
- Alopecia areata
- Etiology: Immunologically mediated.
- Morphology: Noncicatricial, noninflammatory alope
- Presents as discoid areas of hair loss with exclamation mark hair at the edge.
- Course: Unpredictable; spontaneous recovery with regrowth initially of gray hair. Or progresses to
- Alopecia totalis (whole scalp) or Alopecia universalis (whole body).
- Site: Scalp, less frequently beard, eyebrows, and eye- lashes. Sometimes generalized.
- Treatment: Localized lesions: <6 months: no treatment; >6 months: topical steroids.
- Extensive lesions: Response to treatment unpredictable.
- Minoxidil, psoralens with UVA/UVA sol (topical and systemic) and steroids (topical and systemic).
- 2. A transplant from one to another twin is called?
- a. Autograft
- b. Isograft
- c. Xenograft
- d. Allograft
Ans, b, Isograft
- Autografts, Isografts, Allografts, and Xenografts.
- A transplant of a tissue or whole organ
- i. from one part of the same animal to another part is called an autograft;
- ii. from one identical twin to another, an isograft;
- iii. from one human being to another or from any animal to another animal of the same species, an allograft;
- iv. from a non-human animal to a human being or from an animal of one species to one of another species, a xenograft.
- 3. What is the correct sequence of stages of overcoming addiction?
- a. Contemplation, pre-contemplation, action, preparation, maintenance, relapse
- b. Preparation, pre-contemplation, contemplation, action, maintenance, relapse
- c. Pre-contemplation, contemplation, preparation, action, maintenance, relapse
- d. Pre-contemplation, contemplation, action, preparation, maintenance, relapse
Ans, c, Pre-contemplation, contemplation, preparation, action, maintenance, relapse
- Stages of change in overcoming addiction
- Pre-contemplation—denying problem
- Contemplation. —acknowledging problem, but unwilling to change
- Preparation/determination—preparing for behavioral changes
- Action/willpower—changing behaviors
- Maintenance. —maintaining changes
- Relapse. —(if applicable) returning to old behaviors and abandoning changes
-
95
- 1. Earliest radiological sign of Pulmonary Venous Hypertension in Chest X-ray is:
- a. Cephalization of pulmonary vascularity
- b. Pleural effusion
- c. Kerley B lines
- d. Alveolar pulmonary edema
- Correct Ans : a
- Explanation
- Ref:; Textbook of radiology and Imaging By David Sutton, 5th Edition, Pages 288-89
- The first radiographic sign of pulmoi ary venous hypertension is cephalization of the pulmonary vessels due to pulmonary vein and artery dilatation.
- In the normal individual, there is continuous passage of fluid from the pulmonary veins into adjacent interlobular lymphatics that return the fluid to the central mediastinal veins.
- If the lymphatic reserve is overcome by increased transudate as a result of elevated pulmonary venous pressure, the interlobular septa are thickened and become visible radiographically.
- The first radiologic sign of pulmonary venous hypertension is cephalization of the pulmonary vessels.
- 2. Which of the following is an autosomal recessive disorder?
- a. Duchenne muscular dystrophy
- b. Friedreich ataxia
- c. Wiskott Aldrich syndrome
- d. Lesch Nyhan syndrome
- Ans: b
- Ref: Robbins & Cotran, Pathologic basis of disease, South Asia edition, Vol 1, p141, 1
- Duchenne muscular dystrophy, Wiskott Aldrich syndrome and Lesch Nyhan syndrome are X-linked recessive disorders.
- Friedreich ataxia is an autosomal recessive disorder.
- Other autosomal recessive disorders include congenital adrenal hyperplasia, alkaptonuria, alpha 1 antitrypsin deficiency, phenylketonuria and Wilson disease.
- 3. Long term fluctuation is seen with
- a. cyclic trends
- b. epidemics
- c. secular trends
- d. seasonal trends
- Ans, C, secular trends
- Ref, Park’s Textbook of Preventive and Social Medicine 22nd edition, p62
- Explanation
- Long Term Fluctuations [Secular Trends]
- Implies changes in occurrence of a disease (progressive increase or decrease) over a long period of time, generally several years or decades.
- Is the consistent tendency to change in a particular direction or a definite movement in one direction.
- Examples:
- – Communicable diseases (Poliomyelitis, Diphtheria, and Pertussis) are reducing in past few decades
- – Non-communicable diseases (Diabetes, Hypertension, and Obesity) are increasing in past few decades.
-
96
- Q1.Bachman's test is done to diagnose infections with :
- a. Schistosoma japonicum
- b. Trichinella spiralis
- c. Trichuris trichiura
- d. Ancylostoma duodenale
Ans: b,Trichinella
- Trichinella spiralis, tissue n ematode, is the causative agent of trichinosis.
- Diagnosis of trichinosis can be made by direct and indirect methods
- Bachman intra.dermal test: It uses a 1:5,000 or 1:10,000 dilution of the larval antigen. An erythematous wheal appears in positive cases within 15- 20 minutes. The rest remains positive for years after infection.
- Q2.A 6 years old boy presents with passage of frothy urine,anasarca.He was evaluated and diagnosed as Nephrotic Syndrome and was kept under prednisolone therapy however there was no remission at 8 weeks of therapy.What could be the most common cause?
- a.Minimal change disease
- b.Focal segmental glomerulosclerosis(FSGS)
- c.Membranoproliferative Glomerulonephritis
- d.IgA Nephropathy
- Ans:b, FSGS
- Ref:Nelson 21st ed pg 10823
- • Steroid resistance is defined as the failure to achieve remission(Remission consists of a urine protein:creatinine ratio of < 0.2 or < 1+ protein on urine dipstick testing for 3 consecutive days)after 8 wk of corticosteroid therapy.
- • Children with steroid-resistant nephrotic syndrome require further evaluation, including a diagnostic kidney biopsy, evaluation of kidney function, and quantitation of urine protein excretion (in addition to urine dipstick testing).
- • Steroid-resistant nephrotic syndrome is usually caused by FSGS (80%), MCNS, or membranoproliferative glomerulonephritis.
- Q3.Dendritic keratitis is associated with:
- a.Adeno virus keratitis
- b.Fungal keratitis
- c.Pseudomonas keratitis
- d.Herpes simplex keratitis
- Ans:d, Herpes simplex keratitis
- Ref:Nelson textbook of pediatrics 21st edition pg13021
- Dendritic Keratitis
- • Infection of the cornea with the herpes simplex virus produces a characteristic lesion of the corneal epithelium, referred to as a dendrite; it has a branching treelike pattern that can be demonstrated by fluorescein staining .
- • The acute episode is accompanied by pain, photophobia, tearing, blepharospasm, and conjunctival injection.
- • Specific treatment may include mechanical debridement of the involved corneal epithelium to remove the source of infection and eliminate an antigenic stimulus to inflammation in the adjacent stroma. Medical treatment involves the use of trifluridine, topical ganciclovir, or systemic acyclovir. In addition, a cycloplegic agent is useful to relieve pain from spasm of the ciliary muscle.
- • Overly aggressive topical antiviral treatment itself can be toxic to the cornea and should be avoided. Recurrent infection and deep stromal involvement can lead to corneal scarring and loss of vision.
-
97
- 1. Most common complication of Shigellosis is:
- a. Hemolytic Uremic Syndrome (HUS)
- b. Reactive arthritis
- c. Pneumonia
- d. Meningitis
- Ans,a. Hemolytic Uremic Syndrome (HUS)
- Harrison 19th ed/pg.905,906
- Common complications:
- • Bacteremia
- • HUS : It occurs with S. dysenteriae type-1 infection. Manifestations of HUS usually develop at the end of first week of shigellosis.Oliguria, marked drop in hematocrit are the first signs & progresses to anuria with renal failure & severe anemia with congestive heart failure.
- Less common complications: Reactive arthritis
- Rare complications: Pneumonia; Meningitis; Vaginitis (in prepubertal girls);Keratoconjunctivitis & “rose spot” rashes.
- 2. Which one of the following act directly on thromboplastin to activate plasminogen:
- a. Streptokinase
- b. Urokinase
- c. Heparin
- d. rt-PA
- Ans,d. rt-PA
- Ref: Harrison’s 18th ed./p.1001
- • Streptokinase is obtained from cultures of Beta hemolytic streptococci. By itself, streptokinase has no PA activity, but when complexed with plasminogen, it can convert other plasminogen molecules to plasmin. It is not fibrin-selective in that the so-called lytic state resulting from its therapeutic use is due to lysis of fibrinogen as well as fibrin.
- • Recombinant tissue-type plasminogen activator (rt-PA) produced by recombinant technology demonstrates in vitro affinity for fibrin with which it forms a ternary complex with plasminogen. Despite reported fibrin-specificity, the lytic state is produced with dosages currently used. Bleeding complications with rt-PA are similar to those of streptokinase or urokinase.
- 3. If the fracture is intra-articular and extends through the epiphysis, physis and metaphysis,which type of Salter Harris classification does it fall under:
- a. Type II
- b. Type III
- c. Type IV
- d. Type V
- Ans,c. type IV
- Ref: Campbell’s 12th ed./p.1366
- Salter Harris classification : Epiphyseal / Physeal injuries
- • Type I: Complete separation of epiphysis from the metaphysis without fracture. Common in rickets, scurvy and osteomyelitis.
- • Type II: The fracture involves physis and a triangle of metaphyseal bone (Thurston Holland sign). This is the commonest type of epiphyseal injury accounting for 73 percent of cases over 10 years of age.
- • Type III: The fracture is intra- articular and extends along the physis and then along the growth plate. This injury is relatively uncommon.
- • Type IV: The fracture is intra-articular and extends through the epiphysis, physis and metaphysis. Perfect reduction is necessary and open reduction is more often necessary to prevent growth arrest.
- • Type V: Crushing of epiphysis. Growth arrest usually follows.
- • Type VI (Rang’s type) : There is a peripheral physis (perichondrial ring) injury.
-
98
- 1. Deep Brain Stimulation done for Parkinson’s disease primarily targets
- a. Globus Pallidus internus
- b. Globus Pallidus externus
- c. Substantia Nigra compacta
- d. Substantia nigra reticulata
- Ans: a
- Ref: Harrison 20th Edition, Page No: 3129
- • DBS for PD primarily targets the STN or the GPi. It provides dramatic results, particularly with respect to tremor and reducing both “off” time and dyskinesias, but does not provide superior clinical benefits or improve features that do not respond to levodopa such as freezing, falling, and dementia.
- • The procedure is thus primarily indicated for patients who suffer disability resulting from severe tremor, or levodopa-induced motor complications that cannot be satisfactorily controlled with drug manipulation. In such patients, DBS has been shown to provide benefits in comparison to best medical therapy.
- • Recent studies indicate that benefits following DBS of the STN and GPi are comparable, but that GPi stimulation may be associated with a reduced frequency of depression. Although not all PD patients are candidates, the procedure can be profoundly beneficial for many.
- 2. Mantle cell lymphoma are positive for all except:
- a. CD 23
- b. CD 5
- c. CD 43
- d. CD 20
- Ans: a
- Ref: Robbins Basic Pathology 10th Edition, Page No: 469
- • Almost all tumors have an (11;14) translocation that fuses the cyclin D1 gene to the IgH locus. This translocation leads to over expression of cyclin D1, which you will recall stimulates growth by promoting the progression of cells from the G1 phase to the S phase of the cell cycle.
- • The tumor cells express surface IgM and IgD, the B cell antigen CD20, and CD5, and they contain high levels of cyclin D1 protein.
- 3. Achondroplasia is caused by point mutation of:
- a. TCIRG1
- b. Mutation in the genes that encode the α1 and α2 chains
- c. carbonic anhydrase 2
- d. FGFR3
- Ans: d
- Ref: Robbins Basic Pathology 10th Edition, Page No: 800
- • Mutation in the genes that encode the α1 and α2 chains- Osteogenesis Imperfecta
- • TCIRG1 and carbonic anhydrase 2 mutation-Osteopetrosis
- • FGFR3 mutation-Achondroplasia and Thanatophoric Dysplasia
- 4. The use of genetic information to guide the choice of drug and dose on an individual basis:
- a. Pharmacogenetics
- b. Pharmacogenomics
- c. Pharmacovigilance
- d. Pharmacodynamics
- Ans: b
- Ref: KDT Pharmacology 7th Edition, Page No: 82,65,1-2
- Pharmacovigilance
- Pharmacovigilance has been defined by the WHO (2002) as the ‘science and activities relating to the detection, assessment, understanding and prevention of adverse effects or any other drug related problems.’
- Pharmacogenetics: The study of genetic basis for variability in drug response is called ‘Pharmacogenetics’.
- Pharmacogenomics: The use of genetic information to guide the choice of drug and dose on an individual basis.
- Pharmacodynamics (Greek: dynamis—power) —What the drug does to the body.
- • This includes physiological and biochemical effects of drugs and their mechanism of action at organ system/subcellular/macromolecular levels, e.g.—Adrenaline → interaction with adrenoceptors → G-protein mediated stimulation of cell membrane bound adenylyl cyclase → increased intracellular cyclic 3 ́,5 ́AMP → cardiac stimulation, hepatic glycogenolysis and hyperglycaemia, etc.
- Pharmacokinetics (Greek: Kinesis—movement)—What the body does to the drug.
- • This refers to movement of the drug in and alteration of the drug by the body; includes absorption, distribution, binding/localization/storage, bio- transformation and excretion of the drug, e.g. paracetamol is rapidly and almost completely absorbed orally attaining peak blood levels at 30–60 min; 25% bound to plasma proteins, widely and almost uniformly distributed in the body (volume of distribution ~ 1L/kg); extensively metabolized in the liver, primarily by glucuronide and sulfate conjugation into inactive metabolites which are excreted in urine; has a plasma half life (t1⁄2) of 2–3 hours and a clearance value of 5 ml/kg/min.
- 5. The active form of Remdesivir, drug under clinical trial for treatment of COVID 19, is:
- a. GC-376
- b. GS-441524
- c. B-5701
- d. GS-5734
- Ans: b
- Ref: Agostini ML, Andres EL, Sims AC, Graham RL, Sheahan TP, Lu X, et al. (March 2018). "Coronavirus Susceptibility to the Antiviral Remdesivir (GS-5734) Is Mediated by the Viral Polymerase and the Proofreading Exoribonuclease
- • Remdesivir is a prodrug that metabolizes into its active form GS-441524.
- • An adenosine nucleotide analog, GS-441524 interferes with the action of viral RNA-dependent RNA polymerase and evades proofreading by viral exoribonuclease (ExoN), causing a decrease in viral RNA production.
- • Though in some viruses, such as the respiratory syncytial virus but not Ebola virus, it causes the RNA-dependent RNA polymerases to pause, its predominant effect is to induce an irreversible chain termination.
- • Unlike with many other chain terminators, this was not mediated by preventing addition of the immediately subsequent nucleotide, but is instead delayed, occurring after five additional bases have been added to growing RNA chain.
- • During 2020, several clinical trials were underway. The first placebo controlled trial of Remdesivir in China, according to a WHO report that was published early in error, showed that Remdesivir had no clinical or virological benefits, and the drug was stopped early in 11.6% of patients due to adverse effects. In the large industry sponsored study for Remdesivir, for severe COVID-19, the trial was modified, mid trial, increasing in size from 400 subjects to 2,400 participants. The primary end point of the trial was also changed during the trial.
-
99
- 1. A 66-year-old man with chronic cough has an episode of hemoptysis. On physical examination, there are no abnormal findings. A chest radiograph shows a 6-cm mass in the right lung. A sputum cytologic analysis shows neoplastic squamous cells. Metastases from his lung lesion are most likely to be found at which of the following sites?
- a. Cerebral hemisphere
- b. Chest wall muscle
- c. Hilar lymph nodes
- d. Splenic red pulp
- Ans: c
- Ref: Robbins Basic Pathology 9th Edition, Page No: 168–169
- Carcinomas metastasize through lymphatics most of¬ten, usually to regional nodes first. Hematogenous metastases are possible, however, to sites such as bone marrow, liver, or the opposite lung. About half of all cerebral metastases arise from lung primary carcinomas. Soft-tissue metastases to muscle, fat, and connective tissues are rare, as are splenic metastases.
- 2. Thiopentone is used for induction of anaesthesia. It shows marked redistribution which is a characteristic of:
- a. Highly lipid soluble drugs
- b. Highly water soluble drugs
- c. Weak electrolytes
- d. Highly plasma protein bound drugs
- Ans: a
- Ref: KDT Pharmacology, 7th Edition; Page No: 18
- Redistribution
- • Highly lipid-soluble drugs get initially distributed to organs with high blood flow, i.e. brain, heart, kidney, etc. Later, less vascular but more bulky tissues (muscle, fat) take up the drug—plasma concentration falls and the drug is withdrawn from the highly perfused sites.
- • If the site of action of the drug was in one of the highly perfused organs, redistribution results in termination of drug action.
- • Greater the lipid solubility of the drug, faster is its redistribution.
- • Anaesthetic action of thiopentone sod. Injected i.v. is terminated in few minutes due to redistribution.
- 3. Atropine is most sensitive to:
- a. Mucous and pharyngeal secretions
- b. Heart
- c. Pupil
- d. GI tract motility
- Ans: a
- Ref: KDT Pharmacology, 7th Edition; Page No: 115
- The sensitivity of different organs and tissues to atropine varies and can be graded as:
- Saliva, sweat, bronchial secretion > eye, bronchial muscle, heart > smooth muscle of intestine, bladder > gastric glands and smooth muscle.
- • Atropine markedly decreases sweat, salivary, tracheobronchial and lacrimal secretion (M3 blockade). Skin and eyes become dry, talking and swallowing may be difficult.
- • Atropine decreases secretion of acid, pepsin and mucus in the stomach, but the primary action is on volume of secretion so that pH of gastric contents may not be elevated unless diluted by food.
- • Since bicarbonate secretion is also reduced, rise in pH of fasting gastric juice is only modest.
- 4. Major basic protein is formed from:
- a. Neutrophil
- b. Lymphocyte
- c. Basophil
- d. Eosinophil
- Ans: D
- Ref: Robbins Pathology; 10th edition; pg 84
- Eosinophils are abundant in immune reactions mediated by IgE and in parasitic infections. Their recruitment is driven by adhesion molecules similar to those used by neutrophils, and by specific chemokines (e.g., eotaxin) derived from leukocytes and epithelia cells. Eosinophils have granules that contain major basic protein, a highly cationic protein that is toxic to parasites but also injures host epithelial cells. This is why eosinophils are of benefit in controlling parasitic infections, yet also contribute to tissue damage in immune reactions such as allergies.
-
100 (Repeat)
Answers and Winners of QUIZ for CEE MD/MS Based Online Model Test (Baisakh 17, 07:00 PM)
- 1. 9th, 10th and 11th cranial nerves form nucleus ambiguus which is classified as
- a. Special visceral efferent
- b. General somatic efferent
- c. General visceral efferent
- d. General somatic afferent
- Ans: a
- Ref: Inderbir Singh embryology 12th edition; Pg 297-299
- Afferent columns in alar lamina: The four functional columns in alar lamina are:
- • General visceral afferent (GVA): concerned with visceral sensations conveyed through 9th and 10th cranial nerves.
- • Special visceral afferent (SVA): concerned with taste sensation conveyed through 7th, 9th and 10th cranial nerves from the tongue, pharynx and soft palate.
- • General somatic afferent (GSA): contains mesencephalic, chief and spinal nuclei of trigeminal nerve concerned with proprioception, touch, and pain and temperature sensations respectively from the head and neck region.
- • Special somatic afferent (SSA): contains vestibular and cochlear nuclei concerned with equilibrium and hearing.
- Efferent columns of basal lamina: The three functional columns in basal lamina are:
- • General somatic efferent (GSE): contains nuclei of cranial nerves (3rd, 4th, 6th and 12th) that supply the extraocular muscles of eyeball and musculature of tongue.
- • Special visceral efferent/Branchial efferent (SVE): contains the nuclei of cranial nerves (5th, 7th, 9th, 10th and 11th) that supply to musculature derived from pharyngeal arches.
- • General visceral efferent (GVE): contains the cranial nerve nuclei of origin for preganglionic neurons of peripheral parasympathetic ganglia.
- • Edinger-Westphal nucleus (3rd cranial nerve): Ciliary ganglion
- • Superior salivatory nucleus (7th cranial nerve): Pterygopalatine and submandibular ganglia
- • Inferior salivatory nucleus (9th cranial nerve): Otic ganglion
- • Dorsal nucleus of vagus (10th cranial nerve): for supply of thoracic and abdominal viscera.
- 2. In triage the second priority is given to which of the color coding
- a. Black
- b. Blue
- c. Yellow
- d. Green
- Ans: C
- Ref: Park PSM 23rd Edition, Page No: 796-797
- Triage
- When the quantity and severity of injuries overwhelm the operative capacity of health facilities, a different approach to medical treatment must be adopted. The principle of "first come, first treated", is not followed in mass emergencies. Triage consists of rapidly classifying the injured on the basis of the severity of their injuries and the likelihood of their survival with prompt medical intervention. It must be adopted to locally available skills. Higher priority is granted to victims whose immediate or long-term prognosis can be dramatically affected by simple intensive care. Moribund patients who require a great deal of attention, with questionable benefit, have the lowest priority. Triage is the only approach that can provide maximum benefit to the greatest number of injured in a major disaster situation.
- Although different triage systems have been adopted and are still in use in some countries, the most common classification uses the internationally accepted four colour code system.
- • Red indicates high priority treatment or transfer,
- • Yellow signals medium priority,
- • Green indicates ambulatory patients and
- • Black for dead or moribund patients.
Triage should be carried out at the site of disaster, in order to determine transportation priority, and admission to the hospital or treatment centre, where the patient's needs and priority of medical care will be reassessed. Ideally, local health workers should be taught the principles of triage as part of disaster training. Persons with minor or moderate injuries should be treated at their own homes to avoid social dislocation and the added drain on resources of transporting them to central facilities. The seriously injured should be transported to hospitals with specialized treatment facilities.
- 3. Infliximab is:
- a. IgG1 chimeric monoclonal antibody against TNF alpha
- b. IgG1 fully human monoclonal antibody against TNF alpha
- c. IgG4 chimeric monoclonal antibody against TNF alpha
- d. P75 TNF receptor fusion protein
- Ans: a
- Ref: Katzung Pharmacology 12th Edition, Page No: 993
- Infliximab
- Human-mouse chimeric IgG 1 monoclonal antibody possessing human constant (Fc) regions and murine variable regions.
- It is administered intravenously but has the same anti- TNF-α activity as adalimumab and etanercept. Infliximab is currently approved for use in Crohn’s disease, ulcerative colitis, rheumatoid arthritis, ankylosing spondylitis, plaque psoriasis, and psoriatic arthritis.
- 4. Rugger Jersey spine is seen in
- a. Fluorosis
- b. Achondroplasia
- c. Renal osteodystrophy
- d. Marfan’s syndrome
- Ans: c
- Rugger jersey spine describes the prominent endplate densities at multiple contiguous vertebral levels to produce an alternating sclerotic-lucent-sclerotic appearance.
- This mimics the horizontal stripes of a rugby jersey. This term and pattern are distinctive for hyperparathyroidism.
- Seen in osteopetrosis and renal osteodystrophy. In patient with renal osteodystrophy, this appearance is due to hyperparathyroidism and osteosclerosis.
- 5. Ormond’s disease is:
- a. Idiopathic lymphadenopathy
- b. Retractile testis
- c. Idiopathic retroperitoneal fibrosis
- d. Idiopathic mediastinitis
- Ans: c
- Ref: Bailey and Love 27th Edition, Page No: 1064-1065
- Retroperitoneal fibrosis
- • This is a relatively rare diagnosis characterized by the development of a flat grey/white plaque of tissue, which is found first in the low lumbar region but then spreads laterally and upwards to encase the common iliac vessels, ureters and aorta. Histological appearances vary from active inflammation with a high cellular content interspersed with bundles of collagen through to one of acellularity and mature fibrosis/calcification.
- • Its etiology is obscure in most cases (idiopathic-) being allied to other fibromatoses (others being Dupuytren’s contracture and Peyronie’s disease). In other patients the cause is known.
- • The clinical presentation may be one of ill-defined chronic backache or occur as a result of compromise to involved structures, e.g. lower limb or scrotal edema secondary to venous occlusion, or chronic renal failure secondary to ureteric obstruction. Treatment will be directed to the cause, the modification of disease activity when appropriate, e.g. immunomodulation with steroids, tamoxifen and restoration of flow in affected structures, e.g. ureteric stenting.
Causes of retroperitoneal fibrosis: Idiopathic (Ormond’s disease)
- Winners:
- Fastest correct answer respondents:
- - Dr Safalta Yadav
- - Dr Pragya Sharma
- Lucky draw winners:
- - Dr Bijay Ranabhat
- - Dr Kiran Pradhan Shrestha
Winnersa re requested to send email to onlinepgnepal@gmail.com to claim Free Access to Today eveninf CEE MD/MS Based Online Model Test (07:00 PM, Wednesday, Baisakh 17).
-
101
- 1. CABG is preferred over PCI for revascularization in the following situations except
- a. Severe left main disease
- b. Three vessel disease
- c. In patients with DM
- d. Favorable anatomy
- Ans: d
- CABG is preferred over PCI in most severe left main and three-vessel disease or those with diabetes.
- Otherwise PCI is preferred to CABG, specially when there is favorable anatomy.
- Reference: UpToDate
- 2. HDV RNA requires which of the following for its replication?
- a. Host RNA polymerase I
- b. Viral RNA polymerase I
- c. Host RNA polymerase II
- d. Viral RNA polymerase II
- Ans: c
- Ref: Harrison 19th Edition, Page No: 2009
- HDV RNA requires host RNA polymerase II for its replication via RNA-directed RNA synthesis by transcription of genomic RNA to a complementary antigenomic (plus strand) RNA. The antigenomic RNA, in turn, serves as a template for subsequent genomic RNA synthesis.
- 3. Trachea begins at the level of:
- a. Lower border of thyroid cartilage
- b. Lower border of cricoid
- c. Lower border of hyoid
- d. Fourth cervical vertebra
- Ans: b
- Ref: Snell Clinical Anatomy; 9th edition; Page No: 63
- o The trachea is a mobile cartilaginous and membranous tube. It begins in the neck as a continuation of the larynx at the lower border of the cricoid cartilage at the level of the 6th cervical vertebra. It descends in the midline of the neck.
- o In the thorax, the trachea ends below at the carina by dividing into right and left principal (main) bronchi at the level of the sternal angle (opposite the disc between the 4th and 5th thoracic vertebrae).
- o During expiration, the bifurcation rises by about one vertebral level, and during deep inspiration may be lowered as far as the 6th thoracic vertebra.
- 4. A 22 year old patient presents with excessive wax in both ears. This is due to secretion from the gland:
- a. Modified eccrine glands
- b. Modified apocrine glands
- c. Mucous gland
- d. Modified holocrine glands
- Ans: b
- Ref: PL Dhingra and Shruti Dhingra ENT and HNS 6th Edition, Page No: 53
- • Wax is composed of secretion of sebaceous glands, ceruminous glands, hair, desquamated epithelial debris, keratin and dirt.
- • Sebaceous and ceruminous (modified sweat glands) glands open into the space of the hair follicle.
- • Sebaceous glands provide fluid rich in fatty acids while secretion of ceruminous gland is rich in lipids and pigment granules. Secretion of both these glands mixes with the desquamated epithelial cells and keratin shed from the tympanic membrane and deep bony meatus to form wax.
-
102
- 1. A 6 year old child has briefly sustained flexion or extension of predominantly proximal muscles, including truncal muscles. The EEG usually shows diffuse, giant slow waves with a chaotic background of irregular, multifocal spikes and sharp waves. The type of seizure probably is
- a. Epileptic spasm
- b. Atonic seizure
- c. Myoclonic seizure
- d. Absence seizure
- Ans: a
- Ref: Harrison 20th Edition, Page No: 3052
- Epileptic spasms are characterized by a briefly sustained flexion or extension of predominantly proximal muscles, including truncal muscles. The EEG usually shows hypsarrhythmia, which consist of diffuse, giant slow waves with a chaotic background of irregular, multifocal spikes and sharp waves. During the clinical spasm, there is a marked suppression of the EEG background (the “electrodecremental response”). The electromyogram (EMG) also reveals a characteristic rhomboid pattern that may help distinguish spasms from brief tonic and myoclonic seizures. Epileptic spasms occur predominantly in infants and likely result from differences in neuronal function and connectivity in the immature versus mature CNS.
- 2. A 5-year-old child is noted to be iron deficient. Upon questioning, his family reports several episodes of grossly bloody (maroon) stools. Which of the following is a likely explanation for his anemia?
- a. Increased demands for iron due to accelerated growth
- b. Meckel diverticulum
- c. Infestation by hookworm
- d. Recurrent epistaxis
- Ans: b
- Ref: Nelson pediatrics, 20th Edition, Page No: 1804, 1805
- Iron deficiency is most common between 6 months and 3 years of age. During this period, the need for iron is increased because of accelerated growth, and this need is often not met when the diet consists predominantly of cow’s milk, a uniquely iron-poor food. Growth plateaus during the preschool and preadolescent years and the mixed diets of children in this period are more likely to provide them with adequate iron. Chronic loss of blood from the intestinal tract or from nosebleeds can quickly deplete the iron stores of children. In adolescence, there are increased requirements for iron because of rapid growth. The adolescent female has the additional risk of becoming iron deficient from the loss of menstrual blood.
- Grossly positive blood in the stool of this patient requires further evaluation. The Meckel diverticulum results from an embryologic remnant of the vitelline duct. Gastric tissue in the area ulcerates, causing intermittent bleeding. Blood can be microscopic or massive and grossly positive. The bleeding is often painless and is intermittent. The diverticulum can serve as a lead point for intussusception with resultant symptoms of obstruction. The diagnosis is with a Meckel scan (Tc99m pertechnetate nuclear scan), which demonstrates a lesion in the right lower quadrant.
- 3. What is true about Okazaki pieces?
- a. Continuously synthesized
- b. Fragments of RNA
- c. Synthesized by lagging strand
- d. Joined by endonuclease
- Ans: c
- Ref: Satyanarayan Biochemistry, 4th edition, Page No: 525 – 526
- • Okazaki pieces are small fragments of the discontinuously synthesized DNA.
- • These are produced on the lagging strand of the parent DNA.
- • Okazaki pieces are later joined to form a continuous strand of DNA. DNA polymerase I and DNA ligase are responsible for this process.
- 4. Neisseria gonorrhoeae attaches to host cells using which of the following?
- a. IgA1 protease
- b. Lipooligosaccharide
- c. Outer membrane protein
- d. Pili
- Ans: d
- Ref: Jawetz Microbiology, 27th edition, page 282
- • Pili (fimbriae) are hairlike appendages that extend up to several micrometers from the gonococcal surface. They enhance attachment to host cells and resist phagocytosis.
- • The pilins of almost all strains of N gonorrhoeae are antigenically different, and a single strain can make many antigentically distinct forms of pili.
- 5. Lancefield grouping of streptococci is done by using:
- a. M protein
- b. Group C carbohydrate antigen in cell wall
- c. Group C peptidoglycan cell wall
- d. M antigen
- Ans: b
- Ref: Surinder Microbiology; 1st edition; Page No: 241
- • The work of Rebecca Lancefield in 1933 laid the groundwork for the serological classification of B hemolytic streptococci. On the basis of group-specific carbohydrate (C) antigens in the cell wall, B hemolytic streptococci are divided into 21 serological groups from A to W (without I and J). These are known as Lancefield groups. Groups A, B, C, D, and G are most commonly found associated with human infections.
- Griffith Typing
- • Hemolytic streptococci of group A are known as Str. pyogenes. These are further divided into types based on the protein (M, T and R) antigens present on the cell surface (Griffith typing). About eighty types of Str. pyogenes have been recognized so far (types 1, 2, 3 and so on).
- 6. Which of the following helps in entry of SARS-CoV-2 entry into the cells?
- a. Transmembrane protease, serine 1
- b. Transmembrane protease, serine 2
- c. Transmembrane protease, thymine 1
- d. Transmembrane protease, thymine 2
- Ans: a
- • Each SARS-CoV-2 virion is approximately 50–200 nanometres in diameter.
- • Like other coronaviruses, SARS-CoV-2 has four structural proteins, known as the S (spike), E (envelope), M (membrane), and N (nucleocapsid) proteins; the N protein holds the RNA genome, and the S, E, and M proteins together create the viral envelope.
- • The spike protein, which has been imaged at the atomic level using cryogenic electron microscopy, is the protein responsible for allowing the virus to attach to and fuse with the membrane of a host cell.
- • Protein modeling experiments on the spike protein of the virus soon suggested that SARS-CoV-2 has sufficient affinity to the receptor angiotensin converting enzyme 2 (ACE2) on human cells to use them as a mechanism of cell entry.
- • Initial spike protein priming by transmembrane protease, serine 2 (TMPRSS2) is essential for entry of SARS-CoV-2. After a SARS-CoV-2 virion attaches to a target cell, the cell's protease TMPRSS2 cuts open the spike protein of the virus, exposing a fusion peptide. The virion then releases RNA into the cell and forces the cell to produce and disseminate copies of the virus, which infect more cells.
- • SARS-CoV-2 produces at least three virulence factors that promote shedding of new virions from host cells and inhibit immune response.
-
103
- 1. Which of the following is not microbial sensor?
- a. TLRs
- b. NOD-like receptors (NLRs)
- c. RIG-1 like helicases and MDA-5
- d. Transcriptase
- Ans: d
- Ref: Jawetz Microbiology 26th Edition, Page No: 124
- When a pathogen enters the skin, it is confronted with macrophages and other phagocytic cells possessing “microbial sensors.”
- There are three major groups of microbial sensors: (1) TLRs, (2) NOD-like receptors (NLRs), and (3) RIG-1 like helicases and MDA-5. The best studied of the microbial sensors are the TLR.
- 2. IL-12 is secreted by:
- a. Dendritic cells
- b. Macrophages
- c. B cells
- d. All of the above
- Ans: d
- Ref: Jawetz Microbiology 26th Edition, Page No: 130
- Cytokines, Primary cells and their Activity
- • IL-1: Most cells, macrophages, dendritic cells; Induces inflammation, fever and sepsis, activate TNF-α
- • IL-2: T cells; Induces proliferation and maturation of T cells
- • IL-6: Most cells; B cell stimulation, mediator of acute phase reactions
- • IL-10: T cells, monocytes/macrophages; Inhibits IFN-γ and IL-12 production
- • IL-11: Bone marrow stromal cells, mesenchymal cells; Synergistic effects on hematopoiesis and thrombopoiesis, cytoprotective effects on epithelial cells, induces immunosuppression
- • IL-12: Dendritic cells, macrophages, B cells; Induces production of IFN-γ , TNF-α, and IL-2 by resting and activated T and NK cells
- • IL-15: T cells, atrocytes, microglia, fibroblasts, epithelial cells; Biological activities similar to IL-2, induces proliferation of peripheral blood, mononuclear cells, maturation of NK cells (IL-1, IFN-γ, TNF-α)
- • IL-17 (6 members) (IL-17 A-F Th17 cells; Stimulates epithelial, endothelial, and fibroblastic cells to produce IL-6, IL-8, G-CSF, and ICAM-1
- • IL-23: Macrophages, dendritic cells; Similar to IL-12 (induces IFN-γ) helps to differentiate CD4 T cells to TH17
- 3. Replication of coronavirus occurs in:
- a. Cytoplasm
- b. Nucleus
- c. Cell membrane
- d. Mitochondria
- Ans: a
- Ref: Jawetz Microbiology 26th Edition, Page No: 613
- Properties of Coronaviruses
- • Virion: Spherical, 120–160 nm in diameter, helical nucleocapsid genome: Single-stranded RNA, linear, nonsegmented, positive-sense, 27–32 kb, capped and polyadenylated, infectious
- • Proteins: Two glycoproteins and one phosphoprotein. Some viruses contain a third glycoprotein (hemagglutinin esterase) envelope: Contains large, widely spaced, club- or petal-shaped spikes
- • Replication: Cytoplasm; particles mature by budding into endoplasmic reticulum and Golgi
- • Outstanding characteristics: Cause colds and SARS; Display high frequency of recombination; Difficult to grow in cell culture
- 4. The flow of electrons through the respiratory chain and the production of ATP are normally tightly coupled. The processes are uncoupled by which of the following?
- a. Cyanide
- b. Oligomycin
- c. Thermogenin
- d. Carbon monoxide
- Ans: c
- Ref: Harper Biochemistry 30th Edition, Page No: 264
- A thermogenic uncoupling protein, thermogenin, acts as a proton conductance pathway dissipating the electrochemical potential across the mitochondrial membrane.
- 5. In laryngeal trauma, which of the following structure is commonly involved?
- a. Thyroid cartilage
- b. Arytenoid cartilage
- c. Vocal cord
- d. Cricoid cartilage
- Ans: a
- Ref: Jacob Mandell; Visual Approach to Diagnostic Imaging; Core Radiology Page No: 289
- Blunt trauma to the neck may compress the larynx against the cervical spine. The thyroid cartilage and cricoid cartilage are susceptible to fracture, which may be difficult to detect in young patients with incomplete ossification of these structures.
- Laryngeal trauma may also result from intubation, which can cause arytenoid dislocation.
-
104
- 1. A change in the third nucleotide of a codon is likely to produce which type of mutation?
- a. Frameshift mutation
- b. Missense mutation
- c. Nonsense mutation
- d. Silent mutation
- Ans: d
- Ref: Harper’s Biochemistry, 30th edition, page 416 – 417
- Single base change mutation in the mRNA will have the following effects:
- ☆ Silent mutation: There may be no detectable effect because of the degeneracy of the code. This would be most likely if the changed base in mRNA molecule were to be at the third nucleotide of a codon. Because of wobble, the translation of a codon is least sensitive to a change at the third position.
- ☆A missense effect : will occur when a different amino acid is incorporated at the corresponding site in the protein molecule. This mistaken amino-acid – or missense, depending upon its location in the specific protein – might be acceptable, partially acceptable, or unacceptable to the function of that protein molecule.
- ☆A nonsense codon : may appear that would then result in the premature termination of translation and the production of only a fragment of the intended protein molecule. The probability is high that a prematurely terminated protein molecule or peptide fragment will not function in its normal role.
☆Frameshift mutations : result from deletion or insertion of nucleotides in DNA that generates altered mRNAs. The deletion of a single nucleotide from the coding strand of a gene results in an altered reading frame in the mRNA.
- 2. PFK -1 inhibitor:
- a. AMP
- b. Citrate
- c. ATP
- d. Both b and c
- Ans: d
- Ref: Harper biochemistry; 30th edition; Page No: 189
- Phosphofructokinase (phosphofructokinase-1) occupies a key position in regulating glycolysis and is also subject to feedback control. It is inhibited by citrate and by normal intracellular concentrations of ATP and is activated by 5′ AMP. At the normal intracellular [ATP] the enzyme is about 90% inhibited; this inhibition is reversed by 5′AMP.
- 3. Which of the following is not a step in PCR?
- a. Annealing
- b. Denaturation
- c. Transformation
- d. Synthesis
- Ans: c
- Ref: Satyanarayan biochemistry; 4th edition; Page No: 594
- The actual technique of PCR involves repeated cycles for amplification of target DNA.
- Each cycle has three stages.
- 1. Denaturation: On raising the temperature to about 95°C for about one minute, the DNA gets denatured and the two strands separate.
- 2. Renaturation or annealing: As the temperature of the mixture is slowly cooled to about 55°C, the primers base pair with the complementary regions flanking target DNA strands. This process is called renaturation or annealing. High concentration of primer ensures annealing between each DNA strand and the primer rather than the two strands of DNA.
- 3. Synthesis: The initiation of DNA synthesis occurs at 3c-hydroxyl end of each primer. The primers are extended by joining the bases complementary to DNA strands. The synthetic process in PCR is quite comparable to the DNA replication of the leading strand. However, the temperature has to be kept optimal as required by the enzyme DNA polymerase. For Taq DNA polymerase, the optimum temperature is around 75°C (for E. coli DNA polymerase, it is around 37°C). The reaction can be stopped by raising the temperature (to about 95°C).Each cycle of PCR takes about 3-5 minutes. In the normal practice, the PCR is carried out in an automated machine.
- 4. A 7-year-old boy has fallen on an outstretched hand and complains of a painful right elbow. When reviewing the radiographs for evidence of bony injury, which one of the following statements is true?
- a. A posterior fat pad may be a normal finding on a flexed lateral view.
- b. An anterior fat pad is always abnormal.
- c. The line from the anterior cortex of the humerus should pass through the anterior third of the capitellum.
- d. The radiocapitellar line should intersect on all views.
- Ans: d
- Ref: Grainger and Allison's Radiology, 5th Edition, Page No: 1614
- If a line does not intersect the capitellum on all views, a dislocation should be suspected.
- Plain radiograph in supracondylar humeral fracture:
- Lateral and AP radiographs are usually sufficient, and in many instances demonstrate an obvious fracture. Often, however, no fracture line can be identified. In such cases assessing for indirect signs is essential:
- • Anterior fat pad sign (sail sign : the anterior fat pad is elevated by a joint effusion and appears as a lucent triangle on the lateral projection
- • Posterior fat pad sign
- • Anterior humeral line should intersect the middle third of the capitellum in most children although, in children under 4, the anterior humeral line may pass through the anterior third without injury.
- 5. Which is a form of cold sterilization?
- a. Infrared rays
- b. Gamma rays
- c. Beta rays
- d. Autoclave
- Ans: b
- Ref: Surinder Singh microbiology;1st edition; Page No:46
- Ionizing Radiation
- These include X-rays, γ (gamma) rays and cosmic rays. These have very high penetrative power and are highly lethal to all cells including bacteria. Ionizing radiations damage the DNA by various mechanisms and include structural defects in microbial DNA synthesis, leading to cell death. Bacterial spores are generally more resistant than vegetative cells, and spores are among the most radiation resistant microorganisms known.
- Applications
- i. For sterilization in pharmacy and medicine.
ii. Sterilization of packaged disposable articles: Such as plastic syringes, intravenous lines, catheters and gloves that are unable to withstand heat.
- Cold Sterilization
- Since there is no appreciable increase in temperature in this method it is known as cold sterilization. Large commercial plants use gamma radiation emitted from a radioactive element, usually cobalt 60 for this type of sterilization.
iii. Use for antibiotics, hormones, sutures, and vaccines and to prevent food spoilage.
-
105
- 1. Coronavirus genome is:
- a. Segmented genome of positive-sense, single stranded RNA
- b. Unsegmented genome of positive-sense, single stranded RNA
- c. Segmented genome of negative-sense, single stranded RNA
- d. Unsegmented genome of negative-sense, single stranded RNA
- Ans: b
- Ref: Jawetz Microbiology 26th Edition, Page No: 412, 413
- Coronaviruses are enveloped 120- to 160-nm particles containing an unsegmented genome of positive-sense, single stranded RNA, 27–32 kb in size; the nucleocapsid is helical, 9–11 nm in diameter. Coronaviruses resemble orthomyxoviruses but have petal-shaped surface projections arranged in a fringe, similar to a solar corona.
- Coronavirus nucleocapsids develop in the cytoplasm and mature by budding into cytoplasmic vesicles. These viruses have narrow host ranges.
- 2. What is the source of secretion of erythropoietin?
- a. Bone marrow
- b. Liver
- c. Lungs
- d. Kidneys
- Ans: d
- Ref: Guyton & Hall Physiology, 13th edition, page 324
- The kidneys secrete erythropoietin, which stimulates the production of red blood cells by hematopoietic stem cells in the bone marrow. One important stimulus for erythropoietin secretion by the kidneys is hypoxia. The kidneys normally account for almost all the erythropoietin secreted into the circulation. In people with severe kidney disease or who have had their kidneys removed and have been placed on hemodialysis, severe anemia develops as a result of decreased erythropoietin production.
- 3. What is the commonest type of choanal atresia?
- a. Unilateral bony
- b. Unilateral membranous
- c. Bilateral bony
- d. Bilateral membranous
- Ans: a
- Ref: PL Dhingra and Shruti Dhingra ENT and HNS 7th Edition, Page No: 183
- Choanal atresia occurs due to persistence of bucconasal membrane and may be unilateral or bilateral, complete or incomplete, bony (90%) or membranous (10%). Unilateral atresia is more common and may remain undiagnosed until adult life. Bilateral atresia presents with respiratory obstruction as the newborn.
- 4. Receptors used by Rhinovirus is:
- a. Integrins
- b. CD21
- c. CXCR4
- d. ICAM-1
- Ans: d
- Ref: USMLE Step 1 First Aid 2020, Page No: 166
- Receptors used by viruses:
- CMV: Integrins (heparan sulfate)
- EBV: CD21
- HIV: CD4, CXCR4, CCR5
- Parvovirus B19: P antigen on RBCs
- Rabies: Nicotinic AChR
- Rhinovirus: ICAM-1
- 5. An immunologically mediated adverse effect producing a known set of symptoms & signs which are not related to the mechanism of action of the specific drug is known as:
- a. Intolerance
- b. Idiosyncrasy
- c. Hypersensitivity
- d. Toxicity
- Ans: c
- Ref: KDT Pharmacology 8th Edition, Page 94 - 96
- • Idiosyncrasy refers to genetically determined abnormal reactivity to a chemical.
- • Intolerance is the appearance of characteristic toxic effects of a drug in an individual at therapeutic doses.
- • Hypersensitivity is an immunologically mediated reaction producing stereotype symptoms which are unrelated to the pharmacodynamic profile of the drug.
- • Toxicity are the results of excessive pharmacological action of the drug due to overdosage or prolonged usage.
- 6. Carman’s meniscus sign is diagnostic of:
- a. Peptic ulcer
- b. Cholecystitis
- c. Meconium ileus
- d. Carcinoma of stomach
- Ans: d
- Ref: Jacob Mandell; Visual Approach to Diagnostic Imaging; Core Radiology Page No: 133
- Benign gastric ulcer
- Although less commonly encountered in the modern era of proton pump inhibitors and Helicobacter pylori treatment, benign gastric ulcers tend to have typical imaging findings:
- • Radiating gastric folds are smooth and symmetric.
- • Ulcer extends beyond the normal contour of the gastric lumen.
- • The Hampton line represents nonulcerated acid-resistant mucosa surrounding the ulcer crater.
- • Most benign ulcers occur along the lesser curvature of the stomach, although benign ulcers associated with aspirin ingestion can occur in the greater curvature and antrum, which are dependent locations.
- Gastric carcinoma
- Gastric carcinoma may present with malignant ulceration, which can usually be distinguished from a benign ulcer by the following features:
- • Asymmetric ulcer crater, with surrounding nodular tissue.
- • Abrupt transition between normal gastric wall and surrounding tissue.
- • Ulcer crater does not project beyond the expected location of gastric wall.
- • The Carman meniscus sign is considered pathognomonic for tumor. It describes the splaying open of a large, flat malignant ulcer when compression is applied.
-
106
- 1. Which of the following is ethically correct?
- a. A physician should not enter into any emotional or sexual relationship with patient
- b. A physician should not enter into any emotional or sexual relationship with patient but can do with member of patient's family at personal level
- c. A physician can enter into any emotional or sexual relationship with patient, if they have affair
- d. There is no ethical recommendation if the relationship is private
- Ans: a
- A physician should not enter into any emotional or sexual relationship with patient or a member of patient's family, which may disrupt the patient's life, damage, or distress the patient or his or her family.
- 2. The genetic testing in an asymptomatic child with positive family history of Huntington chorea is called:
- a. Predisposional testing
- b. Predictive testing
- c. Diagnostic testing
- d. Pharmacogenetic testing
- Ans: b
- Ref: Nelson pediatrics, 20th Edition, Page No: 581, 582
- Predictive testing
- Predictive genetic testing involves performing a test in a person who is at risk for developing a genetic disorder (presymptomatic), usually on the basis of family history, yet who does not manifest signs or symptoms. This is usually done for disorders that display age-dependent penetrance; the likelihood of manifesting signs and symptoms increases with age, as in cancer or Huntington disease.
- Predispositional testing
- It is expected that genetic tests will become available that will predict risk of disease. Common disorders are multifactorial in etiology; there may be many different genes that contribute to risk of any specific condition. Most of the genetic variants that have been found to correlate with risk of a common disease add small increments of relative risk, probably in most cases too little to guide management.
- Pharmacogenetic testing
- Polymorphisms in drug metabolism genes can result in distinctive patterns of drug absorption, metabolism, excretion, or effectiveness. Knowledge of individual genotypes will guide pharmacologic therapy, allowing customization of choice of drug and dosage to avoid toxicity and provide a therapeutic response. An example of this is testing for polymorphisms within the methylenetetrahydrofolate reductase (MTHFR) gene for susceptibility of potentially increased toxicity to methotrexate antimetabolite therapy for treatment of acute lymphoblastic leukemia.
- 3. The combination of trimethoprim and sulfamethoxazole is effective against which of the following opportunistic infections in the AIDS patient?
- a. Disseminated Herpes simplex
- b. Cryptococcal meningitis
- c. Pneumocystis jiroveci
- d. Tuberculosis
- Ans: c
- Ref: KDT Pharmacology 7th Edition, Page No: 706,707
- COTRIMOXAZOLE
- • The fixed dose combination of trimethoprim and sulfamethoxazole is called cotrimoxazole.
- • Trimethoprim is a diaminopyrimidine related to the antimalarial drug pyrimethamine which selectively inhibits bacterial dihydrofolate reductase (DHFRase).
- • Antibacterial spectra of trimethoprim and sulfonamides overlap considerably.
- • Additional organisms covered by the combination are—Salmonella typhi, Serratia, Klebsiella, Enterobacter, Yersinia enterocolitica, Pneumocystis jiroveci and many sulfonamide-resistant strains of Staph. aureus, Strep. pyogenes, Shigella, enteropathogenic E. coli, H.influenzae, gonococci and meningococci.
- 4. Tract present in middle cerebellar peduncle is:
- a. Dentato-thalamic
- b. Posterior spinocerebellar
- c. Olivocerebellar
- d. Pontocerebellar
- Ans: d
- Ref: USMLE Step 1 First Aid 2020, Page No: 499
- Input in cerebellum:
- Contralateral cortex via middle cerebellar peduncle
- Ipsilateral proprioceptive information via inferior cerebellar peduncle from spinal cord
- Middle cerebellar peduncle has incoming (afferent) fibers from the contralateral pons (ponto-cerebellar) fibers.
- Dentato-thalamic fibers pass through superior cerebellar peduncle from the dentate nucleus to synapse in thalamus.
- Posterior spino-cerebellar and the olivo cerebellar fibers pass through the inferior cerebellar peduncle.
- 5. Forgarty’s catheter is used for:
- a. Embolectomy
- b. Angiography
- c. Drainage
- d. Suprapubic catheterization
- Ans: a
- Ref: Bailey & Love’s Surgery; 27th edition; Pg 955
- Embolectomy
- Local or general anaesthesia may be used. The artery (usually the femoral), bulging with clot, is exposed and held in silastic vessel loops. Through a transverse incision the clot begins to extrude and is removed, together with the embolus, with the help of a Fogarty balloon catheter. The catheter, with its balloon tip, is introduced both proximally and distally until it is deemed to have passed the limit of the clot. The balloon is inflated and the catheter withdrawn slowly,together with any obstructing material. The procedure is repeated until bleeding occurs. An angiogram may be performed in the operating theatre at the end of the procedure to ensure that flow to the distal leg has been restored. Postoperatively, heparin therapy is continued until long-term anticoagulation with warfarin is established to reduce the chance of further embolism.
-
107
Which of the following is not a component of Syndrome Z?
- A. Obstructive sleep apnoea
- B. Blood pressure more than 130/85 mm Hg
- C. Fasting triglyceride more than 150mg/dl
- D. LDL more than 100mg/dl
Correct answer: D. LDL more than 100mg/dl
- Explanation:
- Syndrome Z is a combination of obstructive sleep apnoea and metabolic syndrome. Metabolic syndrome is diagnosed if any three of the following conditions are met:
- 1. Fasting plasma glucose level > 100mg/dl
- 2. Blood pressure > 130/85 mm Hg
- 3. Fasting triglyceride level > 150mg/dl
- 4. HDL cholesterol level < 40mg /dl (men) and < 50mg/dl (women)
- 5. Waist circumference > 102cm (men) and > 88cm (women)
-
108
- Which of the following helps in entry of SARS-CoV-2 entry into the cells?
- a. Transmembrane protease, serine 1
- b. Transmembrane protease, serine 2
- c. Transmembrane protease, thymine 1
- d. Transmembrane protease, thymine 2
- Ans: b
- • Each SARS-CoV-2 virion is approximately 50–200 nanometres in diameter.
- • Like other coronaviruses, SARS-CoV-2 has four structural proteins, known as the S (spike), E (envelope), M (membrane), and N (nucleocapsid) proteins; the N protein holds the RNA genome, and the S, E, and M proteins together create the viral envelope.
- • The spike protein, which has been imaged at the atomic level using cryogenic electron microscopy, is the protein responsible for allowing the virus to attach to and fuse with the membrane of a host cell.
- • Protein modeling experiments on the spike protein of the virus soon suggested that SARS-CoV-2 has sufficient affinity to the receptor angiotensin converting enzyme 2 (ACE2) on human cells to use them as a mechanism of cell entry.
- • Initial spike protein priming by transmembrane protease, serine 2 (TMPRSS2) is essential for entry of SARS-CoV-2. After a SARS-CoV-2 virion attaches to a target cell, the cell's protease TMPRSS2 cuts open the spike protein of the virus, exposing a fusion peptide. The virion then releases RNA into the cell and forces the cell to produce and disseminate copies of the virus, which infect more cells.
- • SARS-CoV-2 produces at least three virulence factors that promote shedding of new virions from host cells and inhibit immune response.
-
109
- 1. The most commonly diagnosed multiple sclerosis belongs to following category
- A.Relapsing-remitting MS (RRMS)
- B.Primary progressive MS (PPMS)
- C.Secondary progressive MS (SPMS)
- DProgressive-relapsing MS (PRMS)
Ans is A
Approximately 85% of all MS cases present as RRMS. The categories are based on frequency of clinical relapses, progression, and lesion development, as detected by MRI. RRMS is characterized by recurrent attacks of neurologic deficits referable to different parts of the central nervous system that resolve completely or almost completely over a short period of time with little to no residual deficit. Clinically isolated syndrome (CIS) and benign MS are sometimes counted as subgroups of RRMS. CIS describes a single episode of neurologic symptoms, whereas patients with benign MS achieve almost complete remission between relapses, with little or no accumulated physical disability over decades.
About half of patients diagnosed initially with RRMS will convert to a secondary progressive pattern within 10-15 years. Once patients have SPMS, they may or may not experience relapses but the disease will continually progress, with increasing disability. PPMS is associated with steady functional decline with no relapses; it accounts for 10%-15% of MS cases. Less than 5% of patients with MS have PRMS, in which the disease is progressive, with occasional relapses.
- 2. The duration of uncontrolled asthma required for the diagnosis of Severe asthma is
- A.1 month
- B.3-6 months
- C.6-9 months
- D.9+ months
Ans B
- According to the GINA report, a diagnosis of severe asthma should be made if difficult-to-treat asthma remains after about 3-6 months of management that includes:
- · Providing asthma education
- · Optimizing treatment
- · Treating comorbidities and modifiable risk factors
- · Considering nonbiologic add-on therapy, nonpharmacologic interventions, and a trial of high-dose inhaled corticosteroids
- 3.Which of the following statement is most accurate regarding deep vein thrombosis?
- A.DVT involving vena cava or pelvic veins produces unilateral leg edema rather than bilateral
- B.Pain and tenderness associated with DVT correlates with the size, location, and extent of thrombus
- C.DVT that is present without causing a net venous outflow obstruction is often asymptomatic
- D.DVT can often be diagnosed or excluded based solely on clinical findings
Ans is C
DVT classically produces pain and limb edema; however, in a given patient, symptoms may be present or absent, unilateral or bilateral, or mild or severe. A thrombus that does not cause a net venous outflow obstruction is often asymptomatic. DVT that involves the iliac bifurcation, pelvic veins, or vena cava produces leg edema that is usually bilateral rather than unilateral. High partial obstruction often produces mild bilateral edema that is mistaken for the dependent edema of right-sided heart failure, fluid overload, or hepatic or renal insufficiency.
Leg pain is commonly associated with DVT, but this is entirely nonspecific. Pain can occur on dorsiflexion of the foot (Homan sign). Tenderness occurs in most patients but is also found in many patients without objectively confirmed DVT. When tenderness is present, it is usually confined to the calf muscles or along the course of the deep veins in the medial thigh. Pain and/or tenderness away from these areas is not consistent with venous thrombosis and usually indicates another diagnosis. The pain and tenderness associated with DVT does not usually correlate with the size, location, or extent of the thrombus. DVT cannot be diagnosed or excluded based on clinical findings.
- 4.The most frequent cause of secondary hypertension is
- A.Hyperthyroidism
- B.Hypothyroidism
- C.Obstructive sleep apnea
- D.Aortic coarctation
Ans is C
- Although the vast majority of patients have primary or essential hypertension, secondary hypertension is identified in approximately 10% of adult patients. Common causes of secondary hypertension include the following:
- Renovascular disease
- Obstructive sleep apnea
- Renal parenchymal disease
- Primary aldosteronism
- Drug- or alcohol-induced
- Uncommon causes include the following:
- · Pheochromocytoma/paraganglioma
- · Cushing syndrome
- · Hypothyroidism
- · Hyperthyroidism
- · Aortic coarctation
- · Primary hyperparathyroidism
- · Congenital adrenal hyperplasia
- · Mineralocorticoid excess syndromes (other than primary aldosteronism)
- · Acromegaly
- 5. Which of the following is the cause for orthostatic hypotension in the patient with Sickle cell disease?
- A.Patients with SCD rarely drink any liquids
- B.Heart failure
- C.Inabality of urine concentration urine because of damage to the kidneys
- D.Pulmonary hypertension
- Ans : C
- Orthostatic hypotension can be common in patients with SCD. Patients with SCD may be susceptible to hypovolemia, leading to vaso-occlusive painful crises due to loss of renal concentrating capacity. Patients with SCD are taught early on about the need to drink a lot of fluids, and it is often the first thing a patient will try if they feel like a crisis is coming on.
Heart failure in patients with SCD is often underrecognized and diagnosed at a late congestive stage.
Pulmonary hypertension is associated with high mortality in patients with SCD.
-
110
- 1. A 25 year old boy presents with renal failure. His uncle died of renal failure three years ago. Slit lamp examination reveals Lenticonus / Keratoconus. The likely diagnosis is:
- a. Autosomal dominant polycystic kidney disease (ADPCKD)
- b. Autosomal recessive polycystic kidney disease (ARPCKD)
- c. Alport’s syndrome
- d. Denysh-Drash syndrome
- Ans: c
- Ref: Harrison’s Principles of Internal Medicine; 19th Edition, Page No: 1847
- ALPORT’S SYNDROME
- • Classically, patients with Alport’s syndrome develop hematuria, thinning and splitting of the GBMs, mild proteinuria (<1–2 g/24 h), which appears late in the course, followed by chronic glomerulosclerosis leading to renal failure in association with sensorineural deafness.
- • Some patients develop lenticonus of the anterior lens capsule, “dot and fleck” retinopathy, and rarely, mental retardation or leiomyomatosis.
- 2. Greater superficial petrosal nerve arises from:
- a. Meatal segment
- b. Labyrinthine segment
- c. Horizontal segment
- d. Vertical segment
- Ans: c
- Ref: PL Dhingra and Shruti Dhingra ENT and HNS 6th Edition, Page No: 90
- BRANCHES OF FACIAL NERVE
- • Greater superficial petrosal nerve : It arises from geniculate ganglion and carries secretomotor fibres to lacrimal gland and the glands of nasal mucosa and palate.
- • Nerve to stapedius : It arises at the level of second genu and supplies the stapedius muscle.
- • Chorda tympani : It arises from the middle of vertical segment, passes between the incus and neck of malleus, and leaves the tympanic cavity through petrotympanic fissure. It carries secretomotor fibres to submandibular and sublingual glands and brings taste from anterior two-thirds of tongue.
- 3. What is the duration needed to label a case of diarrhea as chronic?
- a. > 2 weeks
- b. > 4 weeks
- c. > 6 weeks
- d. > 8 weeks
- Ans: B, > 4 weeks
- Ref: Harrison’s Principles of Internal Medicine; 20th Edition, Page No: 261
- • Diarrhea is loosely defined as passage of abnormally liquid or unformed stools at an increased frequency. For adults on a typical Western diet, stool weight >200 g/d can generally be considered diarrheal.
- • Diarrhea may be further defined as acute if <2 weeks, persistent if 2–4 weeks, and chronic if >4 weeks in duration.
- • Two common conditions, usually associated with the passage of stool totaling <200 g/d, must be distinguished from diarrhea, because diagnostic and therapeutic algorithms differ. Pseudodiarrhea, or the frequent passage of small volumes of stool, is often associated with rectal urgency, tenesmus, or a feeling of incomplete evacuation, and accompanies IBS or proctitis.
- • Fecal incontinence is the involuntary discharge of rectal contents and is most often caused by neuromuscular disorders or structural anorectal problems.
- 4. Which of the following is the most common cause of Cushing syndrome in a child is?
- a. Bilateral adrenal hyperplasia
- b. Exogenous administration of glucocorticoids
- c. Functioning adrenocortical tumor
- d. Pituitary adenoma
- Ans: b
- Ref: Nelson pediatrics, 20th Edition, Page No: 2723
- • The most common cause of Cushing syndrome is prolonged exogenous administration of glucocorticoid hormones, especially at the high doses used to treat lymphoproliferative disorders. This rarely represents a diagnostic challenge, but management of hyperglycemia, hypertension, weight gain, linear growth retardation, and osteoporosis often complicates therapy with corticosteroids.
- • Endogenous Cushing syndrome is most often caused in infants by a functioning adrenocortical tumor.
-
111
- 1. Radical mastoidectomy includes all except:
- a. Cochlea removed
- b. Ossicles removed
- c. Closure of the auditory tube
- d. Exteriorisation of mastoid
- Ans : a, Cochlea removed
- Ref: Diseases of Ear Nose and Throat, PL Dhingra, 6th edition, page no.405
- Radical mastoidectomy is a procedure to eradicate disease from middle ear and mastoid without any attempt to reconstruct hearing.
- It is rarely done these days – Its only indications are:
- •Malignancy of middle ear
- •When cholesteatoma cannot be removed safely e.g, if it invades eustachian tube, round window or perilabrynthine cells
- •If previous attempts to eradicate cholesteatoma have failed
- Following steps are done in radical mastoidectomy:
- •Posterior meatal wall is removed and the entire area of middle ear, attic, antrum and mastoid is converted into a single cavity, by removing the bridge and lowering the facial ridge.
- •All remnants of tympanic membrane, ossicles (except the stapes foot plate) and mucoperiosteal lining are removed (Not cochlea)
- •Eustachian tube is obliterated by a piece of muscle or cartilage
- .The diseased middle ear and mastoid are permanently exteriorised for inspection and cleaning.
- Remember: Bridge is the most posterosuperior part of bony meatal wall lateral to aditus and anturm, which overlies the Notch of Rivinus while facial ridge lies lateral to fallopian canal. Bridge is removed and ridge is lowered in radical or modified radical operation.
- 2.A known case of gall bladder stone landed up in emergency with complaints of pain over epigastric region since two days not relieved by proton pump inhibitors. On examination, you noticed bluish discoloration in the inguinal region. This sign is;
- a.Grey Turner’s sign
- b.Fox sign
- c.Cullen’s sign
- d.Hacking sign
- Ans :b Fox sign
- Ref: Short Practice of Surgery, Bailey and Love, 27th edition, page no.1223
- •Bleeding into the fascial planes can produce bluish discoloration of the flanks (Grey Turner’s sign) or umbilicus (Cullen’s sign). Neither sign is pathognomonic of acute pancreatitis; Cullen’s sign was first described in association with rupture of an ectopic pregnancy. Subcutaneous fat necrosis may produce small, red, tender nodules on the skin of the legs.
- •Fox's sign is a clinical sign in which bruising is seen over the inguinal ligament. It occurs in patients with retroperitoneal bleeding, usually due to acute hemorrhagic pancreatitis.
- 3. There are different types of cells in blood. Which of the following is less likely correct percentage regarding number of blood cells in a normal adult?
- a.Neutrophils around 60%
- b.Basophils around 0.5%
- c.Monocytes around 5%
- d.Lymphocytes around 60%
- Ans, D, Lymphocytes around 60%
- •Ref, Textbook of Medical Physiology, Guyton and Hall, 13th edition, page no.455
- •Lymphocytes are around 30% in a normal human body and not 60%
- •Concentrations of the Different White Blood Cells in the Blood.
- •The adult human being has about 7000 WBCs per microliter of blood (in comparison with 5 million red blood cells [RBCs] per microliter).
- •Of the total WBCs, the normal percentages of the different types are approximately the following:
- Polymorphonuclear neutrophils 62.0%
- Polymorphonuclear eosinophils 2.3%
- Polymorphonuclear basophils 0.4%
- Monocytes 5.3%
- Lymphocytes 30.0%
•The number of platelets, which are only cell fragments, in each microliter of blood is normally about 300,000.
- 4. Immediately following delivery uterus weighs about
- a.500gm
- b.1000gm
- c.60gm
- d.100gm
- Ans;b 1000gm
- Ref,dc dutta 8/e,p168,169
- INVOLUTION OF THE UTERUS:
- ANATOMICAL CONSIDERATION Uterus:
- •Immediately following delivery, the uterus becomes firm and retract with alternate hardening and softening.
- •The uterus measures about 20 × 12 × 7.5 cm3 (length, breadth and thickness) and weighs about 1,000 g.
- •At the end of 6 weeks, its measurement is almost similar to that of the nonpregnant state and weighs about 60 g.
- •The decrease in size of the uterus and cervix has been shown with serial MRI.
- •The placental site contracts rapidly presenting a raised surface which measures about 7.5 cm and remains elevated even at 6 weeks when it measures about 1.5 cm.
- Lower uterine segment:
- •Immediately following delivery, the lower segment becomes a thin, flabby and collapsed structure.
- •It takes a few weeks to revert back to the normal shape and size of the isthmus, i.e. the part between the body of the uterus and internal os of the cervix.
- Cervix:
- •The cervix contracts slowly; the external os admits two fingers for a few days but by the end of 1st week, narrows down to admit the tip of a finger only. The contour of the cervix takes a longer time to regain (6 weeks) and the external os never reverts back to the nulliparous state.
-
112
- 1. Most common complication of Shigellosis is:
- a. Hemolytic Uremic Syndrome (HUS)
- b. Reactive arthritis
- c. Pneumonia
- d. Meningitis
- Ans,a. Hemolytic Uremic Syndrome (HUS)
- Harrison 19th ed/pg.905,906
- Common complications:
- • Bacteremia
- • HUS : It occurs with S. dysenteriae type-1 infection. Manifestations of HUS usually develop at the end of first week of shigellosis.Oliguria, marked drop in hematocrit are the first signs & progresses to anuria with renal failure & severe anemia with congestive heart failure.
- Less common complications: Reactive arthritis
- Rare complications: Pneumonia; Meningitis; Vaginitis (in prepubertal girls);Keratoconjunctivitis & “rose spot” rashes.
- 2. Which one of the following act directly on thromboplastin to activate plasminogen:
- a. Streptokinase
- b. Urokinase
- c. Heparin
- d. rt-PA
- Ans,d. rt-PA
- Ref: Harrison’s 18th ed./p.1001
- • Streptokinase is obtained from cultures of Beta hemolytic streptococci. By itself, streptokinase has no PA activity, but when complexed with plasminogen, it can convert other plasminogen molecules to plasmin. It is not fibrin-selective in that the so-called lytic state resulting from its therapeutic use is due to lysis of fibrinogen as well as fibrin.
- • Recombinant tissue-type plasminogen activator (rt-PA) produced by recombinant technology demonstrates in vitro affinity for fibrin with which it forms a ternary complex with plasminogen. Despite reported fibrin-specificity, the lytic state is produced with dosages currently used. Bleeding complications with rt-PA are similar to those of streptokinase or urokinase.
- 3. If the fracture is intra-articular and extends through the epiphysis, physis and metaphysis,which type of Salter Harris classification does it fall under:
- a. Type II
- b. Type III
- c. Type IV
- d. Type V
- Ans,c. type IV
- Ref: Campbell’s 12th ed./p.1366
- Salter Harris classification : Epiphyseal / Physeal injuries
- • Type I: Complete separation of epiphysis from the metaphysis without fracture. Common in rickets, scurvy and osteomyelitis.
- • Type II: The fracture involves physis and a triangle of metaphyseal bone (Thurston Holland sign). This is the commonest type of epiphyseal injury accounting for 73 percent of cases over 10 years of age.
- • Type III: The fracture is intra- articular and extends along the physis and then along the growth plate. This injury is relatively uncommon.
- • Type IV: The fracture is intra-articular and extends through the epiphysis, physis and metaphysis. Perfect reduction is necessary and open reduction is more often necessary to prevent growth arrest.
- • Type V: Crushing of epiphysis. Growth arrest usually follows.
- • Type VI (Rang’s type) : There is a peripheral physis (perichondrial ring) injury.
-
113
- 1. Which of the following is/are Unprofessional action and misconduct?
- a. Using a 34x20 inch size signboard in front of your personal clinic
- b. Letters in signboard are yellow on a black background to make it look prominent
- c. Using Red Cross symbol in vehicle to make known the fact of being a physician
- d. All of the above
- Ans: d
- Unprofessional actions and misconducts
- A physician should use an 18x14 inch size signboard and write his name, NMC number, qualification, titles, and name of his specialty.
- The letters should be in blue on a white background.
- S/he should not use the International Red Cross symbol to make known the fact of being a physician.
- It is improper to affix a signboard on a chemist's shop.
- 2. What is seen in boxers hemorrhage?
- a. Fracture of base of skull
- b. Fractured orbits
- c. Hemorrhagic sinusitis
- d. Pontine hemorrhage
- Ans: d
- Ref: Reddy Forensic Medicine, 33rd Edition, Page No: 287
- A wide range of injuries are produced from boxing, but the head is frequently injured.
- Fracture of skull is rare, but subdural haemorrhage, usually within one of the middle cranial fossae, due to tearing of dural emissary veins, is the most common cause of death due to prolonged repetitive punching, occurs in about one-third of fatal cases.
- Some victims may show pontine haemorrhage, the so-called "boxers haemorrhage".
- Retinal detachment, choroidal tears and vitreous hemorrhages may occur. Deterioration in speed and coordination is the chief symptom of the onset of punchdrunk (traumatic encephalopathy or dementia pugilistica) condition, which may arise years after the last injury. Punchdrunk syndrome in its final form is recognized by slurred speech, defective memory, slow thought process, stiff limbs, ataxia, broad-based gait, outbursts of violence, Parkinsonian0-like facial appearance and dementia.
- The intracranial findings in boxers are: subdural, subarachnoid and intracerebral haemorrhage, diffuse axonal injury, focal ischaemic lesions, dissecting aneurysms, cortical atrophy, slight hydrocephalus. thinning or tears of corpus callosum, scars or patches of gliosis, loss of neurons from the cerebellum and substantia nigra, and contusions of the brain. The changes are broadly similar to that in Alzheimer’s disease.
- Boxer fractures: Minimally comminuted, transverse fractures of the 5th metacarpal and are the most common type of metacarpal fracture. They typically occur (as the name suggests) when punching and are a common sight in all emergency departments on Friday nights.
- Boxer knuckle: Represents a tendinous and ligamentous disruption of the metacarpal phalangeal joint.
- 3. Whom of the following has the most total body water?
- a. Fetus
- b. Infant
- c. Preterm newborn
- d. Term new born
- Ans: a
- Ref: Nelson Pediatrics, 20th edition, page 346
- • Total body water (TBW) as a percentage of body weight varies with age.
- • The fetus has very high TBW, which gradually decreases to approximately 75% of birthweight for a term infant. Premature infants have higher TBW than term infants. During the 1st yr of life, TBW decreases to approximately 60% of body weight and basically remains at this level until puberty.
- • At puberty, the fat content of females increases more than that in males, who acquire more muscle mass than females. Because fat has very low water content and muscle has high water content, by the end of puberty TBW in males remains at 60%, but TBW in females decreases to approximately 50% of body weight.
- 4. What is the chromosomal location of the MEN1 gene?
- a. 11q13
- b. 12q13
- c. 13q11
- d. 13q12
- Ans: A, 11q13
- Ref: Harrison’s Principles of Internal Medicine, 20th edition, page 2649
- • The molecular basis of some endocrine tumors, such as the multiple endocrine neoplasia (MEN) syndromes (MEN1, 2A, 2B), has provided important insights into tumorigenesis.
- • MEN1 is characterized primarily by the triad of parathyroid, pancreatic islet, and pituitary tumors. The MEN1 gene, located on chromosome 11q13, encodes a putative tumor-suppressor gene, menin. Analogous to the paradigm first described for retinoblastoma, the affected individual inherits a mutant copy of the MEN1 gene, and tumorigenesis ensues after a somatic “second hit” leads to loss of function of the normal MEN1 gene (through deletion or point mutations).
- • MEN2 predisposes to medullary thyroid carcinoma, pheochromocytoma, and hyperparathyroidism. In contrast to inactivation of a tumor-suppressor gene, as occurs in MEN1 and most other inherited cancer syndromes, MEN2 is caused by activating mutations in a single allele.
- 5. A 28 year old sexually active lady presented with creamy white vaginal discharge with fishy odor. She has sexual relationship with multiple partners. Drug of choice is:
- a. Doxycycline
- b. Ofloxacin
- c. Metronidazole
- d. Clindamycin
- Ans: c
- Ref: Shaw Gynecology 16th Edition, Page No: 384
- Bacterial vaginosis is termed vaginosis rather than vaginitis, because it is associated with alteration in the normal vaginal flora rather than due to any specific infection.
- Characteristics of vaginal discharge are as follows according to Amsel’s criteria:
- • White, milky, non-viscous discharge adherent to the vaginal wall.
- • pH of the discharge is more than 4.5. (5–7 pH).
- • Fishy odor when mixed with 10% KOH is due to amino-metabolites from various organisms (amine or whiff rest).
- • Presence of clue cells—the epithelial cells have a fuzzy border due to adherence of bacteria.
- • Increased number of Gardnerella vaginalis and other organisms and reduced number of lactobacilli and leucocytes.
- • Gram-negative stain and culture are additional investigations.
- Treatment
- • The 7-day course of metronidazole 500 mg twice daily is effective in 85% cases, whereas a single dose of 2 g cures only 45%. Ampicillin 500 mg or cephalosporin 500 mg bid for 7 days is also effective. Tetracycline 500 mg four times a day, doxycycline 100 mg twice a day and sulphafurazole 1 g four times daily for 10–14 days are the alternative antibiotics.
- • Clindamycin 2% cream locally is effective in 85%. Oral clindamycin 300 mg daily for 7 days is effective. Ornidazole 500 mg vaginal tablet daily for 7 days is also effective.
- • Vaginal tablets avoid first-pass effect in liver seen with oral route.
- • Lacteal is a protein-free acidifying lactate gel which neutralizes the vaginal pH (lactic acid 5% W/V, 0.1% glycogen)—5 mL is applied daily for 7 days.
-
114
- Q.1. You are a first year resident of Thapathali maternity hospital, you got a call from labour room for retention of placenta for one hour. Cleavage of placental separation after birth of baby occurs through which layer?
- A. Superficial compact layer
- B. Deep spongy layer
- C. Nitabuch’s layer
- D. Basal layer
- Ans ,B, Deep spongy layer
- Ref: Ref: Dc Dutta Obstetrics 8th edition page number 122
- Mechanism of separation of placenta;
- Marked retraction reduces effectively the surface area at the placental site to about its half. But as the placenta is inelastic, it cannot keep pace with such an extent of diminution resulting in its buckling . A shearing force is instituted between the placenta and the placental site which brings about its ultimate separation. The plane of separation runs through deep spongy layer of decidua basalis so that a variable thickness of decidua covers the maternal surface of the
- separated placenta.
- There are two ways of separation of placenta
- (1) Central separation (Schultze) : Detachment of placenta from its uterine attachment starts at the center resulting in opening up of few uterine sinuses and accumulation of blood behind the placenta (retro placental hematoma). With increasing contraction, more and more detachment occurs facilitated by weight of the placenta and retro placental blood until whole of the placenta gets detached.
- (2) Marginal separation (Mathews-Duncan) : Separation starts at the margin as it is mostly unsupported. With progressive uterine contraction, more and more areas of the placenta get separated. Marginal separation is found more frequently.
- Q.2 A 24 year old postpartum female complaining of whitish color PV discharge, which was told to be normal by consulting doctor. What is her probable postpartum day?
- A.1-4 days
- B.5-9 days
- C.10-15 days
- D.15-20 days
- Ans C ,10-15 days
- Ref: Dc Dutta Obstetrics 7th edition page number 142
- In this case according to consulting doctor, the discharge is normal. The discharge which she complained of, is Lochia.
- LOCHIA
- for the first fortnight during puerperium. The discharge originates from the uterine body, cervix and vagina.
- Odor and reaction: It has got a peculiar offensive fishy smell. Its reaction is alkaline tending to become acid towards the end.
- Color: Depending upon the variation of the color of the discharge, it is named as: (1) Lochia rubra (red) 1-4 days. (2) Lochia serosa (5-9 days) — the color is yellowish or pink or pale brownish. (3) Lochia alba — (pale white) — 10-15 days.
- Composition: Lochia rubra consists of blood, shreds of fetal membranes and decidua, vernix caseosa, lanugo and meconium.
- Lochia serosa consists of less RBC but more leukocytes, wound exudate, mucus from the cervix and microorganisms (anaerobic streptococci and staphylococci). The presence of bacteria is not pathognomonic unless associated with clinical signs of sepsis.
- Lochia alba contains plenty of decidual cells, leukocytes, mucus, cholestrin crystals, fatty and granular epithelial cells and microorganisms.
- Amount: The average amount of discharge for the first 5–6 days, is estimated to be 250 mL.
- Normal duration: The normal duration may extend up to 3 weeks. The red lochia may persist for longer duration especially in women who get up from the bed for the first time in later period. The discharge may be scanty, especially following premature labors or may be excessive in twin delivery or hydramnios.
- Clinical importance: The character of the lochial discharge gives useful information about the abnormal puerperal state.
- The vulval pads are to be inspected daily to get information:
- • Odor: If malodorous, indicates infection. Retained plug or cotton piece inside the vagina Amount: Scanty or absent — signifies infection or lochiometra. If excessive — indicates infection.
- • Color: Persistence of red color beyond the normal limit signifies sub involution or retained bits of conceptus.
- • Duration: Duration of the lochia alba beyond 3 weeks suggests local genital lesion
- Q.3. A comatose patient with unidentified poison is brought to ER .You are planning to give coma-cocktail, which of the following is NOT the composition of Coma –cocktail?
- A. Glucose
- B. Thiamine
- C. Naloxone
- D. None
- Ans D,None
- Ref: Reddy, The essential of forensic medicine and toxicology 33rd edition, page number 586
- In a suspected poisoning case, when exact poison is not known, and patient is unconscious or in comatose condition, we give the mixture of glucose, thiamine, Naloxone –which is called COMA-COCKTAIL.
- Coma Cocktail: In comatose patients where the identity of poison is not known, 100 mi. 50% glucose, 100 mg. thiamine, and 2 mg. naloxone should be given i.v.
-
115
- 1. All of the following are the derivatives of dorsal mesogastrium except
- a. Lesser omentum
- b. Greater omentum
- c. Gastrophrenic ligament
- d. Gastrosplenic ligament
- Ans, A
- Ref: Human anatomy BD Chaurasia 4th edition Vol 2 Page no. 224
- The abdominal part of the foregut is suspended by mesenteries both ventrally and dorsally. The ventral mesentery of the foregut is called the ventral mesogastrium, and the dorsal mesentery is called the dorsal mesogastrium (gastrium-stomach).
- The fate of the dorsal mesogastrium is as follows.
- a) The greater or caudal part of the dorsal mesogastrium becomes greatly elongated and forms the greater omentum
- b) The spleen develops in relation to the cranial part of the dorsal mesogastrium, and divides it into dorsal and ventral parts. The ventral part forms the gastrosplenic ligament while the dorsal part forms the lienorenal ligament.
- c) The cranial most part of the dorsal mesogastrium forms the gastrophrenic ligament.
- The midgut and hindgut have only a dorsal mesentery, which forms the mesentery of the jejunum and ileum, the mesoappendix, the transverse mesocolon and the sigmoid mesocolon.
- The mesenteries of the duodenum, the ascending colon, the descending colon and the rectum are lost during development.
- 2. Which drug is used for the treatment of chronic hepatitis C infection?
- a. Ledipasvir
- b. Simeprevir
- c. Sofosbuvir
- d. All of the above
- Ans, D
- Ref: Basic and clinical pharmacology Katzung 14th e Page no 885
- (Please See the picture after this explanations)
- 3. All are third generation cephalosporins except
- a. Cefoperazone
- b. Cefotaxime
- c. Cefuroxime
- d. Moxalactam
- Ans, C
- Ref: Basic and clinical pharmacology Katzung 14th e Page no 802,803
- • Third-generation agents include cefoperazone, cefotaxime, ceftazidime, ceftizoxime, ceftriaxone, cefixime, cefpodoxime proxetil, cefdinir, cefditoren pivoxil, ceftibuten, and moxalactam.
- • Cefuroxime is a second generation cephalosporin.
-
116
- 1. Deja vu means [IOM 2018]
- a. Unfamiliarity of familiar situations
- b. Familiarity of unfamiliar situations
- c. Hearing of unfamiliar events
- d. Complete forgetfulness of past events
- Ans: b
- Ref: Kaplan and Sadock Psychiatry 11th Edition, Page No: 683
- Preictal events (auras) in complex partial epilepsy include autonomic sensations (e.g., fullness in the stomach, blushing, and changes in respiration); cognitive sensations (e.g., déjá vu, jamais vu, forced thinking, dreamy states); affective states (e.g., fear, panic, depression, elation); and, classically, automatisms (e.g., lip smacking, rubbing, chewing).
- • déjà entendu: Illusion that what one is hearing one has heard previously. See also paramnesia.
- • déjà pensé: Condition in which a thought never entertained before is incorrectly regarded as a repetition of a previous thought.
- • déjà vu: Illusion of visual recognition in which a new situation is incorrectly regarded as a repetition of a previous experience.
- 2. Commotio cordis is due to: [IOM 2019]
- a. Hypokalemia
- b. Blunt trauma to chest
- c. Hypomagnesemia
- d. Myocardial infarction
- Ans: b
- Ref: Link MS, Mark Estes NA 3rd, Maron BJ, et al. Eligibility and Disqualification Recommendations for Competitive Athletes With Cardiovascular Abnormalities: Task Force 13: Commotio Cordis: A Scientific Statement From the American Heart Association and American College of Cardiology. Circulation. 2015 Dec 1. 132 (22):e339-42.
- Sudden unexpected cardiac death that occurs in young people during sports participation is usually associated with previously diagnosed or undiagnosed structural or primary electrical cardiac abnormalities. Examples of such abnormalities include hypertrophic cardiomyopathy, anomalous origin of a coronary artery, arrhythmogenic right ventricular cardiomyopathy, and primary electrical disorders, such as congenital prolongation of the QTc interval and catecholaminergic, polymorphic ventricular tachycardia (CPVT).
- Sudden death due to ventricular fibrillation may also occur following a blunt, nonpenetrating blow to the chest, specifically the precordial area, in an individual with no underlying cardiac disease. This is termed commotio cordis.
- 3. Platelets act via [IOM 2013]
- a. cGMP
- b. cAMP
- c. IP3
- d. PIP2
- Ans: a
- Ref: cGMP Signaling in Platelets; Platelets, Haemostasis and Inflammation pp 231-252
- Platelets generate and degrade cGMP by the NO-activated soluble guanylate cyclase and several phosphodiesterases (PDE2, PDE3, and PDE5), respectively.
- An increase of the cGMP concentration activates cGMP-dependent protein kinase type I (cGKI), which phosphorylates several platelet proteins.
- 4. Pinna is not supplied by which one of the following nerve? [IOM 2015]
- a. Greater Auricular Nerve
- b. Posterior Occipital Nerve
- c. Glossopharyngeal Nerve
- d. Vagus Nerve
- Ans: c
- Ref: PL Dhingra and Shruti Dhingra ENT and HNS 7th Edition, Page No: 4
- Nerve supply of the pinna
- Greater auricular nerve (C2, 3) supplies most of the medial surface of pinna and only posterior part of the lateral surface.
- Lesser occipital (C2) supplies upper part of medial surface.
- Auriculotemporal (V3) supplies tragus, crus of helix and the adjacent part of the helix.
- Auricular branch of vagus (CN X), also called Arnold’s nerve, supplies the concha and corresponding eminence on the medial surface.
- Facial nerve, which is distributed with fibres of auricular branch of vagus, supplies the concha and retroauricular groove.
- 5. Thickest layer of cornea is [IOM 2009]
- a. Stroma
- b. Descemet’s membrane
- c. Bowman's membrane
- d. Epithelium
- Ans: a
- Ref: AK Khurana Ophthalmology 6th Edition, Page No: 96 | Ref: Parson Ophthalmology 22ndEdition, Page No: 4, 262
- Stroma (substantia propria )
- This layer is about 0.5 mm in thickness and constitutes most of the cornea (90% of total thickness).
- It consists of collagen fibrils (type-I and type-V fibrillae interwined with filaments of type-VI collagen) embedded in hydrated matrix of proteoglycans (chondroitin sulphate and keratan sulphate).
- The sclera is the ‘white’ supporting wall of the eyeball and is continuous with the clear cornea. It is a dense white tissue, thickest in the area around the optic nerve.
- The lens capsule is a thick, collagenous basement membrane which is transparent, is thickest at the anterior pre equatorial region and thinnest at the posterior pole.
-
117
- 1. What is the minimum recommended body weight required for blood donation? [IOM 2017]
- a. 100 pounds
- b. 115 pounds
- c. 110 pounds
- d. 120 pounds
- Ans: a
- Ref: Nepal Red Cross Society. Blood donation: general information. Kathmandu: Nepal Red Cross Society Central Blood Transfusion Service; 2014. pp 44
- Eligibility for blood donation : Nepal Red Cross Society.
- • Age: 18 – 60 years of both male and female can donate blood.
- • Weight: More than 45 kg (99.2 pounds) can donate blood.
- • Hemoglobin: ≥ 12 g/ dl can donate blood
- • Blood pressure: 100/70 – 160/95 mm Hg can donate blood.
- • Person suffering from epilepsy, tuberculosis, diabetes, hypertension, and HIV/AIDS infected person should not donate blood.
- • Menstrual woman, lactating mother and pregnant women should not donate. Unsafe sex and multi sex partner having people should not donate blood. Substance and drug abused people should not donate blood.
WHO reference: 50 kg (110 pounds). WHO mentions the variability In some countries. Donors of whole blood donations should weigh at least 45 kg to donate 350 ml ± 10% .
Better we follow Nepalese guidelines.
- 2. Most common age group for Tinea capitis is: [IOM 2012]
- a. <10 yrs
- b. 10-20 yrs
- c. 20-30 yrs
- d. >30 yrs
- Ans: a
- Ref: Dermatology, Neena Khanna, 4th edition, Page No: 283
- Tinea capitis (tinea of scalp)
- • Age: Invariably a child. Tinea capitis occurs primarily in children and occasionally in other age groups. It is seen most commonly in children aged between 5 and 10 years. Mean age of onset is in patients aged 6.9-8.1 years
- • Discoid patch of partial alopecia from which the hair can easily and painlessly be plucked.
- • Degree of inflammation varies, depending on the strain of fungus, being more when tinea capitis is caused by zoophilic or geophilic species than with anthropophilic species.
- 3. How much percentage of total body weight is constituted by protein? [IOM 2009]
- a. 15%
- b. 20%
- c. 25%
- d. 30%
- Ans: a
- Ref: Assessment of Fluid Status and Body Composition and Control of Fluid Balance With Intermittent Hemodialysis in the Critically Ill Patient; Israel Campos, Peter Kotanko, in Critical Care Nephrology (Third Edition), 2019
- • Body composition can be viewed from five perspectives: atomic, molecular, cellular, tissue, and whole body levels. At the atomic level, six elements form 98% of the body mass: 61% oxygen, 23% carbon, 10% hydrogen, 2.6% nitrogen, and 1.4% calcium; the remaining 2% of the mass consists of 44 other elements.
- • Water, which accounts for about 60% of a 70-kg “reference male” and about 50% of a “reference female,” is the major chemical component of the body and essential for the interior milieu.
- • The total body water (TBW) is distributed between two major compartments, the intracellular volume (ICV) and the extracellular volume (ECV); the latter can be divided into the interstitial compartment, which constitutes the extracellular environment of the cells, and the vascular space.
- • Body fat depends heavily on nutrition and training status, ranging from less than 10% to more than 50%.
- • Protein and minerals account for 15% and 5% of body composition, respectively.
- 4. Median longitudinal arch is chiefly maintained by: [IOM 2011]
- a. Calcaneum
- b. Talus
- c. Medial cuneiform
- d. 1st metatarsal
- Ans: b
- Ref: Moore's Anatomy 7th Edition, Page No: 654
- • The longitudinal arch of the foot is composed of medial and lateral parts. Functionally, both parts act as a unit with the transverse arch of the foot, spreading the weight in all directions.
- • The medial longitudinal arch is higher and more important than the lateral longitudinal arch. The medial longitudinal arch is composed of the calcaneus, talus, navicular, three cuneiforms, and three metatarsals. The talar head is the keystone of the medial longitudinal arch. The tibialis anterior and posterior, via their tendinous attachments, help support the medial longitudinal arch. The fibularis longus tendon, passing from lateral to medial, also helps support this arch.
- • The lateral longitudinal arch is much flatter than the medial part of the arch and rests on the ground during standing. It is made up of the calcaneus, cuboid, and lateral two metatarsals.
- • The transverse arch of the foot runs from side to side. It is formed by the cuboid, cuneiforms, and bases of the metatarsals. The medial and lateral parts of the longitudinal arch serve as pillars for the transverse arch. The tendons of the fibularis longus and tibialis posterior, crossing under the sole of the foot like a stirrup, help maintain the curvature of the transverse arch. The integrity of the bony arches of the foot is maintained by both passive factors and dynamic supports.
-
118
- 1. What is the mechanism of pseudofolliculitis barbae?
- a. Infection
- b. Collection of sebum
- c. Ingrowing of hairs
- d. Obstruction of sweat glands
- Ans, C, Ingrowing of hairs
- Ref. Dermatology, Andrew’s Diseases of skin, 13th edition, page no. 767
- Pseudofolliculitis Barbae
- Pseudofolliculitis barbae (“razor bumps”) consists of hairs that, after appearing at the surface, curve back and pierce the skin as ingrowing hairs.
- This results in inflammatory papules and pustules, which may scar.
- In severe cases, large deforming keloids may result in the beard area.
- Pseudofolliculitis of the beard is seen in more than 50% of black men, who must sometimes give up shaving to alleviate the disorder.
- 2. Which of the following is true about stabbed wounds?
- a. The length of the wound is equal to the width of the weapon
- b. The length of the wound is slightly more than the width of the weapon
- c. The length of the wound is slightly less than the width of the weapon
- d. The length of the wound is equal to the width of the wound
- Ans, C, The length of the wound is slightly less than the width of the weapon
- Ref, The Essentials of Forensic Medicine and Toxicology, Reddy, 33rd edition, page no.210-211
- STAB OR PUNCTURED WOUNDS
- A stab wound is produced when force is delivered along the long axis of a narrow or pointed object, such as knife, dagger, sword, chisel, scissors, nail, needle, spear, screw driver, etc. into the depths of the body.
- Characters:
- (1) Margins: The edges of the wound are clean-cut and inverted. The margins can be everted when the wound is situated over the fatty area, such as portuberant abdomen or gluteal region.
- (2) Length: The length of the wound is slightly less than the width of the weapon up to which it has been driven in, because of stretching of the skin.
- (3) Width: By examining multiple stab wounds in the body, the length and width of the knife blade can be determined approximately.
- (4) Depth: The depth (length of track) is greater than the width and length of the external injury. It is not safe to find out the depth of a stab wound by introducing a probe, because it may disturb a loose clot and may lead to fatal haemorrhage, or cause serious damage and may produce multiple false wound tracks.
- 3. What is the approximate total surface area of respiratory membrane of normal adult human lungs?
- a. 10 sq meters
- b. 50 sq meters
- c. 70 sq meters
- d. 110 sq meters
- Ans, C, 70 sq meters
- Ref, Textbook of Medical Physiology, Guyton and Hall, 13th edition, page no.522
- The different layers of the respiratory membrane are:
- A layer of fluid containing surfactant that lines the alveolus and reduces the surface tension of the alveolar fluid
- The alveolar epithelium, which is composed of thin epithelial cells
- An epithelial basement membrane
- A thin interstitial space between the alveolar epithelium and the capillary membrane
- A capillary basement membrane that in many places fuses with the alveolar epithelial basement membrane
- The capillary endothelial membrane
- Despite the large number of layers, the overall thickness of the respiratory membrane in some areas is as little as 0.2 micrometer and averages about 0.6 micrometer, except where there are cell nuclei.
- From histological studies, it has been estimated that the total surface area of the respiratory membrane is about 70 square meters in the healthy adult human male, which is equivalent to the floor area of a 25 × 30 foot room.
-
119
- 1. In trigger finger the level of tendon sheath constriction is found at the level of:
- a. Middle phalanx
- b. Proximal interphalangeal joint
- c. Proximal phalanx
- d. Metacarpophalangeal joint
- Ans: d
- Ref: Gray’s anatomy, 41st Edition; Page No: 879.e1
- • Trigger finger is a stenosing tenovaginitis that affects the fibrous flexor sheaths of the fingers or thumb within the palm. The affected sheath thickens and entraps the contained tendons, which become constricted at the site of entrapment and bulge distal to it. This produces a distinct nodule in the palm of the hand and the finger now snaps as the tendon nodule passes through the constriction on flexing the finger. The corresponding extensor muscle is insufficiently powerful to extend the affected finger, so the patient does this passively, accompanied by a painful snap. Treatment frequently requires surgical division of the A1 pulley of the flexor sheath to relieve the stricture.
- • The most commonly affected level is at MCP joint but occasionally it may occur at distal joints also. The index finger is affected rarely.
- • De Quervain’s tenovaginitis is a stenosing tenovaginitis of unknown aetiology that occurs in the first dorsal compartment at the level of the radial styloid. It involves the common extensor sheath containing the tendons of abductor pollicis longus and extensor pollicis brevis. There is palpable thickening of the tendon sheath with painful limitation of extension of the thumb. Treatment frequently requires division of the thickened sheath, care being taken to avoid the adjacent superficial radial nerve; division of the sheath produces no functional impairment.
- 2. Atherosclerosis induced peripheral artery diseases primarily involves which artery?
- a. Iliac arteries
- b. Peroneal arteries
- c. Popliteal arteries
- d. Tibial arteries
- Ans: c
- Ref: Harrison’s Principles of Internal Medicine; 20th Edition, Page No: 1923
- • Peripheral artery disease (PAD) is defined as a clinical disorder in which there is a stenosis or occlusion in the aorta or the arteries of the limbs. Atherosclerosis is the leading cause of PAD in patients >40 years old. Other causes include thrombosis, embolism, vasculitis, fibromuscular dysplasia, entrapment, cystic adventitial disease, and trauma. Segmental lesions that cause stenosis or occlusion are usually localized to large and medium-size vessels.
- • The pathology of the lesions includes atherosclerotic plaques with calcium deposition, thinning of the media, patchy destruction of muscle and elastic fibers, fragmentation of the internal elastic lamina, and thrombi composed of platelets and fibrin.
- • The primary sites of involvement are the abdominal aorta and iliac arteries (30% of symptomatic patients), the femoral and popliteal arteries (80–90% of patients), and the more distal vessels, including the tibial and peroneal arteries (40–50% of patients).
- • Atherosclerotic lesions occur preferentially at arterial branch points, which are sites of increased turbulence, altered shear stress, and intimal injury. Involvement of the distal vasculature is most common in elderly individuals and patients with diabetes mellitus.
- 3. Vitamin K is available as all forms except?
- a. Phytonadione
- b. Menaquinone
- c. Phytoquinone
- d. Acetomenapthone
- Ans: c
- Ref: KDT Pharmacology 8th Edition, Page No: 972
- The available forms of Vitamin K are:
- • Phytonadione (K1) (Phylloquinone)
- • Menaquinone (K2)
- • Menadione (K3)
- • Acetomenapthone
- 4. Neurovegetative symptoms best relate to?
- a. Depressive disorder
- b. Bipolar disorder
- c. Posttraumatic stress disorder (PTSD)
- d. Schizophrenia
- Ans: a
- Ref: Harrison’s Principles of Internal Medicine; 19th Edition, Page No: 2715
- Neuroendocrine abnormalities that reflect the neurovegetative signs and symptoms of depression include:
- • increased cortisol and corticotropin-releasing hormone (CRH) secretion,
- • an increase in adrenal size,
- • a decreased inhibitory response of glucocorticoids to dexamethasone, and
- • a blunted response of thyroid-stimulating hormone (TSH) level to infusion of thyroid-releasing hormone (TRH).
- Antidepressant treatment leads to normalization of these abnormalities.
- Major depression is also associated with changes in levels of Proinflammatory cytokines and neurotrophins.
- 5. Boxer’s fracture is?
- a. Radial styloid fracture
- b. Reverse Colles fracture
- c. 5th metacarpal fracture
- d. 1st metacarpal fracture
- Ans: c
- Ref: Maheshwari Orthopedics 5th Edition Page No: 118
- • Monteggia’s fracture:- fracture of proximal third of ulna with radial head dislocation
- • Galeazzi fracture:- fracture of the distal third of radius with dislocation of distal radio-ulnar joint
- • Colles fracture:- fracture of distal end of radius with dorsal tilt
- • Smith’s fracture:- reverse Colles fracture
- • Boxers fracture:- fracture through neck of 5th metacarpal
-
120
- 1. Piezoelectricity technique is used in:
- a. Ultrasound
- b. MRI
- c. CT Scan
- d. PET Scan
Ans, A, Ultrasound
Ref, Diagnostic Ultrasound Rumack e5/ p7
- Explanation
- Ultrasound transducers use piezoelectricity, a principle discovered by Pierre and Jacques Curie in 1880. Piezoelectric materials have the unique ability to respond to the action of an electric field by changing shape. They also have the property of generating electric potentials when compressed. Changing the polarity of a voltage applied to the transducer changes the thickness of the transducer, which expands and contracts as the polarity changes. This results in the generation of mechanical pressure waves that can be transmitted into the body. The piezoelectric effect also results in the generation of small potentials across the transducer when the transducer is struck by returning echoes.
2. Artificial surfactant is most likely to be used in the treatment of an infant with
- a. Hemolytic disease of the newborn
- b. Hyaline membrane disease
- c. Physiologic jaundice of the newborn
- d. Retrolental fibroplasia
Ans, B , Hyaline membrane disease
Ref, Robbins & Cotran, Pathologic basis of disease, 7e, p 471-473
Explanation
A disorder in newborns that is histologically identical to ARDS is called neonatal respiratory distress syndrome [hyaline membrane disease (HMD)]. HMD, which accounts for 20% of all deaths in the first 28 days of life, is basically a disease of premature infants; most affected infants weigh 1000 to 1500 g. Contributing factors in the development of HMD include diabetes in the mother (maternal diabetes with increased glucose causes increased fetal secretion of insulin, which inhibits the effects of steroids such as lung maturation and production of surfactant) and cesarean section. Infants who develop HMD appear normal at birth, but within minutes to hours their respirations become labored. Grossly the lungs are a mottled, red-purple color, while microscopically there are hyaline membranes in air spaces, similar to those of ARDS.
3. Disease elimination means:
- a. Cure of the disease
- b. Preventing the transmission totally
- c. Eradication of the vector
- d. Complete termination of infective organism
Ans, B, Preventing the transmission totally
Ref, Park’s Textbook of Preventive and Social Medicine 22nd edition, p38
- Explanation
- • Concepts of control of disease:
- – Disease control: Is reducing the transmission of disease agent to such a low level that it ceases to be a public health problem; it aims at reducing,
- 1. Incidence of the disease
- 2. Duration of the disease
- 3. Effects of infection
- 4. Financial burden to the community
- – Disease elimination: Is complete interruption of transmission of disease in a defined geographical area, but the causative organism may be persisting somewhere
- Disease elimination is a ‘geographical term’, i.e. can be used only for a country or a region
- – Disease eradication: Is complete ‘extermination’ of organism
- 1. Is ‘tearing out by roots’ of a disease
- 2. Exhibits ‘All or none phenomenon’
- 3. Disease eradication is a ‘global term’, i.e. can be used only for whole planet
- 4. World has eradicated ONLY 1 disease till date: Small pox (declared eradicated on 8 May, 1980)
- 5. 3 next target diseases for eradication, globally:
- i. Poliomyelitis
- ii. Measles
- iii. Guinea worm
-
121
- Q1.Condition in which corneal scar results in slight opacification allowing the details of the iris to be seen through the opacity is known as:
- a.Macula
- b.Leucoma
- c.Nebula
- d.Adherent leucoma
- Ans: c
- Ref: Parsons diseases of eye 22nd edition page no: 213
- Corneal Opacity
- • Nebula or a nebular corneal opacity: If the corneal scar results in slight opacification allowing the details of the iris to be seen through the opacity.
- • Macula or a macular corneal opacity: If rather more dense, through which the details of the iris cannot be seen but the iris and pupillary margins are visible.
- • Leucoma or a leucomatous corneal opacity: If very dense and white and totally opaque obscuring the view of the iris and pupil.Old central leucomata sometimes show a horizontal pigmented line in the palpebral aperture, the nature of which is obscure but may be due to deposition of iron from the pre-corneal tear film.
- • Adherent leucoma: If the iris is adherent to the back of a leucoma following healing of a perforated corneal ulcer.
- Q2.Which of the following stages of Ancylostoma duodenale is infective to human beings ?
- a. Rhabditiform larva
- b. Filariform larva
- c. Eggs
- d. Adult worm
- Ans: b
- Ref:Panikers textbook of medical parasitology 8th edition pg203
- Infective Form
- Third-stage filariform larva.
- • Adult worm inhabiting the small intestine of man attach themselves to the mucous membrane by means of their mouth parts. The female worm lays eggs.
- • The eggs containing segmented ova with four blastomeres, are passed out in the feces of infected person. Eggs freshly passed in feces are not infective for humans.
- • When deposited in the soil, the embryo develops inside the eggs. Its development takes place optimally in sandy loamy soil with decaying vegetation under a moist, warm, shady environment.
- • In about 2 days, a rhabditiform larva, measuring 250 mcm in length hatches out of the egg. It feeds on bacteria and other organic matter in the soil and grows in size .
- • It moults twice, on the 3rd and 5th days after hatching to become the third-stage infective filariform larva .
- • Filariform larva is about 500-600 µm long, with a sharp pointed tail. The filariform larva is nonfeeding. They can live in the soil for 5-6 weeks, with their heads waving in the air, waiting for their hosts. They can also ascend on blades of grass or other vegetation, being carried in capillary water films on their surface. Direct sun light, drying, or salt water can kill the larva.
- Q3.The greatest risk factor for Necrotizing enterocolitis is with:
- a. Low birth weight
- b. Intestinal ischemia
- c. Enteral nutrition
- d. Prematurity
- Ans: d
- Ref: Nelson’s textbook of pediatrics 20th edition page no:869
- • Although NEC is a multifactorial disease primarily associated with intestinal immaturity, the concept of “risk factors” for NEC is controversial.
- • The triad of intestinal ischemia (injury), enteral nutrition (metabolic substrate), and bacterial translocation has classically been linked to NEC.
- • The greatest risk factor for NEC is prematurity. The disorder probably results from an interaction between loss of mucosal integrity due to a variety of factors (ischemia, infection, inflammation) and the host’s response to that injury (circulatory, immunologic, inflammatory), leading to necrosis of the affected area.
- • Coagulation necrosis is the characteristic histologic finding in intestinal specimens.
-
122
- 1. MRP 2 associated with which of the following?
- a. Rotor syndrome
- b. Dubin-Johnson syndrome
- c. Crigler-Najjar syndrome
- d. Gilbert syndrome
- Ans: b
- Ref: Nelson pediatrics, 20th Edition, Page No: 1938
- Dubin-Johnson Syndrome
- • Dubin-Johnson syndrome is an autosomal recessive inherited defect with variable penetrance in hepatocyte secretion of bilirubin glucuronide.
- • The defect in hepatic excretory function is not limited to conjugated bilirubin excretion but also involves several organic anions normally excreted from the liver cell into bile.
- • Absent function of multiple drug-resistant protein 2 (MRP2), an adenosine triphosphate–dependent canalicular transporter, is the responsible defect.
- • More than 10 different mutations, including compound heterozygous mutation in the CMOAT gene, have been identified and either affect localization of MRP2 with resultant increased degradation or impair MRP2 transport activity in the canalicular membrane.
- • Bile acid excretion and serum bile acid levels are normal. Total urinary coproporphyrin excretion is normal in quantity but coproporphyrin I excretion increases to approximately 80% with a concomitant decrease in coproporphyrin III excretion.
- 2. Linear deposition of immunocomplex is seen in examination of lung or renal tissue by immunofluorescent techniques in?
- a. Systemic lupus erythematous
- b. Goodpasture syndrome
- c. Henoch Schonlein purpura
- d. Wegner granulomatosis
- Ans: b
- Ref: Harrison’s Principles of Internal Medicine; 19th Edition, Page No: 1715
- Goodpasture Syndrome
- • Clinical manifestations Pulmonary hemorrhage and glomerulonephritis are features in most patients with this disease.
- • Autoantibodies to renal glomerular and lung alveolar basement membranes are present.
- • This syndrome can present and recur as DAH without an associated glomerulonephritis. In such cases, circulating anti-basement membrane antibody is often absent, and the only way to establish the diagnosis is by demonstrating linear immunofluorescence in lung tissue.
- • Histologic findings: The underlying histology may be bland hemorrhage or DAH associated with capillaritis.
- • Treatment Plasmapheresis has been recommended as adjunctive treatment.
- 3. Fuch’s spots are seen in:
- a. Fuch’s dystrophy
- b. High myopia
- c. Hypertensive retinopathy
- d. Best disease
- Ans: b
- Ref: Kanski Ophthalmology 8th Edition, Page No: 632
- A Fuchs spot: is a raised, circular, pigmented lesion at the macula developing after a subretinal hemorrhage has absorbed. Seen in high myopia.
- 4. In an in-vitro fertilization, embryo transfer should be done how many times?
- a. 3
- b. 4
- c. 5
- d. no limit
- Ans: a
- Ref: Dutta Gynecology 6th Edition, Page No: 253, 251
- Embryo transfer
- The fertilized ova at the 6–8 blastomere stage are placed into the uterine cavity close to the fundus about 3 days after fertilization through a fine flexible soft catheter transcervically.
- Not more than three embryos are transferred per cycle to minimize multiple pregnancy.
- Principal steps of an Assisted Reproductive Technology (ART) cycle:
- Down regulation using GnRH agonist.
- Controlled ovarian hyperstimulation (COH).
- Monitoring of follicular growth.
- Oocyte retrieval.
- Fertilization in vitro (IVF, ICSI, GIFT).
- Transfer of gametes or embryos.
- Luteal support with Progesterone
-
123
- Nepal has long term goal to end tuberculosis epidemic by:
- a. 2025
- b. 2045
- c. 2050
- d. 2075
- Ans: c
- Ref: Annual Health Report 73/74; Page No: 163-164
Vision, goal, strategy and target of the National TB Programme
Vision: Nepal free of tuberculosis.
Long term goal: End the tuberculosis epidemic by 2050.
Short term goal: Reduce TB incidence by 20% by 2021 compared to 2015 and increase case notifications by a cumulative total of 20,000 from July 2016 to July 2021.
- Objectives:
- • Increase case notification through improved health facility-based diagnosis.
- • Maintain the treatment success rate at 90% of patients (for all forms of TB) through to 2021.
- • Provide drug resistance diagnostic services for 50% of persons with presumptive drug resistant TB by 2018 and 100% by 2021 and successfully treat at least 75% of diagnosed drug resistant patients.
- • Further expand case finding by engaging the private sector.
- • Strengthen community systems for the management, advocacy, support and rights of TB patients in order to create an enabling environment to detect and manage TB cases in 60% of all districts by 2018 and 100% of districts by 2021.
- • Contribute to health system strengthening through TB human resource management, capacity development, financial management, and infrastructure, procurement and supply management.
- • Develop a comprehensive TB surveillance, monitoring and evaluation system
- • Develop a plan to continue NTP services in the aftermath of natural disasters and public health emergencies.
- Guttmann’s sign is:
- a. Flushing of the conjunctiva
- b. Engorged veins of the arm
- c. Stuffiness of the nostril
- d. Warmth of the face
- Ans: c
- Ref: Lee’s Synopsis of Anaesthesia. 13th edition Page No: 444.
- Guttmann’s sign is stuffiness of the nostril which is a sign of successful stellate ganglion block.
- The other signs of successful stellate ganglion block include Horner’s syndrome, engorged veins of the arm, flushing of the conjunctiva and sclera, increase in skin temperature and anhidrosis of the face and neck.
- You are working as a Medical Officer in district hospital in Rupandehi. You are about to screen a patient for COVID 19. What will you ask?
- a. Do you have cough, fever or shortness of breath?
- b. Have you recently returned from, travel in, or been living in India?
- c. Have you been in close contact with anyone with fever or respiratory illness of unknown cause?
- d. All of the above
- Ans: d
- Screening questionnaire for COVID 19: Nepali Guidelines
- All individuals presenting to the OPD or ER entrance should be screened with the following questions:
- a. Symptoms: Do you have any of the following symptoms?
- • Cough? Fever? Shortness of breath? (common)
- • Sore throat, headache or body ache? (less common)
- b. Travel history or contact with traveler: Have you?
- • Recently returned from, travel in, or been living in, an affected area in the past 2 weeks?
- • Been in close contact in the past 2 weeks with someone returning from an affected area?
- c. Exposures: Did you have any exposures to any of the following in last 2 weeks?
- • Close contact with anyone with fever or respiratory illness of unknown cause
- • Known or suspected COVID-positive contact
-
124
- Which of the following is not the discharge criterion of COVID 19 patient?
- a. Afebrile >72 hours without antipyretics
- b. Improvement in respiratory symptoms/signs
- c. More than 7 days since the onset of symptoms
- d. Negative RT-PCR from at least two specimens collected ≥ 6 hrs apart
- Ans: d
- Discharge Criteria:
- Clinical
- • Afebrile >72 hours without antipyretics
- • Improvement in respiratory symptoms/signs
- • More than 7 days since the onset of symptoms
- Laboratory
- • Afebrile >72 hours
- • Negative RT-PCR from at least two specimens collected ≥ 24hrs apart
- Atherosclerosis induced peripheral artery diseases primarily involves which artery?
- a. Iliac arteries
- b. Peroneal arteries
- c. Popliteal arteries
- d. Tibial arteries
- Ans: c
- Ref: Harrison’s Principles of Internal Medicine; 20th Edition, Page No: 1923
- • Peripheral artery disease (PAD) is defined as a clinical disorder in which there is a stenosis or occlusion in the aorta or the arteries of the limbs. Atherosclerosis is the leading cause of PAD in patients >40 years old. Other causes include thrombosis, embolism, vasculitis, fibromuscular dysplasia, entrapment, cystic adventitial disease, and trauma. Segmental lesions that cause stenosis or occlusion are usually localized to large and medium-size vessels.
- • The pathology of the lesions includes atherosclerotic plaques with calcium deposition, thinning of the media, patchy destruction of muscle and elastic fibers, fragmentation of the internal elastic lamina, and thrombi composed of platelets and fibrin.
- • The primary sites of involvement are the abdominal aorta and iliac arteries (30% of symptomatic patients), the femoral and popliteal arteries (80–90% of patients), and the more distal vessels, including the tibial and peroneal arteries (40–50% of patients).
- • Atherosclerotic lesions occur preferentially at arterial branch points, which are sites of increased turbulence, altered shear stress, and intimal injury. Involvement of the distal vasculature is most common in elderly individuals and patients with diabetes mellitus.
-
125
- 1. Birbeck granule is found in:
- a. Langerhans cell
- b. Langhans giant cell
- c. Lepra cell
- d. Clue cell
- Ans: a
- Ref: Robbins Basic Pathology 10th Edition, Page No: 484
- Langerhans cell proliferations take on different clinical forms, but all are believed to be variations of the same basic disorder. The proliferating Langerhans cells express MHC class II antigens, CD1a, and langerin.
- Langerin is a transmembrane protein found in Birbeck granules, cytoplasmic pentalaminar rod-like tubular structures that in electron micrographs have a characteristic periodicity and sometimes a dilated terminal end (“tennis racket” appearance).
- Under the light microscope, the proliferating Langerhans cells do not resemble their normal dendritic counterparts.
- Instead, they have abundant, often vacuolated cytoplasm and vesicular nuclei, an appearance more akin to that of tissue macrophages (called histiocytes by morphologists)—hence the term Langerhans cell histiocytosis.
- 2. Guttmann’s sign is:
- a. Flushing of the conjunctiva
- b. Engorged veins of the arm
- c. Stuffiness of the nostril
- d. Warmth of the face
- Ans: c
- Ref: Lee’s Synopsis of Anaesthesia. 13th edition Page No: 444.
- Guttmann’s sign is stuffiness of the nostril which is a sign of successful stellate ganglion block.
- The other signs of successful stellate ganglion block include Horner’s syndrome, engorged veins of the arm, flushing of the conjunctiva and sclera, increase in skin temperature and anhidrosis of the face and neck.
- 3. What volume of urine is required to be collected in a normal human adult to elicit reflex contraction of bladder?
- a. 50 – 100 ml
- b. 200 – 300 ml
- c. 300 – 400 ml
- d. 500 – 600 ml
- Ans: c
- Ref: Ganong Review of Medical Physiology, Lange; 25th Edition, Page No: 692
- • The bladder smooth muscle has some inherent contractile activity; however, when its nerve supply is intact, stretch receptors in the bladder wall initiate a reflex contraction that has a lower threshold than the inherent contractile response of the muscle.
- • Fibers in the pelvic nerves are the afferent limb of the voiding reflex, and the parasympathetic fibers to the blad¬der that constitute the efferent limb also travel in these nerves.
- • The reflex is integrated in the sacral portion of the spinal cord.
- • In the adult, the volume of urine in the bladder that normally initiates a reflex contraction is about 300–400 ml.
- 4. Nepal has long term goal to end tuberculosis epidemic by:
- a. 2025
- b. 2045
- c. 2050
- d. 2075
- Ans: c
- Ref: Annual Health Report 73/74; Page No: 163-164
Vision, goal, strategy and target of the National TB Programme
Vision: Nepal free of tuberculosis.
Long term goal: End the tuberculosis epidemic by 2050.
Short term goal: Reduce TB incidence by 20% by 2021 compared to 2015 and increase case notifications by a cumulative total of 20,000 from July 2016 to July 2021.
- Objectives:
- • Increase case notification through improved health facility-based diagnosis.
- • Maintain the treatment success rate at 90% of patients (for all forms of TB) through to 2021.
- • Provide drug resistance diagnostic services for 50% of persons with presumptive drug resistant TB by 2018 and 100% by 2021 and successfully treat at least 75% of diagnosed drug resistant patients.
- • Further expand case finding by engaging the private sector.
- • Strengthen community systems for the management, advocacy, support and rights of TB patients in order to create an enabling environment to detect and manage TB cases in 60% of all districts by 2018 and 100% of districts by 2021.
- • Contribute to health system strengthening through TB human resource management, capacity development, financial management, and infrastructure, procurement and supply management.
- • Develop a comprehensive TB surveillance, monitoring and evaluation system
- • Develop a plan to continue NTP services in the aftermath of natural disasters and public health emergencies.
- 5. You are working as a Medical Officer in district hospital in Rupandehi. You are about to screen a patient for COVID 19. What will you ask?
- a. Do you have cough, fever or shortness of breath?
- b. Have you recently returned from, travel in, or been living in India?
- c. Have you been in close contact with anyone with fever or respiratory illness of unknown cause?
- d. All of the above
- Ans: d
- Screening questionnaire for COVID 19: Nepali Guidelines
- All individuals presenting to the OPD or ER entrance should be screened with the following questions:
- a. Symptoms: Do you have any of the following symptoms?
- • Cough? Fever? Shortness of breath? (common)
- • Sore throat, headache or body ache? (less common)
- b. Travel history or contact with traveler: Have you?
- • Recently returned from, travel in, or been living in, an affected area in the past 2 weeks?
- • Been in close contact in the past 2 weeks with someone returning from an affected area?
- c. Exposures: Did you have any exposures to any of the following in last 2 weeks?
- • Close contact with anyone with fever or respiratory illness of unknown cause
- • Known or suspected COVID-positive contact
- 6. Which of the following is not the discharge criterion of COVID 19 patient?
- a. Afebrile >72 hours without antipyretics
- b. Improvement in respiratory symptoms/signs
- c. More than 7 days since the onset of symptoms
- d. Negative RT-PCR from at least two specimens collected ≥ 6 hrs apart
- Ans: d
- Discharge Criteria:
- Clinical
- • Afebrile >72 hours without antipyretics
- • Improvement in respiratory symptoms/signs
- • More than 7 days since the onset of symptoms
- Laboratory
- • Afebrile >72 hours
- • Negative RT-PCR from at least two specimens collected ≥ 24hrs apart
-
126
- 1.True statement about gliobastoma multiforme:
- a. Common in 3rd decade
- b. Most common location is supratentorial
- c. Mostly located in posterior fossa
- d. It has good prognosis
- Ans,b. Most common location is supratentorial
- Ref:Harrison 19th/p.91,17thed./p.2603
- Glioblastoma Multiforme (GBM)
- • GBM is the most common and most aggressive type of primary brain tumor in humans, involving glial cells and accounting for 52% of all parenchymal brain tumor cases and 20% of all intracranial tumors.
- • Excepting the brainstem gliomas, glioblastoma has the worst prognosis of any CNS malignancy
- • Composed of a heterogenous mixture of poorly differentiated neoplastic astrocytes, glioblastomas primarily affect adults, and they are located preferentially in the cerebral hemispheres (so supratentorial)
- • It occurs most often in the subcortical white matter of the cerebral hemispheres. The most frequently affected sites are the temporal (31%), parietal (24%), frontal (23%), and occipital (16%) lobes. Combined fronto-temporal location is particularly typical.
- • Sites for glioblastomas that are much less common are the brainstem (which often is found in affected children), the cerebellum, and the spinal cord.
- 2.Which is the best investigation best to confirm diagnosis of anaphylaxis:
- a. IgA levels
- b. IgD levels
- c. Serum tryptase
- d. Serum precipitins
- Ans,c. Serum Tryptase
- Ref: Harrison 19th ed./p.2116
- • Elevations of tryptase levels in serum implicate mast cell activation in a systemic reaction and are particularly informative for anaphylaxis with episodes of hypotension during general anesthesia or when there has been a fatal outcome.
- • However, because of the short half-life of tryptase, elevated levels are best detected within 4 hours of a systemic reaction
- • Immunoassays using purified antigens can demonstrate the presence of specific IgE in the serum of patients with anaphylactic reactions, and intracutaneous skin testing may be performed after the patient has recovered to elicit a local wheal and flare in response to the putative antigen.
- 3.All of the following are the causes of sudden increase in pain in osteochondroma, except:
- a. Sarcomatous change
- b. Fracture
- c. Bursitis
- d. Degenerative changes
- Ans. is ‘d’ i.e. Degenerative changes
- Ref: Maheshwari, 3rd ed./p.225
- • Osteochondroma usually presents as a painless swelling.
- • Occurrence of pain suggests some complications such as : bursitis at the tip of the swelling, fracture through stalk of the osteochondroma, impingement on or tethering of neighbouring structures such as a tendon, muscle or nerve, malignant transformation
- • A malignant transformation of osteochondroma should be suspected if the osteochondroma continues to grow after skeletal maturity or shows a rapid increase in the size or appearance of sudden pain.
- • The malignancy associated with osteochondroma is most often chondrosarcoma, although malignant fibrous histiocytoma and osteosarcoma have also been reported.
-
127
- 1. Reversibility of asthma is defined as an increase in FEV1 15 minutes after inhaled short-acting beta2-agonist of:
- a. 12 % or 50 mL
- b. 12 % or 100 mL
- c. 12 % or 150 mL
- d. 12 % or 200 mL
Ans: d
Ref: Harrison’s Principles of Internal Medicine; 19th Edition, Page No: 1675
The diagnosis of asthma is usually apparent from the symptoms of variable and intermittent airways obstruction, but must be confirmed by objective measurements of lung function.
Lung Function Tests Simple spirometry confirms airflow limitation with a reduced FEV1, FEV1/FVC ratio, and PEF.
Reversibility is demonstrated by a >12% and 200-mL increase in FEV1 15 min after an inhaled short-acting β2-agonist or in some patients by a 2- to 4-week trial of oral corticosteroids (OCS) (prednisone or prednisolone 30–40 mg daily).
Measurements of PEF twice daily may confirm the diurnal variations in airflow obstruction.
Flow-volume loops show reduced peak flow and reduced maximum expiratory flow.
Further lung function tests are rarely necessary, but whole-body plethysmography shows increased airway resistance and may show increased total lung capacity and residual volume.
Gas diffusion is usually normal, but there may be a small increase in gas transfer in some patients.
- 2. A 7-year-old child is brought to the emergency room by his parents in severe pain. They state that he fell on his outstretched right arm while playing with his friends. He is unable to move his right arm which is being supported by his left. On exam, his vitals are normal. His right extremity reveals normal pulses without swelling in any compartments, but there is crepitus above the elbow upon movement. The child is able to flex and extend his wrist, but this is limited by pain. The child has decreased sensation along his thumb and is unable to make the "OK" sign with his thumb and index finger. What is the most likely diagnosis?
- a. Mid humerus fracture
- b. Scaphoid fracture
- c. Distal radius fracture
- d. Supracondylar humerus fracture
Ans: d
This patient has a clinical presentation suggestive of a supracondylar humerus fracture with concomitant median nerve palsy. Patients are typically treated non-operatively with closed reduction or operatively with open reduction and internal fixation with pinning.
Pediatric patients with falls on outstreched hands commonly present with supracondylar fractures of the humerus. These fractures are important to diagnose because they can often concur with neurovascular injuries. Specifically, the anterior interosseous nerve, a branch of the median nerve, can be stretched due to traction and lead to a palsy preventing thump opposition and making the "OK" sign. Additionally, injuries to vasculature can lead to permanent ischemic contracture.
A midhumerus fracture is associated with a radial nerve palsy that results in wrist drop and sensory deficits in the posterior compartments of the arm.
A scaphoid fracture is a common injury in falls and is typically characterized by pain over the anatomic snuffbox (dorsal aspect of the proximal thumb).
Distal radial and ulnar fractures would present with crepitus below the elbow and would not results in extensive pain above the arm.
- 3. Lining epithelium of vagina is:
- a. Pseudostratified columnar epithelium
- b. Keratinized stratified squamous epithelium
- c. Non keratinized stratified squamous epithelium
- d. Ciliated columnar epithelium
Ans: c
Ref: Gray’s anatomy, 41st Edition; Page No: 1291
VAGINA
• The vagina is a fibromuscular tube lined by non-keratinized stratified epithelium. It extends from the vestibule (the opening between the labia minora) to the uterus. The upper end of the vagina surrounds the vaginal projection of the uterine cervix. The anular recess between the cervix and vagina is the fornix; the different parts of the fornix are given separate names, i.e. anterior, posterior and right and left lateral, but they are continuous.
• The apex of the vagina is directed posteriorly towards the ischial spines. The width of the vagina increases as it ascends. The inner surfaces of the anterior and posterior vaginal walls are in contact with each other, forming a transverse slit. The vaginal mucosa is attached to the uterine cervix higher on the posterior cervical wall than on the anterior; the anterior wall is approximately 7.5 cm long and the posterior wall is approximately 9 cm long.
• The fibromuscular anterior wall of the vagina supports the base of the bladder in its middle and upper portions, and the urethra (which is embedded in it) inferiorly. The fibromuscular posterior wall of the vagina supports the rectum.
• The upper quarter of the posterior vagina is separated from the rectum by the peritoneum of the recto-uterine pouch, and by moderately dense fibromuscular tissue (Denonvilliers’s fascia) in its middle half.
• In its lower quarter, it is separated from the anal canal by the fibromuscular perineal body.
• The upper part of the vagina is supported laterally by levator ani, together with the transverse cervical, pubocervical and uterosacral ligaments. Pubovaginalis provides a U-shaped muscular sling around the mid-vagina.
• The lower vagina is surrounded by the skeletal muscle fibres of bulbospongiosus. As the ureters pass anteromedially to reach the fundus of the bladder, they pass close to the lateral Fornices, where care must be taken to avoid damage during hysterectomy. As they enter the bladder, the ureters are usually anterior to the vagina; at this point, each ureter is crossed transversely by a uterine artery.
- 4. Luxatio erecta is:
- a. Tear of the glenoid labrum
- b. Inferior dislocation of shoulder
- c. Anterior dislocation of shoulder
- d. Defect in the humeral head
Ans: b
Ref: Maheshwari Orthopedics 5th Edition Page No: 367
DISLOCATIONS WITH EPONYMS:
• Chopart’s: dislocation through talo-navicular joints
• Divergent: elbow dislocation where ulna and radius dislocate in opposite directions
• Lisfranc’s: dislocation through inter tarsal joint
• Lunate: wrist injury where lunate bone comes out to lie in front of other carpal bones
• Luxatio erecta: inferior dislocation of shoulder
• Otto pelvis: gradual shift of the acetabulum into the pelvis (e.g. in osteomalacia)
• Perilunate: wrist injury where the lunate remains in its place and the other carpal bones dislocate around it dorsally
• Spondylolisthesis: movement of one vertebra over another (usually L4 over L5)
• Sprain: A break in the continuity of a ligament
• Strain: A break in muscle fibres
-
137
- Q1.Vascular resistance is low in which type of shock?
- a. Hypovolemic shock
- b. Distributive shock
- c. Cardiogenic shock
- d. Obstructive shock
- Ans: b, Distributive shock
- Ref: Short Practice of Surgery, Bailey and Love, 27th edition, page no.15
- Explanation:
- Cardiovascular and metabolic characteristics of shock
- Hypovolaemic vs Cardiogenic vs Obstructive vs Distributive
- Cardiac output : Low vs Low vs Low vs High
- Vascular resistance : High vs High vs High vs Low
- Venous pressure : Low vs High vs High vs Low
- Mixed venous saturation : Low vs Low vs Low vs High
- Base deficit : High vs High vs High vs High
- Q2. The anterior group of axillary lymph nodes lies;
- a. Along the axillary vein
- b. Along the lateral thoracic vessels
- c. Along the subscapular vessels
- d. Embedded in fat in the centre of the axilla
- Ans: b, Along the lateral thoracic vessels
- Ref: Short Practice of Surgery, Bailey and Love, 27th edition, page no.860-61
- Explanation:
- The lymphatics of the breast drain predominantly into the axillary and internal mammary lymph nodes. The axillary nodes receive approximately 85% of the drainage and are arranged in the following groups:
- • lateral, along the axillary vein;
- • anterior, along the lateral thoracic vessels;
- • posterior, along the subscapular vessels;
- • central, embedded in fat in the centre of the axilla; interpectoral, a few nodes lying between the pectoralis major and minor muscles;
- • apical, which lie above the level of the pectoralis minor tendon in continuity with the lateral nodes and which receive the efferents of all the other groups.
- Q3. A final year MBBS student is asked by the examiner to perform abdominal examination during his final year assessment. While palpating the abdomen, the examiner asked him the direction of enlargement of spleen which is;
- a. Downwards, forwards and medially
- b. Downwards, backwards and medially
- c. Downward, forward and laterally
- d. Downward, backward and laterally
- Ans: a, Downwards, forwards and medially
- Ref: Short Practice of Surgery, Bailey and Love, 27th edition, page no.1021
- Examination of Spleen
- •In a healthy patient the spleen is not normally palpable.
- •An enlarged spleen descends downwards, forwards and medially.
- •Palpation for an enlarged spleen is best performed in a supine patient. The examining hand should start in the right lower abdomen, with the tips of the fingers pointing upwards and pressed inwards. The patient is then asked to take a deep breath, and if the spleen is enlarged the lower edge with the characteristic notch will touch the fingers. If it is not palpable, then the hand is gradually moved upwards in the direction of the position of the edge of the normal-sized spleen with each breath.
- •If the spleen is still not palpable, the patient is moved to the right lateral position and the examination repeated.
- Q4. You are doing autopsy examination of a 45 years old male body, who died due to lightening stroke, you find filigree burns over his body. Filigree burns are:
- a. Deep, thick, regular markings on the skin
- b. Deep, thick, irregular markings on the skin
- c. Superficial, thin, regular markings on the skin
- d. Superficial, thin, irregular markings on the skin
- Ans: d, Superficial, thin, irregular markings on the skin
- Ref: Reddy, The essential of forensic medicine and toxicology 33rd edition, page number 334
- Arborescent or Filigree Burns: (Lichtenberg's flowers):
- •They are superficial, thin, irregular and tortuous markings on the skin.
- •These markings have a general pattern resembling the branches of a tree.
- •This fern-like pattern of erythema in the skin is usually found over the shoulders or the flanks.
- •It appears within few minutes to one hour of accident.
- Mechanism:
- •The mechanism by which this pattern hi occurs is not known.
- •It may be caused due to the slight staining of the tissues by haemoglobin from lysed red blood cells along the path of the electric current or rupture of smaller blood vessels at several places giving rise to ecchymoses.
- •They may be superficial bums producing mere erythema of skin which indicate the path taken by the current.
- •They are also said to be due to minute depositions of copper in the dermis or from boiling of the intercellular fluid following the fascial planes.
-
134 (Repeat)
- Insular carcinoma is a variety of
- a. Papillary
- b. Medullary
- c. Anaplastic
- d. Follicular
- Ans: d
- Ref: Harrison’s Principles of Internal Medicine; 19th Edition, Page No: 2303
-
130
- Q1.Termination of pregnancy by Menstrual regulation method consists of aspiration of the contents of the uterine cavity by means of a plastic cannula (Karman’s cannula). It is carried out effectively within:
- a 14 days of the beginning of the last menstrual period (LMP)
- b.72 hours of the beginning of the last menstrual period (LMP)
- c. 28 days of the beginning of the last menstrual period (LMP)
- d. 42 days of the beginning of the last menstrual period (LMP)
Answer d. 42 days of the beginning of the last menstrual period (LMP)
- Ref: Shaw’s text book of gynecology, 15th edition, page number 288
- Menstrual Regulation method for termination of pregnancy:
- • Menstrual regulation consists of aspiration of the contents of the uterine cavity by means of a plastic cannula (Karman’s cannula).
- • It has a plastic 50 mL syringe capable of creating a vacuum of of 65 cm Hg.
- • It has a simple thumb-operated pressure control valve and piston locking handle.
- • It is independent of electricity, is portable and washable.
- • It is carried out effectively within 42 days of the beginning of the last menstrual period (LMP).
- • A paracervical local anaesthetic block or preoperative sedative alone usually suffices but sometimes
- • in an apprehensive patient, general anaesthesia with intravenous thiopentone sodium may be necessary.
- • This procedure can be performed in an office set-up, outpatient clinic, or day-care centre.
- • Since 1972, this method has been extensively evaluated and found to be efficient, safe, and easy to use in terminating early pregnancy.
- • It is a good practice to examine the products of conception.
- • The occasional complications encountered include failure to evacuate leading to continuation of pregnancy, incomplete evacuation, haemorrhage, cervical laceration, perforation, infection and anaesthetic complications.
- A failure to evacuate is due to:
- 1. Too early pregnancy.
- 2. Ectopic pregnancy.
- 3. Uterus bicornuate, aspiration being carried out in nonpregnant horn. Sometimes, tip of the cannula breaks but comes out in the next menstrual bleeding—and it may not be necessary to retrieve it.
- • RH anti-D globulin 100 mcg IM should be given to a RH negative non-immunized woman.
- Q2. The incidence of heterotopic pregnancy is higher in IVF programmes. Heterotopic pregnancy is:
- a. Ectopic pregnancy at two different site
- b. Combined abdominal and tubal pregnancy
- c. Twin pregnancy of different sex
- d. Combined uterine and ectopic tubal pregnancy
Answer d. Combined uterine and ectopic tubal pregnancy
Ref: Shaw’s text book of gynecology, 15th edition, page number 309
- Heterotopic Pregnancy
- • Heterotopic pregnancy, i.e. combined uterine and ectopic tubal pregnancy, is very rare in spontaneous conception cycles; the incidence is not more than 1:4000 to 1:7000 pregnancies.
- • The incidence is however higher in IVF programmes because of the higher number of embryo transfer, with one embryo migrating to the tube.
- • The incidence is also related to the amount of fluid injected with the embryo.
- • At present, IVF centres present an incidence of 1–3% of heterotopic pregnancy.
- The diagnosis is not easy.
- • The serum b-hCG may not be proportionately high.
- • Ultrasound can visualize multiple pregnancy in early pregnancy.
- • It is therefore mandatory to search for adnexal mass when pregnancy occurs in ART.
- Treatment
- • Medical treatment with mTX, mifepristone and prostaglandin are contraindicated because of their adverse effects on the normal uterine pregnancy.
- • Glucose and KCl may be used or surgery performed.
- • The treatment is preferably laparoscopic minimal invasive surgery, allowing uterine pregnancy to grow.
- In IVF programme, the following prophylactic measures have been suggested:
- • Bilateral tubectomy prior to IVF.
- • Transfer of not more than two embryos
- • A small amount of fluid medium to be transferred.
- • A routine ultrasound scanning in early pregnancy, incase conception follows
- Q3. A 65-year-old male is fall from tree and suddenly collapses. He is Unresponsive, you are doing cardiopulmonary resuscitation. What method should be used to open his airway if you suspect he had neck injury?
- a. Chin-lift
- b. Jaw thrust
- c. Head-tilt/ chin-lift
- d. Head-tilt
Answer b. Jaw thrust
Ref: Basic life support provider hand book (2015-2020 guidelines and standards) page number 10
- The jaw thrust is the maneuver of choice to open this patient’s airway given the concern of a fall and potential for traumatic injuries.
- Cardiopulmonary resuscitation steps:
- 1. Check for the carotid pulse on the side of the neck. Keep in mind to not waste time trying to feel for a pulse; feel for no more than 10 seconds. If you are not sure you feel a pulse, begin CPR with a cycle of 30 chest compressions and two breaths (Figure a).
- 2. Use the heel of one hand on the lower half of the sternum in the middle of the chest (Figure b).
- 3. Put your other hand on top of the first hand (Figure c).
- 4. Straighten your arms and press straight down (Figure d). Compressions should be at least two inches into the person’s chest and at a rate of 100 to 120 compressions per minute.
- 5. Be sure that between each compression you completely stop pressing on the chest and allow the chest wall to return to its natural position. Leaning or resting on the chest between compressions can keep the heart from refilling in between each compression and make CPR less effective.
- 6. After 30 compressions, stop compressions and open the airway by tilting the head and lifting the chin (Figure e & f).
- a. Put your hand on the person’s forehead and tilt the head back
- b. Lift the person’s jaw by placing your index and middle fingers on the lower jaw; lift up.
- c. Do not perform head-tilt/ chin lift if you suspect the person may have a neck injury. In that case the jaw-thrust is used.
- d. For the jaw-thrust maneuver, grasp the angles of the lower jaw and lift it with both hands, one on each side, moving the jaw forward. If their lips are closed, open the lower lip using your thumb.
- 7. Give a breath while watching the chest rise. Repeat while giving a second breath. Breaths should be delivered over one second.
- 8. Resume chest compressions. Switch quickly between compressions and rescue breaths to minimize interruptions in chest compressions.
-
131
- 1. Multiple motor tics often accompanied by vocalization, predominantly in males and patient can voluntarily suppress tics for short period of time but then experience an irresistible urge to express them. This is seen in:
- a. Tardive dyskinesia
- b. Huntington's disease
- c. Tourette's syndrome
- d. Psychogenic disorder
- Ans: c
- Ref: Harrison’s Principles of Internal Medicine; 20th Edition, Page No: 3138
- • TS is a neurobehavioral disorder named after the French neurologist Georges Gilles de la Tourette.
- • TS is characterized by multiple motor tics often accompanied by vocalizations (phonic tics).
- • Patients characteristically can voluntarily suppress tics for short periods of time, but then experience an irresistible urge to express them.
- • Tics vary in intensity and may be absent for days or weeks only to recur, occasionally in a different pattern.
- • Tics tend to present between ages 2 and 15 years (mean 7 years) and often lessen or even disappear in adulthood, particularly in males.
- • Associated behavioral disturbances include anxiety, depression, attention deficit hyperactivity disorder, and obsessive-compulsive disorder.
- 2. Inspiratory stridor is found in what kind of lesions:
- a. Supraglottic
- b. Subglottic
- c. Tracheal
- d. Bronchus
- Ans: a
- Ref: PL Dhingra and Shruti Dhingra ENT and HNS 7th Edition, Page No: 333
- Inspiratory stridor is often produced in obstructive lesions of supraglottis or pharynx, e.g. laryngomalacia or retropharyngeal abscess.
- Expiratory stridor is produced in lesions of thoracic trachea, primary and secondary bronchi, e.g. bronchial foreign body, and tracheal stenosis.
- Biphasic stridor is seen in lesions of glottis, subglottis and cervical trachea, e.g. laryngeal papillomas, vocal cord paralysis and subglottis stenosis.
- 3. Glycogenin is a:
- a. Polypeptide
- b. Polysaccharide
- c. Lipid
- d. Glycosaminoglycan
- Ans: a
- Ref: Harper Biochemistry 30th Edition, Page No: 177
- • Glycogenin is a protein
- • 37 K Da protein
- • Glycogenin catalyzes transfer of 7 glucosyl residues from UDPGlucose, in alpha 1 → 4 linkage
- • Glucosyl residues are added on specific tyrosine residues of Glycogenin
- • Glycogenin remains at the core of Glycogen granule.
- 4. Empyema necessitans is defined as so when -
- a. Plural empyema is under pressure
- b. Pleural empyema has ruptured into bronchus
- c. Pleural empyema has ruptured into the pericardium
- d. Pleural empyema is showing extension to the subcutaneous tissue
- Ans: d
- Ref: SRB's Manual of Surgery; 5th Edition, Page No: 1120
- It is a complication of empyema thoracis, where in empyema which is not drained perforates through the chest wall presenting as subcutaneous collection of pus communicating directly or often with a tortuous route with the pleural cavity.
- Causes
- • Neglected empyema of tuberculous or non tuberculous etiology.
- • Following incomplete aspiration of pus of empyema through thin needle—needle track itself allows pus to form a track and leads to empyema necessitans.
-
132
- 1. Pricking injury to finger leads to: [KU 2019]
- a. Implantation dermoid
- b. Sequestration dermoid
- c. Sebaceous cyst
- d. Corn
- Ans: a
- Ref: SRB's Manual of Surgery; 5th Edition, Page No: 77
- Implantation dermoid is due to minor pricks or trauma, epidermis gets buried into the deeper subcutaneous tissue which causes reaction and cyst formation (trauma is forgotten often). It is an acquired cyst. It is common in fingers (common in tailors, gardeners), toes and feet.
- 2. KatG mutation occurs in: [KU 2018]
- a. Rifampicin
- b. Isoniazid
- c. Pyrazinamide
- d. Ethambutol
- Ans: b
- Ref: Katzung Pharmacology 12th Edition, Page No: 840
- Isoniazid inhibits synthesis of mycolic acids, which are essential components of mycobacterial cell walls. Isoniazid is a prodrug that is activated by KatG, the mycobacterial catalase-peroxidase. The activated form of isoniazid forms a covalent complex with an acyl carrier protein (AcpM) and KasA, a beta-ketoacyl carrier protein synthetase, which blocks mycolic acid synthesis and kills the cell.
- Resistance to isoniazid is associated with mutations resulting in overexpression of inhA , which encodes an NADH-dependent acyl carrier protein reductase; mutation or deletion of the katG gene; promoter mutations resulting in overexpression of ahpC , a putative virulence gene involved in protection of the cell from oxidative stress; and mutations in kasA . Overproducers of inhA express low-level isoniazid resistance and cross-resistance to ethionamide.
- KatG mutants express high-level isoniazid resistance and often are not cross-resistant to ethionamide.
- 3. Intercalary staphyloma is [KU 2011]
- a. Ectasia of sclera with incarceration of root of iris
- b. Ectasia of sclera with incarceration of choroid
- c. Ectasia of sclera with incarceration of ciliary body
- d. Ectasia of sclera with incarceration of iris
- Ans: a
- Ref: Parson Ophthalmology 22ndEdition, Page No: 229
- Intercalary Staphyloma
- • This is located at the limbus and is lined by the root of the iris and the anteriormost part of the ciliary body.
- • It is seen externally from the limbus to up to 2 mm behind the limbus.
- • The usual causes are lesions that produce weakening of the globe in this region such as perforating injuries of the peripheral cornea, marginal corneal ulcer, anterior scleritis, scleromalacia perforans, complicated cataract surgery with poor wound apposition and secondary glaucoma.
- 4. Zinc phosphide is a: [KU 2013]
- a. Contact poison
- b. Respiratory poison
- c. Stomach poison
- d. Hemotoxin
- Ans: b
- Ref: Gautam Biswas Forensic Medicine, 3rdEdition, Page No: 512
- • Zinc is normally present in our body. Poisonous salts are compounds of chloride, phosphide, sulfate (white vitriol), oxide and stearate.
- • Zinc chloride is used to clean metals before soldering. Zinc phosphide is used as rodenticide. Zinc stearate is used as a cosmetic (baby powder).
- • Zinc phosphide releases phosphine gas under acidic conditions in the stomach (similar to aluminum phosphide). In addition to the above features, the vomitus gives the smell of garlic. Dyspnea, lethargy, hypotension, cardiac arrhythmias, pulmonary edema, metabolic acidosis, convulsions, circulatory collapse, coma and death may be occur.
- • Zinc phosphide is hydrolyzed by the gastric acid and is transformed into phosphine gas. Phosphine is a respiratory toxin that inhibits cytochrome C oxidase system.
-
142 (Repeat)
- 1. Post partum blues commonly occurs : [KU 2018]
- a. Immediately after delivery
- b. 4-5 days after delivery
- c. 4-5 weeks after delivery
- d. 6 month after delivery
- Ans: b
- Ref: USMLE Step 1 First Aid 2020, Page No: 562
- Peripartum mood disturbances: Onset during or shortly after pregnancy or within 4 weeks of delivery. risk with history of mood disorders.
- Maternal (postpartum) blues:
- • 50–85% incidence rate.
- • Characterized by depressed affect, tearfulness, and fatigue starting 2–3 days after delivery.
- • Usually resolves within 2 weeks.
- • Treatment: supportive. Follow up to assess for possible MDD with peripartum onset.
- MDD with peripartum onset
- • 10–15% incidence rate.
- • Formerly called postpartum depression.
- • Meets MDD criteria with onset no later than 1 year after delivery.
- • Treatment: CBT and SSRIs are first line.
- Postpartum psychosis
- • 0.1–0.2% incidence rate.
- • Characterized by mood-congruent delusions, hallucinations, and thoughts of harming the baby or self.
- • Risk factors include first pregnancy, family history, bipolar disorder, psychotic disorder, recent medication change.
- • Treatment: hospitalization and initiation of atypical antipsychotic; if insufficient, ECT may be used.
- 2. The most common type of elbow dislocation is: [KU 2015]
- a. Posterior
- b. Postero - Medial
- c. Postero - Lateral
- d. Anterior
- Ans: a
- Ref: Ebnezar Orthopedics 4th Edition, Page No: 157
- Stimson’s classification
- Proximal radioulnar joint intact
- A. Posterior (90%)
- a. Posterolateral
- b. Posteromedial
- B. Anterior
- C. Medial
- D. Lateral
- Proximal radioulnar joint disrupted (divergent dislocation)
- A. Anteroposterior
- a. Radius is anterior
- b. Ulna is posterior
- B. Medial lateral (transverse)
- c. Radius is lateral
- d. Ulna is medial
- Terrible triad” of the elbow:
- Posterior dislocation of elbow.
- Radial head fracture.
- Fracture coronoid process of ulna.
- 3. As per the WHO guideline tubectomy can be done in ....... days of delivery
- a. Less than 3
- b. 3-7
- c. 7-14
- d. More than 45 days
- Ans: a
- Ref: Shaw Gynecology 16th Edition, Page No: 285 | Ref: Postpartum Tubal Sterilization by Author: Nan G O'Connell in Medscape.
- • A multiparous woman may be counselled on sterilization or vasectomy. This is done any time after 24 h of delivery, so the woman need not return to the hospital for tubectomy later, and this is cost effective and convenient. Minilaparot¬omy is a simple and a quick procedure done under local or a short general anaesthesia.
- • Postpartum tubal sterilization is indicated in any patient who is medically stable after a vaginal delivery (usually within 48 hours) and desires permanent contraception. The patient should have been properly counseled about the procedure. Consideration must also be given to regulations regarding timing of consent and adequacy of staff to perform an elective procedure.
- 4. Tympanic membrane and ossicles efficiently transmit sound of frequency [KU 2009]
- a. 3000-5000 Hz
- b. 200-300 Hz
- c. 500-3000 Hz
- d. 200-1000 Hz
- Ans: c
- Ref: PL Dhingra and Shruti Dhingra ENT and HNS 7th Edition, Page No: 17
- Frequencies most efficiently transmitted by ossicular chain are between 500 and 2000 Hz while that by tympanic membrane is 800–1600 Hz. Thus greatest sensitivity of the sound transmission is between 500 and 3000 Hz and these are the frequencies most important to man in day-to-day conversation.
- 5. A drug binding to a receptor but producing a response opposite to that of the endogenous agonist is called: [KU 2014]
- a. Agonist
- b. Antagonist
- c. Inverse agonist
- d. Inverse antagonist
- Ans: c
- Ref: KD Tripathi Essentials of Medical Pharmacology; 7th Edition, Page No: 40
- • Agonist: An agent which activates a receptor to produce an effect similar to that of the physiological signal molecule.
- • Inverse agonist: An agent which activates a receptor to produce an effect in the opposite direction to that of the agonist.
- • Antagonist: An agent which prevents the action of an agonist on a receptor or the subsequent response, but does not have any effect of its own.
-
135
- 1. Which of the following does not pass through the deep inguinal ring?
- a. Spermatic cord
- b. Round ligament of uterus
- c. Ilioinguinal nerve
- d. None
- Ans: C, Ilioinguinal nerve
- Ref: Human anatomy BD Chaurasia 4th edition Vol 2 Page no. 208
- Structures Passing through the Canal
- 1. The spermatic cord in males, or the round ligament of the uterus in females, enters the inguinal canal through the deep inguinal ring and passes out through the superficial inguinal ring.
- 2. The ilioinguinal nerve enters the canal through the interval between the external and internal oblique muscles and passes out through the superficial inguinal ring
- 2. Adverse effects of heparin are all except
- a. Bleeding
- b. Reversible alopecia
- c. Osteoporosis
- d. Thrombocytosis
- Ans: D, Thrombocytosis
- Ref: Basic and clinical pharmacology Katzung 14th e Page no 613
- Toxicity of Heparin
- A. Bleeding and Miscellaneous Effects
- The major adverse effect of heparin is bleeding. This risk can be decreased by scrupulous patient selection, careful control of dosage, and close monitoring. Elderly women and patients with renal failure are more prone to hemorrhage. Heparin is of animal origin and should be used cautiously in patients with allergy. Increased loss of hair and reversible alopecia have been reported. Long-term heparin therapy is associated with osteoporosis and spontaneous fractures. Heparin accelerates the clearing of postprandial lipemia by causing the release of lipoprotein lipase from tissues, and long-term use is associated with mineralocorticoid deficiency.
- B. Heparin-Induced Thrombocytopenia
- 3. Common type of collagen found in lens is
- a. Type I
- b. Type II
- c. Type III
- d. Type IV
- Ans: D, Type IV
- Please see the picture below.
- Ref: First Aid for the USMLE 2019 P 50
-
136
- 1. Which of the following neurons do not have acetylcholine as the neurotransmitter?
- a. Preganglionic sympathetic neurons
- b. Postganglionic sympathetic neurons
- c. Preganglionic parasympathetic neurons
- d. Neurons innervating sweat glands
- Ans : B. Postganglionic sympathetic neurons
- Explanation:
- * Preganglionic neurons
- * Sympathetic – acetylcholine
- * Parasympathetic – acetylcholine
- * Postganglionic neurons
- * Sympathetic – norepinephrine
- * Parasympathetic – acetylcholine
- * Exception: In sympathetic neurons innervating eccrine sweat glands, acetylcholine is the neurotransmitter.
- 2. Pseudo-isomorphic phenomenon is characteristic of?
- a. Vitiligo
- b. Psoriasis
- c. DLE
- d. Plane warts
- Ans : D. Plane warts
- PSEUDO-ISOMORPHIC PHENOMENON
- * To learn about pseudo-isomorphic phenomenon, we first need to know what is isomorphic phenomenon.
- * Isomorphic phenomenon (also known as Koebner phenomenon) is a phenomenon in which new skin lesions appear at the site of injury.
- * It is named after Heinrich Köbner, a German dermatologist.
- * Isomorphic phenomenon is seen in conditions like psoriasis, vitiligo and lichen planus.
- * Pseudo-isomorphic phenomenon (Pseudo-Koebner phenomenon) is characterised by the appearance of similar skin lesions at the site of injury in infective lesions like plane warts and molluscum contagiosum.
- * It occurs due to the spread of the infective agent (virus) to the new site.
- 3. All are features of autistic disorders except ?
- a. Stereotypic movements
- b. Impairment of social interaction
- c. Visual impairment
- d. Delay in speech development
- Ans : C. Visual impairment
- Autistic disorders are characterised by:
- * Impairement of social interaction
- * Disorder of communication and language (as in delayed development of speech)
- * Stereotypic movements
- Visual impairment is not seen in autism.
-
138
Cause of death in Judicial hanging
- Ref: Reddy Forensic Medicine, 33rd Edition, Page No: 339, 346, 347
- In judicial hanging, the sudden stoppage of the moving body associated with the position of the knot causes the head to be jerked violently. This causes fracture-dislocation usually at the level of the second and third, or third and fourth cervical vertebrae. Bilateral fractures of either the pedicles or laminae of the arch of the second, third or fourth cervical vertebrae occur (hangman's fracture).
- Ref: Gautam Biswas Forensic Medicine, 3rdEdition, Page No: 167
- The ligature around the neck causes a forceful jerky impact on the neck at the end of the fall, so as to cause fracture of cervical column (fracture dislocation of C2 from C3, rarely C3 and C4 vertebrae—hangman fracture) with stretching or tearing of cervical spinal cord, but not decapitation. In juidicial hangings, odontoid process is usually not fractured.
- Ref: JUDICIAL HANGING-CAUSE OF DEATH by Dr Mrinal Kanti Jha; J Punjab Acad Forensic Med Toxicol 2012;12(2)
- The World is divided on the issue of Capital punishment. Many western countries have abolished death as a judicial punishment, with the argument; eye for an eye is barbaric. India however has death penalty, by hanging in statute. It is awarded in rarest of the rare cases. It seems death penalty is there to stay in Indian statute. On the cause of death in cases of judicial hanging the popular book teaching is- facture of second and third or third and fourth cervical vertebra is responsible for quick and painless death. Studies carried out on later dates on the body of victims of judicial hanging showed otherwise.
-
139 (Repeat)
- As per the WHO guideline tubectomy can be done in ....... days of delivery
- a. Less than 3
- b. 3-7
- c. 7-14
- d. More than 45 days
- Ans: a
- Ref: Shaw Gynecology 16th Edition, Page No: 285 | Ref: Postpartum Tubal Sterilization by Author: Nan G O'Connell in Medscape.
- • A multiparous woman may be counselled on sterilization or vasectomy. This is done any time after 24 h of delivery, so the woman need not return to the hospital for tubectomy later, and this is cost effective and convenient. Minilaparot¬omy is a simple and a quick procedure done under local or a short general anaesthesia.
- • Postpartum tubal sterilization is indicated in any patient who is medically stable after a vaginal delivery (usually within 48 hours) and desires permanent contraception. The patient should have been properly counseled about the procedure. Consideration must also be given to regulations regarding timing of consent and adequacy of staff to perform an elective procedure.
-
140 (Repeat)
- A drug binding to a receptor but producing a response opposite to that of the endogenous agonist is called: [KU 2014]
- a. Agonist
- b. Antagonist
- c. Inverse agonist
- d. Inverse antagonist
- Ans: c
- Ref: KD Tripathi Essentials of Medical Pharmacology; 7th Edition, Page No: 40
- • Agonist: An agent which activates a receptor to produce an effect similar to that of the physiological signal molecule.
- • Inverse agonist: An agent which activates a receptor to produce an effect in the opposite direction to that of the agonist.
- • Antagonist: An agent which prevents the action of an agonist on a receptor or the subsequent response, but does not have any effect of its own.
-
141 (Repeat)
- Multiple motor tics often accompanied by vocalization, predominantly in males and patient can voluntarily suppress tics for short period of time but then experience an irresistible urge to express them. This is seen in:
- a. Tardive dyskinesia
- b. Huntington's disease
- c. Tourette's syndrome
- d. Psychogenic disorder
- Ans: c
- Ref: Harrison’s Principles of Internal Medicine; 20th Edition, Page No: 3138
- • TS is a neurobehavioral disorder named after the French neurologist Georges Gilles de la Tourette.
- • TS is characterized by multiple motor tics often accompanied by vocalizations (phonic tics).
- • Patients characteristically can voluntarily suppress tics for short periods of time, but then experience an irresistible urge to express them.
- • Tics vary in intensity and may be absent for days or weeks only to recur, occasionally in a different pattern.
- • Tics tend to present between ages 2 and 15 years (mean 7 years) and often lessen or even disappear in adulthood, particularly in males.
- • Associated behavioral disturbances include anxiety, depression, attention deficit hyperactivity disorder, and obsessive-compulsive disorder.
-
143
1. Sensation travels from the thumb and index finger via the C6 nerve root, from the middle finger via the C7 root, and from the fourth and fifth fingers via the C8 root.
2. For the remainder of the upper extremity, the root innervation "fans out" from the hand.
3. These innervation patterns cover both the anterior and posterior aspects of the upper extremity.
4. The 1,5 dermatome includes the large toe and the lateral lower leg; the S1 dermatome includes the small toe and sole. Roots L4, L3, and L2 cover the region fanning out medially and proximally from the L5 dermatome. The S2 dermatome extends from the S1 dermatome up the back of the leg.
5. The median nerve carries sensation from all fingers except the fifth finger and half of the fourth, which are served by the ulnar nerve. These nerve territories extend proximally up to the wrist on the palmar aspect of the hand. On the dorsal aspect, the ulnar nerve territory still extends to the wrist, but the median nerve territory fades into radial nerve territory at the metacarpal-phalangeal joints.
-
114
- 1. Inverse ratio ventilation refers to?
- a. Prone-position ventilation
- b. Inspiratory time is longer than expiratory time
- c. Expiratory time is longer than inspiratory time
- d. Ventilating at extremely high respiratory rates
- Ans: b
- Ref: Harrison’s Principles of Internal Medicine; 19th Edition, Page No: 1738
- In inverse ratio ventilation technique, inspiratory time is lengthened so that it is longer than expiratory time. With diminished time to exhale, dynamic hyperinflation leads to increased end expiratory pressure, similar to ventilator-prescribed PEEP.
- 2. BCYE medium is used to culture:
- a. Mycoplasma
- b. T. Pallidum
- c. Legionella
- d. H. Pylori
- Ans: c
- Ref: Jawetz Microbiology 26th Edition, Page No: 305
- • Legionellae can be grown on complex media such as buffered charcoal yeast extract agar with α-ketoglutarate and iron (BCYE) at a pH of 6.9, temperature of 35°C, and 90% humidity.
- • Antibiotics can be added to make the medium selective for Legionella species. Th e charcoal acts as a detoxifying agent.
- • A biphasic BCYE medium can be used for blood cultures. Legionellae grow slowly; visible colonies are usually present aft er 3 days of incubation. Colonies that appear after overnight incubation are not Legionella species.
- • Colonies are round or fl at with entire edges. They vary in color from colorless to iridescent pink or blue and are translucent or speckled. Variation in colony morphology is common, and the colonies may rapidly lose their color and speckles.
- • Many other genera of bacteria grow on BCYE medium and must be differentiated from Legionella by Gram staining and other tests. Legionellae in blood cultures usually require 2 weeks or more to grow. Colonies can be seen on the agar surface of the biphasic medium.
- 3. In Krebs’s cycle aconitase is inhibited by:
- a. Arsenite
- b. Fluoroacetate
- c. Malonate
- d. Ammonia
- Ans: b
- Ref: Harper Biochemistry 30th Edition, Page No: 163-164
- • Aconitase- Fluoroacetate
- • Succinate dehydrogenase- Malonate
- • α-Ketoglutarate dehydrogenase complex-Arsenite and High concentrations of ammonia
- 4. Laron dwarfism is caused by:
- a. Deficiency of GH
- b. GH receptor defect
- c. Deficiency of thyroxine
- d. Thyroxine receptor defect
- Ans: b
- Ref: OP Ghai Pediatrics 8th Edition, Page No: 511
- Growth Hormone Deficiency
- Growth hormone deficiency (GHD) may be caused by congenital CNS malformations, genetic defects or acquired neurological insults. These children have normal growth at birth. Growth retardation becomes apparent around one year of age. Midfacial crowding, round facies, mild obesity, depressed nasal bridge, single central incisor tooth and micropenis are common. Body proportions are normal. The development of teeth is delayed. The facial appearance is 'doll like' and these children look much younger than their actual age. Bone age is delayed. Newborns may present with severe hypoglycemic seizures due to concomitant ACTH deficiency. Associated gonadotropin deficiency causes delay in sexual development and small genitalia. Resistance to growth hormone action (growth hormone insensitivity or Laron syndrome) presents with almost similar features with severe growth retardation and elevated baseline GH levels.
-
115
- 1. Most common hematologic problem in sarcoidosis is?
- a. Anemia
- b. Lymphopenia
- c. Leukopenia
- d. Thrombocytopenia
- Ans: b
- Ref: Harrison’s Principles of Internal Medicine; 19th Edition, Page No: 2208
- One or more bone marrow manifestations can be identified in many sarcoidosis patients. The most common hematologic problem is lymphopenia, which is a reflection of sequestration of the lymphocytes into the areas of inflammation.
- 2. Lateral traction test is used for:
- a. Habitual passive agent of sodomy
- b. Active agent of sodomy
- c. Non-habitual passive agent of sodomy
- d. Rape accused
- Ans: a
- Ref: Reddy Forensic Medicine, 33rd Edition, Page No: 429
- Habitual Passive Agent
- • There may be shaving of anal hair.
- • Bloodstains are usually not observed.
- • Loose foreign hair and smears of lubricant may be present.
- • Perianal skin may be thickened and keratinized with mucocutaneous eversion. Shiny silvery hyperkeratinized skin may also be due to scratching from chronic irritation associated with hemorrhoids, threadworms or viral infections.
- • Person does not experience any pain or tenderness during digital examination. Anal sphincter is lax, opening is patulous, canal is dilated and there may be loss of fine symmetric rugal pattern, along with congested or dilated veins.
- • Lateral traction test: External anal sphincter relaxes reflexly when bimanual traction is applied to the buttocks.
- 3. Which of the following has highest MAC value?
- a. Desflurane
- b. Halothane
- c. Isoflurane
- d. Sevoflurane
- Ans: a
- Ref: Ajay Yadav, Short Textbook of Anesthesia, 6th edition, page 90
- Best estimate for dete rmining the potency of inhalational anesthetic is minimum alveolar concentration (MAC) which is defined as minimum concentration of agent required to produce immobility in 50% of the subjects given noxious stimuli, which is skin incision in human beings.
- Common MAC values are:
- • Halothane 0.74%
- • Isoflurane 1.15%
- • Sevoflurane 2.05%
- ��� Desflurane 6.0%
- • Nitrous oxide 104%
- 4. Highest cardiac output is seen:
- a. After delivery
- b. Second trimester
- c. First trimester
- d. Near term
- Ans: a
- Ref: Dutta Obstetrics 8th Edition, Page No: 60
- CARDIAC OUTPUT: The cardiac output (CO) starts to increase from 5th week of pregnancy and reaches its peak 40–50% at about 30–34 weeks. Thereafter the CO remains static till term when the observation is made at lateral recumbent position. CO is lowest in the sitting or supine position and highest in the right or left lateral or knee chest position. Cardiac output increases further during labor (+50%) and immediately following delivery (+70%) over the pre-labor values. MAP also rises. There is squeezing out of blood from the uterus into the maternal circulation (auto transfusion) during labor and in the immediate postpartum. CO returns to pre-labor values by 1 hour following delivery and to the pre-pregnant level by another 4 weeks time.
- 5. Polycyclic erosions are seen in:
- a. Molluscum contagiosum
- b. Actinomycosis
- c. Herpes genitalis
- d. Syphilis
- Ans: c
- Ref: Dermatology, Neena Khanna, 4th edition, Page No: 279
- Herpes genitalis is caused by HSV II. Lesions start with multiple grouped vesicles, when rupture coalesce with each other and form an ulcer with polycyclic margin.
-
116
Ans: CCA (to yesterday's questions)
- 1. Which one of the following tumors shows calcification on CT Scan:
- a. Ependymoma
- b. Meduloblastoma
- c. Meningioma
- d. CNS lymphoma
Ans, C, Meningioma
Ref, Diagnostic Radiology Essentials, Grainger & Allison 2e/ p734
- Explanation
- CT Features of Meningioma is :
- A hyperdense extra-axial mass (60%), calcification (20%) intense and uniform enhancement bone hyperostosis indicates the site of meningeal attachment (and is particularly common when the skull base or anterior cranial fossa is involved)
- 2. The most frequent cause of aortic valve incompetence and regurgitation is
- a. Latent syphilis
- b. Infective endocarditis
- c. Rheumatic fever
- d. Aortic dissection
Ans, C, Rheumatic fever
Ref, Robbins & Cotran, Pathologic basis of disease, 7e, p 1257-1272
- Explanation
- Aortic regurgitation (AR) is rheumatic in origin in approximately 70% of cases. Much less frequently it is due to syphilis, ankylosing spondylitis (rarely), infective endocarditis, aortic dissection, or aortic dilation from cystic medial necrosis. Congenital forms of aortic stenosis occur fairly frequently, but AR is rarely congenital in origin. In chronic AR, patients remain asymptomatic for many years, but clinical manifestations include exertional dyspnea, angina, and left ventricular failure. Owing to the rapidly falling arterial pressure during late systole and diastole, there is often wide pulse pressure, Corrigan’s “water-hammer” pulse, capillary pulsations at the nail beds, and a pistolshot sound over the femoral arteries. A blowing diastolic murmur is heard along the left sternal border. Volume overload of the heart is the basic defect and results in left ventricular dilation and hypertrophy.
- 3. True about citrate in ORS:
- a. Increases shelf life
- b. Nutritious
- c. Cheaper
- d. Tastier
Ans, A, Increases shelf life
Ref, Park’s Textbook of Preventive and Social Medicine 22nd edition, p203
Explanation
- • In WHO ORS, sodium bicarbonate has been replaced by trisodium citrate:
- – Makes the product more stable
- – Results in less stool output (especially in high-output diarrhoea like cholera) as it increases intestinal absorption of sodium and water
-
117
DDB (Correct answers to Wednesday's questions)
- Q1. Miller-Fisher syndrome (MFS) is characterized by all except:
- a.External ophthalmoplegia
- b.Ataxia
- c.Areflexia
- d.Proximal parasthesias
- Ans: d, proximal parasthesias
- Ref: Nelson textbook of pediatrics 21st edition pg12899
- • Miller-Fisher syndrome (MFS) is an uncommon GBS variant associated with acute external (and occasionally internal) ophthalmoplegia, ataxia, and areflexia.
- • The 6th cranial nerve is most often involved in MFS.
- • Although areflexia is seen in MFS, patients have no or only very mild lower extremity weakness, compared with GBS.
- • Distal paresthesias are common in MFS.
- • Urinary incontinence or retention is a complication in approximately 20% of cases but is usually transient.
- • MFS overlaps clinically with Bickerstaff brainstem encephalitis.
- Q2. Spore of which bacteria is most commonly used as sterilization control of Plasma sterilization?
- a.B. subtilis
- b.B. pumilis
- c.Cl. tetani
- d.B. stearothermophilus
- Ans: d, B. stearothermophilus
- Ref: Apurba Sastry’s Essentials of Medical Microbiology, 1st Edition, Pg 38
- Plasma sterilizers:
- • It is a special device use to create the plasma state (commercial brands, such as Sterrad and Plazlyte). They maintain a uniform vacuum inside the chamber.
- • Chemical sterilant: Such as H202 alone or a mixture of H202 and peracetic acid are used which provide O and OH.
- • Low temperature is maintained (<5O°C) throughout the process which preserves the integrity of heat labile items.
- • It is used for sterilization of surgical instruments.
- • Biological Sterilization Indicator of plasma sterilization: B. stearothermophilus(most common),B.subtilis subspecies niger.
- Q3. The normal IOP is:
- a.0-10 mm Hg
- b.10-20 mm Hg
- c.20-30 mmHg
- d.25-35mm Hg
- Ans:b ,10-20 mm Hg
- Ref: : Parson’s disease of eye 22nd edition pg31
- • The intraocular pressure within the eye normally varies from 10 to 20 mm Hg.
- • It is most accurately measured by manometry, wherein a small cannula is inserted into the anterior chamber and connected with a small-bore mercury or saline manometer.
- • Such a technique is used experimentally on animals but its clinical application is obviously impossible.
- • The sclera is only very slightly elastic and is rendered tense by the internal pressure, allowing the intraocular pressure to be measured by the degree to which it can be indented on the application of a standard weight or flattened by a measured pressure with considerable accuracy. Such a method is used clinically in tonometry.
-
118
- 1. Pilon fracture is:
- a. Avulsion fracture of lateral tibial condyle; associated with ACL injury
- b. Intraarticular fracture of tibial condyle
- c. Intraarticular comminuted distal tibia fracture
- d. Salter-Harris III of lateral distal tibia (because of later epiphyseal fusion)
- Ans: c
- Ref: Apley’s Orthopedics, 9th Edition; Page No: 916
- Pilon fractures
- Unlike the twisting injuries that cause the common ankle fractures, this injury to the ankle joint occurs when a large force drives the talus upwards against the tibial plafond, like a pestle (pilon) being struck into a mortar. There is considerable damage to the articular cartilage and the subchondral bone may be broken into several pieces; in severe cases, the comminution extends some way up the shaft of the tibia.
- • Segond: Avulsion fracture of lateral tibial condyle; associated with ACL injury
- • Bumper: Intraarticular fracture of tibial condyle
- • Pilon: Intraarticular comminuted distal tibia fracture
- • Tillaux: Salter-Harris III of lateral distal tibia (because of later epiphyseal fusion)
- • Triplane: Salter III/IV fracture of distal tibia
- 2. Which of the following milestone develops first?
- a. Mirror play
- b. Crawling
- c. Creeping
- d. Pincer grasp
- Ans: a
- Ref: OP Ghai Pediatrics 8th Edition, Page No: 52
- • By 6 months, a child vocalizes and smiles at his mirror image and imitates acts such as cough or tongue protrusion.
- • By the age of 8 months, he crawls (with abdomen on the ground) and by 10 months, creeps (abdomen off the ground, with weight on knees and hands).
- • By 9-10 months, the child approaches the pellet by an index finger and lifts it using finger thumb apposition, termed 'pincer' grasp.
- 3. Which of the following lasers is used for treatment of benign prostatic hyperplasia as well as urinary calculi?
- a. CO2 laser
- b. Excimer laser
- c. Ho: YAG laser
- d. Nd-YAG laser
- Ans: c
- Ref: Schwartz’s Principles of Surgery, Eleventh Edition, Page No: 1763, 1764
- • Surgical modalities for BPH continue to evolve towards less invasive endoscopic procedures. Transurethral resection of the prostate (TURP) remains the mainstay of endoscopic procedures, with low treatment failure and complication rates.
- • TUR syndrome is associated with prolonged use of hypotonic irrigation fluid, resulting in fluid overload and dilutional hyponatremia.
- • Symptoms include nausea/vomiting, bradycardia and hypertension, pulmonary edema, mental status changes, and rarely death. Other endoscopic modalities used today include bipolar TURP and various laser procedures (e.g., Ho:YAG laser enucleation of the prostate, Ho:YAG laser ablation of the prostate, and photoselective vaporization of the prostate) with the goal of enucleating or vaporizing prostatic tissue.
- • Normal saline is used for irrigation with these modalities, which greatly reduces the risk of TUR syndrome. Generally, laser procedures have been associated with shorter catheterization time and length of stay with comparable improvements in LUTS to open prostatectomy or TURP.
- • In urolithiasis, this procedure involves advancing a semi-rigid or flexible ureteroscope to the level of the stone and fragmenting it under direct visualization, often using a holmium:YAG laser.
- 4. SI unit of strength of a magnetic field is:
- a. Gauss
- b. Hounsfield unit
- c. Tesla
- d. Sievert
- Ans: c
- The tesla (symbol T) is the derived SI unit of magnetic flux density, which represents the strength of a magnetic field. One tesla represents one weber per square meter. The equivalent, and superseded, cgs unit is the gauss (G); one tesla equals exactly 10,000 gauss.
- 5. Which of the following most likely cause necrotizing enteritis?
- a. Clostridium botulinum
- b. Clostridium tetani
- c. Clostridium perfringens A
- d. Clostridium perfringens C
- Ans: d
- Ref: Jawetz Microbiology 27th Edition, Page No: 186
- C perfringens type C produces necrotizing enteritis (pigbel) that can be highly fatal in children.
- Clostridium perfringens A produce Lecithinase and is responsible for gas gangrene.
- Enterotoxin-producing strains of C perfringens may also play a role in antibiotic associated diarrhea and necrotizing enterocolitis in infants.
- 6. A person who has lost his foot in an accident and is not able to walk is an example of:
- a. Disease
- b. Disability
- c. Impairment
- d. Handicap
- Ans: b
- Ref: Park's Textbook of Preventive and Social Medicine; 24th Edition; Page No: 44
- • Impairment: An impairment is defined as "any loss or abnormality of psychological, physiological or anatomical structure or function", e.g., loss of foot, defective vision or mental retardation. Impairment may be visible or invisible, temporary or permanent, progressive or regressive. Further, one impairment may lead to the development of "secondary" impairments as in the case of leprosy where damage to nerves (primary impairment) may lead to plantar ulcers (secondary impairment).
- • Disability: Because of impairment, the affected person may be unable to carry out certain activities considered normal for his age, sex, etc. This inability to carry out certain activities is termed "disability". A disability has been defined as "any restriction or lack of ability to perform an activity in the manner or within the range considered normal for a human being".
- • Handicap: As a result of disability, the person experiences certain disadvantages in life and is not able to discharge the obligations required of him and play the role expected of him in the society. This is termed "handicap", and is defined as "a disadvantage for a given individual, resulting from impairment or a disability, that limits or prevents the fulfilment of a role that is normal (depending on age, sex, and social and cultural factors) for that individual".
-
119
Answers to yesterday’s daily Questions :
- 1. Water hammer pulse is seen in:
- a. Mitral stenosis
- b. Aortic stenosis
- c. Aortic incompetence
- d. Myocardial infarction
- Ans,c.Aortic incompetence
- Ref: Ganong 22nd ed./pg.568
- Water Hammer Pulse or Corrigan Pulse is seen in :
- • Physiological - Exercise, Emotion, Pregnancy, Alcohol
- • Hyperkinetic circulatory states: Anaemia, Thyrotoxicosis, Paget’s disease of bone, Beriberi, Anoxic corpulmonale.
- • Leak in Atrial side of Circulation: AR, PDA, AV fistula, Large VSD
- • Complete Heart Block
- 2. Polysaccharide pneumococcal vaccine used in all except:
- a. Sickle cell anemia
- b. Less than 2 years of age
- c. More than 65 years of man
- d. Asplenia
- Ans,b.Less than 2 years of age
- Harrison’s 18th edition/pg.1158-1159 ;Nelson 19th edition/pg.914
- • Immunologic responsiveness and efficacy following administration of pneumococcal polysaccharide vaccines is unpredictable in children < 2 years of age.
- • High risk children > 2 years of age, such as those with asplenia, sickle cell disease, some types of immune deficiency (eg.antibody deficiencies),HIV infections, cochlear implants,CSF leaks, Diabetes Mellitus and chronic lung, heart or kidney disease (including nephritic syndrome) may also benefit from PPSV-23 administered after 2 years of age following priming with the scheduled dose of 13-valent pneumococcal conjugated vaccine (PCV-13).
- 3. Total duration of antibiotics in acute osteomyelitis is:
- a. 4 weeks
- b. 2 weeks
- c. 6 weeks
- d. 8 weeks
- Ans,c. 6 weeks
- Ref: Maheshwari 4th ed./p.162-165
- Treatment of acute osteomyelitis:
- • If acute osteomyelitis is suspected on clinical grounds, blood and fluid samples should be taken for laboratory investigation and then treatment is started immediately without waiting for final confirmation of the diagnosis
- • There are four important aspects to the management of the patient:
- • Supportive treatment of pain (analgesic) and dehydration.
- • Splintage of the affected part to provide comfort and to prevent contracture.
- • Appropriate antimicrobial therapy: 2 weeks intravenous and 4 weeks oral (total 6 weeks).
- • Surgical drainage : Surgical drainage is indicated :
- o If there is no improvement within 24-48 hours of iv antibiotics.
- o Formation of abscess: Surgical methods like Aspiration, Incision and drainage, Multiple drill holes, or small bone window depending on the situation.
-
120
- 1. A 50 years female presented in OPD with fatigue, pruritus and bone pain. On examination, hyperpigmentation and xanthelasma was found. On investigation, Anti mitochondrial antibodies were present. The probable diagnosis is
- a. Primary biliary cholangitis
- b. Primary sclerosing cholangitis
- c. Autoimmune hepatitis
- d. Idiopathic adulthood ductopenia
- Ans: a
- Ref: Harrison’s Principles of Internal Medicine; 20th Edition, Page No: 2407
- • PBC is seen in about 100-200 individuals per million, with a strong female preponderance and a median age of around 50 years at the time of diagnosis. Antimitochondrial antibodies (AMA) are present in about 90% of patients with PBC.
- • When symptoms are present, they most prominently include a significant degree of fatigue out of proportion to what would be expected for either the severity of the liver disease or the age of the patient. Pruritus is seen in ~50% of patients.
- • Physical examination can show jaundice and other complications of chronic liver disease including hepatomegaly, splenomegaly, ascites, and edema. Other features that are unique to PBC include hyperpigmentation, xanthelasma, and xanthomata, which are related to the altered cholesterol metabolism seen in this disease. Hyperpigmentation is evident on the trunk and the arms and is seen in areas of exfoliation and lichenification associated with progressive scratching related to the pruritus. Bone pain resulting from osteopenia or osteoporosis is occasionally seen at the time of diagnosis.
- 2. Wound contraction is mediated by:
- a. Epithelial cells
- b. Myofibroblast
- c. Collagen
- d. Elastin
- Ans: b
- Ref: Robbins Basic Pathology 10th Edition, Page No: 92
- • Wound strength increases because of cross-linking of collagen and increased size of collagen fibers.
- • In addition, there is a shift of the type of collagen deposited, from type III collagen early in repair to more resilient type I collagen. In well-sutured skin wounds, strength may recover to 70% to 80% of normal skin by 3 months.
- • Wound contraction is initially caused by myofibroblasts and later by cross-linking of collagen fibers.
- 3. Which of the following statements regarding Parkinson disease is true?
- a. Cigarette smoking reduces the risk of developing the disease.
- b. Parkinson disease has been identified as a monogenetic disorder related to mutations in the α-synuclein protein.
- c. The typical age of onset of symptoms is about 70 years.
- d. The hallmark pathologic feature of Parkinson disease is presence of neurofibrillary tangle and tau protein in the substantia nigra pars compacta.
- Ans: a
- Ref: Harrison’s Principles of Internal Medicine; 19th Edition, Page No: 2611
- • PD affects men and women equally, with a typical age of symptom onset around age 60. The frequency of PD increases with age but can present as early as the third decade of life.
- • Most cases of PD occur sporadically, although genetic factors play a role in some individuals. These individuals are more likely to present at a younger age. There is no single gene found to be associated with PD. The most likely genes to be altered in PD patients include α-synuclein, PINK1/Parkin, and LRRK2, but many others have been identified. Other epidemiologic risk factors for PD include exposure to pesticides, rural living, and drinking well water.
- • Cigarette smoking and caffeine are associated with reduced risk of PD. Pathologically, the characteristic finding in PD is degeneration of the dopaminergic neurons in the substantia nigra pars compacta. Lewy bodies, which are intracytoplasmic inclusions containing primarily α- synuclein, may also be seen.
- 4. Non absorbable sutures are all except:
- a. Silk
- b. Catgut
- c. Polypropylene
- d. Polyester
- Ans: b
- Ref: Bailey and Love’s Short Practice of Surgery 27th Edition, Page No: 92-93
- Absorbable sutures
- • Catgut (Plain and chromic)
- • Polyglactin
- • Polyglyconate
- • Polyglycolic acid
- • Polydioxanone
- • Polyglycaprone
- Non-absorbable sutures
- • Silk
- • Linen
- • Surgical steel
- • Nylon
- • Polyester
- • Polybutester
- • Polypropylene
-
121
- 1. Sex cells are present in which layer on newborn ovary?
- a. Surface epithelium
- b. Subepithelial connective tissue layer
- c. Parenchymatous zone
- d. Central fluid
- Ans: c
- Ref: Shaw Gynecology 16th Edition, Page No: 25
- The Ovary of the Newborn
- At term, the fetal ovary measures 10–16 mm in length and is situated at the level of the brim of the pelvis. If a section is taken through the ovary and examined histologically, the following can be recognized.
- • The surface epithelium. This is a single layer of cuboi¬dal cells, which later gives rise to the surface epithelium of the adult ovary. It is morphologically continuous with the mesothelium of the peritoneum.
- • The subepithelial connective tissue layer. This layer gives rise to the tunica albuginea of the adult ovary and to the basement membrane beneath the surface epithelium.
- • The parenchymatous zone. This area is the cortex and also the most important area, as it contains the sex cells.
- 2. The estimated num¬ber of spermatids formed from a single spermatogonium is:
- a. 16
- b. 64
- c. 512
- d. 1024
- Ans: c
- Ref: Ganong Review of Medical Physiology, Lange; 25th Edition, Page No: 418
- Spermatogenesis
- Spermatogonia, the primitive germ cells next to the basal lamina of the seminiferous tubules, mature into primary spermatocytes.
- This process begins during ado¬lescence.
- The primary spermatocytes undergo meiotic division, reducing the number of chromosomes. In this two-stage pro¬cess, they divide into secondary spermatocytes and then into spermatids, which contain the haploid number of 23 chromo¬somes.
- The spermatids mature into spermatozoa (sperm).
- As a single spermatogonium divides and matures, its descendants remain tied together by cytoplasmic bridges until the late sper¬matid stage. This arrangement helps ensure synchrony of the differentiation of each clone of germ cells.
- The estimated num¬ber of spermatids formed from a single spermatogonium is 512.
- The formation of a mature sperm from a primitive germ cell by spermatogenesis in humans spans approximately 74 days.
- 3. Which of the following is considered against code of medical ethics?
- a. Publishing professional service information in practice website
- b. Publishing your educational qualification in practice website
- c. Post medical activities in Facebook with an intent of procuring patients
- d. All of the above
- Ans: c
- A medical practitioner may publish his professional service information in either his practice website or the website of medical practice group.
- The website may carry only the service information which is permitted by the council.
- Medical practitioner should not post her/his medical activities in social medias (like Facebook) with an intent of procuring patients or with a sense of self-grandiosity as it is against the code of medical ethics.
- 4. Morison’s pouch is:
- a. Right subphrenic space
- b. Right subhepatic space
- c. Left subhepatic space
- d. Left subphrenic space
- Ans: b
- Ref: Gray’s anatomy, 41st Edition; Page No: 1107
- Right subhepatic space (hepatorenal pouch)
- • The right subhepatic space lies between the inferior surface of the right lobe of the liver and the upper pole of the right kidney.
- • It is bounded superiorly by the inferior layer of the coronary ligament, laterally by the right lateral abdominal wall, posteriorly by the anterior surface of the upper pole of the right kidney, and inferomedially by the hepatic flexure, transverse mesocolon, second part of the duodenum, and part of the head of the pancreas.
- • In the supine position, the hepatorenal pouch (of Morison) is more dependent than the right paracolic gutter.
- • It is a site where a pathological fluid collection may develop.
- 5. When is imperforate hymen usually detected?
- a. In utero
- b. During neonatal age
- c. At 3 – 6 years
- d. At 14 – 16 years
- Ans: d
- Ref: Dutta Gynecology 6th Edition, Page No: 41
- • Imperforate hymen is due to failure of disintegration of the central cells of the Müllerian eminence that projects into the urogenital sinus.
- • The existence is almost always unnoticed until the girl attains the age of 14–16 years.
- • As the uterus is functioning normally, the menstrual blood is pent up inside the vagina behind the hymen (cryptomenorrhea). Depending upon the amount of blood so accumulated, it first distends the vagina (hematocolpos). The uterus is next involved and the cavity is dilated (hematometra). In the late and neglected cases, the tubes may also be distended after the fimbrial ends are closed by adhesions (hematosalpinx).
- 6. Rotigotine is a:
- a. GABA antagonist
- b. GABA agonist
- c. Dopamine agonist
- d. Dopamine antagonist
- Ans: C
- Ref: Katzung Pharmacology; 14th edition; Page 499
- Rotigotine
- The dopamine agonist rotigotine, delivered daily through a skin patch, is approved for treatment of early Parkinson’s disease. It supposedly provides more continuous dopaminergic stimulation than oral medication in early parkinsonism; its efficacy in more advanced disease is less clear. Benefits and side effects are similar to those of other dopamine agonists but reactions may also occur at the application site and are sometimes serious.
-
122
- Q.1 Under Southhampton wound grading system, Clear or haemoserous discharge fall under which grade?
- a. Grade II
- b. Grade III
- c. Grade IV
- d. Grade V
- Ans: b, Grade III
- Ref: Short Practice of Surgery, Bailey and Love, 25th edition, page no.36
- Explanation:
- Southampton wound grading system
- Grade Appearance
- 0 Normal healing
- I Normal healing with mild bruising or erythema
- Ia Some bruising
- Ib Considerable bruising
- Ic Mild erythema
- II Erythema plus other signs of inflammation
- IIa At one point
- IIb Around sutures
- IIc Along wound
- IId Around wound
- III Clear or haemoserous discharge
- IIIa At one point only ( 2 cm)
- IIIb Along wound (> 2 cm)
- IIIc Large volume
- IIId Prolonged (> 3 days)
- Major complication
- IV Pus
- IVa At one point only ( 2 cm)
- IVb Along wound (> 2 cm)
- V Deep or severe wound infection with or without tissue breakdown; haematoma requiring aspiration
- Q.2 All of the following are known complications of ulcerative colitis except:
- a. Stricture
- b. Perforation
- c. Toxic megacolon
- d. Carcinoma
- Ans: a. Stricture
- Ref: Short Practice of Surgery, Bailey and Love, 26th edition, page no.1145.
- Explanation:
- Complications of ulcerative colitis
- ■ Acute
- Toxic dilatation
- Perforation
- Haemorrhage
- ■ Chronic
- Cancer
- Extra-alimentary manifestations: skin lesions, eye problems and liver disease
- Q3. A 10 years old child had sudden onset of gross hematuria, edema, hypertension, and renal dysfunction.On further inquiry there is history of pyoderma and was diagnosed as PSGN.The strain of group A β-hemolytic streptococcI responsible is:
- a.M1
- b.M4
- c.M25
- d.M49
- Ans: d, M49
- Ref:Nelson textbook of Pediatrics 21st edition page no:10682
•Group A β-hemolytic streptococcal infections are common in children and can lead to the postinfectious complication of acute glomerulonephritis (GN).
•Acute poststreptococcal glomerulonephritis (APSGN) is a classic example of the acute nephritic syndrome characterized by the sudden onset of gross hematuria, edema, hypertension, and renal dysfunction.
•It is one of the most common glomerular causes of gross hematuria in children and is a major cause of morbidity in group A β-hemolytic streptococcal infections.
•APSGN follows infection of the throat or skin by certain nephritogenic strains of group A β-hemolytic streptococci.
•Poststreptococcal GN commonly follows streptococcal pharyngitis during cold-weather months and streptococcal skin infections or pyoderma during warm-weather months. Although epidemics of nephritis have been described in association with throat (serotypes M1, M4, M25, and some strains of M12) and skin (serotype M49) infections, this disease is most commonly sporadic.
- Q4. 35 years old HIV positive Rama with 20 weeks of gestation came to OPD of Thapathali Maternity hospital and not taking antiretroviral medications. You have to counsel about perinatal transmission of HIV. Which of the following is true regarding perinatal transmission of HIV?
- a. Transmission of HIV 1 is less frequent than for HIV 2
- b. Maternal anti-retroviral therapy reduces the risk of vertical transmission by 10%
- c. Breast feeding doubles the risk of transmission
- d. All of the above
- Ans: c, Breast feeding doubles the risk of transmission
- Ref: Dc Dutta Obstetrics 8th edition, page no. 351
- Perinatal transmission of HIV:
- •Vertical transmission to the neonates is about 14–25%.
- •Transmission of HIV 2 is less frequent (1–4%) than for HIV 1 (15-40%).
- •Transplacental transmission occurs: 20% before 36 weeks, over 80% of transmissions occur around the time of labor and delivery.
- •Vertical transmission is more in cases with preterm birth and with prolonged membrane rupture.
- •Risks of vertical transmission is directly related to maternal viral load (measured by HIV RNA) and inversely to maternal immune status (CD4 + count).
- •Maternal anti-retroviral therapy reduces the risk of vertical transmission by 70%.
- •Breastfeeding doubles the risk of MTCT transmission (14% to 28%).
-
123
- 1. Graham steel murmur is heard in: [NAMS 2017]
- a. Aortic incompetence
- b. Pulmonary incompetence
- c. Aortic stenosis
- d. Tricuspid stenosis
- Ans: b
- Ref: Harrison’s Principles of Internal Medicine; 19th Edition, Page No: 51e-5
- Pulmonic regurgitation (PR) results in a decrescendo, early to middiastolic murmur (Graham Steell murmur) that begins after the pulmonic component of S2 (P2), is best heard at the second left interspace, and radiates along the left sternal border. The intensity of the murmur may increase with inspiration.
- 2. Skin pigmentation in bronze diabetes is due to: [NAMS 2014]
- a. Hemosiderin
- b. Lipofuscin
- c. Melanin
- d. Carotenoids
- Ans: c
- Ref: Robbins Basic Pathology 10th Edition, Page No: 656, 657 | Ref: Goljan Pathology 5th Edition, Page No: 546
- Hemochromatosis is caused by excessive absorption of iron, which is primarily deposited in parenchymal organs such as the liver and pancreas, as well as in the heart, joints, and endocrine organs.
- Although skin pigmentation is partially attributable to hemosiderin deposition in dermal macrophages and fibroblasts, most of the coloration results from increased epidermal melanin production.
- Bronze diabetes
- • Type I diabetes mellitus develops in 60% of cases and is caused by destruction of the ����-islet cells in the pancreas.
- • Hyperpigmentation occurs in the skin in 75% of cases because excess iron in the skin increases melanin production
- 3. Privation of any member of joint is a [NAMS 2012]
- a. Serious hurt
- b. Grievous hurt
- c. Simple hurt
- d. Dangerous hurt
- Ans: b
- Section 14 of Criminal (Code) Act, 2074 (2017)
- 4. Beta-oxidation of odd carbon fatty acid chain produces [NAMS 2011]
- a. Succinyl CoA
- b. Propionyl CoA
- c. Butyryl CoA
- d. Malonyl CoA
- Ans: b
- Ref: Harper Biochemistry 30th Edition, Page No: 146
- The (relatively rare) fatty acids with an odd number of carbon atoms yield propionyl CoA as the product of the final cycle of β-oxidation, and this can be a substrate for gluconeogenesis, as can the glycerol released by lipolysis of adipose tissue triacylglycerol reserves.
- 5. Tylosis may be associated with carcinoma of [NAMS 2019]
- a. Eyelid
- b. Colon
- c. Esophagus
- d. Small intestine
- Ans: c
- Ref: SRB's Manual of Surgery; 5th Edition, Page No: 804
- Tylosis
- • Autosomal dominant condition seen from childhood
- • Soles and palms are involved called as palmoplantar keratoderma
- • Waxy, yellow lesions, which does not itch
- • 60% of members of families develop carcinoma oesophagus after the age of 60
- • Systemic retinoids are the drugs used for tylosis
-
124
- ANSWERS OF YESTERDAY’S QUESTIONS
- CORRECT ANSWER SEQUENCES: (BAD)
Q1.Rita a26 years old primi with history of rheumatic heart disease came to your clinic for antenatal checkup. Which of the following is the commonest heart lesion met during pregnancy?
- a. Mitral regurgitation
- b. Mitral stenosis
- c. Tricuspid regurgitation
- d. Aortic stenosis
- Ans b. Mitral stenosis
Ref: Dc Dutta Obstetrics 8th edition page no 319,323,324
- HEART DISEASE IN PREGNANCY
- INCIDENCE AND TYPES:
- • The incidence of cardiac lesion is less than 1% amongst hospital deliveries.
- • The commonest cardiac lesion is of rheumatic origin followed by the congenital ones.
- • The ratio between the two has fallen over the past two decades from 10: 1 to about 3: 1 or even 1: 1 in advanced countries.
- • Adequate treatment of rheumatic fever by appropriate antibiotics to cope with the group A β-hemolytic streptococcal infection, pari passu with the advancement in cardiac surgery to rectify the congenital heart lesions, are responsible for the change in the profile.
- • Rheumatic valvular lesion predominantly includes mitral stenosis (80%).
- • Predominant congenital lesions include patent ductus arteriosus, atrial or ventricular septal defect, pulmonary stenosis, coarctation of aorta and Fallot’s tetralogy.
- • Rare causes are hypertensive, thyrotoxic, syphilitic or coronary cardiac diseases.
- MITRAL STENOSIS:
- • Mitral stenosis is the commonest heart lesion met during pregnancy.
- • Normal mitral valve area ranges between 4 and 6 cm2.
- • Symptoms usually appear when stenosis narrows this to less than 2.5 cm2.
- • Women with mitral valve area ≤1 cm2, have the high rate of pulmonary edema (55%) and arrhythmia (33%).
- • In asymptomatic cases, the mortality is < 1% but once it is significantly symptomatic, mortality ranges between 5% and 15%.
- • Diagnosis and management has been mentioned earlier.
- • During labor continuous epidural analgesia is ideal and intravenous fluid overload is to be avoided.
- AORTIC STENOSIS:
- • Most cases of aortic stenosis are congenital, some are rheumatic in origin.
- • Normal aortic valve area is 3–4 cm2.
- • When it is reduced to less than or equal to 1 cm2, stenosis is significant.
- • Maternal mortality of significant aortic stenosis is about 15–20% with perinatal loss of about 30%.
- • Epidural anesthesia is contraindicated.
- • During labor, fluid therapy (125–150 ml/h) should not be restricted.
- • Left ventricular after load is high and the pregnant patient is sensitive to hemorrhage.
- Q2. Delivery of an antimicrobial process (gas or liquid) indirectly to the internal surfaces of an enclosed area is called:
- a. Fumigation
- b. Sanitization
- c. Aseptic processing
- d. Cleaning
- Ans a. Fumigation
Ref gerald,E-mcdonnel antisepsis ,page 3,4
- Fumigation:
- • Delivery of an antimicrobial process (gas or liquid) indirectly to the internal surfaces of an enclosed area.
- ����� An example is fogging, which is the indirect application of a biocidal liquid to a given area.
- Sanitization:
- • The removal or inactivation of microorganisms that pose a threat to public health.
- Aseptic processing:
- • The act of handling materials in a controlled environment in which the air supply, materials, equipment, and personnel are regulated to control microbial and particulate contamination within acceptable levels.
- Cleaning:
- • Removal of contamination (often referred to as “soil”) from a surface to the extent necessary for further processing or for the intended use.
- Q3. Date of bone can be corroborated by analyzing amino acid content of that bone. A fresh bone shows about:
- a. Five amino acids
- c. Seven amino acids
- b. Ten amino acids
- d. Fifteen amino acids
- Ans d. Fifteen amino acids
- Ref: Reddy, The essential of forensic medicine and toxicology 33rd edition, page number 131
- Dating of bones:
- • The date of a bone can be corroborated by following methods (Knight).
- • A recent bone will have about 4 to 5 g.% of nitrogen which gradually diminishes with decay.
- • Between 50 to 100 years, the nitrogen content is more than 3.5 g. %, and if it is more than 2.5 g.%, the age will be less than 350 years.
- • A fresh bone shows about 15 amino acids mostly derived from collagen.
- • Glycine and analine are predominant.
- • A bone more than 100 years old will contain 7 amino acids.
- • Proline and hydroxyproline tend to disappear after 50 years.
- • A bone less than 100 years old fluoresces in ultraviolet light over most of its cut surface. There is progressive loss, and it is absent in 5OO to 800 years.
- • Blood pigments persist up to 100 years since death in temperate zones.
- • Precipitin tests are negative after about 10 years.
- • There is no significant fall in the C14 content of bones during the first century after death.
- • After prolonged burial, probably over 5O years, histological examination of ground-sections of bone may show globular pockets of resorption, which result from the acid balance of the bacteria and their by-products.
- • Ancient bones tend to be dry, brittle, chalky and the marrow cavity is dry, free of fat andoften contains particles of earth or sand. Minerals in the bone may be replaced chemically by minerals in thesediments, while maintaining the original shape of thebones, a process termed diagenesis.
- • This occurs over along time periods and leads to fossilisation.
-
125
Answers and Winners of QUIZ for NAMS PAST QUESTION Based Model test (07:00 PM, Jestha 12)
- 1. Chance of HIV by needle stick injury [NAMS 2010]
- a. 0.3%
- b. 5%
- c. 0.9%
- d. 10%
- Ans: a
- Ref: Harrison’s Principles of Internal Medicine; 19th Edition, Page No: 1220
- Estimate d Per-Act Probability of Acquiring HIV from an Infected Source, by Exposure Act
- Type of Exposure Risk per 10,000 Exposures
- • Blood transfusion: 9250
- • Needle-sharing during injection drug use: 63
- • Percutaneous (needle-stick): 23
- • Receptive anal intercourse: 138
- • Insertive anal intercourse: 11
- • Receptive penile-vaginal intercourse: 8
- • Insertive penile-vaginal intercourse: 4
- 2. Least common type of finger prints is: [NAMS 2009]
- a. Loop
- b. Whorls
- c. Arches
- d. Composite
- Ans: d
- Ref: Gautam Biswas Forensic Medicine, 3rdEdition, Page No: 85
- Types of fingerprint ridges
- • Loop6 (Ulnar/radial): 60–70 %
- • Whorl: 30–35 %
- • Arch: 5���10 %
- • Composite: 2–3 %
- 3. Half life of Cobalt 60 is: [NAMS 2013]
- a. 5.2 hours
- b. 5.2 months
- c. 5.2 weeks
- d. 5.2 years
- Ans: d
- Ref: NDT Resource centre, radioactive half-life
- Radioisotope Half-life
- • Polonium-215: 0.0018 seconds
- • Bismuth-212: 60.5 seconds
- • Sodium-24: 15 hours
- • Iodine-131: 8.07 days
- • Cobalt-60: 5.26 years
- • Radium-226: 1600 years
- • Uranium-238: 4.5 billion years
- 4. Linitis plastica is which type of gastric carcinoma according to Bormann classification? [NAMS 2019]
- a. Type 1
- b. Type 2
- c. Type 3
- d. Type 4
- Ans: d
- Ref: Gastric cancer: Classification, histology and application of molecular pathology by Bing Hu et al; J Gastrointest Oncol. 2012 Sep; 3(3): 251–261.
- Based on Borrmann’s classification, the gross appearance of advanced gastric carcinomas can be divided into type I for polypoid growth, type II for fungating growth, type III for ulcerating growth, and type IV for diffusely infiltrating growth which is also referred to as linitis plastica in signet ring cell carcinoma when most of gastric wall is involved by infiltrating tumor cells.
- 5. Oxygen masks have a breathing bag for: [NAMS 2016]
- a. To store a supply of oxygen to allow deep breathing without waste of oxygen
- b. To serve extra source oxygen when source is switched off for automatic respiration
- c. To increase the blood solubility of oxygen
- d. To control the energy wasted in normal respiration
- Ans: a
- Ref: Ajay Yadav Anesthesia 6th Edition, Page No: 24
- Ref: Morgan & Mikhail’s Clinical Anesthesiology, 5th Edition, Page No: 35
- • Reservoir bags function as a reservoir of anesthetic gas and a method of generating positive-pressure ventilation. They are designed to increase in compliance as their volume increases. Three distinct phases of reservoir bag filling are recognizable.
- • After the nominal 3-L capacity of an adult reservoir bag is achieved (phase I), pressure rises rapidly to a peak (phase II). Further increases in volume result in a plateau or even a slight decrease in pressure (phase III). This ceiling effect provides some minimal protection of the patient’s lungs against high airway pressures, if the APL valve is unintentionally left in the closed position while fresh gas continues to flow into the circuit.
- • Some types of oxygen masks have a breathing bag made of plastic or rubber attached to the mask or oxygen supply hose to store a supply of oxygen to allow deep breathing without waste of oxygen with use of simple fixed flow regulators.
- Reservoir (Breathing) Bag are attached lo anesthesia breathing circuits to ventilate the patient. Bags with different capacities are available for various age groups.
- • Neonates: 250 mL
- • Children (up to 3 years): 500 mL
- • Children> 3 years: 1,000 mL
- • Adults: 2,000 mL
- The capacity of bag should be more than the patient tidal volume.
- Winners:
- Fastest correct answer respondents:
- - Dr Prerana Mallik
- - Dr Pragya Sharma
- Lucky draw winners:
- - Dr Bimal Pokharel
- - Dr Anjwani Rimal
Winners are requested to send an email to onlinepgnepal@gmail.com to cliam Free Access to Today evening NAMS Past Question Based Modle test (07:00 PM, Jestha 12, Monday)
-
126
- Ans: BDA (to yesterday’s questions)
- 1. The gene that is abnormal in cystic fibrosis is located on the long arm of chromosome 7 and encodes the cystic fibrosis transmembrane conductance regulator (CFTR). There are different types of mutations on the CFTR gene, according to which CF is grouped as CF Class I to CF Class V (five groups). Which one of them is commonest?
- a. Class I
- b. Class II
- c. Class III
- d. Class V
- Ans: B
- Ref, Ganong’s Review of Medical Physiology, 26th edition, page no. 612
- Cystic Fibrosis
- -Among whites, cystic fibrosis is one of the most common genetic disorders; greater than 3% of the United States population are carriers for this autosomal recessive disease.
- -The gene that is abnormal in cystic fibrosis is located on the long arm of chromosome 7 and encodes the cystic fibrosis transmembrane conductance regulator (CFTR), a regulated Cl– channel located on the apical membrane of various secretory and absorptive epithelia.
- -The number of reported mutations in the CFTR gene that cause cystic fibrosis is large (> 1000) and the mutations are now grouped into five classes (I–V) based on their effects on cellular function.
- 1. Class I mutations do not allow for synthesis of the protein.
- 2. Class II mutations have protein processing defects.
- 3. Class III mutations have a block in their channel regulation.
- 4. Class IV mutations display altered conductance of the ion channel.
- 5. Class V mutations display reduced synthesis of the protein.
- 2. A 45 year old lady has painful small joints of hands which are symmetrical bilaterally. She wakes up in the morning with stiffness of those joints which gradually improves over time with household works. Previously diagnosed as Rheumatoid arthritis, the lady now complains of reddening of her palms and swelling/lump in her right hand which is firm to touch like an unripe fruit. These lumps aka rheumatoid nodules mostly occur in which part of the hand/leg?
- a. Dorsa of hands
- b. Pressure points
- c. Dorsa of knees
- d. Along ulnar border of forearm
- Ans: D
- Ref, Illustrated Synopsis of Dermatology and Sexually Transmitted Diseases, Neena Khanna, 4th edition, page no. 240 Cutaneous manifestation in Rheumatoid Arthritis Though it is unusual for patients with rheumatoid arthritis to present initially to dermatologist, cutaneous manifestations are not uncommon and include:
- 1. Rheumatoid nodules
- - Seen in 20% of seropositive patients.
- - Marble-like nodules which may ulcerate.
- - Most commonly along ulnar border of forearm. Less commonly, on dorsa of hands and knees and pressure points.
- 2. Tiny finger-tip infarcts.
- 3. Purpura and vasculitic ulcers.
- 4. Palmar erythema.
- 3. A child was born to a mother who had prolonged labour during his delivery. Later in his childhood, he was diagnosed to have cerebral palsy. What is true about this condition?
- a. UMN type paralysis i.e. spastic
- b. LMN type paralysis i.e. flaccid
- c. LMN type paralysis i.e. spastic
- d. UMN type paralysis i.e. flaccid
- Ans: A
- Ref, A Short Textbook of Psychiatry, Niraj Ahuja, 7th edition, page no. 159 Cerebral Palsy
- - This is a syndrome consisting of a conglomeration of perinatal disorders of various aetiologies, presenting with a common feature of paralysis of limbs.
- - The paralysis may be monoplegia, hemiplegia, paraplegia, triplegia or quadriplegia.
- - It is usually of upper motor neuron type, presenting with spasticity.
- - The extrapyramidal symptoms may be present and seizures may occur often.
- - Mental retardation is present in about 70% of all cases, and ranges from mild to severe.
-
127
Answers and Winners of QUIZ for CEE MD/MS Based Online Model test (07:00 PM, Jestha 14)
- 1. Function of Magnesium
- a. Coagulation
- b. Release of PTH
- c. Excretion of Phosphate
- d. Increased intracellular sodium
- Ans: b
- Ref: Satyanarayan Biochemistry, 3rd edition, Page No: 613
- • Kidneys filter about 250 mmol of Ca++ every day, some 95 per cent of which is reabsorbed by the tubules. The major portion of this filtered Ca+2 is taken up by proximal tubule without hormonal regulation.
- • A fine adjustment to the amount reabsorbed occurs in distal tubules under the influence of PTH (PTH-uptake).
- • Plasma level of ionised calcium conc. is the principal regulator of PTH secretion by a simple negative feedback mechanism.
- • A threshold level of magnesium is required for PTH release. Hypermagnesaemia inhibits PTH secretion. PTH secretion is also subject to negative feedback by the vit D metabolite 1,25 (OH)2D3. PTH rapidly stimulates osteoclast activity, the increased bone resorption causing an increase in plasma Ca+2 and PO4; Vit D3 plays a permissive role for this effect.
- 2. E-2020 initiative is related to:
- a. Corona virus
- b. Elephantiasis
- c. Malaria
- d. Dengue
- Ans: c
- Ref: Official WHO website
- What is the E-2020 initiative?
- In May 2015, the World Health Assembly endorsed a new Global Technical Strategy for Malaria 2016-2030, setting ambitious goals aimed at dramatically lowering the global malaria burden over this 15-year period, with milestones along the way to track progress. A key milestone for 2020 is the elimination of malaria in at least 10 countries that had the disease in 2015. To meet this target, countries must report zero indigenous cases in 2020.
- According to a WHO analysis published in 2016, 21 countries have the potential to eliminate malaria by 2020. They were selected based on an analysis that looked at the likelihood of elimination across 3 key criteria:
- • trends in malaria case incidence between 2000 and 2014;
- • declared malaria objectives of affected countries; and
- • informed opinions of WHO experts in the field.
- Together, these 21 malaria-eliminating countries are part of a concerted effort known as the E-2020 initiative, supported by WHO and other partners, to eliminate malaria in an ambitious but technically feasible time frame.
- 3. A 29-year-old woman is found, on routine annual blood testing, to have an increase in unconjugated bilirubin. There is no evidence of hemolysis and liver tests are otherwise normal. Which of the following is the most likely diagnosis?
- a. Crigler-Najjar syndrome
- b. Dubin-Johnson syndrome
- c. Rotor’s syndrome
- d. Gilbert’s syndrome
- Ans: d
- Ref: Nelson Textbook of Pediatrics, Elsevier, 20th Edition, Page No: 399
- • Gilbert’s syndrome may be associated with impaired hepatic uptake of bilirubin.
- • It is caused by hereditary decrease in the activity of glucoronosyltransferase in the Uridine diphosphate glycosyltransferase 1 (UGT1) family. More severe enzyme deficits are the cause of the two variants of Crigler-Najjar syndrome.
- 4. A 19-year-old man has had progressive ataxia of gait and great difficulty in running. In the past year, he has developed hand clumsiness. Physical examination reveals pes cavus, kyphoscoliosis, and both cerebellar and sensory changes in the legs. There is a positive family history of Friedreich’s ataxia. Where are the pathologic changes seen in this condition most likely to be found?
- a. Spinal cord tracts
- b. Basal ganglia
- c. Cerebral cortex
- d. Peripheral autonomic nerves
- Ans: a
- Ref: Nelson Textbook of Pediatrics, Elsevier, 20th Edition, Page No: 602
- • This young man has Friedreich’s ataxia, associated with a gene defect on chromosome 9.
- • The pathologic changes are found in the spinal cord tracts. Degeneration is seen in the posterior columns, the lateral corticospinal tract, and the spinocerebellar tracts.
- • Ataxia, sensory loss, nystagmus, reflex changes, clubfeet, and kyphoscoliosis are the characteristic findings.
- • The heart is frequently involved, and cardiac disease is a common cause of death.
-
128
Answers: DCB (to yesterday's questions)
1. Routine radiological examination of a middle aged man shows "Spongy appearance" with central sunburst calcification. This is seen in:
- a. Pancreatic adenocarcinoma
- b. Mucinous cystadenocarcinoma
- c. Somatostatinoma
- d. Serous cystadenoma
Ans, D, Serous cystadenoma
Ref, The Pancreas: An Integrated Textbook of Basic Science, Medicine, and Surgery edited by Hans-Gunther Beger, 2009, Page 933.
- Explanation
- Serous cystadenoma is a rare solitary, benign cystic neoplasm of the pancreas. It consists of multiple small locules lined by cuboidal epithelium, the cells of which contain abundant glycogen (also called microcystic, serous, and glycogen-rich cystadenoma).
- Radiological appearance of serous cystadenoma of pancreas shows characteristic central sunburst calcification which is pathognomonic.
2. Adrenal cortical adenoma has following finding?
- a. Hyponatremia
- b. Hyperglycemia
- c. Low serum renin level
- d. Hyperkalemia
Ans, C, Low serum renin level
Ref, Robbins & Cotran, Pathologic basis of disease, 7e, p 1210 -1211
Explanation
An adrenal cortical adenoma secrets aldosterone (Conn syndrome). Hyperaldosteronism reduces the synthesis of renin by the juxtaglomerular apparatus in the kidney. Adrenal adenomas can be nonfunctional or can secrete glucocorticoids or mineralocorticoids. Aldosterone does not exhibit feedback suppression of the anterior pituitary, and corticotropin levels are not affected. Patients with hyperaldosteronism have low serum potassium levels and sodium retention. There is no effect on blood glucose.
- 3. ‘Demographic Processes’ does not include:
- a. Fertility
- b. Morbidity
- c. Mortality
- d. Social mobility
Ans, B, Morbidity
- Ref, Park’s Textbook of Preventive and Social Medicine 22nd edition, p441
- Explanation
- Demographic Processes: 5 processes continuously on work in a population, thus determining its’ size, composition and distribution
- • Fertility
- • Marriage
- • Mortality
- • Migration
- • Social mobility
-
129
- Answers to yesterday’s Questions (BDC)
- 1.Acute Infective Endocarditis with abscess formation is most commonly associated with:
- a. Listeria
- b. Staphylococcus
- c. Streptococcus
- d. Enterococcus
- Ans,b. Staphylococcus
- Ref: Harrison’s 17th /e.p.790
- • Acute Infective Endocarditis with abscess formation is most commonly associated with Staphylococcus.
- • The most common organism causing acute infective endocarditis overall is Staphylococcus aureus. Staphyloccoccus aureus endocarditis is particularly virulent and associated with annular and myocardial abscess formation and a higher mortality.
- • Abscesses may also occur with enterococcus but are more common with staphylococcus.
- 2. In vitro DNA amplification is done by:
- a. Blotting technique
- b. Recombinant technique
- c. Electrophoresis
- d. Polymerase chain reaction
- Ans,d. Polymerase chain reaction
- Harrison’s 18th ed./pg.507
- DNA amplification is done by PCR
- • Southern blotting is a technique for detecting specific DNA fragments in a complex mixture.
- • Northern blotting is used for detecting RNA fragments, instead of DNA fragments.
- • Western blotting is used to detect a particular protein in a mixture. The probe used is therefore not DNA or RNA, but antibodies. The technique is also called “immunoblotting”
- • PCR: In vitro techniques such as the polymerase chain reaction allow the amplification of specific DNA sequences so that minute quantities of foreign nucleic acids can be recognized in host specimens.
- 3. Fracture of the femoral head is called as:
- a. Gartland’s fracture
- b. Rolando Fracture
- c. Pipkin Fracture
- d. Bennet Fracture
- Ans,c. Pipkin Fracture
- Ref: Apley's 9th /e.p. 844
- Pipkin’s classification of femoral head fracture
- Type I: Femoral head fracture inferior (caudal) to fovea
- Type II: Femoral head fracture superior (cephalad) to fovea
- Type III: Femoral head fracture with associated femoral neck fracture
- Type IV: Type I, II or III with associated acetabular fracture
-
130
Answers and Winners of QUIZ for CEE MD/MS Based Online Modle test (11:00 AM, Jestha 17)
- 1. The specific antidote for fibrinolytic agents are:
- a. Vitamin K
- b. Protamine sulfate
- c. Epsilon amino-caproic acid
- d. Tranexamic acid
- Ans: c
- Ref: KD Tripathi Essentials of Medical Pharmacology; 8th Edition, Page No: 676
- Antifibrinolytic drugs inhibit plasminogen activation and are used to check fibrinolysis associated bleeding. They include:
- a. Epsilon amino-caproic acid (EACA): It is a lysine analogue which combines with lysine-binding sites of plasminogen and plasmin so that latter is not able to bind to fibrin and lyse it. It is a specific antidote for fibrinolytic agents and has been used in many hyperplasmninaemic states associated with excessive intravascular fibrinolysis resulting in bleeding. EACA is active orally and can be infused i.v. as well.
- b. Transexamic acid, like EACA binds to the lysine binding site on plasminogen and prevents its combination with fibrin leading to fibrinolysis. It is 7 times more potent than EACA and is preferred for prevention/control of excessive bleeding.
- Vitamin K is warfarin antagonist.
- Protamine sulfate is heparin antagonist.
- 2. A 14-year-old female with no past medical history presents to the emergency department of BPKIHS with nausea and abdominal pain. On physical examination, her blood pressure is 78/65, her respiratory rate is 30, her breath has a fruity odor, and capillary refill is > 3 seconds. Serum glucose is 820 mg/dL. After starting IV fluids, what is the next best step in the management of this patient?
- a. Intravenous regular insulin
- b. Subcutaneous insulin glargine
- c. Subcutaneous insulin lispro
- d. Intravenous Dextrose in water
- Ans: a
- This patient's presentation is consistent with diabetic ketoacidosis (DKA). Regular insulin is the most appropriate initial treatment for DKA.
- DKA is a medical emergency that occurs in both type I and type II diabetics, although it is more common in type I. In DKA, insulin deficiency and glucagon excess promote hyperglycemia and ketogenesis, resulting in an anion gap metabolic acidosis, osmotic diuresis and volume depletion. Left untreated, the condition may progress to coma and death. Regular insulin is short-acting, especially when given intravenously, where its effect is almost immediate and the half-life is very short. For the initial treatment of DKA, regular insulin is given as an IV bolus and then as a continuous drip. Glargine is the longest acting insulin; it lasts 18-24 hours without a distinct peak in action with subcutaneous administration. Insulin lispro is the shortest acting insulin; it starts working in 15 minutes and peaks in 30 minutes to 1 hour when administered subcutaneously.
- • Dextrose would increase this patient's serum glucose and is not the appropriate initial step in the treatment of this patient. Note, however, that dextrose plays an important role later in DKA therapy by preventing hypoglycemia.
- • Glucagon would increase this patient's serum glucose and is contraindicated in DKA.
- 3. A 3-year-old male who fell from a tree complains of severe pain over the right side of his chest because of a rib fracture at the midaxillary line. He is admitted to the Dadeldhura Sub-Regional Hospital due to his difficulty breathing. Radiographic and physical examinations reveal atelectasis, resulting from the accumulation of blood in his pleural space and resulting hemothorax. What is the most likely the source of bleeding to cause the hemothorax?
- a. Left common carotid artery
- b. Intercostal vessels
- c. Pulmonary arteries
- d. Pulmonary veins
- Ans: b
- Ref: Gray’s Anatomy for Students, 146-148
- Due to rib fracture, the intercostal vessels are damaged, parietal pleura is torn, and blood flows into the pleural space. The loss of negative pressure within the pleural cavity results in collapse of the lung. The carotid vessels would not be affected by the described injury. The pulmonary vessels are found within the parenchyma of the lungs and would not be injured due to an external injury such as that described. The internal thoracic artery is well protected by the sternum and is not the cause of this hemothorax.
- 4. LCAT activity is associated with which of the lipoprotein complex?
- a. VLDL
- b. Chylomicrons
- c. LDL
- d. HDL
- Ans: d
- Ref: Harper Biochemistry 30th Edition, Page No: 272
- Lecithin: cholesterol acyltransferase (LCAT) activity is associated with HDL containing apo A-I. As cholesterol in HDL becomes esterified, it creates a concentration gradient and draws in cholesterol from tissues and from other lipoproteins, thus enabling HDL to function in reverse cholesterol transport.
-
131
- 1. A phase of avid bone mineralization, with hypocalcemia due to rapid movement of calcium from the circulation into the skeletal compartment is
- a. Hungry bone syndrome
- b. Osteopetrosis
- c. Osteoporosis
- d. Pseudohypoparathyroidism
- Ans: a
- • Hungry bone syndrome refers to a phase of avid bone mineralization, with hypocalcemia due to rapid movement of calcium from the circulation into the skeletal compartment; this tends to occur during the early phases of recovery from a severe mineralization defect or after a prolonged period of calcium resorption from bone.
- • A parallel increase in bone uptake of magnesium leading to hypomagnesemia may increase the severity of hypocalcemia. The hungry bone syndrome may be triggered by vitamin D therapy in the setting of severe rickets, parathyroidectomy, or rarely by thyroidectomy.
- Ref: UpToDate
- 2. Which of the following is a malignant tumor?
- a. Glomus tumor
- b. Chondroma
- c. Chordoma
- d. Chondroblastoma
- Ans: c
- Ref: Harrison’s Principles of Internal Medicine; 20th Edition, Page No: 119e-3
- The most common malignant tumors of bone are plasma cell tumors. The four most common malignant nonhematopoietic bone tumors are osteosarcoma, chondrosarcoma, Ewing’s sarcoma, and UPS. Rare malignant tumors include chordoma (of notochordal origin), malignant giant-cell tumor and adamantinoma (of unknown origin), and hemangioendothelioma (of vascular origin).
- Chordoma originate from embryonic remnants of the primitive notochord (earliest fetal axial skeleton, extending from the Rathke's pouch to the tip of the coccyx). Since chordomas arise in bone, they are usually extradural and result in local bone destruction. They are locally aggressive but uncommonly metastasize.
- 3. Biomedical concept of health is based on:
- a. Germ theory of disease
- b. Absence of pain
- c. Social and psychological factors
- d. Equilibrium between man and environment
- Ans: a
- Ref: Park's Textbook of Preventive and Social Medicine; 24th Edition; Page No: 13
- Traditionally, health has been viewed as an "absence of disease", and if one was free from disease, then the person was considered healthy. This concept, known as the "biomedical concept" has the basis in the "germ theory of disease" which dominated medical thought at the turn of the 20th century.
- 4. Which of the following drug is useful in acute attack of migraine?
- a. Bromocriptine
- b. Cinnarizine
- c. Sumatriptan
- d. Ondansetron
- Ans: c
- Ref: KD Tripathi Essentials of Medical Pharmacology; 7th Edition, Page No: 178
- Sumatriptan:
- • It is the first selective 5-HT1D/1B receptor agonist; activates other subtypes of 5-HT1 receptors only at very high concentrations, and does not interact with 5-HT2, 5-HT3, 5-HT4-7, α or β adrenergic, dopaminergic, cholinergic or GABA receptors.
- • Administered at the onset of an attack of migraine, sumatriptan is as effective and better tolerated than ergotamine. About 3/4 patients obtain complete/significant relief within 2–3 hours.
-
132
- 1. All of the following drugs can cause gynaecomastia except:
- a. Digoxin
- b. Amiloride
- c. Cimetidine
- d. Spironolactone
- Ans: b
- Ref: KD Tripathi Essentials of Medical Pharmacology; 7th Edition, Page No: 589
- Amiloride:
- • It is 10 times more potent than triamterene (dose 5–10 mg OD–BD).
- • At higher doses it also inhibits Na+ reabsorption in PT, but this is clinically insignificant. It decreases Ca2+ and Mg2+ excretion but increases urate excretion.
- • Usual side effects are nausea, diarrhoea and headache.
- • Amiloride blocks entry of Li+ through Na+ channels in the CD cells and mitigates diabetes insipidus induced by lithium.
- • Given as an aerosol it affords symptomatic improvement in cystic fibrosis by increasing fluidity of respiratory secretions.
- 2. Molotov cocktail is:
- a. Mixture device of bomb
- b. Simple petrol bomb thrown by hand
- c. Molotov, foreign minister of Russia died after having the cocktail
- d. Type of tank
- Ans: b
- Ref: Gautam Biswas Forensic Medicine, 3rdEdition, Page No: 290
- Classification of explosives (based on material used)
- High-order explosives (HEs) undergo detonation producing an instantaneous blast wave under extremely high pressure causing severe primary blast injury, e.g. TNT, dynamite, ammonium nitrate and C-4 ‘plastic’ explosives.
- Low-order explosives (LEs) undergo deflagration rather than detonation, and thus lacking in blast wave—uncommonly to cause the pulmonary and central nervous system injuries unique to primary blast injury. They are composed of propellants, such as black powder and pyrotechnics, such as fireworks and oil- or petroleum-based explosives such as Molotov cocktails.
- 3. What is the most important factor producing the disinfecting action of chlorine tablets?
- a. Chloride ions
- b. Hypochlorite ions
- c. Hypochlorous acid
- d. Hypoxide free radicals
- Ans: c
- Ref: Park's Textbook of Preventive and Social Medicine; 23rd Edition; Page No: 714
- • When chlorine is added to water, there is formation of hydrochloric and hypochlorous acids.
- • The disinfecting action of chlorine is mainly due to the hypochlorous acid, and to a small extent due to the hypochlorite ions. The hypochlorous acid is the most effective form of chlorine for water disinfection. It is more effective (70-80 times) than the hypochlorite ion.
- • Chlorine acts best as a disinfectant when the pH of water is around 7 because of the predominance of hypochlorous acid.
- 4. Liver cirrhosis in chronic alcohol consumers is associated with which chronic infection?
- a. Hepatitis A
- b. Hepatitis B
- c. Hepatitis C
- d. Hepatitis D
- Ans: C, Hepatitis C
- Ref: Harrison’s principles of internal medicine, 20th edition, page 2399
- • Chronic infection with hepatitis C virus (HCV) is an important comorbidity in the progression of alcoholic liver disease to cirrhosis in chronic drinkers.
- • Even light to moderate alcohol intake of 15–30 g/d increases the risk of cirrhosis and hepatocellular cancer in HCV-infected individuals. Patients with both alcoholic liver injury and HCV infection develop decompensated liver disease at a younger age and have poorer overall survival.
- • Increased liver iron stores and, rarely, porphyria cutanea tarda can occur as a consequence of the overlapping injurious processes secondary to alcohol and HCV infection.
- 5. COVID-19 specific VTM kits are being produced by Everest Parenterals Pvt Ltd in Nepal. This kit is used for:
- a. Rapid diagnostic test
- b. Polymerase chain reaction
- c. Transport of sample
- d. Purification of sample
Ans: c
- Commonly used Transport medium:
- Cary and Blair Medium: semi-solid, white colored transport medium for faeces that may contain Salmonella, Shigella, Vibrio or Campylobacter
Amies medium with charcoal: Charcoal helps eliminate metabolic products of bacterial growth, which may be especially useful in the isolation of fastidious organisms like However it is suggested that, some other pathogens like Campylobacter can also survive in such medium.
Amies medium without charcoal: Are ideal for the isolation of Mycoplasma and Ureaplasma
Stuarts medium: Commonly used for transporting specimens suspected of having gonococci. Also used for transporting Throat, wound and skin swabs that may contain fastidious organisms.
Venkatraman Ramakrishnan (VR) medium: Used to transport feces from suspected cholera patients.
Sach’s buffered glycerol saline: Used to transport faeces from patients suspected to be suffering from bacillary dysentery.
Viral Transport Medium (VTM): Viral Transport Medium (VTM) is ideal for diagnosis of viral infection. Ocular, respiratory and tissue swabs can be submitted in this medium. Fluid samples such as tracheal wash specimens or peritoneal fluid should be submitted as is, in sterile vials which prevent desiccation. In the absence of viral transport medium, submit swabs in sterile, sealed vials with several drops of saline added, to prevent desiccation. Cotton, plastic, wood-handled, and Dacron and other synthetic swabs are all acceptable. Calcium alginate swabs should be avoided. Bacterial transport media are not appropriate for virology.
-
133
- 1. Aspirin is recommended in patient with Diabetes mellitus if
- a. Age>50 years even without any cardiovascular risk
- b. Age>50 years even with any one cardiovascular risk factors
- c. Age>50 years even with at least any two cardiovascular risk factors
- d. Contraindicated in patient with DM and age >50 years
- Ans: b
- Ref: Harrison’s Principles of Internal Medicine; 20th Edition, Page No: 2881
- • Antiplatelet therapy reduces cardiovascular events in individuals with DM who have CHD and is recommended.
- • The ADA recommends considering the use of aspirin for primary prevention of coronary events in individuals with diabetes with an increased cardiovascular risk (>50 years with at least one risk factor such as hypertension, dyslipidemia, smoking, family history, or albuminuria).
- • ASA is not recommended for primary prevention in those with a low cardiovascular risk (<50 years with no risk factors).
- • The aspirin dose is the same as in nondiabetic individuals.
- 2. Simmonds’ test helps to check integrity of the following tendon?
- a. Brachioradialis
- b. Flexor hallucis longus
- c. Achilles
- d. Quadriceps
- Ans: c
- Ref: Bailey and Love’s Short Practice of Surgery 27th Edition, Page No: 460
- Achilles tendon test
- • Feel the gastrocnemius and soleus bellies and the whole length of the tendon for gaps (rupture), tenderness or swelling.
- • Also identify the posterolateral (Haglund’s) prominence of the calcaneus and palpate the retro-Achilles bursa.
- • The test for integrity of the tendon is the Thompson’s or Simmonds’ test. Do not be misled by the patient’s ability to stand on tiptoes – some people can do this using their long toe flexors alone. Lie the patient prone and allow their calves to rest on your forearms. Squeeze each calf in turn and watch for movement at the ankle joint. Lack of movement may indicate a rupture.
- 3. One who has delivered twins is:
- a. Nullipara
- b. Primipara
- c. Multipara
- d. Parturient
- Ans: b
- Ref: Dutta Obstetrics 8th Edition, Page No: 107
- Nullipara is one who has never completed a pregnancy to the stage of viability. She may or may not have aborted previously.
- Primipara is one who has delivered one viable child. Parity is not increased even if the fetuses are many (twins, triplets).
- Multipara is one who has completed two or more pregnancies to the stage of viability or more.
- Multigravida is one who has previously been pregnant. She may have aborted or have delivered a viable baby.
- Parturient is a women in labor.
- 4. Schwann cells ensheathe axons on what ratio basis?
- a. 1:1
- b. 1:5
- c. 1:50
- d. 1:1000
- Ans: a
- Ref: Gray’s anatomy, 41st Edition; Page No: 51
- Myelin is formed by oligodendrocytes (CNS) and Schwann cells (PNS).
- A single oligodendrocyte may ensheathe up to 50 separate axon segments, depending on calibre, whereas myelinating Schwann cells ensheathe axons on a 1 : 1 basis.
- 5. A 22 year old male presented with syncope during exercise. He has symptoms of dyspnea on exertion, palpitations. On physical exam, S4 gallop with systolic murmur without radiation to carotids seen. Maneuvers that decrease the murmur of obstructive HCM includes all except?
- a. Squatting
- b. Sustained handgrip
- c. Nitroglycerin
- d. Passive leg raising
- Ans: c
- Ref: Harrison Medicine 18th Edition, Page No: 1968
- Interventions that increase myocardial contractility like exercise, sympathomimetic amines & digitalis glycosides, and those that reduce ventricular volume like Valsalva maneuver, sudden standing, nitroglycerin, amyl nitrite or tachycardia, may all cause an increase in the gradient & the murmur.
- Conversely, elevation of arterial pressure by phenylephrine, squatting, sustained handgrip, augmentation of venous return by passive leg raising & expansion of the blood volume all increase ventricular volume & ameliorate the gradient & murmur.
-
134
- Q1. A patient was diagnosed to have direct inguinal hernia was taken to the operation theatre for repair. After laparotomy, it was found that he also had indirect inguinal hernia. What is your diagnosis?
- a. Mydl’s hernia
- b. Velpeau hernia
- c. Petit hernia
- d. Saddle bag hernia
- Ans: d, Saddle bag hernia
- Ref: Essentials of general surgery. Lippincott Williams & Wilkins.
- •A pantaloon hernia (dual hernia, Romberg hernia or saddle bag hernia) is defined as ipsilateral, concurrent direct and indirect inguinal hernias.
- •Hernial sacs are present on both sides of the inferior epigastric vessels, and separated by the posterior wall of the inguinal canal brought down by the direct hernia.
- Q2. As compared to Benzocaine, Prilocaine has which of the following features(s)?
- a. Produce anesthesia for short duration
- b. Hydrolysed by plasma esterase
- c. Bind to α1 acid glycoprotein in plasma
- d. All of the above
Ans: c, Bind to α1acid glycoprotein in plasma
- Ref: Kd Tripathi, Essentials of medical pharmacology 7th edition, page number 361
- •Benzocaine is an ester- linked local anesthetic, while Prilocaine is Amide-linked local anesthetic. So Prilocaine has features of Amide-linked local anesthetics.
- •Answer is option C.
- Chemical structure of local anesthetics agents:
- •The clinically useful LAs are weak bases with amphiphilic property.
- •A hydrophilic secondary or tertiary amine on one side and a lipophilic aromatic residue on the other are joined by an alkyl chain through an ester or amide linkage.
- Ester-linked local anesthetics:
- •Cocaine,
- •Procaine,
- •Chloroprocaine,
- •Tetracaine,
- •Benzocaine.
- Q3. You are seeing a patient with suspected CSF rhinorrhea. All of the following points are in favour of CSF while differentiating it from nasal secretion except;
- a. Thin, watery and clear
- b. Sugar content less than 10 mg/dl
- c. Presence of 2 transferrin
- d. Cannot be sniffed back
- Ans: b, Sugar content less than 10 mg/dl
- Ref: Disease of Ear, Nose and Throat, PL Dhingra, 6th edition, Page no.164
- Q4. Thyroid neoplasm with follicular epithelium and poorly differentiated cells belong to which type?
- a. Follicular
- b. Papillary
- c. Anaplastic
- d. Medullary
- Ans: c, Anaplastic
- Ref: Ref: Short Practice of Surgery, Bailey and Love, 27th edition, page no.816
-
135
Answers and Winners of QUIZ for BPKIHS Past Question Based Model test (07:00 PM, Jestha 19, Monday).
- 1. Hatters' Shakes are seen in relation with: [BPKIHS 2015]
- a. Lead poisoning
- b. Mercury Poisoning
- c. Lithium Toxicity
- d. Arsenic Poisoning
- Ans: b
- Ref: Gautam Biswas Forensic Medicine, 3rdEdition, Page No: 496
- Intention tremors (Danbury tremors/shaking palsy) in Chronic Mercury Poisoning (Hydrargyrism)
- • It occurs first in the hands, then progresses to the lips and tongue, and finally involves the arms and legs.
- • Tremor is moderately coarse and is interspersed by jerky movements. The patient may not display much tremor during an accustomed job, but if he is being observed, he may begin to shake violently.
- ��� In the advanced stage, the person is unable to dress himself, write legibly or walk properly.
- • They are also called hatter’s shakes or glass blower’s shakes, as they are common in persons working with mercury in glass-blowing and hat industries. The most severe form of tremors is known as concussion mercurilis.
- Other compounds causing tremors:
- • Alcohol
- • Phosphorus
- • Phenothiazines
- • Carbon monoxide
- • Antidepressants (tricyclic)
- • Caffeine and theophylline
- 2. The nozzle of the Ear Syringe in syringing is pointed: [BPKIHS 2015]
- a. Posterosuperiorly
- b. Posterioinferiorly
- c. Anterosuperiorly
- d. Anteroinferiorly
- Ans: a
- Ref: PL Dhingra and Shruti Dhingra ENT and HNS 7th Edition, Page No: 56
- Technique of syringing the ear
- • Patient is seated with ear to be syringed towards the examiner. A towel is placed round his neck. A kidney tray is placed over the shoulder and held snugly by the patient. Patient’s head is slightly tilted over the tray to collect the return fluid.
- • Pinna is pulled upwards and backwards and a stream of water from the ear syringe is directed along the posterosuperior wall of the meatus. Pressure of water, built up deeper to the wax, expels the wax out.
- • If wax is tightly impacted, it is necessary to create a space between it and the meatal wall for the jet of water to pass, otherwise syringing will be ineffective or may even push the wax deeper. Ear canal should be inspected from time to time to see if all wax has been removed. Unnecessary syringing should be avoided.
- 3. The antral nipple sign is seen in which of the following? [BPKIHS 2016]
- a. Peptic Ulcer
- b. Intussusception
- c. Pyloric Stenosis
- d. Duodenal Atresia
- Ans: c
- Ref: Antral nipple sign by Dr Yusra Sheikh et al. in Radiopedia
- The antral nipple sign refers to redundant pyloric mucosa protruding into the gastric antrum and is seen in hypertrophic pyloric stenosis on ultrasound examination.
- The hypertrophied muscle is hypoechoic, and the central mucosa is hyperechoic. Diagnostic measurements include (mnemonic "number pi"):
- • Pyloric muscle thickness, i.e. diameter of a single muscular wall (hypoechoic component) on a transverse image: >3 mm (most accurate)
- • Length, i.e. longitudinal measurement: >15-17 mm
- • Pyloric volume: >1.5 cm3
- • Pyloric transverse diameter: >13 mm
- • With the patient's right side down the pylorus should be watched and should not be seen to open.
- Described sonographic signs include:
- • Antral nipple sign
- • Cervix sign
- • Target sign
- 4. Berger nephropathy is chiefly due to mesangial deposition of: [BPKIHS 2017]
- a. Fibrin and C3
- b. IgM and C3
- c. IgE and C3
- d. IgA and C3
- Ans: d
- Ref: Robbins Basic Pathology 10th Edition, Page No: 561
- IgA Nephropathy
- • IgA nephropathy is one of the most common causes of recurrent microscopic or gross hematuria and is the most common glomerular disease revealed by renal biopsy worldwide. This condition usually affects children and young adults and begins as an episode of gross hematuria that occurs within 1 or 2 days of a nonspecific upper respiratory tract infection.
- • An abnormally glycosylated IgA1 (i.e., galactose-deficient IgA1 [Gd-IgA1]) immunoglobulin is thought to play a central role in the pathogenesis. This abnormal IgA may elicit an autoimmune response, and autoantibodies may form large immune complexes with circulating IgA.
- • These complexes deposit in the glomerular mesangium; this unusual location may be related to physicochemical features of the IgA and may be facilitated by an IgA1 receptor (CD71) on mesangial cells.
- • The presence of C3 in the mesangium and the absence of C1q and C4 points to activation of the alternative complement pathway in the pathogenesis.
- 5. Koch postulates are not followed by: [BPKIHS 2019]
- a. M tuberculosis
- b. M leprae
- c. M avium
- d. M intercellulare
- Ans: b
- Ref: Jawetz’s Medical Microbiology; Lange; 27th Edition, Page No: 154
- • Koch’s postulates have remained a mainstay of microbiology; however, since the late 19th century, many microorganisms that do not meet the criteria of the postulates have been shown to cause disease. For example, Treponema pallidum (syphilis) and Mycobacterium leprae (leprosy) cannot be grown in vitro; however, there are animal models of infection with these agents.
- • In another example, Neisseria gonorrhoeae (gonorrhea), there is no animal model of infection even though the bacteria can readily be cultured in vitro; experimental infection in humans has been produced that substitutes for an animal model.
-
136
- 1. Basal body temperature increases around the time of ovulation in a female primarily due to:
- a. Estrogen
- b. Progesterone
- c. Cytokines
- d. Both a and b
- Ans: B, Progesterone
- Ref, Ganong’s Review of Medical Physiology, 26th edition, page no. 396
- Indicators of Ovulation
- • Knowing when during the menstrual cycle ovulation occurs is important in increasing fertility or, conversely, in fam¬ily planning.
- • A convenient and reasonably reliable indica¬tor of the time of ovulation is a change—usually a rise—in the basal body temperature. The rise starts 1–2 days after ovulation.
- • Women interested in obtaining an accurate temperature chart should use a digital thermometer and take their temperatures (oral or rectal) in the morning before getting out of bed.
- • The cause of the temperature change at the time of ovulation is probably the increase in progester¬one secretion, since progesterone is thermogenic.
- • A surge in LH secretion triggers ovulation, and ovulation normally occurs about 9 h after the peak of the LH surge at mid¬cycle.
- The ovum lives for approximately 72 h after it is extruded from the follicle, but it is fertilizable for a much shorter time than this.
- 2. Which of the following is the site of highest airway resistance?
- a. Largest bronchi
- b. Medium-sized bronchi
- c. Smallest bronchi
- d. Alveoli
- Ans: B, Medium-sized bronchi
- Ref, BRS Physiology, 6th edition, page no. 144
- • The medium-sized bronchi actually constitute the site of highest resistance along the bronchial tree.
- • Although the small radii of the alveoli might predict that they would have the highest resistance, they do not because of their parallel arrangement.
- • In fact, early changes in resistance in the small airways may be “silent” and go undetected because of their small overall contribution to resistance.
- 3. In active chicken pox, skin lesions of various stages can be seen simultaneously distribution of which follows one of the following pattern. Which one is the most correct one?
- a. Centrifugal
- b. Centripetal
- c. Both pattern exists
- d. None of the above
- Ans: B, Centripetal
- Ref, Illustrated Synopsis of Dermatology and Sexually Transmitted Diseases, Neena Khanna, 4th edition
- Varicella-Zoster Infections
- Varicella
- • Synonym: Chicken pox.
- • Etiology: Varicella-zoster virus.
- • Prodrome: Fever and malaise.
- • Morphology: Crops of papules with erythematous halo; rapidly become vesicular (dew drops on rose petal appearance), then pustular. Eruption at different stages present. Heal with minimal scarring unless complicated by secondary infection or hemorrhagic lesions (as seen in immunocompromised). Adults usually have a more severe eruption.
- • Site: Centripetal distribution.
- • Prophylaxis: Live attenuated vaccine.
- • Treatment: Specific antiviral therapy (acyclovir 800 mg, five times daily × 7–10 days) in adults and in immunocompromised individuals. None needed in children.
- 4. Of the following defense mechanisms, which is considered the most mature?
- a. Sublimation
- b. Dissociation
- c. Regression
- d. Intellectualization
- Ans: A, Sublimation
- Ref. FA, USMLE step 1, 2020, Psychiatry
- • Sublimation, expressing an unacceptable emotion in a socially acceptable way, is classified as a mature defense mechanism.
- • Denial, dissociation, regression, and intellectualization are all classified as less mature defense mechanisms.
-
137
- 1. Latent autoimmune diabetes of adults (LADA) has an average onset at what age?
- a. < 35 years
- b. < 50 years
- c. 40 – 70 years
- d. 70 years
- Ans: b
- Ref: Harrison’s Principles of Internal Medicine; 19th Edition, Page No: 2407
- Some individuals (5–10%) with the phenotypic appearance of type 2 DM do not have absolute insulin deficiency but have autoimmune markers (GAD and other ICA autoantibodies) suggestive of type 1 DM (termed latent autoimmune diabetes of the adult). Such individuals are more likely to be <50 years of age, thinner, and have a personal or family history of other autoimmune disease than individuals with type 2 DM.
- 2. Pseudoparalysis occurs due to deficiency of which of the following vitamin?
- a. A
- b. C
- c. D
- d. K
- Ans: b
- Ref: Nelson Textbook of Pediatrics, Elsevier, 20th Edition, Page No: 329
- The early manifestations are irritability, loss of appetite, low-grade fever, musculoskeletal pain, and tenderness in the legs. These signs and symptoms are followed by leg swelling—most marked at the knees and the ankles—and pseudoparalysis.
- 3. Which zone of liver is first affected by viral hepatitis?
- a. Zone I
- b. Zone II
- c. Zone III
- d. No specific order
- Ans: a
- Ref: USMLE Step 1 First Aid 2020, Page No: 361
- Liver is divided into 3 zones:
- • Zone I (Periportal zone) is the best oxygenated zone and is most resistant to circulatory compromise. It is first affected by viral hepatitis.
- • Zone II (Intermediate zone) is first affected by Yellow fever.
- • Zone III (Pericentral vein/centrilobular zone) Least oxygenated zone and first affected by ischemia. It contains high concentration of cytochrome P-450. It is most sensitive to metabolic toxins (eg, ethanol, CCl4, halothane, rifampin). And is the site of alcoholic hepatitis.
- 4. ‘Tram line’ calcification seen in skull X –ray is characteristic of:
- a. Congenital cytomegalovirus injection
- b. Congenital toxoplasmosis
- c. Craniopharyngioma
- d. Sturge Weber syndrome
- Ans: d
- Ref: Berry and Sury Diagnostic Radiology, 2nd Edition, Page No: 3, 14
- Sturge-Weber syndrome is another important cause of intracranial calcification. Patients present with epilepsy and mental retardation and often have cutaneous hemangioma in the distribution of trigeminal nerve on the same side as calcification. Calcification has a typical tram-track appearance and is seen in the cerebral cortex.
- Tramline calcification is a feature of Sturge-Weber syndrome, and describes the pattern of calcification seen on skull radiology.
-
112
- 1. Angle of the neck of femur to shaft is:
- a. 110°
- b. 125°
- c. 145°
- d. 100°
- Ans: b
- Ref: Moore's Anatomy 7th Edition, Page No: 518
- The proximal femur is “bent” (L-shaped) so that the long axis of the head and neck projects superomedially at an angle to that of the obliquely oriented shaft. This obtuse angle of inclination is greatest (most nearly straight) at birth and gradually diminishes (becomes more acute) until the adult angle is reached (115–140°, averaging 126°).
- The angle of inclination is less in females because of the increased width between the acetabula (a consequence of a wider lesser pelvis) and the greater obliquity of the femoral shaft.
- 2. Malignant tumour of thyroid in FNAC is reported as:
- a. Thy 2
- b. Thy 3
- c. Thy 4
- d. Thy 5
- Ans: d
- Ref: Bailey and Love’s Short Practice of Surgery 27th Edition, Page No: 746, 751
- Classification of FNAC cytology report:
- • Thy 1: Non-diagnostic
- • Thy 1c: Non-diagnostic cystic
- • Thy 2: Non-neoplastic
- • Thy 3: Follicular
- • Thy 4: Suspicious of malignancy
- • Thy 5: Malignant
- 3. What is the average tidal volume of an average healthy new born?
- a. 1.5 ml
- b. 15 ml
- c. 50 ml
- d. 80 ml
- Ans: b
- Ref: OP Ghai Pediatrics 9th Edition, Page No: 371
- • Before birth, the lungs are filled with fluid that needs to be replaced by air. Somefluid is extruded from the mouth and some is absorbed by the lymphatics.
- • Intrapleural negative pressure required for first breath is 40 to 100 cm water. This pressure is higher initially, because of low compliance of the newborn lung (1.5 mL/ cm water at birth). The compliance increases in the first few hours to 6mL/cm of water and resistance to air flow decreases.
- • The tidal volume of a 3 kg infant is approximately l6 mL at 28 breaths per minute. Resting lung volume gradually increases in the first few hours to reach a maximum of 80 mL within 24 hours.
- 4. Whose consent is needed to perform abortion in 9 weeks gestation?
- a. Pregnant woman
- b. Pregnant woman and her husband
- c. Pregnant woman, her husband or her family
- d. Pregnant woman, her husband and her family
- Ans: a
- Fetus (gestation) up to twelve weeks, with the consent of the pregnant woman,
- Abortion law update Nepal
- Ref: Lawcommission Gov website
- The Right to Safe Motherhood and Reproductive Health Act, 2075 (2018)
- Chapter 4: Safe Abortion
- To perform safe abortion: A pregnant woman shall have the right to get safe abortion performed in any of the following circumstances:
- Fetus (gestation) up to twelve weeks, with the consent of the pregnant woman,
- Fetus (gestation) up to twenty-eight weeks, as per the consent of such woman, after the opinion of the licensed doctor that there may be danger upon the life of the pregnant woman or her physical or mental health may deteriorate or disabled infant may be born in case the abortion is not performed,
- Fetus (gestation) remained due to rape or incest, fetus (gestation) up to twenty-eight weeks with the consent of the pregnant woman,
- Fetus (gestation) up to twenty-eight weeks with the consent of the woman who is suffering from H.I.V. or other incurable disease of such nature,
- Fetus (gestation) up to twenty eight weeks with the consent of the woman, as per the opinion of the health worker involved in the treatment that damage may occur in the womb due to defects occurred in the fetus (gestation), or that there is such defect in the fetus of the womb that it cannot live even after the birth, that there is condition of disability in the fetus (gestation) due to genetic defect or any other cause.
-
112
1. The MR imaging in multiple sclerosis will show lesions in:
- a. White matter
- b. Grey matter
- c. Thalamus
- d. Basal ganglia
Ans, A, White Matter
- Ref, Diagnostic Radiology Essentials, Grainger & Allison 2e/ p772
- Explanation
- Multiple sclerosis is a demyelinating disorder. Hence the lesions will be seen in the white matter (due to loss of myelin).
2. The most common primary tumor of the heart in adults is usually located in the left atrium and is called a
- a. Chordoma
- b. Rhabdomyoma
- c. Leiomyoma
- d. Myxoma
Ans, D, Myxoma
Ref, Robbins & Cotran, Pathologic basis of disease, 7e, p 589-591
- Explanation
- Most tumors involving the heart are secondary to metastases, most commonly from bronchogenic carcinoma or breast carcinoma, and they usually involve the pericardium. Primary tumors of the heart are quite rare; the most common in the adult is the myxoma. These tumors occur most often in the left atrium, and if pedunculated they may interfere with the mitral valve by a “ball valve” effect. Histologically they are composed of stellate cells in a loose myxoid background. Rhabdomyomas are the most common primary cardiac tumors in infants and children and often occur in association with tuberous sclerosis. Histologically, so-called spider cells may be seen. Papillary fibroelastomas usually are incidental lesions found at the time of autopsy and are probably hamartomas rather than true neoplasms.
3. The systematic distortion of retrospective studies that can be eliminated by a prospective design is:
- a. Confounding
- b. Effect modification
- c. Recall bias
- d. Measurement bias
Ans, C, Recall bias
Ref, Park’s Textbook of Preventive and Social Medicine 22nd edition, p70-71
Explanation
In a Nested case control study, interviews are performed at the beginning of the study (at baseline), and data are ob¬tained before the disease has developed, thereby eliminating the problem of Recall bias
-
134
- 1. Basal body temperature increases around the time of ovulation in a female primarily due to:
- a. Estrogen
- b. Progesterone
- c. Cytokines
- d. Both a and b
- Ans: B, Progesterone
- Ref, Ganong’s Review of Medical Physiology, 26th edition, page no. 396
- Indicators of Ovulation
- • Knowing when during the menstrual cycle ovulation occurs is important in increasing fertility or, conversely, in fam¬ily planning.
- • A convenient and reasonably reliable indica¬tor of the time of ovulation is a change—usually a rise—in the basal body temperature. The rise starts 1–2 days after ovulation.
- • Women interested in obtaining an accurate temperature chart should use a digital thermometer and take their temperatures (oral or rectal) in the morning before getting out of bed.
- • The cause of the temperature change at the time of ovulation is probably the increase in progester¬one secretion, since progesterone is thermogenic.
- • A surge in LH secretion triggers ovulation, and ovulation normally occurs about 9 h after the peak of the LH surge at mid¬cycle.
- The ovum lives for approximately 72 h after it is extruded from the follicle, but it is fertilizable for a much shorter time than this.
- 2. Which of the following is the site of highest airway resistance?
- a. Largest bronchi
- b. Medium-sized bronchi
- c. Smallest bronchi
- d. Alveoli
- Ans: B, Medium-sized bronchi
- Ref, BRS Physiology, 6th edition, page no. 144
- • The medium-sized bronchi actually constitute the site of highest resistance along the bronchial tree.
- • Although the small radii of the alveoli might predict that they would have the highest resistance, they do not because of their parallel arrangement.
- • In fact, early changes in resistance in the small airways may be “silent” and go undetected because of their small overall contribution to resistance.
- 3. In active chicken pox, skin lesions of various stages can be seen simultaneously distribution of which follows one of the following pattern. Which one is the most correct one?
- a. Centrifugal
- b. Centripetal
- c. Both pattern exists
- d. None of the above
- Ans: B, Centripetal
- Ref, Illustrated Synopsis of Dermatology and Sexually Transmitted Diseases, Neena Khanna, 4th edition
- Varicella-Zoster Infections
- Varicella
- • Synonym: Chicken pox.
- • Etiology: Varicella-zoster virus.
- • Prodrome: Fever and malaise.
- • Morphology: Crops of papules with erythematous halo; rapidly become vesicular (dew drops on rose petal appearance), then pustular. Eruption at different stages present. Heal with minimal scarring unless complicated by secondary infection or hemorrhagic lesions (as seen in immunocompromised). Adults usually have a more severe eruption.
- • Site: Centripetal distribution.
- • Prophylaxis: Live attenuated vaccine.
- • Treatment: Specific antiviral therapy (acyclovir 800 mg, five times daily × 7–10 days) in adults and in immunocompromised individuals. None needed in children.
- 4. Of the following defense mechanisms, which is considered the most mature?
- a. Sublimation
- b. Dissociation
- c. Regression
- d. Intellectualization
- Ans: A, Sublimation
- Ref. FA, USMLE step 1, 2020, Psychiatry
- • Sublimation, expressing an unacceptable emotion in a socially acceptable way, is classified as a mature defense mechanism.
- • Denial, dissociation, regression, and intellectualization are all classified as less mature defense mechanisms.
-
135
- A 63-year-old male develops a brochopleural fistula following a pneumonectomy. He is ventilated on the ICU, but achieving adequate tidal volume is proving to be difficult due to an air leak of 2.5L/min through the fistula. Which one of the following would be most effective in achieving adequate ventilation in this patient?
- a. Adding PEEP of 7.5cm of H2O.
- b. Decreasing the inflation pressure.
- c. Increasing the flow rate by 2.5L/min.
- d. High frequency jet ventilation.
- Answer: D. High frequency jet ventilation.
- Bronchopleural fistula can occur after a pneumonectomy due to the failure
- of the bronchial stump to heal. As the bronchus is in direct connection with
- the pleural cavity, some of the tidal volume is lost into the pleural cavity
- affecting achievable lung ventilation. Management of a ventilated patient
- with a bronchopleural fistula is particularly challenging and it is often
- difficult to wean a patient from the ventilator. High frequency ventilation
- with small tidal volumes, low airway pressure and a high respiratory rate
- provides the best chance of ventilating the lungs in these patients if
- conventional ventilation fails.
-
136
- Q1. What percentage of patients with polyarticular Juvenile idiopathic arthritis (JIA) are seropositive for Rheumatoid factor (RF)?
- a.10-20%
- b.5-15%
- c.20-40%
- d.1-2%
- Ans:b, 5-15%
- Ref:Nelson textbook of Pediatrics 21st edition page no:5196
- • Elevated ANA titers are present in 40–85% of children with oligoarticular or polyarticular JIA, but are rare with sJIA.
- • ANA seropositivity is associated with increased risk of chronic uveitis in JIA.
- • Approximately 5–15% of patients with polyarticular JIA are seropositive for RF.
- • Anti–cyclic citrullinated antibody, as with RF, is a marker of more aggressive disease.
- • Both ANA and RF seropositivity can occur in association with transient events, such as viral infection.
- Q2. Which of the following corneal dystrophies is autosomal recessive?
- a. Macular dystrophy
- b. Lattice dystrophy type 1
- c. Granular dystrophy
- d. Fuchs endothelial corneal dystrophy
- Ans: a, Macular dystrophy
- Ref:Kanski 6th edition page no:290
- • Macular corneal dystrophy is autosomal recessive, whereas lattice dystrophy type 1 and granular dystrophy are autosomal dominant.
- • Fuchs endothelial corneal dystrophy can show an autosomal dominant inheritance in some families, but in the majority of cases the pattern of inheritance is yet to be elucidated.
- Q3. The following is an example of capsulated gram-positive bacilli :
- a. Corynebacterium diphtheria
- b. Mycobacterium tuberculosis
- c. Actinomyces Israeli
- d. Bacillus anthracis
- Ans:d,Bacillus anthracis
- Ref.: CP Bevjea, 3rd edition page no:211
- - Bacillus anthracis is a gram positive, bacillus. In infected tissues, the bacilli are found singly, in pairs or in short chains, the entire chin being surrounded by capsule. The capsule is polypeptide (polymer of D-glutamic acid) in nature.
- - Corynebacterium diphtheria are gram positive non encapsulated bacilli.
- - Mycobacterium tuberculosis is acid fast bacillus; nonmotile, nonsporing, non encapsulated.
- - Actinomycetes are gram positive filamentous bacteria intermediate in properties between bacteria and fungi.
-
137
- 1. Most common site of metastasis of Ca cervix is:
- a. Lymph node
- b. Lungs
- c. Bone
- d. Abdominal cavity
- Ans: b
- Ref: Dutta Gynecology 6th Edition, Page No: 346
- Metastases to the distant organs commonly observed are—lung (36%), lymph nodes (30%), bone (16%) and abdominal cavity (7%). These may be fatal.
- 2. What is seen in remodeling of surgical wound?
- a. Type I collagen replacing type II collagen
- b. Type I collagen replacing type III collagen
- c. Type III collagen replacing type I collagen
- d. Type III collagen replacing type II collagen
- Ans: b
- Ref: Bailey and Love’s Short Practice of Surgery 27th Edition, Page No: 25
- • The wound tissue formed in the early part of this phase is called granulation tissue. In the latter part of thisphase, there is an increase in the tensile strength of the wound due to increased collagen, which is at first deposited in a random fashion and consists of type III collagen.
- • This proliferative phase with its increase of collagen deposition is associated with wound contraction, which can considerably reduce the surface area of a wound over the first 3 weeks of healing.
- • The remodelling phase is characterised by maturation of collagen (type I replacing type III until a ratio of 4:1 is achieved).
- • There is a realignment of collagen fibres along the lines of tension, decreased wound vascularity, and wound contraction due to fibroblast and myofibroblast activity. This maturation of collagen leads to increased tensile strength in the wound which is maximal at the 12th week post injury and represents approximately 80% of the uninjured skin strength.
- 3. Which laryngoscope blade is used for neonates?
- a. Macintosh
- b. Magill
- c. McCoy
- d. Miller
- Ans: b
- Ref: Ajay Yadav, Short Textbook of Anesthesia, 6th edition, page 33
- • Laryngoscope is used for visualizing the glottis to facilitate intubation.
- • Direct rigid laryngoscope consists of a handle (containing 2 batteries), a blade with a bulb. They are named on the shape of the blade. Although there are number of laryngoscopes available in the market, however, the most common in use are:
- • Macintosh: It is most commonly used. It has curved blade and is available in 4 sizes; smallest for children and largest for adults with long necks.
- • McCoy: It has got a movable lip, which can be used to maneuver the glottis.
- • Miller: It has a straight blade with curve at the tip.
- • Magill: Magill is straight blade used for neonates. Neonatal epiglottis is large, leafy and more anterior, therefore it need to be lifted by straight blade to visualize glottis (Adult's epiglottis just need to be pushed anteriorly, therefore curved blade is used).
- • Oxford infant blade: Used for infants.
- 4. Borrmann’s type IV for advanced gastric cancer means:
- a. Ulcerated carcinoma with clear cut margin
- b. Ulcerated carcinoma without clear cut margin
- c. Diffuse carcinoma
- d. Unclassified
- Ans: c
- Ref: SRB's Manual of Surgery; 5th Edition, Page No: 847
- Advanced gastric cancer is defined as involvement of muscularis and/or serosa with or without involvement of lymph nodes.
- It is classified as: (Borrmann’s classification )
- I. Single, polypoid carcinoma.
- II. Ulcerated carcinoma with clear cut margin.
- III. Ulcerated carcinoma without clear cut margin.
- IV. Diffuse carcinoma—linitis plastica.
- V. Unclassified.
- 5. Which of the following is USG feature of the fatty liver?
- a. Increased echotexture
- b. Decreased echotexture
- c. Coarse echotexture
- d. Nodular outline
- Ans: a
- Ref: Sutton Radiology, 7th Edition, Page No: 24
- Fatty infiltration or steatosis
- This is the metabolic complication of various insults to the liver. It is commonly seen in patients with diabetes mellitus and in alcoholics.
- Other causes include obesity, hyperlipidemia, parenteral nutrition, glycogen storage disease, severe hepatitis and chemotherapy.
- The normal liver parenchyma reflectivity should be midway between renal cortex and pancreas.
- Fatty infiltration will cause an increase in reflectivity and hence increased echotexture.
-
138
Answers and Winners of QUIZ for Today's CEE MD/MS Based Online Model test (11:00 AM, Jestha 24, Saturday):
- 1. Adson test involves contraction of which of the following muscle?
- a. Scalenus anterior
- b. Scalenus posterior
- c. Sternocleidomastoid
- d. Platysma
- Ans: a
- Ref: SRB's Manual of Surgery; 5th Edition, Page No: 173
- Adson’s test (Scalene manoeuvre):
- In a patient sitting on a stool, the radial pulse is felt. The patient is then asked to take a deep breath (to allow the rib cage to move upwards so as to narrow the cervicoaxillary channel) and turn the face to same side (to contract scalenus anterior muscle so as to narrow the scalene triangle).
- If the radial pulse disappears or become feeble it signifies cervical rib or scalenus anticus syndrome.
- 2. What is the commonest mechanism of uptake of large macromolecules within the cells?
- a. Diffusion
- b. Osmosis
- c. Pinocytosis
- d. Phagocytoses
- Ans: c
- Ref: Guyton and Hall Textbook of Medical Physiology, Elsevier; 13th Edition, Page No: 19
- • If a cell is to live and grow and reproduce, it must obtain nutrients and other substances from the surrounding fluids. Most substances pass through the cell membrane by diffusion and active transport. Diffusion involves simple movement through the membrane caused by the random motion of the molecules of the substance; substances move either through cell membrane pores or, in the case of lipid-soluble substances, through the lipid matrix of the membrane. Active transport involves the actual carrying of a substance through the membrane by a physical protein structure that penetrates all the way through the membrane.
- • Very large particles enter the cell by a specialized function of the cell membrane called endocytosis. The principal forms of endocytosis are pinocytosis and phagocytosis.
- • Pinocytosis means ingestion of minute particles that form vesicles of extracellular fluid and particulate constituents inside the cell cytoplasm.
- • Pinocytosis is the only means by which most large macromolecules, such as most protein molecules, can enter cells. In fact, the rate at which pinocytotic vesicles form is usually enhanced when such macromolecules attach to the cell membrane.
- • Phagocytosis means ingestion of large particles, such as bacteria, whole cells, or portions of degenerating tissue. Phagocytosis occurs in much the same way as pinocytosis occurs, except that it involves large particles rather than molecules. Only certain cells have the capability of phagocytosis, most notably the tissue macrophages and some white blood cells.
- 3. Inhibitor of complex I of ETC?
- a. H2S
- b. 2,4 DND
- c. Rotenone
- d. BAL
- Ans: c
- Ref: Harper Biochemistry 30th Edition, Page No: 234
- Inhibitors of ETC:
- • Complex I: Barbiturates (amobarbital), piercidin A, rotenone, chlorpromazine, guanethidine.
- • Complex II: Carboxin, TTFA, Malonate
- • Complex III: Dimercaprol, BAL, actinomycin A, Naphthyl quinone
- • Complex IV (Cytochrome C Oxidase) Carbon monoxide (CO), Cyanide (CN), H2S, Azide(n3)
- 4. In horse shoe shaped kidney, ascent is prevented by:
- a. Superior mesenteric artery
- b. Inferior mesenteric artery
- c. Superior mesenteric vein
- d. Inferior mesenteric vein
- Ans: b
- Ref: Bailey and Love’s Short Practice of Surgery 27th Edition, Page No: 1399
- Horseshoe kidney
- In a horseshoe kidney, the two renal units are low lying and the lower poles fuse to form an isthmus. Further ascent of the fused kidneys is prevented by the inferior mesenteric artery. This condition is detected in 1 in 1000 autopsies and has a male preponderance (2:1). Horseshoe kidneys have an unpredictable vascular supply from nearby major vessels. Horseshoe kidneys are prone to reflux, obstruction and stone formation.
- 5. 2 week old neonate with history of stridor in supine position; what is treatment ?
- a. Oral calcium
- b. Nebulization
- c. Wait & watch
- d. Antibiotic
- Ans: c
- Ref: Nelson Textbook of Pediatrics, Elsevier, 20th Edition, Page No: 2036
- • Laryngomalacia is the most common congenital laryngeal anomaly and the most common cause of stridor in infants and children. Sixty percent of congenital laryngeal anomalies in children with stridor are due to laryngomalacia. Stridor is inspiratory, low-pitched, and exacerbated by any exertion: crying, agitation, or feeding.
- • Expectant observation is suitable for most infants because most symptoms resolve spontaneously as the child and airway grow.
- Winners:
- Fastest correct answer respondents:
- - Dr Krishna Hari Shrestha Gaikhure
- - Dr Prabin Paudyal
- Luck draw winners:
- - Dr Alina Chhetri
- - Dr Sandeep Shrestha
Winners are requested to send an email to onlinepgnepal@gmail.com to claim Free Access to Today CEE MD/MS Based Online Model test (11 AM, Jestha 24, Saturday).
-
139
Answers to yesterday’s questions:
- 1.Enzyme inhibited in cyanide poisoning:
- a. Cytochrome oxidase
- b. Xanthine oxidase
- c. Cytochrome reductase
- d. None
- Ans,a.Cytochrome oxidase
- Ref. Harrison’s 18th ed./Ch. 35
- • Cyanide binds irreversibly to Cytochrome a/a3 (Complex IV) & this inhibits ETC.
- • Cyanide poisoning inhibits mitochondrial cytochrome oxidase →blocks electron transport → prevents O2 utilization and oxidative metabolism.
- • Cyanide poisoning during nitroprusside therapy can be prevented by prophylactic mechanism of thiosulphate.
- • Treatment: High dose O2, amyl nitrate inhalation, sodium nitrate, sodium thiosulphate and hydroxycobalalmine.
- 2.The most common ocular infection after renal transplantation is by:
- a. Cytomegalovirus
- b. Toxoplasma
- c. Herpes virus
- d. Epstein Barr virus
- Ans,a. Cytomegalovirus
- Ref: Harrison 18th/e.p. 2332
- • Cytomegalovirus (CMK) is a significant cause of morbidity and mortality in renal transplant recipients.
- • CMV is present in more than two thirds of donors and recipients prior to transplantation and can be transmitted from the donor by the transplanted kidney.
- • The concurrent administration of immunosuppressive drugs to prevent rejection increases the risk of clinically relevant CMV disease.
- • Both the donor and the recipient are routinely tested for anti-CMV antibodies prior to transplantation.
- 3.In a man lifting up suitcase, posterior dislocation of glenohumeral joint is prevented by:
- a. Deltoid
- b. Latissimus dorsi
- c. Coracobrachialis
- d. Short head of biceps
- Ans,c. Coracobrachialis
- Ref: Maheshwari 5th ed. /pg. 88
- • Glenohumeral joint is the most important joint of shoulder complex.
- • It is a synovial ball and socket articulation between the head of humerus and glenoid cavity of scapula.
- • Movements at this joint includes flexion, extension, abduction, adduction, medial rotation, lateral rotation and circumduction.
- • Posterior dislocation of glenohumeral (shoulder) joint would be prevented by the muscle which originate posteriorly and is inserted anteriorly.
- • For Example: Corachobrachialis muscle originate from coracoid process of scapula and inserted at medial aspect of the shaft of humerus.
- • The net vector of pull of this muscle would be anterior, opposing the posterior dislocation of shoulder joint.
-
140
- Answers to Daily Questions, Jestha 23, Friday
- Q1. All of the following spaces are seen in relation to larynx except:
- a. Space of boyer
- b. Space of tucker
- c. Reinke‘s space
- d. Space of gillette
- Ans is d i.e. Space of Gillette
- Ref: Diseases of Ear Nose and Throat, PL Dhingra, 6th edition, page no.265
- Space of Gillette is seen in pharynx in relation to retropharyngeal space and not in larynx.
- Also know: Spaces in relation to larynx
- 1. Pre-epiglottic space (Space of Boyer)
- Location: Between the thyroid and Thyrohyoid membrane anteriorly and epiglottis posteriorly. Laterally it is continuous with Paraglottic space.
2. Paraglottic space (space of Tucker): It is bounded by the thyroid cartilage laterally, conus elasticus inferomedially, the ventricle and quadrangular membrane medially, pyriform sinus posteriorly and superomedially with pre-epiglottic space. Growths which invade this space can present in the neck through cricothyroid space.
3. Reinke’s space: Under the epithelium of vocal cords is a potential space with scanty subepithelial connective tissues. It is bounded above and below by the arcuate lines, in front by anterior commissure, and behind by vocal process of arytenoid. Oedema of this space causes fusiform swelling of the membranous cords (Reinke’s oedema).
- Q2. In modified Wells criteria for predicting pulmonary embolism (PE), maximum score of 3 is given for which component?
- a. Clinical signs and symptoms of DVT
- b. Heart rate of more than 100
- c. Previous DVT or PE
- d. Hemoptysis
- Ans: a, Clinical signs and symptoms of DVT
- Ref: Short Practice of Surgery, Bailey and Love, 27th edition, page no.989
- Q3. Warning signs of burns to the respiratory system comprises of;
- a. Burns around the face and neck
- b. Change in voice
- c. Stridor
- d. All of the above
- Ans: d, All of the above
- Ref: Short Practice of Surgery, Bailey and Love, 27th edition, page no.617
- Warning signs of burns to the respiratory system
- •Burns around the face and neck
- •A history of being trapped in a burning room
- •Change in voice
- •Stridor
-
141
- 1. Which is the best time for the diagnosis of gestational diabetes mellitus? [Patan-PAHS 2018]
- a. 10-14 wks
- b. 16-20 wks
- c. 20-24 wks
- d. 24-28 wks
- Ans: d
- Ref: Williams Obstetrics 24th Edition, Chapter 57 | Ref: Harrison’s Principles of Internal Medicine; 19th Edition, Page No: 48
- Gestational diabetes
- • Gestational diabetes occurs in approximately 4% of pregnancies.
- • All pregnant women should be screened for gestational diabetes unless they are in a low-risk group.
- • Women at low risk for gestational diabetes are those <25 years of age; those with a body mass index <25 kg/m2, no maternal history of macrosomia or gestational diabetes, and no diabetes in a first-degree relative; and those who are not members of a high risk ethnic group (African American, Hispanic, Native American).
- • A typical two-step strategy for establishing the diagnosis of gestational diabetes involves administration of a 50-g oral glucose challenge with a single serum glucose measurement at 60 min.
- • If the plasma glucose is <7.8 mmol/L (<130 mg/dL), the test is considered normal.
- • Plasma glucose >7.8 mmol/L (>130 mg/dL) warrants administration of a 100-g oral glucose challenge with plasma glucose measurements obtained in the fasting state and at 1, 2, and 3 h. Normal plasma glucose concentrations at these time points are <5.8 mmol/L (<105 mg/dL), 10.5 mmol/L (190 mg/dL), 9.1 mmol/L (165 mg/dL), and 8.0 mmol/L (145 mg/dL), respectively.
- • Two elevated glucose values indicate a positive test.
- • Screening should be performed between 24 and 28 weeks’ gestation in those women not known to have glucose intolerance earlier in pregnancy.
- 2. A male patient with known case of bipolar disorder was under lithium therapy. After few months, he came to you with features of lithium toxicity. What is the ECG finding in such case? [Patan-PAHS 2019]
- a. Prolonged QT interval
- b. Short QT interval
- c. Absent P wave
- d. Wide QRS complex
- Ans: a
- Ref: Katzung’s Basic & Clinical Pharmacology; Lange; 14th Edition, Page No: 524, 527 | Ref: Lithium poisoning by Jeanmarie Perrone er al in UpToDate.
- Pharmacokinetics of lithium:
- • Absorption: Virtually complete within 6–8 hours; peak plasma levels in 30 minutes to 2 hours
- • Distribution: In total body water; slow entry into intracellular compartment. Initial volume of distribution is 0.5 L/kg, rising to 0.7–0.9 L/kg; some sequestration in bone. No protein binding.
- • Metabolism: None
- • Excretion: Virtually entirely in urine. Lithium clearance about 20% of creatinine. Plasma half-life about 20 hours.
- • Target plasma concentration: 0.6–1.4 mEq/L
- • Dosage: 0.5 mEq/kg/d in divided doses
- • Cardiac Adverse Effects: The bradycardia-tachycardia (“sick sinus”) syndrome is a definite contraindication to the use of lithium because the ion further depresses the sinus node. T-wave flattening is often observed on the electrocardiogram but is of questionable significance.
- • Although lithium toxicity can cause changes in the electrocardiogram (ECG), dangerous arrhythmias or other important clinical effects are rare. Flattened T waves, prolonged QTc intervals, and bradycardia have been reported.
- 3. In gall stone ileus classically the stone is impacted at: [Pokhara Cancelled paper]
- a. Ileocaecal valve
- b. Duodenum
- c. Jejunum
- d. 60 cm proximal to ileocaecal valve
- Ans: d
- Ref: Bailey and Love’s Short Practice of Surgery 27th Edition, Page No: 1282
- Bolus obstruction in the small bowel may be caused by gallstones, food, trichobezoar, phytobezoar, stercoliths and worms.
- Gallstones
- • This type of obstruction tends to occur in the elderly secondary to erosion of a large gallstone directly through the gall bladder into the duodenum.
- • Classically, there is impaction about 60 cm proximal to the ileocaecal valve.
- • The patient may have recurrent attacks as the obstruction is frequently incomplete or relapsing as a result of a ball-valve effect.
- • The characteristic radiological sign of gallstone ileus is Rigler’s triad, comprising: small bowel obstruction, pneumobilia and an atypical mineral shadow on radiographs of the abdomen.
- 4. Critical temps of N20 is: [Pokhara PAHS 2nd set]
- a. 20
- b. 34.5
- c. 35.5
- d. 36.5
- Ans: d
- Ref: Morgan & Mikhail’s Clinical Anesthesiology, 5th Edition, Page No: 12
- Nitrous Oxide
- Nitrous oxide is manufactured by heating ammonium nitrate (thermal decomposition).
- It is almost always stored by hospitals in large H-cylinders connected by a manifold with an automatic crossover feature.
- Bulk liquid storage of nitrous oxide is economical only in very large institutions.
- Because the critical temperature of nitrous oxide (36.5°C) is above room temperature, it can be kept liquefied without an elaborate refrigeration system. If the liquefied nitrous oxide rises above its critical temperature, it will revert to its gaseous phase. Because nitrous oxide is not an ideal gas and is easily compressible, this transformation into a gaseous phase is not accompanied by a great rise in tank pressure.
- 5. Third window effect is seen in: [Pokhara PAHS 2nd set]
- a. Round window
- b. Superior semicircular canal dehiscence
- c. Oval window
- d. Perforated tympanic membrane
- Ans: b
- Ref: Spectrum of Third Window Abnormalities: Semicircular Canal Dehiscence and Beyond by M.-L. Ho et al in American Journal of Neuroradiology January 2017, 38 (1) 2-9
- • Third window abnormalities are defects in the integrity of the bony structure of the inner ear, classically producing sound-/pressure-induced vertigo (Tullio and Hennebert signs) and/or a low-frequency air-bone gap by audiometry.
- • Specific anatomic defects include semicircular canal dehiscence, perilabyrinthine fistula, enlarged vestibular aqueduct, dehiscence of the scala vestibuli side of the cochlea, X-linked stapes gusher, and bone dyscrasias.
-
142
- 1. Alkalinization of urine is required for decreasing the poisoning due to:
- a. Barbiturates
- b. Amphetamine
- c. Alcohol
- d. Morphine
- Ans. (a) Barbiturates (Ref: KDT 7/e p29)
- For acidic drug poisonings (like barbiturates, salicylates and methotrexate), urinary alkalinizing agents are prescribed
- whereas for basic drug poisonings, (morphine, amphetamine, atropine etc.) urinary acidifying agents are administered.
- 2. True about the teratogenicity of a drug is all except
- a. Characteristic set of malformations indicating selectivity for certain target organs is seen
- b. Heparin is highly teratogenic drug
- c. Related to the dose of the teratogenic drug
- d. Affects specifically at a particular phase of development of the fetus
- Ans. (b) Heparin is highly teratogenic drug (Ref: KDT 7/e p89)
- Teratogenicity is the development of characteristic set of malformations by the use of a drug during pregnancy. Different
- drugs have specificity for a particular phase of development of fetus. The risk of teratogenesis is dose dependent. Heparin
- cannot cross the placenta and is thus safe during pregnancy.
- 3.Which of the following drugs increases gastro-intestinal motility?
- Glycopyrrolate
- Atropine
- Neostigmine
- Fentanyl
- Ans. (c) Neostigmine (Ref: KDT 6/e p104)
- • Cholinergic drugs increase GI motility. Neostigmine is an inhibitor of the enzyme AChE (which is responsible for the
- breakdown of ACh). It thus acts like a cholinergic agent.
- • Glycopyrrolate and atropine are anticholinergic drugs; thus reduce GI motility. Fentanyl is an opioid and can cause
- constipation.
- 4. A highway truck driver has profuse rhinorrhoea and sneezing. Which amongst the following drugs would you prescribe him?
- a. Pheniramine
- b. Promethazine
- c. Dimenhydrinate
- d. Cetirizine
- Ans. (d) Cetirizine (Ref: KDT 6/e p157)
- It is a non-sedating second generation antihistaminic agent and is preferred for the patients requiring constant attention.
- It does not cause sedation or impairment of psychomotor function. Other drugs listed in the question are first generation
- antihistaminics.
- 5. Which of the following surgical procedures has the highest incidence of postoperative respiratory complications?
- a. Abdominal aortic surgery
- b. Peripheral vascular surgery
- c. Abdominal surgery for bowel resection
- d. Neurosurgery
- Answer: A. Abdominal aortic surgery.
- The risk of overall postoperative respiratory complications is about 25%
- with open abdominal aortic surgery. The incidence of postoperative
- pneumonia is higher with abdominal aortic surgery compared to thoracic
- surgery (odds ratio 4.3 versus 3.9).
- Endovascular repair is associated with a much lower risk of postoperative
- pneumonia as compared to an open procedure.
- Ref.
- 1. Moppett IK. Respiratory risk. In: Consent, benefit, and risk in
- anaesthetic practice. Hardmann JG, Moppett IK, Aitkenhead AR, Eds.
- Oxford: Oxford University Press, 2009; Chapter 12: 173-87.
-
143
- 1. A 30-year-old male patient is undergoing a laparoscopic cholecystectomy. Halfway through the surgery, despite adequate inspired oxygen and ventilation, his SpO2 decreases to 90%. Clinical examination reveals distended neck veins, and reduced movement and breath sounds on the left side of the chest. What should be the next step in the management of this patient?
- a. Increasing minute ventilation
- b. Arterial blood gas analysis.
- c. Endotracheal suction
- d. Needle decompression and insertion of a chest drain.
- Answer: D. Needle decompression and insertion
- of a chest drain.
- Pneumothorax is a recognised complication of laparoscopy. Gas passes
- from the peritoneum directly into the pleural space either through preexisting
- channels, or it may spread from the mediastinum along the bronchi
- until it breaks through the weak spot into the pleural cavity. The distended
- neck veins (increased CVP), hypotension and desaturation suggest
- development of a tension pneumothorax. The specific treatment is needle
- decompression followed by insertion of a chest drain.
- Increasing minute ventilation may improve the desaturation due to
- hypoventilation. An endotracheal tube blocked by a mucous plug can
- result in high airway pressures and desaturation which may be improved
- by endotracheal suction. PEEP is useful in improving oxygenation due to
- atelectasis.
- Ref.
- 1. Farn J, Hammerman A, Brunt LM. Intraoperative pneumothorax during
- laparoscopic cholecystectomy: a complication of prior
- transdiaphragmatic surgery. Surgical Laparoscopy Endoscopy &
- Percutaneous Techniques 1993; 3: 219-22.
- 2. Complications and contraindications of laparoscopic surgery. In:
- Anaesthesia for minimally invasive surgery. Crozier TA, Ed.
- Cambridge: Cambridge University Press, 2004.
- 2. A 60-year-old female patient is admitted to the neurosurgical unit with a subarachnoid haemorrhage secondary to a ruptured intracranial aneurysm. Which of the following is the most serious complication that could occur during the subsequent 3 days?
- a. Re-bleeding.
- b. Cerebral vasospasm.
- c. Hypertension
- d. Hydrocephalus
- Answer: A. Re-bleeding.
- Although cerebral vasospasm is the most common problem, the most
- serious complication of subarachnoid haemorrhage (SAH) is re-bleeding;
- the occurrence is about 15% during the first week. The management of
- cerebral vasospasm consists of nimodipine and triple H therapy (induced
- hypertension, hypervolaemia and haemodilution). SAH is also frequently
- associated with systemic and pulmonary hypertension, cardiac
- arrhythmias and neurogenic pulmonary oedema. Other complications
- include hydrocephalus and electrolyte disturbances. Hyponatraemia
- develops as a result of either cerebral salt wasting syndrome or the
- syndrome of inappropriate ADH secretion.
- Ref
- 1. Solenski NJ, Haley E, et al. Medical complications of aneurysmal
- subarachnoid hemorrhage: a report of the multicenter, cooperative
- aneurysm study. Critical Care Medicine 1995; 23: 1007-17.
- 2. Priebe H-J. Aneurysmal subarachnoid haemorrhage and the
- anaesthetist. British Journal of Anaesthesia 2007; 99: 102-18.
- 3. An anaesthetist administers 10ml of 0.5% bupivacaine as part of an epidural top-up for a Caesarean section. One minute after completing the injection, the patient complains of dizziness, difficulty in breathing and then starts to convulse. She then suffers a VF cardiac arrest. The most appropriate management in the first 4 minutes would be:
- a. Defibrillation, CPR, adrenaline, amiodarone
- b. CPR, adrenaline, 20% Intralipid, Caesarean section.
- c. CPR, defibrillation, 20% Intralipid, Caesarean section.
- d. CPR, defibrillation, adrenaline, 20% Intralipid.
- Answer: D. CPR, defibrillation, adrenaline, 20%
- Intralipid.
- The cause of this woman’s symptoms and cardiac arrest is likely to be due
- to intravascular injection of local anaesthetic (LA) leading to LA toxicity.
- The principles of managing cardiac arrest associated with LA toxicity are:
- w To start cardiopulmonary resuscitation (CPR) using standard
- protocols.
- w To manage arrhythmias using the same protocols, recognising that
- they may be very refractory to treatment.
- w Prolonged resuscitation may be necessary; it may be appropriate to
- consider other options (cardiopulmonary bypass and continuing
- treatment with 20% Intralipid).
- CPR should be continued throughout the treatment with the 20%
- Intralipid. After 4 minutes, preparations to perform an immediate
- Caesarean section should commence, aiming to deliver the foetus 5
- minutes after cardiac arrest has occurred. A gravid uterus causes
- aortocaval compression which will only hamper resuscitation efforts;
- emptying the uterus affords the mother the best chance of survival.
- Ref.
- 1. Guidelines for the management of severe local anaesthetic toxicity.
- Association of Anaesthetists of Great Britain and Ireland, 2007.
- 2. Cardiopulmoary resuscitation in the non-pregnant and pregnant
- woman. In: Managing obstetric emergencies and trauma course
- manual, 2nd ed. Grady K, Howell C, Cox C, Eds. Advanced Life
- Support Group 2007; Chapter 4; 21-9.
- 4. A 63-year-old male develops a brochopleural fistula following a pneumonectomy. He is ventilated on the ICU, but achieving adequate tidal volume is proving to be difficult due to an air leak of 2.5L/min through the fistula. Which one of the following would be most effective in achieving adequate ventilation in this patient?
- a. Adding PEEP of 7.5cm of H2O.
- b. Decreasing the inflation pressure.
- c. Increasing the flow rate by 2.5L/min.
- d. High frequency jet ventilation.
- Answer: D. High frequency jet ventilation.
- Bronchopleural fistula can occur after a pneumonectomy due to the failure
- of the bronchial stump to heal. As the bronchus is in direct connection with
- the pleural cavity, some of the tidal volume is lost into the pleural cavity
- affecting achievable lung ventilation. Management of a ventilated patient
- with a bronchopleural fistula is particularly challenging and it is often
- difficult to wean a patient from the ventilator. High frequency ventilation
- with small tidal volumes, low airway pressure and a high respiratory rate
- provides the best chance of ventilating the lungs in these patients if
- conventional ventilation fails.
- Ref.
- 1. Lois N, Noppen M. Bronchopleural fistulas: an overview of the
- problem with special focus on endoscopic management. Chest 2005;
- 128: 3955-65.
- 5. A 26-year-old woman, on her first postnatal day, suffers a fit on the postnatal ward. The first step in the immediate management should be:
- a. Intravenous magnesium sulphate, with a loading dose of 4g over 10 minutes.
- b. Intravenous lorazepam 4mg
- c. Rapid sequence induction with thiopentone and suxamethonium.
- d. Oxygen via a face mask with a reservoir bag at 15L/min.
- Answer: D. Oxygen via a face mask with a
- reservoir bag at 15L/min.
- Although magnesium sulphate is the treatment of choice for eclampsia, it
- is most important to administer oxygen immediately before implementing
- other therapy. Do not leave the woman alone but call for help, including a
- senior obstetrician. Ensure that it is safe to approach the woman and aim
- to prevent maternal injury during the convulsion. Place the woman in the
- left lateral position and administer oxygen. Assess the airway and
- breathing and check the pulse and blood pressure. Benzodiazepines and
- phenytoin should no longer be used as first-line drugs. A loading dose of
- 4g of magnesium should be administered by an infusion pump over 5-10
- minutes, followed by a further infusion of 1g/hour, maintained for 24 hours following the most recent seizure. Recurrent seizures should be treated with a further bolus of 2g of magnesium sulphate.
- Ref
- 1. RCOG Green-Top 10A guideline: The management of severe preeclampsia/
- eclampsia (www.rcog.org.uk).
- 2. The Eclampsia Trial Collaborative Group: Which anticonvulsant for
- women with eclampsia? Evidence from the Collaborative Eclampsia
- Trial. Lancet 1995; 345: 1455-6
-
144
- 1. You have been asked to review a 67-year-old patient on the fourth postoperative day, who is complaining of severe back pain and increasing numbness in both legs developing over the previous few hours. He has had an epidural in situ since his operation; the epidural infusion was switched off 8 hours ago as his blood pressure had been low. For the last few hours his temperature has been 38°C. Your first step in the management of this patient should be:
- a. Give an epidural top-up using 0.5% bupivacaine.
- b. Remove the epidural catheter and start morphine PCA.
- c. Arrange for an MRI scan of the spine.
- d. Arrange for surgical review as soon as possible
- Answer: C. Arrange for an MRI scan of the spine.
- The clinical triad of fever, back pain, and neurologic deficit is suggestive of
- an epidural abscess. A sequential evolution of symptoms and signs has
- been described, with localised spinal pain, radicular pain and
- paresthesiae, muscular weakness, sensory loss, sphincter dysfunction,
- and, finally, paralysis. The incidence is extremely rare but is partly affected
- by the time the epidural catheter has been in situ, and the general health
- of the patient. In order to prevent permanent neurological sequelae, an
- early definitive diagnosis by MRI scan and surgical decompression of the
- spinal cord and drainage of the abscess is essential. Consultation with a
- neurosurgeon or spinal surgeon should be requested when a spinal
- epidural abscess is detected or strongly suspected. Increasing
- neurological deficit, persistent severe pain, or persistent fever and
- leukocytosis are all indications for surgery.
- Ref.
- 1. Grewal S, Hocking G, Wildsmith JA. Epidural abscesses. British
- Journal of Anaesthesia 2006; 96: 292-302.
- 2. A 12-year-old girl is undergoing scoliosis correction. Anaesthesia is maintained with isoflurane in nitrous oxide and oxygen. A total of 10mg morphine has been administered as intermittent boluses. About 30 minutes into the procedure, the patient develops a tachycardia which is not responsive to a bolus of intravenous fluids or intravenous morphine. The EtCO2 is 7.2kPa despite adequate ventilation and the temperature is recorded as 39°C. The first step in the immediate treatment should be:
- a. Dantrolene sodium 1mg/kg as an initial bolus.
- b. Dantrolene sodium 2-3mg/kg as an initial bolus.
- c. Send urine sample for myoglobin.
- d. Measurement of arterial blood pH.
- Answer: B. Dantrolene sodium 2-3mg/kg as an
- initial bolus.
- The clinical scenario is suggestive of malignant hyperthermia (MH):
- unexplained tachycardia, an increase in EtCO2 and hyperthermia. The
- correct dose of dantrolene is 2-3mg/kg as an initial bolus followed by
- 1mg/kg PRN. The other immediate measures include removing the trigger
- agent, and maintaining anaesthesia with total intravenous anaesthesia.
- Active cooling measures and intravenous infusion of cold I.V. fluids should
- be performed. End-tidal CO2, invasive arterial BP, CVP, core and
- peripheral temperature, urine output and pH, arterial blood gases,
- potassium, haematocrit, platelets, clotting indices, and creatine kinase
- should be monitored. The systemic effects of MH include hyperkalaemia,
- cardiac arrhythmias, myoglobinaemia and disseminated intravascular
- coagulation.
- Ref.
- 1. Guidelines for the management of malignant hyperthermia crisis. The
- Association of Anaesthetists of Great Britain and Ireland, 2007.
- 3. A 29-year-old woman (gravida 2, para 1) has had a ventouse vaginal delivery of a baby boy weighing 4.2kg. Intramuscular syntometrine has been administered by the midwife. Thirty minutes after delivery of the placenta, she suffers a primary postpartum haemorrhage (PPH) of about 400ml. The most common cause of PPH in this scenario is:
- a. An atonic uterus
- b. Coagulopathy.
- c. Retained placental tissue.
- d. A vaginal tear.
- Answer: A. An atonic uterus.
- The most common cause of primary (within 24 hours) postpartum
- haemorrhage is an atonic uterus, occurring in about 70% of cases. The
- other options mentioned above are possible causes which need to be
- excluded. In particular, tears may cause significant haemorrhage and can
- be more difficult to diagnose if high in the genital tract, often requiring an
- examination under anaesthesia. Factors predisposing to an atonic uterus
- include a large baby (in this case), multiple pregnancy, prolonged labour
- (especially if augmented with syntocinon), abnormal placentation,
- multiparity, and chorioamnionitis
- 4. A 60-year-old male patient is admitted to the emergency department. He was working on his car in a garage and was found unconscious by his wife, with the garage door almost shut and the car engine running. On assessment, his GCS is 7, oxygen saturation is 99% and mucous membranes are ‘cherry red’ in colour. Which of the following actions is most appropriate in the immediate management?
- a. Take a full history from his wife to confirm the facts.
- b. Arrange for a CT of the brain to precisely diagnose the cause of unconsciousness.
- c. Arrange for urgent transfer to a neuro-intensive care unit.
- d. Intubate and ventilate the patient with 100% oxygen.
- Answer: D. Intubate and ventilate the patient
- with 100% oxygen.
- The history and examination findings indicate that the most likely cause of
- unconsciousness in this patient is carbon monoxide (CO) poisoning.
- Carboxy-haemoglobin (COHb) has a similar absorption spectrum to
- oxyhaemoglobin and therefore oxygen saturation is falsely raised. Carbon
- monoxide binds with haemoglobin about 250 times as avidly as oxygen
- and this adversely affects the oxygen content of blood. The half-life of
- COHb is 4 hours; it is reduced to an hour with 100% oxygen and to 20-
- 30 minutes with hyperbaric oxygen therapy. This patient is unconscious
- which indicates severe CO poisoning. Airway protection and oxygenation
- of tissue is an absolute priority and this will be best achieved by tracheal
- intubation and ventilation with 100% oxygen.
- Ref.
- 1. Piantadosi CA. Carbon monoxide poisoning. Undersea Hyperb Med
- 5. An elderly hypertensive has diabetes mellitus and bilateral renal artery stenosis. The best management is:
- a. Enalapril
- b. Verapamil
- c. Beta blockers
- d. Thiazides
Ans : b
-
145
- 1. Pure vertical nystagmus is caused by: [Pokhara 2nd set]
- a. Medulla
- b. Labyrinthitis
- c. Cochlear damage
- d. Middle ear damage
- Ans: a
- Ref: PL Dhingra and Shruti Dhingra ENT and HNS 7th Edition, Page No: 43
- Nystagmus of central origin cannot be suppressed by optic fixation.
- Purely torsional nystagmus indicates lesion of the brainstem/vestibular nuclei and is seen in syringomyelia.
- Vertical downbeat nystagmus indicates lesion at craniocervical region such as Arnold–Chiari malformation or degenerative lesion of the cerebellum.
- Vertical upbeat nystagmus is seen in lesions at the junction of pons and medulla or pons and midbrain.
- Pendular nystagmus is either congenital or acquired. The latter is seen in multiple sclerosis. Pendular nystagmus may also be disconjugate, i.e. vertical in one eye and horizontal in the other.
- 2. Most common site of histiocytosis is: [PAHS Pokhara 2nd set]
- a. Skin
- b. Bone
- c. Liver
- d. Kidney
- Ans: b
- Ref: Robbins Basic Pathology 10th Edition, Page No: 484 | Ref: Goljan Rapid Review of Pathology; 5th Edition, Page No: 384
- • The term histiocytosis is an “umbrella” designation for a variety of proliferative disorders of dendritic cells or macrophages. Some, such as very rare histiocytic lymphomas, are highly malignant neoplasms. Others, such as most histiocytic proliferations in lymph nodes, are completely benign and reactive.
- • Between these two extremes lie a group of relatively rare tumors comprised of Langerhans cells, the Langerhans cell histiocytoses.
- • Langerhans cells are immature dendritic cells found in the epidermis; similar cells are found in many other organs, and they function to capture antigens and display them to T cells.
- • Langerin is a transmembrane protein found in Birbeck granules, cytoplasmic pentalaminar rodlike tubular structures that in electron micrographs have a characteristic periodicity and sometimes a dilated terminal end (“tennis racket” appearance).
- • LCH (replacement term for histiocytosis X) is a multifaceted disorder that presents with isolated bone lesions (old term, eosinophilic granuloma), bone lesions with diabetes insipidus and exophthalmos (old term, Hand-Schuller-Christian disease), or bone lesions with disseminated disease (old term, Letterer-Siwe disease). Each will be discussed separately, with the understanding that they are now all part of LCH.
- 3. Saddle back type of temperature is seen in: [PAHS Pokhara cancelled paper]
- a. Brucellosis
- b. Dengue fever
- c. Viral fever
- d. Enteric fever
- Ans: b
- Dengue is a mosquito-borne viral disease widely spread in tropical and subtropical regions.
- Clinical features
- • Up to 40–80% of all dengue infections are asymptomatic.
- • Commonly reported clinical symptoms include sudden onset of high fever, severe headache and retro-orbital pain, myalgia, arthralgia, a maculo-papular rash and minor haemorrhage. Fever and other symptoms often follow a ‘saddleback’ sequence, with a brief remission after the third day.
- • Illness rarely lasts for more than ten days, but convalescence can be prolonged and debilitating.
- • A portion of cases, usually < 5%, can be severe and a fraction of these may be fatal. Most severe cases and fatalities occur among children and adolescents.
- • Severe dengue — commonly referred to as ‘Dengue haemorrhagic fever/Dengue shock syndrome (DHF/DSS)’ to distinguish it from ‘classic’ dengue fever (DF) — is characterised by an increase of vascular permeability that can lead to life-threatening hypovolemic shock.
- • The causes of this severe dengue condition have been debated for decades, but remain unresolved. A hotly contested hypothesis is that after a ‘primary’ infection with one serotype, ‘secondary’ infections by one or more of the other serotypes can precipitate ‘antibody dependant enhancement’ (ADE). Resolution of this issue is hampered by the absence of a reliable animal model. In addition, controversy remains with regards to the validity of a graded set of criteria to categorise severity. Both issues are of prime importance for the management and treatment of patients, and to future acceptance of dengue vaccines.
- 4. A 28 year old woman presents with hearing loss. Hearing loss is bilateral, slowly progressive, pure tone audiometry bone conduction hearing loss with an apparent bone conduction hearing loss at 2000 Hz. What is the most likely diagnosis? [PAHS Patan 2019]
- a. Otosclerosis
- b. Acoustic neuroma
- c. Otitis media with effusion
- d. Sigmoid sinus thrombosis
- Ans: a
- Ref: PL Dhingra and Shruti Dhingra ENT and HNS 7th Edition, Page No: 96
- Otosclerosis:
- • Hearing loss: This is the presenting symptom and usually starts in twenties. It is painless and progressive with insidious onset. Often it is bilateral conductive type.
- • Paracusis Willisii. An otosclerotic patient hears better in noisy than in quiet surroundings. This is because a normal person will raise his voice in noisy surroundings.
- • Tinnitus. It is more commonly seen in cochlear otosclerosis and in active lesions.
- • Vertigo. It is an uncommon symptom.
- • Speech. Patient has a monotonous, well-modulated soft speech.
- • Pure tone audiometry shows loss of air conduction, more for lower frequencies.
- • Bone conduction is normal. In some cases, there is a dip in bone conduction curve. It is different at different frequencies but maximum at 2000 Hz and is called Carhart’s notch (5 dB at 500 Hz, 10 dB at 1000 Hz, 15 dB at 2000 Hz and 5 dB at 4000 Hz). Carhart’s notch disappears after successful stapedectomy.
- 5. Paralysis of axillary nerve impairs? [PAHS Patan 2018]
- a. Adduction
- b. Abduction
- c. Flexion
- d. Extension
- Ans: b
- Ref: Moore's Anatomy 7th Edition, Page No: 815
- Axillary Nerve Injury
- • The axillary nerve may be injured when the glenohumeral joint dislocates because of its close relation to the inferior part of the joint capsule.
- • The subglenoid displacement of the head of the humerus into the quadrangular space damages the axillary nerve.
- • Axillary nerve injury is indicated by paralysis of the deltoid (manifest as an inability to abduct the arm to or above the horizontal level) and loss of sensation in a small area of skin covering the central part of the deltoid.
-
146
- Q1.Guillain Barre syndrome may follow administration vaccines against following EXCEPT
- a.Rabies
- b.Influenza
- c.Measles
- d.Conjugated meningococcal vaccine type C
- Ans: c, Measles
- Ref: Nelson textbook of pediatrics 21st edition page no:12900
- •West Nile virus also can mimic Guillain-Barré–like syndrome, but more often causes a motor neuron disease similar to poliomyelitis.
- •GBS may follow administration of vaccines against rabies, influenza, and conjugated meningococcal vaccine, particularly serogroup C.
- •Other infectious precursors of GBS include mononucleosis, Lyme disease, cytomegalovirus, and the Zika virus.
- •Initial symptoms include numbness and paresthesia, followed by weakness
- •Radicular back pain and myalgia are common in the initial stages; affected children can be very irritable. Weakness usually begins in the lower extremities and progressively involves the trunk, the upper limbs, and finally the bulbar muscles, but weakness is sometimes proximally prominent.
- •Extraocular muscle involvement is rare, but many patients develop facial weakness. In most patients, weakness is essentially symmetric.
- •Weakness progresses over days or weeks, the clinical nadir occurring in less than 4 wk. Approximately 60% of children lose the ability to walk at some point in their illness; a small proportion progress to flaccid tetraplegia. The maximal severity of weakness is reached by 4 wk after onset.
- Q2. Munson sign is seen in:
- a.Pterygium
- b.Keratoconus
- c.Sclerocornea
- d.Trachoma
Ans: b, Keratoconus
- Ref:Nelson textbook of pediatrics 21st edition pg13016
- Keratoconus
- • This is a disease of unclear pathogenesis characterized by progressive thinning and bulging of the central cornea, which becomes cone shaped. Although familial cases are known, most cases are sporadic.
- • Eye rubbing and contact lens wear have been implicated as pathogenic, but the evidence to support this is equivocal. The incidence is increased in individuals with atopy, Down syndrome, Marfan syndrome, and retinitis pigmentosa.
- • Most cases are bilateral, but involvement may be asymmetric. The disorder usually presents and progresses rapidly during adolescence; progression slows and stabilizes when patients reach full growth. Descemet membrane may occasionally be stretched beyond its elastic breaking point, causing an acute rupture in the membrane with resultant sudden and marked corneal edema (acute hydrops ) and decrease in vision.
- •The corneal edema resolves as endothelial cells cover the defective area. Some degree of corneal scarring occurs, but the visual acuity is often better than before the initial incident.
- •Signs of keratoconus include Munson sign (bulging of the lower eyelid on looking downward) and the presence of a Fleischer ring (a deposit of iron in the epithelium at the base of the cone).
- •Glasses and contact lenses are the first step in treating the visual distortion caused by keratoconus. Corneal cross linking is a relatively new procedure using riboflavin and UV light, and may arrest the progression of keratoconus.
- •If the cornea vaults too severely for the vision to be corrected with contact lenses then a corneal transplant must be performed to restore vision.
- Q3. Each ml of PRBC contains:
- a.1 mg iron
- b. 2 mg iron
- c. 3 mg iron
- d. 4 mg iron
- Ans: a , 1 mg iron
- Ref:Nelson textbook of pediatrics 21st edition page no:10040
- •Transfusion-induced hemosiderosis becomes the major clinical complication of transfusion-dependent thalassemia.
- •Each mL of packed red cells contains approximately 1mg of iron.
- •Physiologically, there is no mechanism to eliminate excess body iron.
- •Iron is initially deposited in the liver and is followed by deposition in the endocrine organs and the heart. This leads to a high rate of hypothyroidism,hypogonadotrophic gonadism, growth hormone deficiency, hypoparathyroidism,and diabetes mellitus.
- •Iron deposition in the heart causes heart failure and arrhythmias, and heart disease is the leading cause of death in inadequately chelated patients. Eventually, most patients not receiving adequate iron chelationtherapy die from cardiac failure or cardiac arrhythmias secondary to hemosiderosis.
- •Hemosiderosis-induced morbidity can be prevented by adequate iron chelation therapy.
- Q4. Monitoring of plasma concentration is of no value for:
- a.Drugs activated in the body
- bIn case of poisoning
- c.In case of failure of response without any apparent reason
- d.Drugs with low safety margin
- Ans:a,Drugs activated in the body
- Ref:KD Tripathi 7th edition page no:34
- Therapeutic drug monitoring (TDM) is particularly useful in the following situations:
- 1. Drugs with low safety margin, e.g. —digoxin, anticonvulsants, antiarrhythmics, theophylline, aminoglycoside antibiotics, lithium, tricyclic antidepressants.
- 2. If individual variations are large, e.g.—antidepressants, lithium.
- 3. Potentially toxic drugs used in the presence of renal failure, e.g. —aminoglycoside antibiotics, vancomycin.
- 4. In case of poisoning.
- 5. In case of failure of response without any apparent reason, e.g. —antimicrobials.
- Monitoring of plasma concentration is of no value for:
- 1. Drugs whose response is easily measurable, e.g.— antihypertensives, hypoglycaemics, diuretics, oral anticoagulants, general anaesthetics.
- 2. Drugs activated in the body, e.g.—levodopa.
- 3. ‘Hit and run drugs’ (whose effect lasts much longer than the drug itself), e.g.—reserpine, guanethidine, MAO inhibitors, omeprazole.
- 4. Drugs with irreversible action, e.g.—organophosphate anticholinesterases, phenoxybenzamine.
-
147
DAILY QUESTIONS (JESTHA 24):
- Q1.“Quickening” (feeling of life) denotes the perception of active fetal movements by the women. At about what period of gestation it is usually felt by primigravidae?
- a. 16 weeks
- b. 18 weeks
- c. 20 weeks
- d. Near Term
Ans : b
- Q2. For muscle relaxation, you give intravenous d-tubocurarine to a patient who is planned for orthopedic surgery. You notice there is decrease in blood pressure after giving d-tubocurarine. D-Tubocurarine produces significant fall in BP due to:
- a. Ganglionic blockade
- b. Increased venous return
- c. Inhibiting histamine release
- d. All of the above
Ans : a
- Q3. Delivery of an antimicrobial process (gas or liquid) indirectly to the internal surfaces of an enclosed area is called:
- a. Fumigation
- b. Sanitization
- c. Aseptic processing
- d. Cleaning
Ans : a
-
148
Answers to yesterday's Questions (BBD)
- 1.Which of the following is an autosomal recessive disorder?
- a. Duchenne muscular dystrophy
- b. Friedreich ataxia
- c. Wiskott Aldrich syndrome
- d. Lesch Nyhan syndrome
Ans: b
- Ref: Robbins & Cotran, Pathologic basis of disease, South Asia edition, Vol 1, p141, 142.
- Image
- Duchenne muscular dystrophy, Wiskott Aldrich syndrome and Lesch Nyhan syndrome are X-linked recessive disorders.
- Friedreich ataxia is an autosomal recessive disorder.
- Other autosomal recessive disorders include congenital adrenal hyperplasia, alkaptonuria, alpha 1 antitrypsin deficiency, phenylketonuria and Wilson disease.
- 2. Radiological appearance of thimble' bladder is seen in the following condition:
- a. Cystitis cystica
- b. Chronic tuberculous cystitis
- c. Neurogenic bladder
- d. Acute tuberculous cystitis
Correct Ans: b
- Explanation
- Early tuberculosis of the bladder commences around the ureteric orifice or trigone, the
- earliest evidence being pallor of the mucosa due to submucosal oedema. Subsequently,
- tubercles may be seen and, in long standing cases, there is marked fibrosis and the capacity
- of the bladder is greatly reduced giving the radiological appearance of thimble bladder'.
- Ref: Bailey & Love's Short Practice of Surgery 26E edited by
- Norman Williams, Christopher Bulstrode, P Ronan O'Connell,
- 2013, Page 1327,1328
- 3. The standard normal distribution:
- a. Is skewed to the left
- b. Has mean = 1.0
- c. Has standard deviation = 0.0
- d. Has variance = 1.0
Ans : d. Has variance = 1.0
- • A standard normal deviation is not skewed to any side. It is symmetrical
- • Standard deviation = 1.0
- • Variance = SD^2 = 10
-
149
- Ans: DAD (to yesterday’s questions)
- 1. Which is a true statement regarding inhibin?
- a. Increases the secretion of GnRH.
- b. Molecular weight is between 5000 and 10,000.
- c. It is secreted by hypothalamus.
- d. It is a glycoprotein.
- Ans: D, It is a glycoprotein.
- Ref: Textbook of Medical Physiology, Guyton and Hall, 11th edition, page 1007.
- * Inhibin is secreted by the sertoli cells in males and by the granulosa cells in females.
- * It inhibits the secretion of Gonadotrophin Releasing Hormone.
- * It is a glycoprotein with molecular weight between 10,000 and 30,000.
- * It has potent inhibitory feedback effect on the anterior pituitary gland which in turn provides negative feedback mechanism for the control of spermatogenesis.
- 2. Blaschko’s lines represent?
- a. Lines of development
- b. Dermatomes
- c. Lines along blood vessels
- d. Lines along lymphatics
- Ans: A, Lines of development
- BLASCHKO’S LINES
- * Blaschko’s lines are invisible developmental lines of the skin
- * They are thought to represent the pathways of epithelial migration and proliferation in the foetus
- * These were first demonstrated by German dermatologist Alfred Blaschko in 1901.
- * They do not correspond to the path of vessels / nerves / lymphatics
- * Many skin lesions follow Blaschko’s lines. eg: Incontinentia pigmenti, Linear lichen planus, Lichen striatus Naevus achromicus
- 3. Most common cause of premature death in schizophrenia is?
- a. Drug toxicity
- b. Nosocomial infection
- c. Homicide
- d. Suicide
- Ans: D, Suicide
- PREMATURE DEATH IN SCHIZOPHRENIA
- * Life expectancy of patients with schizophrenia is decreased by approximately 15 to 25 years.
- * Suicide is the most important cause of premature death in patients with schizophrenia.
- * There is a 20 fold increased risk of suicide compared to the general population.
- * Suicide attempts may occur without warning or expression of intent.
- * Cancer is the second most frequent cause of mortality in schizophrenics.
- * The rates of metabolic syndrome and cardiovascular disorders are also increased.
-
150
- 1. Hürthle cells are associated with which of the following?
- a. deQuervain thyroiditis
- b. Graves disease
- c. Hashimoto thyroiditis
- d. Riedel thyroiditis
- Ans: c
- Ref: Robbins Basic Pathology 10th Edition, Page No: 758 – 759
- • Hashimoto thyroiditis is the most common cause of hypothyroidism in areas of the world where iodine levels are sufficient. Hashimoto thyroiditis is an autoimmune disease caused by an immune response to thyroid autoantigens.
- • The thyroid usually is diffusely and symmetrically enlarged. Microscopic examination reveals widespread infiltration of the parenchyma by a mononuclear inflammatory infiltrate containing small lymphocytes, plasma cells, and well-developed germinal centers.
- • The thyroid follicles are atrophic and are lined in many areas by epithelial cells distinguished by the presence of abundant eosinophilic, granular cytoplasm, termed Hürthle, or oxyphil, cells.
- • This is a metaplastic response of the normal low cuboidal follicular epithelium to ongoing injury; on ultrastructural examination, the Hürthle cells are characterized by numerous prominent mitochondria. Interstitial connective tissue is increased and may be abundant. Less commonly, the thyroid is small and atrophic as a result of more extensivefibrosis (fibrosing variant).
- 2. Human development index includes all except:
- a. Life expectancy at birth
- b. Life expectancy at 1 yr of age
- c. Knowledge
- d. Income
- Ans: B
- Ref: Park's Textbook of Preventive and Social Medicine; 23rd Edition; Page No: 17
- Human Development Index (HDI)
- • Human development index (HDI) is defined as "a composite index combining indicators representing three dimensions - longevity (life expectancy at birth); knowledge (mean years of schooling and expected years of schooling. Before the year 2009, the indicators used were adult literacy rate and gross enrolment ratio) and income (GNI per capita in purchasing power parity in US dollars)".
- • Thus the concept of HDI reflects achievements in the most basic human capabilities, viz, leading a long life, being knowledgeable and enjoying a decent standard of living. Hence, these three variables have been chosen to represent those dimensions.
- • The HDI is a more comprehensive measure than per capita income. Income is only a means to human development, not an end. Nor is it a sum total of human lives. Thus by focusing on areas beyond income and treating income as a proxy for a decent standard of living, the HD! provides a more comprehensive picture of human life than income does.
- • The HDI values range between 0 to 1.
- 3. Agar concentration required to prepare nutrient agar is:
- a. 2%
- b. 6%
- c. C. 0.25%
- d. 0.5%
- Ans: a
- Ref: Anantnarayan and Paniker 7th Edition, Page No: 37
- Simple/Basal Media
- They contain minimum ingredients that support the growth of non-fastidious bacteria. Examples include-
- • Peptone water: It contains peptone (1%)+ NaCl (0.5%) + water
- • Nutrient broth: It is made up of peptone water+ meat extract (1%). lt is available in three forms: 1) meat extract, (2) meat infusion, (3) meat digest broth.
- • Nutrient agar: It is made up of nutrient broth+ 2% agar.
- • Semisolid medium: It is prepared by reducing the concentration of agar to 0.2 to 0.5 %.
- 4. Which of the following feature of thyroid nodule on USG is not suggestive of malignancy?
- a. Hypoechogenecity
- b. Hyperechogenecity
- c. Solid nodules
- d. Micro calcification
- Ans: B
- Ref: Harrison’s Principles of Internal Medicine; 19th Edition, Page No: 2289
- Thyroid Ultrasound
- • Certain sonographic patterns are highly suggestive of malignancy (e.g., hypoechoic solid nodules with infiltrative borders and microcalcifications), whereas other features correlate with benignity (e.g., spongiform nodules defined as those with multiple small internal cystic areas).
- • In addition to evaluating thyroid nodules, ultrasound is useful for monitoring nodule size and for the aspiration of nodules or cystic lesions.
- • Ultrasound-guided FNA biopsy of thyroid lesions lowers the rate of inadequate sampling and decreases sample error, thereby reducing the false-negative rate of FNA cytology.
- • Ultrasonography of the central and lateral cervical lymph node compartments is indispensable in the evaluation thyroid cancer patients, preoperatively and during follow-up.
-
151
- 1. A 27-year-old woman presents to her primary care physician due to headache, chest pain, and food intolerance. These symptoms have been very distressing for her and reports that these symptoms have been present for approximately 8 months. In somatization disorder, which of the following is true to fulfill the diagnostic criteria?
- a. 2 pain, 2 GI, 2 sexual, & 2 pseudoneurologic symptom
- b. 3 pain, 2 GI, 2 sexual, & 1 pseudoneurologic symptom
- c. 3 pain, 2 GI, 1 sexual, & 2 pseudoneurologic symptom
- d. 4 pain, 2 GI, 1 sexual, & 1 pseudoneurologic symptom
- Ans: d
- Ref: Harrison’s Principles of Internal Medicine; 18th Edition, Page No: 3541
- Formal diagnostic criteria for somatization disorder requires the recording of at least four pain, two gastrointestinal, one sexual, and one pseudoneurologic symptom.
- 2. A 15 year old male presents with bone pain, pathological fractures, hepatomegaly, splenomegaly and hematological disturbances. Which of the following metabolic disorder results in Erlenmeyer flask deformity of distal femur and accumulation of glucosylceramide?
- a. Niemann Pick disease
- b. Gaucher disease
- c. Hunter syndrome
- d. Galactosemia
- Ans: b
- Ref: Nelson Textbook of Pediatrics, Elsevier, 20th Edition, Page No: 709-710
- Gaucher disease
- • This disease is a multisystemic lipidosis characterized by hematologic abnormalities, organomegaly, and skeletal involvement, the latter usually manifesting as bone pain and pathologic fractures
- • Gaucher disease results from the deficient activity of the lysosomal hydrolase, acid β-glucosidase, which is encoded by a gene located on chromosome 1q21-q31.
- • The enzymatic defect results in the accumulation of undegraded glycolipid substrates, particularly glucosylceramide, in cells of the reticuloendothelial system. This progressive deposition results in infiltration of the bone marrow, progressive hepatosplenomegaly, and skeletal complications
- • Type I Gaucher disease -Most patients develop radiologic evidence of skeletal involvement, including an Erlenmeyer flask deformity of the distal femur
- 3. Tailor’s bursa occurs between:
- a. Head of first metatarsal and skin
- b. Lateral malleolus and skin
- c. Gluteus maximus, ischial tuberosity and skin
- d. Calcaneum and heel
- Ans: b
- Ref: SRB's Manual of Surgery; 5th Edition, Page No: 84, 85
- • Bunion is adventitious bursa in patient with hallux valgus occurring between head of first metatarsal and skin.
- • Tailor’s bursa occurs between lateral malleolus and skin.
- • Porter’s bursa occurs between skin over shoulder and clavicle.
- • Weaver’s bursa occurs between gluteus maximus, ischial tuberosity and skin.
- • Retro-Achillis bursitis occurs between skin and Achilles tendon.
- • Subcalcaneal bursitis occurs between calcaneum and heel in long distance runners.
- • Billing gate hump appears over 7th cervical spine deep to overlying skin in people carrying weight over it. Billing gate is a large fish market in London.
- 4. A psychiatry inpatient routinely misbehaves with one particular doctor every time she attends him. Upon further evaluation, it was found that he suspects her to be his wife who has been trying to kill him accompanied by her lover. What is the likely disorder?
- a. Capgras syndrome
- b. Fregoli syndrome
- c. Intermetamorphosis
- d. Syndome of subjective doubles
- Ans: b
- Ref: Niraj Ahuja Psychiatry 7th Edition, Page No: 87
- There are several delusional misidentification syndromes, of which there are four types described:
- Typical Capgras’ syndrome (Illusion des sosies): Here the patient sees a familiar person as a complete stranger who is imposing on him as a familiar person.
- Illusion de Fregoli: The patient falsely identifies stranger(s) as familiar person(s).
- Syndrome of subjective doubles: The patient’s own self is perceived as being replaced by a double.
- Intermetamorphosis: Here the patient’s misidentification is complete and the patient misidentifies not only the ‘external appearance’ (as in the previous three types) but also the complete personality.
- 5. An infant is born via NVD. The Maternal Streptococcus agalactiae serology was unknown at the time of delivery. The mother's past medical history is notable only for cocaine and IV drug use; however, the patient denies any illicit substances for over a year. During delivery there was a prolonged second stage of labor complicated by uterine atony, which was managed with bimanual massage, IV fluids and oxytocin in a rural hospital. On day 4 of life, the newborn presents with a severe bilateral mucopurulent discharge from her eyes. She is crying and inconsolable. Blood pressure is 65/40 mmHg and pulse is 118/min. Which of the following is the best next step in management for this patient?
- a. Apply topical erythromycin eye ointment
- b. Apply topical silver sulfadiazine
- c. Administer oral erythromycin
- d. Administer IM ceftriaxone
- Ans: d
- Ref: Nelson Textbook of Pediatrics, Elsevier, 20th Edition, Page No: 3038
- • This newborn is presenting with mucopurulent discharge from the eye within 5 days of birth consistent with ophthalmia neonatorum (gonococcal conjunctivitis). The treatment of choice is IM ceftriaxone to cover N. gonorrhea.
- • Newborns that present within 5 days of birth with a mucopurulent discharge from the eye should be considered to have bacterial conjunctivitis caused by Neisseria gonorrhea (ophthalmia neonatorum). Typically erythromycin drops are given to newborns to prevent gonococcal conjunctivitis; however, the history in this case suggests that this routine prophylaxis did not occur. In contrast, conjunctivitis from Chlamydia trachomatis tends to occur days to weeks later and is less purulent in nature than gonococcal conjunctivitis. This mother's limited prenatal care predisposes her to having untreated sexually transmitted infections that could affect her newborn, especially when the mode of delivery is vaginal. The best treatment for gonococcal conjunctivitis is IM ceftriaxone and irrigation of the eye.
-
152
- 1. Which of the following is considered as “Contact” as per National testing guideline of Nepal?
- a. Face-to-face contact with a probable or confirmed case within 1 meter and for more than 15 minutes
- b. Direct physical contact with a probable or confirmed case
- c. Direct care for a patient with probable or confirmed COVID-19 disease without using proper personal protective equipment
- d. All of the above
- Ans: d
- Ref: National Testing Guidelines for COVID-19
- Contact
- • Face-to-face contact with a probable or confirmed case within 1 meter and for more than 15 minutes;
- • Direct physical contact with a probable or confirmed case;
- • Direct care for a patient with probable or confirmed COVID-19 disease without using proper personal protective equipment.
- 2. In a resource-constrained environment, imaging for COVID-19 patient is indicated for:
- a. Mild clinical features
- b. Moderate-severe clinical features
- c. Low pretest probability of disease
- d. PCR positive status
- Ans: b
- Ref: Rubin Geoffrey D., Linda B. Haramati and Jeffrey P. Kanne et al. "The Role of Chest Imaging in Patient Management during the COVID-19 Pandemic: A Multinational Consensus Statement from the Fleischner Society". Radiology (2020): 201365
- According to a Fleischner Society consensus:
- • Imaging is not indicated in patients with suspected COVID-19 and mild clinical features unless they are at risk for disease progression
- • Imaging is indicated in a patient with COVID-19 and worsening respiratory status
- • In a resource-constrained environment, imaging is indicated for medical triage of patients with suspected COVID-19 who present with moderate-severe clinical features and a high pretest probability of disease
- 3. Which of the following serves as the entry point into cells for SARS-CoV-2?
- a. CXCR4
- b. Na-K-ATPase
- c. ACE2
- d. Dystrophin
- Ans: c
- Ref: Gene: ACE2, angiotensin I converting enzyme 2". National Center for Biotechnology Information (NCBI). U.S. National Library of Medicine. 2020-02-28.
- Angiotensin-converting enzyme 2 (ACE2):
- • Enzyme attached to the cell membranes of cells in the lungs, arteries, heart, kidney, and intestines.
- • ACE2 lowers blood pressure by catalysing the hydrolysis of angiotensin II (a vasoconstrictor peptide) into angiotensin (1–7) (a vasodilator).
- • ACE2 also serves as the entry point into cells for some coronaviruses, including HCoV-NL63, SARS-CoV, and SARS-CoV-2. The human version of the enzyme is often referred to as hACE2.
- 4. Which of the following anti-parasitic drug was taken into consideration for treatment of COVID-19?
- a. Remdesivir
- b. Ribavirin
- c. Umifenovir
- d. Mefloquine
- Ans: d
- Ref: Weston S, Haupt R, Logue J, Matthews K, Frieman MB (27 March 2020). "FDA approved drugs with broad anti-coronaviral activity inhibit SARS-CoV-2 in vitro"
- COVID-19 drug repurposing research drugs:
- Antiviral drugs
- • Favipiravir
- • Remdesivir
- • Ribavirin
- • Umifenovir
- Antiparasitics
- • Chloroquine
- • Hydroxychloroquine
- • Mefloquine
- • Ivermectin
- Antibiotics
- • Teicoplanin
- • Oritavancin
- • Dalbavancin
- • Monensin
- • Azithromycin
- Immunologicals
- • Type I Interferons (Interferon-β, peginterferon alpha-2a and -2b)
- • Anti-IL-6 agents (tocilizumab)
- 5. When was first case of COVID 19 confirmed in Nepal?
- a. Dec 2019
- b. Jan 2020
- c. Feb 2020
- d. Mar 2020
- Ans: b
- Ref: Shrestha, Ranish; Shrestha, Sunil; Khanal, Pratik; Kc, Bhuvan (18 May 2020). "Nepal's first case of COVID-19 and public as health response". Journal of Travel Medicine. 27 (3)
- The first case in Nepal was confirmed on 23 January 2020 when a 31-year-old student, who had returned to Kathmandu from Wuhan on 9 January, tested positive for the diseases.
-
152
- 1. Maladie de roger, which creates a very loud murmur refers to
- a. Small VSD
- b. Large VSD
- c. Small ASD
- d. Large ASD
- Ans: a
- Ref: Harrison’s Principles of Internal Medicine; 20th Edition, Page No: 244
- • The holosystolic murmur of a VSD is loudest at the mid- to lower-left sternal border and radiates widely.
- • A thrill is present at the site of maximal intensity in the majority of patients. There is no change in the intensity of the murmur with inspiration.
- • The intensity of the murmur varies as a function of the anatomic size of the defect. Small, restrictive VSDs, as exemplified by the maladie de Roger, create a very loud murmur due to the significant and sustained systolic pressure gradient between the left and right ventricles. With large defects, the ventricular pressures tend to equalize, shunt flow is balanced, and a murmur is not appreciated.
- 2. What is the mutation in Familial adenomatous polyposis (FAP)?
- a. APC
- b. PTEN
- c. SMAD4
- d. STK11
- Ans: a
- Ref: Robbins Basic Pathology, 10th edition, page 627 – 629
- • Familial adenomatous polyposis (FAP) is an autosomal dominant disorder marked by the appearance of numerous colorectal adenomas by the teenage years. It is caused by mutations of the adenomatous polyposis coli gene (APC).
- • A count of at least 100 polyps is necessary for a diagnosis of classic FAP, and as many as several thousand may be present.
- 3. Undescended testes are unlikely to spontaneously descend after what age?
- a. At birth
- b. 3 months of age
- c. 9 months of age
- d. 15 months of age
- Ans: b
- Ref: Bailey and Love’s Short Practice of Surgery 27th Edition, Page No: 1498
- • Incompletely descended testes are often macroscopically normal in early childhood, but by puberty the testis is typically smaller compared with its intrascrotal counterpart.
- • Microscopic changes are apparent from 1–2 years including loss of Leydig cells, degeneration of Sertoli cells and decreased spermatogenesis. The higher the testis, the greater the degree of histological change.
- • Testes that are absent from the scrotum after 3 months of age are unlikely to descend.
- • An incompletely descended testis tends to atrophy as puberty approaches. Boys with undescended testes are at greater risk of infertility, testicular malignancy, hernia and testicular torsion.
- 4. Baclofen is used in the treatment of:
- a. Schizophrenia
- b. Depression
- c. Anxiety
- d. Spasticity
- Ans: d
- Ref: KD Tripathi Essentials of Medical Pharmacology; 7th Edition, Page No: 357
- Classification of Centrally acting Muscle relaxants:
- Mephenesin: Mephenesin, congeners Carisoprodol, Chlorzoxazone, Chlormezanone, Methocarbamol.
- Benzodiazepines: Diazepam and others
- GABA mimetic: Baclofen, Thiocolchicoside
- Central α2 agonist: Tizanidine
- 5. In status asthmaticus, the anesthetic agent used as bronchodilator is
- a. Morphine
- b. Thiopentone sodium
- c. Ketamine
- d. Halothane
- Ans: c
- Ref: Ajay Yadav Anesthesia 6th Edition, Page No: 85
- Salient features of ketamine
- Phencyclidine derivative
- Acts on NMDA receptor
- Increases all pressures ( BP, ICP, intra abdominal and intrathoracic pressure)
- ��� Causes bronchodilatation and preserves airway reflexes
- Analgesic effect excellent
-
154
- Which of the following is not permeable through Blood-brain barrier?
- a. Water
- b. Co2
- c. Protein
- d. lipid-soluble free forms of steroid hormones
- Ans: C
- Ref: Ganong Review of Medical Physiology, Lange; 25th Edition, Page No: 604-605
- Blood-brain barrier
- • The tight junctions between capillary endothelial cells in the brain and between the epithelial cells in the choroid plexus effectively prevent proteins from entering the brain in adults and slow the penetration of some smaller molecules as well. An example is the slow penetration of urea.
- • This uniquely limited exchange of substances into the brain is referred to as the blood-brain barrier, a term most commonly used to encompass this barrier overall and more specifically the barrier in the choroid epithelium between blood and CSF.
- • Passive diffusion across the tight cerebral capillaries is very limited, and little vesicular transport takes place. However, there are numerous carrier-mediated and active transport systems in the cerebral capillaries. These move substances out of as well as into the brain, though movement out of the brain is generally freer than movement into it.
- PENETRATION OF SUBSTANCES INTO THE BRAIN
- • Water, CO2, and 02 penetrate the brain with ease, as do the lipid-soluble free forms of steroid hormones, whereas their protein-bound forms and, in general; all proteins and polypeptides do not.
- False about Brown- Séquard syndrome is:
- a. The lesion to fasciculus gracilus or fasciculus cuneatus leads to ipsilateral loss of discriminative touch, vibration, and proprioception below the level of the lesion.
- b. The loss of the spinothalamic tract leads to contra¬lateral loss of pain and temperature sensation beginning one or two segments below the lesion.
- c. Damage to the cor¬ticospinal tract produces weakness and spasticity in certain muscle groups on the same side of the body.
- d. The loss of the spinocerebellar tract leads to ipsilateral loss of vibration and discriminative touch sensation beginning one or two segments below the lesion.
- Ans: d
- Ref: Ganong Review of Medical Physiology, Lange; 25th Edition, Page No: 172
- Brown-Séquard Syndrome
- A functional hemisection of the spinal cord causes a char¬acteristic and easily recognized clinical picture that reflects damage to ascending sensory (dorsal column pathway, ventrolateral spinothalamic tract) and descending motor (corticospinal tract) pathways, which is called the Brown- Séquard syndrome.
- The lesion to fasciculus gracilus or fasciculus cuneatus leads to ipsilateral loss of discriminative touch, vibration, and proprioception below the level of the lesion.
- The loss of the spinothalamic tract leads to contra¬lateral loss of pain and temperature sensation beginning one or two segments below the lesion.
- Damage to the cor¬ticospinal tract produces weakness and spasticity in certain muscle groups on the same side of the body.
- Although a precise spinal hemisection is rare, the syndrome is fairly common because it can be caused by a spinal cord tumor, spinal cord trauma, degenerative disk disease, and ischemia.
- Long acting sulfonamide is:
- a. Sulfadiazine
- b. Sulfamethoxazole
- c. Sulfamethopyrazine
- d. Silver sulfadiazine
- Ans: c
- Ref: KDT Pharmacology 7th Edition, Page No: 704
- Sulfonamides that are still of clinical interest are:
- Short acting (4–8 hr): Sulfadiazine
- Intermediate acting (8–12 hr): Sulfamethoxazole
- Long acting (~7 days): Sulfadoxine, Sulfamethopyrazine
- Special purpose sulfonamides: Sulfacetamide sod., Mafenide, Silver sulfadiazine, Sulfasalazine
- Which of the following is not permeable through Blood-brain barrier?
- a. Water
- b. Co2
- c. Protein
- d. lipid-soluble free forms of steroid hormones
- Ans: C
- Ref: Ganong Review of Medical Physiology, Lange; 25th Edition, Page No: 604-605
- Blood-brain barrier
- • The tight junctions between capillary endothelial cells in the brain and between the epithelial cells in the choroid plexus effectively prevent proteins from entering the brain in adults and slow the penetration of some smaller molecules as well. An example is the slow penetration of urea.
- • This uniquely limited exchange of substances into the brain is referred to as the blood-brain barrier, a term most commonly used to encompass this barrier overall and more specifically the barrier in the choroid epithelium between blood and CSF.
- • Passive diffusion across the tight cerebral capillaries is very limited, and little vesicular transport takes place. However, there are numerous carrier-mediated and active transport systems in the cerebral capillaries. These move substances out of as well as into the brain, though movement out of the brain is generally freer than movement into it.
- PENETRATION OF SUBSTANCES INTO THE BRAIN
- • Water, CO2, and 02 penetrate the brain with ease, as do the lipid-soluble free forms of steroid hormones, whereas their protein-bound forms and, in general; all proteins and polypeptides do not.
-
155
Q1.Rita a 26 years old primi with history of rheumatic heart disease came to your clinic for antenatal checkup. Which of the following is the commonest heart lesion met during pregnancy?
- a. Mitral regurgitation
- b. Mitral stenosis
- c. Tricuspid regurgitation
- d. Aortic stenosis
- Ans, b. Mitral stenosis
Ref: Dc Dutta Obstetrics 8th edition page no 319,323,324
- HEART DISEASE IN PREGNANCY
- INCIDENCE AND TYPES:
- • The incidence of cardiac lesion is less than 1% amongst hospital deliveries.
- • The commonest cardiac lesion is of rheumatic origin followed by the congenital ones.
- • The ratio between the two has fallen over the past two decades from 10: 1 to about 3: 1 or even 1: 1 in advanced countries.
- • Adequate treatment of rheumatic fever by appropriate antibiotics to cope with the group A β-hemolytic streptococcal infection, pari passu with the advancement in cardiac surgery to rectify the congenital heart lesions, are responsible for the change in the profile.
- • Rheumatic valvular lesion predominantly includes mitral stenosis (80%).
- • Predominant congenital lesions include patent ductus arteriosus, atrial or ventricular septal defect, pulmonary stenosis, coarctation of aorta and Fallot’s tetralogy.
- • Rare causes are hypertensive, thyrotoxic, syphilitic or coronary cardiac diseases.
- MITRAL STENOSIS:
- • Mitral stenosis is the commonest heart lesion met during pregnancy.
- • Normal mitral valve area ranges between 4 and 6 cm2.
- • Symptoms usually appear when stenosis narrows this to less than 2.5 cm2.
- • Women with mitral valve area ≤1 cm2, have the high rate of pulmonary edema (55%) and arrhythmia (33%).
- • In asymptomatic cases, the mortality is < 1% but once it is significantly symptomatic, mortality ranges between 5% and 15%.
- • Diagnosis and management has been mentioned earlier.
- • During labor continuous epidural analgesia is ideal and intravenous fluid overload is to be avoided.
- AORTIC STENOSIS:
- • Most cases of aortic stenosis are congenital, some are rheumatic in origin.
- • Normal aortic valve area is 3–4 cm2.
- • When it is reduced to less than or equal to 1 cm2, stenosis is significant.
- • Maternal mortality of significant aortic stenosis is about 15–20% with perinatal loss of about 30%.
- • Epidural anesthesia is contraindicated.
- • During labor, fluid therapy (125–150 ml/h) should not be restricted.
- • Left ventricular after load is high and the pregnant patient is sensitive to hemorrhage.
- Q2. Halothane is considered a potent bronchodilator, as it oft en reverses asthma-induced bronchospasm. This action is:
- a. Inhibited by adrenergic blocking agent
- b. Not inhibited by adrenergic blocking agent
- c. By increasing intracellular calcium mobilization
- d. By inhibiting relaxation of bronchial smooth muscle
- Ans; b. Not inhibited by adrenergic blocking agent
- Ref: Morgan and Mikhail’s clinical anesthesiology 5th edition, page number 167
- RESPIRATORY ACTIONS OF HALOTHANE:
- • Halothane typically causes rapid, shallow breathing.
- • The increased respiratory rate is not enough to counter the decreased tidal volume, so alveolar ventilation drops, and resting PaCO 2 is elevated.
- • Apneic threshold, the highest Pa co 2 at which a patient remains apneic, also rises because the difference between it and resting Pa co 2 is not altered by general anesthesia. Similarly, halothane limits the increase in minute ventilation that normally accompanies a rise in PaCO2.
- • Halothane’s ventilatory effects are probably due to central (medullary depression) and peripheral (intercostal muscle dysfunction) mechanisms.
- • These changes are exaggerated by preexisting lung disease and attenuated by surgical stimulation.
- • The increase in Pa co 2 and the decrease in intra-thoracic pressure that accompany spontaneous ventilation with halothane partially reverse the depression in cardiac output, arterial blood pressure, and heart rate described above.
- • Hypoxic drive is severely depressed by even low concentrations of halothane (0.1 MAC).
- • Halothane is considered a potent bronchodilator, as it often reverses asthma-induced bronchospasm.
- • This action is not inhibited by adrenergic blocking agents.
- • Halothane attenuates airway reflexes and relaxes bronchial smooth muscle by inhibiting intracellular calcium mobilization.
- • Halothane also depresses clearance of mucus from the respiratory tract (mucociliary function), promoting postoperative hypoxia and atelectasis.
- Q3. The act of handling materials in a controlled environment in which the air supply, materials, equipment, and personnel are regulated to control microbial and particulate contamination within
- acceptable levels is called:
- a. Antisepsis
- b. Aseptic processing
- c. Decontamination
- d. Sanitization
- Ans; b. Aseptic processing
Ref: Gerald E. McDonnell Antisepsis, Disinfection, and Sterilization page number 4
- Antisepsis (noun and adj., antiseptic):
- • Destruction or inhibition of microorganisms in or on living tissue, e.g., on the skin.
- • An antiseptic is a biocidal product used on the skin.
- • Aseptic (noun, asepsis): Free of, or using methods to keep free of microorganisms.
- Aseptic processing:
- • The act of handling materials in a controlled environment in which the air supply, materials, equipment, and personnel are regulated to control microbial and particulate contamination within acceptable levels.
- Decontamination:
- • Physical and/or chemical means to render a surface or item safe for handling, use, or disposal. Decontamination can refer to either chemical and biological removal or inactivation, with emphasis on biological decontamination.
- • Decontamination is generally a combination of cleaning and disinfection or sterilization.
- Sanitization:
- • The removal or inactivation of microorganisms that pose a threat to public health.
-
156
- 1. Which of the following opioid is the most potent one?
- a. Methadone
- b. Heroin
- c. Morphine
- d. All are equally potent
- Ans: B
- OPIOIDS - Overview
- • Narcotics or opioid drugs include agents used medically as analgesics (e.g., morphine) as well as drugs used illegally (e.g., heroin).
- • Compared to medically used opioids such as morphine and methadone, illegal opioids such as heroin are more potent, cross the blood–brain barrier more quickly, have a faster onset of action, and have more euphoric action.
- • In contrast to barbiturate withdrawal, which may be fatal, death from withdrawal of opioids is rare unless a serious physical illness is present.
- 2. A female came to gynae opd with complains of milky discharge from both breasts for last 1 month. She needs to use 2-3 pads a day under her upper inner ware and is frustrated with her symptoms. On lab investigation, prolactinemia was found. All of the following may be the cause for her condition except?
- a. Prolactin
- b. Estrogen
- c. TRH
- d. Dopamine antagonist
- Ans: Increased prolactin is the cause behind the patient’s condition which can be caused by options b, c and d.
- Option A mentions prolactin itself, it would be the answer if the question asked, “Which of the following increase the level of Prolactin?” But the answer to question asked above is none as all of the options provided can cause given symptoms to the patient.
- To announce the winners, this question was not taken into account.
- Ref, BRS Physiology, 6th edition, page no. 236
- Factors that Increase Prolactin Secretion
- Estrogen (pregnancy)
- Breast-feeding
- Sleep
- Stress
- TRH
- Dopamine antagonists
- Factors that Decrease Prolactin Secretion
- Dopamine
- Bromocriptine (dopamine agonist)
- Somatostatin
- Prolactin (by negative feedback)
- 3. What is incorrect among the following options?
- a. You cannot practice as a specialist if you fail NMC exam after residency.
- b. You can practice in any private clinic as a medical officer during your residency.
- c. A medical practitioner should not practice beyond the scope of her/his Speciality.
- d. Only medical practitioners on the Specialist Register are recognized as specialist, and can use the title of “specialist”.
- Ans, B, You can practice in any private clinic as a medical officer during your residency.
- Ref, Code of Ethics and Professional Conduct-2017, Nepal Medical Council
- Specialist title
- • Only medical practitioners on the Specialist Register are recognized as specialist, and can use the title of “specialist”. Medical practitioners who are not on the specialist register cannot claim to identify themselves as specialists.
- • A nonspecialist is not allowed to set up a dedicated specialist practice and publicize as such. However, providing a generalist care or a lifesaving treatment in a community setting or when delegated by Government is a duty of every registered medical practitioner.
- • A medical practitioner should not practice beyond the scope of her/his speciality.
- • If there is a doubt about the specialist or the training of the specialist medical practitioner, the question of speciality will be judged by the ethical committee of the council which will involve peer group and the respective professional society to make a final decision about the appropriate action.
- • Involvement of a medical practitioner during her/his residency, fellowship (MD, MS, DM, MCh), in private practice or practice at any other hospital not attached to his primary program is highly unethical and will be liable for disciplinary action.
- • Medical practitioners who have completed a speciality training program and academic studies but have failed the specialist exam of the Nepal Medical Council should not be allowed to work as a specialist.
- 4. Which of the following glands is apocrine gland?
- a. Sweat gland
- b. Mammary gland
- c. Sebaceous gland
- d. Parotid gland
- Ans: B, Mammary gland
- Mechanisms of secretion
- • In merocrine glands (e.g., parotid gland), the secretory cells release their contents by exocytosis.
- • In apocrine glands (e.g., lactating mammary gland), part of the apical cytoplasm of the secretory cell is released along with the contents.
- In holocrine glands (e.g., sebaceous gland), the entire secretory cell along with its contents is released.
-
157
- 1. False about Brown- Séquard syndrome is:
- a. The lesion to fasciculus gracilus or fasciculus cuneatus leads to ipsilateral loss of discriminative touch, vibration, and proprioception below the level of the lesion.
- b. The loss of the spinothalamic tract leads to contra¬lateral loss of pain and temperature sensation beginning one or two segments below the lesion.
- c. Damage to the cor¬ticospinal tract produces weakness and spasticity in certain muscle groups on the same side of the body.
- d. The loss of the spinocerebellar tract leads to ipsilateral loss of vibration and discriminative touch sensation beginning one or two segments below the lesion.
- Ans: d
- Ref: Ganong Review of Medical Physiology, Lange; 25th Edition, Page No: 172
- Brown-Séquard Syndrome
- A functional hemisection of the spinal cord causes a char¬acteristic and easily recognized clinical picture that reflects damage to ascending sensory (dorsal column pathway, ventrolateral spinothalamic tract) and descending motor (corticospinal tract) pathways, which is called the Brown- Séquard syndrome.
- The lesion to fasciculus gracilus or fasciculus cuneatus leads to ipsilateral loss of discriminative touch, vibration, and proprioception below the level of the lesion.
- The loss of the spinothalamic tract leads to contra¬lateral loss of pain and temperature sensation beginning one or two segments below the lesion.
- Damage to the cor¬ticospinal tract produces weakness and spasticity in certain muscle groups on the same side of the body.
- Although a precise spinal hemisection is rare, the syndrome is fairly common because it can be caused by a spinal cord tumor, spinal cord trauma, degenerative disk disease, and ischemia.
- 2. Long acting sulfonamide is:
- a. Sulfadiazine
- b. Sulfamethoxazole
- c. Sulfamethopyrazine
- d. Silver sulfadiazine
- Ans: c
- Ref: KDT Pharmacology 7th Edition, Page No: 704
- Sulfonamides that are still of clinical interest are:
- Short acting (4–8 hr): Sulfadiazine
- Intermediate acting (8–12 hr): Sulfamethoxazole
- Long acting (~7 days): Sulfadoxine, Sulfamethopyrazine
- Special purpose sulfonamides: Sulfacetamide sod., Mafenide, Silver sulfadiazine, Sulfasalazine
- 3. True about cluster sampling all except:
- a. Sample size same as simple random
- b. It is two stage sampling
- c. Cheaper than other methods
- d. It is a method for rapid assessment
- Ans: a
- Ref: Mahajan Biostatistics 8th Edition, page No: 125
- Cluster Sampling
- A cluster is a randomly selected group. This method is used when units of population are natural groups or clusters such as villages, wards, blocks, slums of a town, factories, work-shops or children of a school, etc.
- From the chosen clusters, 30 in number, the entire population is surveyed. Cluster sampling gives a higher standard error but the data collection in this method is simpler and involves less time and cost than in other sampling techniques.
- 4. Which of the following is not permeable through Blood-brain barrier?
- a. Water
- b. Co2
- c. Protein
- d. lipid-soluble free forms of steroid hormones
- Ans: C
- Ref: Ganong Review of Medical Physiology, Lange; 25th Edition, Page No: 604-605
- Blood-brain barrier
- • The tight junctions between capillary endothelial cells in the brain and between the epithelial cells in the choroid plexus effectively prevent proteins from entering the brain in adults and slow the penetration of some smaller molecules as well. An example is the slow penetration of urea.
- • This uniquely limited exchange of substances into the brain is referred to as the blood-brain barrier, a term most commonly used to encompass this barrier overall and more specifically the barrier in the choroid epithelium between blood and CSF.
- • Passive diffusion across the tight cerebral capillaries is very limited, and little vesicular transport takes place. However, there are numerous carrier-mediated and active transport systems in the cerebral capillaries. These move substances out of as well as into the brain, though movement out of the brain is generally freer than movement into it.
- PENETRATION OF SUBSTANCES INTO THE BRAIN
- • Water, CO2, and 02 penetrate the brain with ease, as do the lipid-soluble free forms of steroid hormones, whereas their protein-bound forms and, in general; all proteins and polypeptides do not.
-
158
- 1. A 54-year-old man with no significant medical history comes to the emergency department complaining of constipation, rectal pain during defecation, and bright red blood on the toilet paper. He is found to have an external hemorrhoid. Which of the following best describes the characteristics of external hemorrhoids?
- a. They are associated with increased risk of adenocarcinoma.
- b. They arise from the endoderm during embryonic development.
- c. They arise from the mesoderm during embryonic development.
- d. They occur inferior to the pectinate line.
- Ans: d
- Ref: USMLE Step 1 First Aid, Page No: 360
- Hemorrhoids are varicosities, or swelling of veins, in the anal canal. They are differentiated into internal or external hemorrhoids depending on their location relative to the pectinate line.
- The pectinate line is the region of the mucosa covering the anal sphincters that is formed where hindgut meets ectoderm. This line divides the anal canal into superior and inferior portions.
- External hemorrhoids occur distal to the pectinate line and arise from ectoderm. They are painful because the area receives somatic innervation by the inferior anal nerves, which are branches of the pudendal nerve. External hemorrhoids are associated with squamous cell carcinoma.
- Internal hemorrhoids arise from endoderm and are associated with adenocarcinoma. These are not painful because they receive visceral innervation from the inferior hypogastric plexus.
- Internal hemorrhoids can be associated with portal hypertension because the venous drainage of the superior rectal veins leads to the inferior mesenteric vein, which drains into the portal system. Symptoms of internal hemorrhoids include rectal mass or prolapse, itching, bright red blood on stool or toilet paper; and fecal incontinence.
- 2. Highest mean and lowest mode indicates:
- a. Positive skewing
- b. Negative skewing
- c. Normal distribution
- d. Symmetrical
- Ans: a
- Ref: USMLE Step 1 First Aid, Page No: 257
- Normal distribution
- • Gaussian, also called bell-shaped.
- • Mean = median = mode.
- Positive skew
- • Typically, mean > median > mode. Asymmetry with longer tail on right.
- Negative skew
- • Typically, mean < median < mode. Asymmetry with longer tail on left.
- 3. What is the maximum duration for which glycogen stored in muscle can sustain muscle contraction?
- a. 1 – 2 seconds
- b. 5 – 8 seconds
- c. 1 minute
- d. 2 – 4 hours
- Ans: c
- Ref: Guyton and Hall Textbook of Medical Physiology, Elsevier; 13th Edition, Page No: 82
- • The concentration of ATP in the muscle fiber, about 4 millimolar, is sufficient to maintain full contraction for only 1 to 2 seconds at most.
- • The first source of energy that is used to reconstitute the ATP is the substance hosphocreatine, which carries a high-energy phosphate bond similar to the bonds of ATP. The combined energy of both the stored ATP and the phosphocreatine in the muscle is capable of causing maximal muscle contraction for only 5 to 8 seconds.
- • The second important source of energy, which is used to reconstitute both ATP and phosphocreatine, is “glycolysis” of glycogen previously stored in the muscle cells. The importance of this glycolysis mechanism is twofold. First, the glycolytic reactions can occur even in the absence of oxygen, so muscle contraction can be sustained for many seconds and sometimes up to more than a minute, even when oxygen delivery from the blood is not available. Second, the rate of formation of ATP by the glycolytic process is about 2.5 times as rapid as ATP formation in response to cellular foodstuffs reacting with oxygen. However, so many end products of glycolysis accumulate in the muscle cells that glycolysis also loses its capability to sustain maximum muscle contraction after about 1 minute.
- • The third and final source of energy is oxidative metabolism, which means combining oxygen with the end products of glycolysis and with various other cellular foodstuffs to liberate ATP. More than 95 percent of all energy used by the muscles for sustained, long-term contraction is derived from oxidative metabolism. For extremely long-term maximal muscle activity—over a period of many hours—by far the greatest proportion of energy comes from fats, but for periods of 2 to 4 hours, as much as one half of the energy can come from stored carbohydrates.
- 4. What is correct about Rose’s position?
- a. Patient is placed prone
- b. Rubber ring is placed under the face
- c. Head should be hyperextended by placing a pillow under the shoulders.
- d. None
- Ans: d
- Ref: PL Dhingra and Shruti Dhingra ENT and HNS 7th Edition, Page No: 487
- In Rose’s position for tonsillectomy, the patient lies supine with head extended by placing a pillow under the shoulders. A rubber ring is placed under the head to stabilize it.
- Hyperextension should always be avoided.
-
159
Answers and Winners of QUIZ for CEE MD/MS Based Model test of Today noon (11 AM, Asar 6, Saturday)
- 1. Neonatal seizure may be seen due to deficiency of:
- a. Riboflavin
- b. Pantothenic acid
- c. Pyridoxine
- d. Thiamine
- Ans: c
- Ref: Treatable neonatal epilepsy by Robert Surtees et al; Arch Dis Child. 2007 Aug; 92(8): 659–661.
The seizures and EEG abnormalities respond promptly (within minutes) to 100 mg of intravenous pyridoxine. However, in about 20% of infants with pyridoxine‐dependent epilepsy the first dose of pyridoxine can also cause cerebral depression, which is more likely if the infant is receiving anticonvulsant drugs.
- Treatable metabolic causes of early onset epilepsy
- Vitamin‐responsive epilepsies
- • Pyridoxine‐dependent epilepsy
- • Pyridox(am)ine phosphate oxidase deficiency (pyridoxal phosphate‐ responsive epilepsy)
- • Folinic acid‐responsive seizures
- • Biotinidase deficiency
- 2. Angular conjunctivitis is caused by:
- a. Haemophilus
- b. Adenovirus
- c. Moraxella
- d. Bacteroides
- Ans: c
- Ref: AK Khurana Ophthalmology 6th Edition, Page No: 66
- Angular bacterial conjunctivitis
- • It is a type of chronic conjunctivitis characterized by mild grade inflammation confined to the conjunctiva and lid margins near the angles (hence the name) associated with maceration of the surrounding skin.
- • Causative organisms. Moraxella Axenfeld (MA) is the commonest causative organism. MA bacilli are placed end to end, so the disease is also called ‘diplobacillary conjunctivitis’. Rarely, staphylococci may also cause angular conjunctivitis.
- 3. Forgarty’s catheter is used for:
- a. Embolectomy
- b. Angiography
- c. Drainage
- d. Suprapubic catheterization
- Ans: a
- Ref: Bailey and Love’s Short Practice of Surgery 27th Edition, Page No: 955
- Embolectomy
- Local or general anaesthesia may be used. The artery (usually the femoral), bulging with clot, is exposed and held in silastic vessel loops. Through a transverse incision the clot begins to extrude and is removed, together with the embolus, with the help of a Fogarty balloon catheter. The catheter, with its balloon tip, is introduced both proximally and distally until it is deemed to have passed the limit of the clot. The balloon is inflated and the catheter withdrawn slowly,together with any obstructing material. The procedure is repeated until bleeding occurs. An angiogram may be performed in the operating theatre at the end of the procedure to ensure that flow to the distal leg has been restored. Postoperatively, heparin therapy is continued until long-term anticoagulation with warfarin is established to reduce the chance of further embolism.
- Malecot’s Catheter
- • Self retaining catheter with an umbrella or flower at the tip
- • Used for draining urine from the urinary bladder, when the urethra is damaged as in suprapubic catheterization; drainage of
- • urine from the kidney by percutaneous nephrostomy
- • Used for drainage of fluid collections, e.g. an abdominal abscess
- • Used as intercostal drain
Nelaton Catheter: Nelaton catheters can be used for one-time emptying of the bladder, for instance during/after surgery or to determine the amount of urine in the bladder.
- 4. Inverse ratio ventilation refers to?
- a. Prone-position ventilation
- b. Inspiratory time is longer than expiratory time
- c. Expiratory time is longer than inspiratory time
- d. Ventilating at extremely high respiratory rates
- Ans: b
- Ref: Harrison’s Principles of Internal Medicine; 19th Edition, Page No: 1742
- In inverse ratio ventilation technique, inspiratory time is lengthened so that it is longer than expiratory time. With diminished time to exhale, dynamic hyperinflation leads to increased end expiratory pressure, similar to ventilator-prescribed PEEP.
- 5. Liver recipients with what MELD scores experienced higher post transplantation mortality rates?
- a. < 15
- b. < 20
- c. < 25
- d. < 30
- Ans: a
- Ref: Harrison’s Principles of Internal Medicine; 19th Edition, Page No: 2070
- Liver recipients with MELD scores <15 experienced higher post transplantation mortality rates. The MELD scale is continuous, with 34 levels ranging between 6 and 40. Donor organs usually do not become available unless the MELD score exceeds 20.
- Winners:
- Fastest correct answer respondnets:
- - Dr Arvind Shah
- - Dr Harish Bogati
- Lucky Draw winners:
- - Dr Ashish Subba
- - Dr Prerana Mallika
Winners are requested to send email to onlinepgnepal@gmail.com to claim Free access to Today's CEE MD/MS based Online Model Test (11 AM, Asar 6, Saturday)
-
160
Answers and Winners of SURGERY QUIZ
- Surgery QUIZ
- 1. A patient in burn ward has complete involvement of bilateral lower limbs and right upper limb. What amount of lactated Ringer’s fluid should be given to him according to latest consensus formula?
- a. 6 mL/kg per % burn of Lactated Ringer’s
- b. 4 mL/kg per % burn of Lactated Ringer’s
- c. 3 mL/kg per % burn of Lactated Ringer’s
- d. 2 mL/kg per % burn of Lactated Ringer’s
- Ans: d
- Ref: Schwartz’s Principles of Surgery, 11th edition, page 254
- • A myriad of formulas exist for calculating fluid needs during burn resuscitation, suggesting that no one formula benefits all patients.
- • The most commonly used formula, the Parkland or Baxter formula, consists of 3 to 4 mL/kg per % burn of Lactated Ringer’s, of which half is given during the first 8 hours
- • after burn and the remaining half is given over the subsequent 16 hours.
- • The most recent American Burn Association consensus formula recommends 2 mL/kg per % burn of Lactated Ringers given the tendency toward excessive fluid administration with the traditional formulas.
- 2. A child was found to have atrial septal closure. Closure of the defect by what age has a better chance of him achieving a normal right ventricular function?
- a. < 1 year
- b. < 5 years
- c. < 7 years
- d. < 10 years
- Ans: d
- Ref: Schwartz’s Principles of Surgery, 11th edition, page 753
- The ability of the RV to recover normal function is related to the duration of chronic overload because those undergoing ASD closure before age 10 years have a better likelihood of achieving normal RV volumes and function in the postoperative period.
- 3. The prostatic urethra extends from?
- a. Bladder neck to superficial perineal membrane
- b. Bladder neck to verumontanum
- c. Verumontanum to deep perineal membrane
- d. Deep to superficial perineal membrane
- Ans: b
- Ref: Bailey and Love’s Short Practice of Surgery 27th Edition, Page No: 1477
- • The male urethra is a tubular structure extending from the bladder neck to the external urinary meatus at the tip of the glans penis. It has four components, which are named (from proximal to distal) the prostatic, membranous, bulbar and penile urethra.
- • The prostatic urethra extends from the bladder neck to the verumontanum and is compressed on either side by the lateral lobes of the prostate, giving it a slit-like configuration.
- • The verumontanum is a small hillock of tissue indented at its crown by a pit called the utriculus masculinus, which marks the proximal extent of the external urethral sphincter and is an important landmark for urologists performing transurethral resection of the prostate.
- • The membranous urethra lies just distal to the verumontanum and is located where the urethra penetrates the pelvic floor and it is the usual site of urethral rupture at the time of a pelvic fracture.
- 4. A 60-year-old woman runs her car off the road near Chakrapath and it hits the traffic station. She presents to the emergency department of TUTH with severe anterior chest pain and a blood pressure of 110/80 mm Hg. A chest x-ray shows a questionably widened mediastinum. The next step in management should be which of the following?
- a. Transthoracic echocardiogram
- b. Pericardiocentesis
- c. Aortogram
- d. Central venous access line
- Ans: c
- The most definitive test for aortic injury is the aortogram, even though only 20–30% of patients with widened mediastinum demonstrate it. A transthoracic echocardiogram does not image the aorta wall; however, a transesophageal echocardiogram may have more value in experienced hands.
- Winners:
- Fastest correct answer respondents:
- - Dr Abinash Yadav
- - Dr Subash Lamichhane
- Lucky draw winners among respondents:
- - Dr Naresh Gharti
- - Dr Pawan Rai
Winners are requested to send email to onlinepgnepal@gmail.com to claim Free Access to Asar 8 Subjectwise [Surgery] Based Model test [07:00 PM, Monday].
-
161
Answers to yesterday's Questions (BBD)
- 1.Which of the following is an autosomal recessive disorder?
- a. Duchenne muscular dystrophy
- b. Friedreich ataxia
- c. Wiskott Aldrich syndrome
- d. Lesch Nyhan syndrome
Ans: b
- Ref: Robbins & Cotran, Pathologic basis of disease, South Asia edition, Vol 1, p141, 142.
- Image
- Duchenne muscular dystrophy, Wiskott Aldrich syndrome and Lesch Nyhan syndrome are X-linked recessive disorders.
- Friedreich ataxia is an autosomal recessive disorder.
- Other autosomal recessive disorders include congenital adrenal hyperplasia, alkaptonuria, alpha 1 antitrypsin deficiency, phenylketonuria and Wilson disease.
- 2. Radiological appearance of thimble' bladder is seen in the following condition:
- a. Cystitis cystica
- b. Chronic tuberculous cystitis
- c. Neurogenic bladder
- d. Acute tuberculous cystitis
Correct Ans: b
- Explanation
- Early tuberculosis of the bladder commences around the ureteric orifice or trigone, the
- earliest evidence being pallor of the mucosa due to submucosal oedema. Subsequently,
- tubercles may be seen and, in long standing cases, there is marked fibrosis and the capacity
- of the bladder is greatly reduced giving the radiological appearance of thimble bladder'.
- Ref: Bailey & Love's Short Practice of Surgery 26E edited by
- Norman Williams, Christopher Bulstrode, P Ronan O'Connell,
- 2013, Page 1327,1328
- 3. The standard normal distribution:
- a. Is skewed to the left
- b. Has mean = 1.0
- c. Has standard deviation = 0.0
- d. Has variance = 1.0
Ans : d. Has variance = 1.0
- • A standard normal deviation is not skewed to any side. It is symmetrical
- • Standard deviation = 1.0
- • Variance = SD^2 = 10
-
162
- Ans: DAD (to yesterday’s questions)
- 1. Which is a true statement regarding inhibin?
- a. Increases the secretion of GnRH.
- b. Molecular weight is between 5000 and 10,000.
- c. It is secreted by hypothalamus.
- d. It is a glycoprotein.
- Ans: D, It is a glycoprotein.
- Ref: Textbook of Medical Physiology, Guyton and Hall, 11th edition, page 1007.
- * Inhibin is secreted by the sertoli cells in males and by the granulosa cells in females.
- * It inhibits the secretion of Gonadotrophin Releasing Hormone.
- * It is a glycoprotein with molecular weight between 10,000 and 30,000.
- * It has potent inhibitory feedback effect on the anterior pituitary gland which in turn provides negative feedback mechanism for the control of spermatogenesis.
- 2. Blaschko’s lines represent?
- a. Lines of development
- b. Dermatomes
- c. Lines along blood vessels
- d. Lines along lymphatics
- Ans: A, Lines of development
- BLASCHKO’S LINES
- * Blaschko’s lines are invisible developmental lines of the skin
- * They are thought to represent the pathways of epithelial migration and proliferation in the foetus
- * These were first demonstrated by German dermatologist Alfred Blaschko in 1901.
- * They do not correspond to the path of vessels / nerves / lymphatics
- * Many skin lesions follow Blaschko’s lines. eg: Incontinentia pigmenti, Linear lichen planus, Lichen striatus Naevus achromicus
- 3. Most common cause of premature death in schizophrenia is?
- a. Drug toxicity
- b. Nosocomial infection
- c. Homicide
- d. Suicide
- Ans: D, Suicide
- PREMATURE DEATH IN SCHIZOPHRENIA
- * Life expectancy of patients with schizophrenia is decreased by approximately 15 to 25 years.
- * Suicide is the most important cause of premature death in patients with schizophrenia.
- * There is a 20 fold increased risk of suicide compared to the general population.
- * Suicide attempts may occur without warning or expression of intent.
- * Cancer is the second most frequent cause of mortality in schizophrenics.
- * The rates of metabolic syndrome and cardiovascular disorders are also increased.
-
163
- ANSWERS TO DAILY QUESTIONS OF YESTERDAY (JESTHA 31):
- ANSWER SEQUENCE (BBB)
- Q1.Rita a 26 years old primi with history of rheumatic heart disease came to your clinic for antenatal checkup. Which of the following is the commonest heart lesion met during pregnancy?
- a. Mitral regurgitation
- b. Mitral stenosis
- c. Tricuspid regurgitation
- d. Aortic stenosis
- Ans, b. Mitral stenosis
Ref: Dc Dutta Obstetrics 8th edition page no 319,323,324
- HEART DISEASE IN PREGNANCY
- INCIDENCE AND TYPES:
- • The incidence of cardiac lesion is less than 1% amongst hospital deliveries.
- • The commonest cardiac lesion is of rheumatic origin followed by the congenital ones.
- • The ratio between the two has fallen over the past two decades from 10: 1 to about 3: 1 or even 1: 1 in advanced countries.
- • Adequate treatment of rheumatic fever by appropriate antibiotics to cope with the group A β-hemolytic streptococcal infection, pari passu with the advancement in cardiac surgery to rectify the congenital heart lesions, are responsible for the change in the profile.
- • Rheumatic valvular lesion predominantly includes mitral stenosis (80%).
- • Predominant congenital lesions include patent ductus arteriosus, atrial or ventricular septal defect, pulmonary stenosis, coarctation of aorta and Fallot’s tetralogy.
- • Rare causes are hypertensive, thyrotoxic, syphilitic or coronary cardiac diseases.
- MITRAL STENOSIS:
- • Mitral stenosis is the commonest heart lesion met during pregnancy.
- • Normal mitral valve area ranges between 4 and 6 cm2.
- • Symptoms usually appear when stenosis narrows this to less than 2.5 cm2.
- • Women with mitral valve area ≤1 cm2, have the high rate of pulmonary edema (55%) and arrhythmia (33%).
- • In asymptomatic cases, the mortality is < 1% but once it is significantly symptomatic, mortality ranges between 5% and 15%.
- • Diagnosis and management has been mentioned earlier.
- • During labor continuous epidural analgesia is ideal and intravenous fluid overload is to be avoided.
- AORTIC STENOSIS:
- • Most cases of aortic stenosis are congenital, some are rheumatic in origin.
- • Normal aortic valve area is 3–4 cm2.
- • When it is reduced to less than or equal to 1 cm2, stenosis is significant.
- • Maternal mortality of significant aortic stenosis is about 15–20% with perinatal loss of about 30%.
- • Epidural anesthesia is contraindicated.
- • During labor, fluid therapy (125–150 ml/h) should not be restricted.
- • Left ventricular after load is high and the pregnant patient is sensitive to hemorrhage.
- Q2. Halothane is considered a potent bronchodilator, as it oft en reverses asthma-induced bronchospasm. This action is:
- a. Inhibited by adrenergic blocking agent
- b. Not inhibited by adrenergic blocking agent
- c. By increasing intracellular calcium mobilization
- d. By inhibiting relaxation of bronchial smooth muscle
- Ans; b. Not inhibited by adrenergic blocking agent
- Ref: Morgan and Mikhail’s clinical anesthesiology 5th edition, page number 167
- RESPIRATORY ACTIONS OF HALOTHANE:
- • Halothane typically causes rapid, shallow breathing.
- • The increased respiratory rate is not enough to counter the decreased tidal volume, so alveolar ventilation drops, and resting PaCO 2 is elevated.
- • Apneic threshold, the highest Pa co 2 at which a patient remains apneic, also rises because the difference between it and resting Pa co 2 is not altered by general anesthesia. Similarly, halothane limits the increase in minute ventilation that normally accompanies a rise in PaCO2.
- • Halothane’s ventilatory effects are probably due to central (medullary depression) and peripheral (intercostal muscle dysfunction) mechanisms.
- • These changes are exaggerated by preexisting lung disease and attenuated by surgical stimulation.
- • The increase in Pa co 2 and the decrease in intra-thoracic pressure that accompany spontaneous ventilation with halothane partially reverse the depression in cardiac output, arterial blood pressure, and heart rate described above.
- • Hypoxic drive is severely depressed by even low concentrations of halothane (0.1 MAC).
- • Halothane is considered a potent bronchodilator, as it often reverses asthma-induced bronchospasm.
- • This action is not inhibited by adrenergic blocking agents.
- • Halothane attenuates airway reflexes and relaxes bronchial smooth muscle by inhibiting intracellular calcium mobilization.
- • Halothane also depresses clearance of mucus from the respiratory tract (mucociliary function), promoting postoperative hypoxia and atelectasis.
- Q3. The act of handling materials in a controlled environment in which the air supply, materials, equipment, and personnel are regulated to control microbial and particulate contamination within
- acceptable levels is called:
- a. Antisepsis
- b. Aseptic processing
- c. Decontamination
- d. Sanitization
- Ans; b. Aseptic processing
Ref: Gerald E. McDonnell Antisepsis, Disinfection, and Sterilization page number 4
- Antisepsis (noun and adj., antiseptic):
- • Destruction or inhibition of microorganisms in or on living tissue, e.g., on the skin.
- • An antiseptic is a biocidal product used on the skin.
- • Aseptic (noun, asepsis): Free of, or using methods to keep free of microorganisms.
- Aseptic processing:
- • The act of handling materials in a controlled environment in which the air supply, materials, equipment, and personnel are regulated to control microbial and particulate contamination within acceptable levels.
- Decontamination:
- • Physical and/or chemical means to render a surface or item safe for handling, use, or disposal. Decontamination can refer to either chemical and biological removal or inactivation, with emphasis on biological decontamination.
- • Decontamination is generally a combination of cleaning and disinfection or sterilization.
- Sanitization:
- • The removal or inactivation of microorganisms that pose a threat to public health.
-
164
- Results of last quiz are out for free access on upcoming CEE based MDMS exam at 7 pm on 7th Asar, Sunday.
- The winners for free access to the upcoming CEE based MDMS (at 7 pm on 7th Asar, Sunday) are:
- Fastest correct responders:
- Dr. Subash Lamichhane
- Dr. Dinesh Neupane
- Luckiest correct responders among all the candidates:
- Dr. Sanjeet Jaiswal
- Dr. Aakanshya Shrestha
Congratulations to the winners. Please contact us in viber, messenger or email for free tokens. Thank you.
- 1. Which of the following opioid is the most potent one?
- a. Methadone
- b. Heroin
- c. Morphine
- d. All are equally potent
- Ans: B
- OPIOIDS - Overview
- • Narcotics or opioid drugs include agents used medically as analgesics (e.g., morphine) as well as drugs used illegally (e.g., heroin).
- • Compared to medically used opioids such as morphine and methadone, illegal opioids such as heroin are more potent, cross the blood–brain barrier more quickly, have a faster onset of action, and have more euphoric action.
- • In contrast to barbiturate withdrawal, which may be fatal, death from withdrawal of opioids is rare unless a serious physical illness is present.
- 2. A female came to gynae opd with complains of milky discharge from both breasts for last 1 month. She needs to use 2-3 pads a day under her upper inner ware and is frustrated with her symptoms. On lab investigation, prolactinemia was found. All of the following may be the cause for her condition except?
- a. Prolactin
- b. Estrogen
- c. TRH
- d. Dopamine antagonist
- Ans: Increased prolactin is the cause behind the patient’s condition which can be caused by options b, c and d.
- Option A mentions prolactin itself, it would be the answer if the question asked, “Which of the following increase the level of Prolactin?” But the answer to question asked above is none as all of the options provided can cause given symptoms to the patient.
- To announce the winners, this question was not taken into account.
- Ref, BRS Physiology, 6th edition, page no. 236
- Factors that Increase Prolactin Secretion
- Estrogen (pregnancy)
- Breast-feeding
- Sleep
- Stress
- TRH
- Dopamine antagonists
- Factors that Decrease Prolactin Secretion
- Dopamine
- Bromocriptine (dopamine agonist)
- Somatostatin
- Prolactin (by negative feedback)
- 3. What is incorrect among the following options?
- a. You cannot practice as a specialist if you fail NMC exam after residency.
- b. You can practice in any private clinic as a medical officer during your residency.
- c. A medical practitioner should not practice beyond the scope of her/his Speciality.
- d. Only medical practitioners on the Specialist Register are recognized as specialist, and can use the title of “specialist”.
- Ans, B, You can practice in any private clinic as a medical officer during your residency.
- Ref, Code of Ethics and Professional Conduct-2017, Nepal Medical Council
- Specialist title
- • Only medical practitioners on the Specialist Register are recognized as specialist, and can use the title of “specialist”. Medical practitioners who are not on the specialist register cannot claim to identify themselves as specialists.
- • A nonspecialist is not allowed to set up a dedicated specialist practice and publicize as such. However, providing a generalist care or a lifesaving treatment in a community setting or when delegated by Government is a duty of every registered medical practitioner.
- • A medical practitioner should not practice beyond the scope of her/his speciality.
- • If there is a doubt about the specialist or the training of the specialist medical practitioner, the question of speciality will be judged by the ethical committee of the council which will involve peer group and the respective professional society to make a final decision about the appropriate action.
- • Involvement of a medical practitioner during her/his residency, fellowship (MD, MS, DM, MCh), in private practice or practice at any other hospital not attached to his primary program is highly unethical and will be liable for disciplinary action.
- • Medical practitioners who have completed a speciality training program and academic studies but have failed the specialist exam of the Nepal Medical Council should not be allowed to work as a specialist.
- 4. Which of the following glands is apocrine gland?
- a. Sweat gland
- b. Mammary gland
- c. Sebaceous gland
- d. Parotid gland
- Ans: B, Mammary gland
- Mechanisms of secretion
- • In merocrine glands (e.g., parotid gland), the secretory cells release their contents by exocytosis.
- • In apocrine glands (e.g., lactating mammary gland), part of the apical cytoplasm of the secretory cell is released along with the contents.
- In holocrine glands (e.g., sebaceous gland), the entire secretory cell along with its contents is released.
-
165
- Answers to daily questions (Ashar 3):
- Q1. Late postoperative complications are those arising;
- a.After 24 hours of surgery
- b. After 72 hours of surgery
- c. After one week of surgery only
- d. Only after 15 days of surgerys
- Ans: b, After 72 hours of surgery
- Ref: Short Practice of Surgery, Bailey and Love, 27th edition, page no.292
- Classification of postoperative complications
- There are three common approaches for the classification of postoperative complications of surgery:
- • Linked to time after surgery:
- o immediate (within 6 h of procedure);
- o early (6–72 h);
- o late (>72 h).
- • Generic and surgery specific.
- • Clavian-Dindo: this system relates to surgical complications only and is used to objectively and reproducibly measure the impact of the surgical complication on the outcome of the procedure. It is included here for completeness and will be discussed no further.
- Q2. During reconstruction of an amputed limb, which of the following is done first?
- a. Arterial repair
- b. Venous repair
- c. Nerve anastomoses
- d. Fixation of the bone
- Ans: d, Fixation of the bone
- Ref: Campbell’s Operativ3 Orthopaedics 11th edition, page no.3718-24
- During reconstruction of an amputated limb, the order of repair is as follows;
- 1. Shorten and internally fix bone
- 2. Repair extensor tendons
- 3. Repair flexor tendons (2 or 3 may be reversed, or flexor tendon repair may be delayed)
- 4. Repair arteries
- 5. Repair nerves
- 6. Repair veins
- Q3. At a distance of 1 meter, intensity of normal conversation is:
- a. 30 dB
- b. 50 dB
- c. 90 dB
- d. 60 dB
- Ans: d, 60 dB
- Ref: Diseases of Ear Nose and Throat, PL Dhingra, 6th edition, page no.19
- Explanation:
- At a distance of 1 m, intensity of
- • Whisper = 30 dB
- • Normal conversation = 60 dB
- • Shout = 90 dB
- • Discomfort of the ear = 120 dB
- • Pain in the ear = 130 dB
-
166
- Answers to daily questions (Asar 4,2077) :
- 1. A 28 years old man has anterior lenticonus and now ESRD. His maternal uncle also died of similar illness. Diagnosis is:
- a.ARPKD
- b.ADPKD
- c.Oxalosis
- d.Alport’s syndrome
- Ans,d.Alport's syndrome
- Ref: Harrison’s 19th /e.p2513
- •The given clinical picture is typical of Alport's syndrome, which is characterized by triad of hereditary nephritis (ESRD), sensorineural deafness and ocular abnormalities (anterior lenticonus).
- •Primary treatment is control of systemic hypertension and use of ACE inhibitors to slow renal disease progression.
- 2.Lambda - Panda sign is typically seen in:
- a.Sarcoidosis
- b.Tuberculosis
- c.Histoplasmosis
- d.Leishmaniasis
- Ans,a.Sarcoidosis
- Ref: Harrison 17th/e p. 2149
- •Lambda sign and Panda sign on Gallium scan are typically described for sarcoidosis.
- •Active pulmonary and/or mediastinal sarcoidosis is gallium avid and a positive gallium scan can support the diagnosis of sarcoidosis. Typical patterns of uptake have been described as ‘panda’ and ‘lambda’signs.
- •Lambda sign → Formed from increased uptake in bilateral hilar and right paratracheal nodes.
- •Panda sign → Formed from increased uptake in the parotids and lacrimal glands.
- •A Lambda sign in combination with a so-called Panda sign (Lambda-Panda Sign) is a highly specific pattern for sarcoidosis.
- •The degree of uptake typically depends on the activity of disease and gallium scan is positive only in the setting of active parenchymal disease and negative in remission.
- 3.Luxatio erecta is another name for:
- a.Inferior shoulder dislocation
- b.Anterior shoulder dislocation
- c.Posterior shoulder dislocation
- d.Superior shoulder dislocation
- Ans,a. Inferior shoulder dislocation
- Ref: Maheshwari 5th/e.p. 367
- Inferior dislocation (Luxation Erecta)
- • Here the head of the humerus is below the glenoid cavity and the humeral shaft is pointing overhead.
- • It is due to hyperabduction injury.
- • It is rare and also called luxatio erecta because the humeral head is subluxated (dislocated inferiorly) and humerus shaft points upwards (erected).
- • Potentially serious consequences e.g., neurovascular damage is quite common.
- • It is reduced by pulling upwards in the line of abducted arm with counter traction downwards.
-
167
- Answers to Yesterday’s questions:
- Answers: BCA
- Q1.Most common complication of scrub typhus is:
- a.Meningoencephalitis
- b.Pneumonitis
- c.Myocarditis
- d.Acute renal failure
- Ans: b,Pneumonitis
- Ref:Nelson textbook of pediatrics 21st ed,pg 6548
- Scrub Typhus (Orientia tsutsugamushi)
- •Serious complications include pneumonitis in 20–35% and meningoencephalitis in approximately 10–25% of children.
- •Acute renal failure, myocarditis, and a septic shock–like syndrome occur much less often.
- •Cerebrospinal fluid examination shows a mild mononuclear pleocytosis with normal glucose levels. Chest radiographs reveal transient perihilar or peribronchial interstitial infiltrates in most children who are examined.
- Q2. Malignant pustule is caused by:
- a. Carbuncle
- b. Ulcerating melanoma
- c. Anthrax of skin
- d. CMV infection.
- Ans: c, Anthrax of skin
- Ref: Ref: Jawetz Melnick and Adelberg’s medical microbiology 27th edition page no:166
- -Cutaneous anthrax generally occurs on exposed surfaces of the arms or hands, followed in frequency by the face and neck.
- -A pruritic papule develops 1–7 days after entry of the organisms or spores through a scratch.
- -Initially it resembles an insect bite. The papule rapidly changes into a vesicle or small ring of vesicles that coalesce, and a necrotic ulcer develops.
- -This lesion is known as malignant pustule
- -The lesions typically are 1–3 cm in diameter and have a characteristic central black eschar.
- -Marked edema occurs. Lymphangitis and lymphadenopathy and systemic signs and symptoms of fever, malaise, and headache may occur. After 7–10 days the eschar is fully developed.
- -Eventually it dries, loosens, and separates; healing is by granulation and leaves a scar. It may take many weeks for the lesion to heal and the edema to subside.
- -Antibiotic therapy does not appear to change the natural progression of the disease. In as many as 20% of patients, cutaneous anthrax can lead to sepsis, the consequences of systemic infection—including meningitis—and death.
- Q3. Which of the following corneal dystrophies is autosomal recessive?
- a. Macular dystrophy
- b. Lattice dystrophy type 1
- c. Granular dystrophy
- d. Fuchs endothelial corneal dystrophy
- Ans: a, Macular dystrophy
- Ref:Kanski 6th edition,page no:290
- -Macular corneal dystrophy is autosomal recessive, whereas lattice dystrophy type 1 and granular dystrophy are autosomal dominant.
- -Fuchs endothelial corneal dystrophy can show an autosomal dominant inheritance in some families, but in the majority of cases the pattern of inheritance is yet to be elucidated.
-
168
- Answers to yesterday’s questions
- Correct answers:ADA
- Q1.Falling leaf motility is seen in
- a. Giardia lambia
- b.Trichomonas vaginalis
- c.Trypanosoma cruzi
- d.Enteromonas hominis
- Ans: a, Giardia lambia
- Ref:: Panikers textbook of medical parasitology 8th edition pg 54
- •Giardia lambia:The trophozoite is motile, with a slow oscillation about its long axis, often resembling falling leaf.
- •Trichomonas:It is motile with a rapid jerky or twitching type movement.
- Q2.Night blindness is associated with:
- a.Xeropthalmia
- b.Pathological myopia
- c.Retinitis pigmentosa
- d.All of the above
- Ans: d, all of the above
- Ref: Parson’s disease of eye,22nd edition pg 102
- Night Blindness or Nyctalopia
- •The inability to see in low light conditions occurs most frequently in retinitis pigmentosa, xerophthalmia, pathological myopia, and in rare cases it is a familial congenital affection.
- •Night blindness is to be attributed to interference with the functions of the retinal rods.
- •In xerophthalmia the symptom is a manifestation of a deficiency of fat soluble vitamin A in the diet.
- •It also occurs in diseases of the liver, especially cirrhosis, or with the use of phenothiazines, and may appear as a functional nervous disorder associated with other symptoms of neurosis or malingering.
- Q3.Pharmacovigilence has an important role in the rational use of medicines, as it provides the basis for:
- a.Assessing safety of medicines.
- b.Assessing potency of medicines
- c. Causality assessment.
- d.None of the above
- Ans:a, Assessing safety of medicines
- Ref:KD Tripathi Essentials of Medical Pharmacology 7th edition page no:83
- •Pharmacovigilance has been defined by the WHO (2002) as the ‘science and activities relating to the detection, assessment, understanding and prevention of adverse effects or any other drug related problems.’
- •The information generated by pharmacovigilance is useful in educating doctors about ADRs(Adverse Drug Reactions) and in the official regulation of drug use.
- •Its main purpose is to reduce the risk of drug-related harm to the patient.
- •It has an important role in the rational use of medicines, as it provides the basis for assessing safety of medicines.
-
169
- 1. A 16-year-old male presents to his pediatrician with complaints of malaise, fatigue, sore throat, and fever over the last several days. His vital signs are as follows: T 39.1 C, HR 82, BP 122/76, RR 14, and SpO2 99%. Physical examination is significant for splenomegaly, tonsillar exudate, and posterior auricular lymphadenopathy. Laboratory work-up shows an elevated lymphocyte count, atypical lymphocytes on a peripheral blood smear, and a positive heterophile antibody screen. Which of the following is the best management of this patient's condition?
- a. Bed rest and activity limitation
- b. Acyclovir
- c. Amoxicillin
- d. Prednisone
- Ans: a
- • This patient has infectious mononucleosis. Infectious mononucleosis is best treated with supportive treatment, including bed rest and avoidance of contact sports due to the risk of splenic rupture.
- • Infectious mononucleosis is an acute infection caused by the Epstein-Barr virus (EBV).
- • It is most common in patients 15-20 years of age and is transmitted by body fluids, particularly saliva ('kissing disease'). The heterophile antibody (Monospot) test is most commonly used to make the diagnosis.
- • An EBV-specific antibody test is also available to confirm the diagnosis; however, this test is costly with a lengthy turn-around time.
- • Complications associated with the disease include splenic rupture secondary to trauma, Guillain-Barre syndrome, encephalitis, hemolytic anemia, and maculopapular rash with ampicillin treatment (when infectious mononucleosis is mistakenly treated with antibiotics).
- 2. The most common site of angiodysplasia:
- a. Duodenum
- b. Jejunum
- c. Caecum
- d. Sigmoid colon
- Ans: c
- Ref: Sabiston Surgery 20th Edition, Page No: 1153
- Angiodysplasia
- • Angiodysplasia of the intestine, also referred to as arteriovenous malformations, are distinct from hemangiomas and true congenital arteriovenous malformations. They are thought to be acquired degenerative lesions secondary to progressive dilation of normal blood vessels within the submucosa of the intestine.
- • Angiodysplasia have an equal gender distribution and are almost uniformly found in patients older than 50 years. These lesions are notably associated with aortic stenosis and renal failure, especially in the elderly.
- • The hemorrhage tends to arise from the right side of the colon, with the cecum being the most common location, although they can occur in the rest of the colon and small bowel. Most patients present with chronic bleeding, but in up to 15%, hemorrhage may be massive. Bleeding stops spontaneously in most cases, but approximately 50% will rebleed within 5 years.
- • These lesions can be diagnosed by either colonoscopy or angiography. During colonoscopy, they appear as red stellate lesions with a surrounding rim of pale mucosa and can be treated with sclerotherapy or electrocautery.
- • Angiography demonstrates dilated, slowly emptying veins and sometimes early venous filling. If these lesions are discovered incidentally, no further therapy is indicated.
- • In acutely bleeding patients, they have been successfully treated with intra-arterial vasopressin, selective gel foam embolization, endoscopic electrocoagulation, or injection with sclerosing agents.
- • If these measures fail or bleeding recurs and the lesion has been localized, segmental resection, most commonly right colectomy, is effective.
- 3. Appendix in inguinal hernial sac is known as:
- a. Littre’s
- b. Amyand’s
- c. Maydl’s hernia
- d. Ritcher’s
- Ans: b
- Ref: SRB's Manual of Surgery; 5th Edition, Page No: 784-785
- • Gibbon’s hernia—It is hernia with hydrocoele
- • Berger’s hernia—Hernia in pouch of Douglas
- • Romberg hernia—Saddle hernia
- • Obturator hernia—Hernia through obturator foramen (canal)
- • Grynfelt’s hernia—Upper lumbar triangle hernia
- • Petit’s hernia—Lower lumbar triangle hernia
- • Femoral hernia—Hernia medial to femoral vein
- • Cloquet’s hernia—Hernia through pectineal fascia
- • Narath’s hernia—Behind femoral artery, in congenital dislo-cation of hip
- • Hesselbach’s hernia—Lateral to femoral artery
- • Serofini’s hernia—Behind femoral vessels
- • Laugier’s hernia—Through lacunar ligament
- • Teale’s hernia—In front of femoral vessels
- • Richter’s hernia—Part of circumference of bowel wall is gangrenous
- • Littre’s hernia—Hernia with Meckel’s diver ticulum as the content
- • Sliding hernia—Posterior wall of the sac is formed by colon or bladder
- • Maydl’s hernia—‘W’ hernia
- • Phantom hernia—Localised muscle bulge following muscular paralysis
- • Spigelian hernia—Hernia through spigelian fascia
- • Mery’s hernia—Perineal hernia
- • Sciatic hernia—Hernia through greater or lesser sciatic foramen
- • Amyand hernia—Appendix in inguinal hernial sac
- • Beclard’s hernia—Femoral hernia through the saphenous opening
- • Barth’s hernia—Hernia between abdominal wall and persis-tentvitellointestinal duct
- • Holthouse’s hernia—Inguinal hernia that has turned outwards into the groin
- • De Garengeot’s hernia is incarcerated appendix in femoral hernia.
- 4. A 25 year old man is brought to the ER following a motorbike accident. His eyes open in response to pain, there is no verbal response and his posture is decerebrate. What is the GCS score in this patient?
- a. 6
- b. 5
- c. 4
- d. 7
- Ans: b
- Ref: Bailey and Love’s Short Practice of Surgery 27th Edition, Page No: 325
- In this case:
- • Opens eyes in response to painful stimuli: 2
- • Makes no sounds: 1
- • Extension to painful stimuli (decerebrate response ) : 2
- Glasgow Coma Scale
- Eye Opening Response
- • Spontaneous--open with blinking at baseline 4 points
- • To verbal stimuli, command, speech 3 points
- • To pain only (not applied to face) 2 points
- • No response 1 point
- Verbal Response
- • Oriented 5 points
- • Confused conversation, but able to answer questions 4 points
- • Inappropriate words 3 points
- • Incomprehensible speech 2 points
- • No response 1 point
- Motor Response
- • Obeys commands for movement 6 points
- • Purposeful movement to painful stimulus 5 points
- • Withdraws in response to pain 4 points
- • Flexion in response to pain (decorticate posturing) 3 points
- • Extension response in response to pain (decerebrate posturing) 2 points
- • No response 1 point
- Categorization:
- • Coma: No eye opening, no ability to follow commands, no word verbalizations (3-8)
- Head Injury Classification:
- • Severe Head Injury----GCS score of 8 or less
- • Moderate Head Injury----GCS score of 9 to 12
- • Mild Head Injury----GCS score of 13 to 15
-
170
- 1. An 88-year male patient presented with end stage renal disease with coronary artery block and metastasis in the lungs. Now presents with acute cholecystitis, patient’s relatives need treatment to do something:
- a. Open cholecystectomy
- b. Tube cholecystostomy
- c. Laparoscopic cholecystectomy
- d. Antibiotics then elective cholecystectomy
- Ans: b
- Ref: Tube Cholecystostomy Before Cholecystectomy for the Treatment of Acute Cholecystitis by Kei Suzuki; JSLS. 2015 Jan-Mar; 19(1): e2014.00200.
- In high-risk patients receiving cholecystostomy tubes for acute cholecystitis, only about one third will undergo surgical cholecystectomy. Laparoscopic cholecystectomy performed in this circumstance has a higher rate of conversion to open surgery and higher hepatobiliary morbidity rate.
- 2. A 40-years old male, chronic alcoholic, diagnosed as cirrhosis, presents with a lump in the right lobe of liver. Serum AFP level is normal. On CT scan, lesion enhances on arterial phase and washes out on portal venous phase. Most probable diagnosis is:
- a. Fibrohyperplasia
- b. Hepatocellular carcinoma
- c. Secondaries
- d. Hepatocellular adenoma
- Ans: b
- Ref: Bailey and Love’s Short Practice of Surgery 27th Edition, Page No: 1173 | Ref: Jacob Mandell; Visual Approach to Diagnostic Imaging; Core Radiology Page No: 94
- Hepatocellular carcinoma
- • HCC comprises the overwhelming majority of primary liver cancers, with a steadily rising global burden.
- • There is wide variation in the geographical incidence of HCC, with >80% of cases occurring in Asia and sub-Saharan Africa, with an incidence of 99 per 100 000. By contrast, the incidence in Europe is considerably lower (approximately 5 per 100 000).
- • The wide variations in distribution reflect the varying incidence of aetiological factors known to be integral to the development of hepatocellular carcinogenesis.
- • Chronic hepatitis B virus (HBV) infection accounts for >50% of cases of HCC worldwide. HBV as a risk factor for HCC is supported by strong evidence that HBV vaccination programmes have led to falls in the incidence of HCC in high-risk areas such as Hong Kong.
- • Hepatitis C virus (HCV) increases the risk of HCC by 17 times, primarily by promoting end-stage liver disease.
- • There is clear evidence that lifetime alcohol exposure correlates with the incidence of HCC.
- • The classic CT or MRI appearance of HCC is an encapsulated mass that enhances on arterial phase and washes out on portal venous phase. HCC may be difficult to detect on non-contrast or portal venous phase CT. On unenhanced MRI, HCC is characteristically slightly hyperintense on T2-weighted images relative to surrounding liver.
- 3. Which of the following represents the remodeling phase of maturation of collagen in a surgical wound?
- a. Type I replacing type II until a ratio of 3:1 is achieved
- b. Type I replacing type II until a ratio of 4:1 is achieved
- c. Type I replacing type III until a ratio of 3:1 is achieved
- d. Type I replacing type III until a ratio of 4:1 is achieved
- Ans: d
- Ref: Bailey and Love’s Short Practice of Surgery 27th Edition, Page No: 25
- • In the latter part of proliferative phase, there is an increase in the tensile strength of the wound due to increased collagen, which is at first deposited in a random fashion and consists of type III collagen.
- • The remodelling phase is characterised by maturation of collagen (type I replacing type III until a ratio of 4:1 is achieved). There is a realignment of collagen fibres along the lines of tension, decreased wound vascularity, and wound contraction due to fibroblast and myofibroblast activity. This maturation of collagen leads to increased tensile strength in the wound which is maximal at the 12th week post injury and represents approximately 80% of the uninjured skin strength.
- 4. What is the most commonly affected gene mutation in Lynch syndrome?
- a. APC
- b. MLH1
- c. P53
- d. RPC
- Ans: b
- Ref: Bailey and Love’s Short Practice of Surgery 27th Edition, Page No: 1260
- • Hereditary non-polyposis colorectal cancer (HNPCC) is characterised by an increased risk of colorectal cancer and also cancers of the endometrium, ovary, stomach and small intestine.
- • It is an autosomal dominant condition caused by a mutation in one of the DNA mismatch repair genes. The most commonly affected genes are MLH1 and MSH2.
- • The lifetime risk of developing colorectal cancer is 80%, and the mean age of diagnosis is 45 years. Most cancers develop in the proximal colon. Females have a 30–50% lifetime risk of developing endometrial cancer.
-
171
Answers and Winners of QUIZ for Today evening CEE MD/MS Based Online Model test [07:00 PM, Asar 10, Wednesday]
- 1. Methadone is used to treat withdrawal symptoms of:
- a. Cocaine
- b. Heroin
- c. Amphetamine
- d. Barbiturate
- Ans: b
- Ref: Reddy Forensic Medicine, 33rd Edition, Page No: 587
- Treatment of Heroin poisoning
- • Methadone 40 mg. daily will usually prevent withdrawal symptoms; in a chronic addict 80 mg. will usually prevent heroin-induced euphoria. The dose is gradually reduced by 20% daily. If signs of withdrawal appear, dose reduction should proceed more slowly. Heroin addicts should never be given 20 mg methadone at one time.
- • Detoxification.
- • Narcotic antagonist, such as naltrexone, naloxone, haloperidol, clonidine and cyclazocine.
- 2. Universal marker for Limbal stem cells:
- a. Elastin
- b. Collagen
- c. Keratin
- d. ABCG2
- Ans: d
- Ref: ABCG2 Transporter Identifies a Population of Clonogenic Human Limbal Epithelial Cells; Cintia S. de Paiva; Stem Cells.
- ABCG2, a member of the ATP binding cassette (ABC) transporters, has been identified as a molecular determinant for bone marrow stem cells and proposed as a universal marker for stem cells. This study investigates ABCG2 expression and its potential as a marker that identifies human limbal epithelial stem cells. ABCG2 expression was evaluated by immunofluorescent and immunohistochemical staining, laser scanning confocal microscopy, flow cytometry, and semiquantitative reverse transcription–polymerase chain reaction.
- 3. Early post operative complication usually refers to time after surgery of:
- a. Within 1 hr
- b. 1 hr to 6 hr
- c. 6 hr to 72 hr
- d. More than 72 hr
- Ans: c
- Ref: Bailey and Love’s Short Practice of Surgery 27th Edition, Page No: 292
- There are three common approaches for the classification of postoperative complications of surgery:
- 1 Linked to time after surgery:
- Immediate (within 6 h of procedure);
- Early (6–72 h);
- Late (>72 h).
- 2 Generic and surgery specific.
- 3 Clavian-Dindo: this system relates to surgical complications only and is used to objectively and reproducibly measure the impact of the surgical complication on the outcome of the procedure.
- 4. Phantom limb pain are best described by:
- a. Weber Fencher law
- b. Bell Magendie law
- c. Law of projection
- d. Power law
- Ans: c
- Ref: Comprehensive Textbook Of Medical Physiology by GK Pal; 1St edition; Pg 1000,1004
- Weber-Fechner Law
- • The magnitude of sensation felt is proportionate to the log of the intensity of the stimulus. This is called Weber-Fechner law.
- Law of Projection
- • No matter where a specific sensory pathway is stimulated along its course to the cortex, the sensation formed is referred to the location of the receptors. This is called the law of projection.
- • This means, irrespective of the site of application of the stimulus in the sensory pathway, the sensation evoked is felt at the nerve endings (the receptors).
- • This forms the basis of phantom limb. In phantom limb phenomenon, the limb actually does not exist (as the limb has been amputated), but the patient complains that he feels pain or itch in the limb.
• Charles Bell (1774–1842), a brilliant anatomist and neurosurgeon, Charles Bell demonstrated the motor functions of ventral spinal nerve roots. He was the first scientist to study and differentiate the sensory and motor functions of nerves roots. A similar observation was also noted by his contemporary Physiologist, Francois Magendie. Hence, the theory proposing “Dorsal root of spinal cord is sensory and ventral root of spinal cord is motor in function” is popularly known as Bell-Magendie Law.
- Winners:
- Fastest correct answer respondents:
- - Dr Aakankshya Shrestha
- - Dr Isha Gyawali
- Lucky draw winners:
- - Dr Anita Bhatta
- - Dr Kanchan KC
Winners are requested to send an email to onlinepgnepal@gmail.com to claim Free Access to Today evening CEE MD/MS Based Online Model test [07:00 PM, Asar 10, Wednesday]
-
172
- 1. Pernicious anaemia is most likely to cause lesions in which part of the central nervous system?
- a. Central gray matter of the spinal cord
- b. Hippocampus
- c. Cerebellum
- d. Posterior columns of the spinal cord
- Ans: d
- Ref: Robbins and Cotran Pathologic basis of disease 9/e p 645,646
- Pernicious anemia refers to anemia that results from lack of intrinsic factor leading to vitamin B12 deficiency. Lack of intrinsic factor is most commonly due to an autoimmune attack on the cells that create it in the stomach. It can also occur following the surgical removal of part of the stomach or from an inherited disorder.
- Paraesthesias, weakness and unsteady gait may be seen in pernicious anaemia. These symptoms are due to myelin degeneration and loss of nerve fibres in the lateral and dorsal columns of the spinal cord (posterior columns of spinal cord).
- 2. Milan criteria is used in transplantation of
- a. Kidney
- b. Liver
- c. Cornea
- d. Pancreas islets
- Ans: b
- Milan criteria is used in transplantation of liver with HCC. The criteria are:
- • Single tumor diameter less than 5 cm
- • Not more than three foci of tumor, each one not exceeding 3 cm
- • Nomacrovascular invasion
- 3. Maximum absorption of short chain fatty acid produced by bacteria occurs in?
- a. Duodenum
- b. Jejunum
- c. Ileum
- d. Colon
- Ans: d
- Ref: Ganong Review of Medical Physiology, Lange; 25th Edition, Page No: 483, 493
- Short chain fatty acids (SCFAs) are 2 – 5 carbon weak acids that have an average normal concentration of about 80 mmol/L in the lumen. They are formed by the action of colonic bacteria on complex carbohydrates, resistant starches, and other components of the dietary fiber, that is, the material that escapes digestion in the upper gastrointestinal tract and enters colon. SCFAs that are produced in the colon (by bacterial action) are absorbed from it.
- 4. Legal age of marriage in Nepal for girls is?
- a. 18 years
- b. 16 years
- c. 20 years
- d. 22 years
- Ans: c
- Ref: Muluki Ain; Registration of Marriage Act Nepal
- In Nepal the following age restrictions are present to prohibit child marriage:
- • Legal age for marriage is 20 years for both male and female.
- • Legal definition of children is not reached 16 years. (In Human trafficking and control act: Children means age not reached 18 years)
-
173
Answers and Winners of QUIZ for Today's CEE MD/MS Based Model test (11:00 AM, Asar 13, Saturday)
- 1. Ticlopidine act by:
- a. Decreasing ADP mediated cAMP activation
- b. Inhibiting COX enzyme irreversibly
- c. GP IIb/IIIa antagonist
- d. Phosphodiesterase inhibition
- Ans: a
- Ref: KDT Pharmacology 7th Edition, page No: 630
- Ticlopidine:
- • It is the first thienopyridine which alters surface receptors on platelets and inhibits ADP as well as fibrinogen-induced platelet aggregation. The Gi coupled P2Y12 (also labeled P2YAC) type of purinergic receptors which mediate adenylyl cyclase inhibition due to ADP are blocked irreversibly by the active metabolite of ticlopidine. As a result, activation of platelets is interfered.
- • Fibrinogen binding to platelets is prevented without modification of GPIIb/IIIa receptor. There is no effect on platelet TXA2, but bleeding time is prolonged and platelet survival in extra-corporeal circulation is increased. Because of different mechanism of action, it has synergistic effect on platelets with aspirin. Their combination is a potent platelet inhibitor.
- 2. Early neonatal sepsis occurs within [hours]
- a. 8
- b. 12
- c. 36
- d. 72
- Ans: d
- Ref: OP Ghai Pediatrics 9th Edition, Page No: 160
- Early-onset sepsis (EOS) (less than 72 hr) infections are caused by organisms prevalent in the maternal genital tract or in the delivery area. The predisposing factors include LBW, prolonged rupture of membranes, foul smelling liquor, multiple per vaginal examinations, maternal fever, difficult or prolonged labor and aspiration of meconiurn. EOS frequently manifests as pneumonia and less commonly as septicemia or meningitis.
- Late-onset sepsis (LOS) (72 hr or later) infections are caused by the organisms thriving in the external environment of the home or the hospital. The infection is often transmitted through the hands of the care-providers. The presentation is that of septicemia, pneumonia or meningitis. The predisposing factors include LBW, lack of breastfeeding, poor cord care, superficial infections (pyoderma, umbilical sepsis), aspiration of feeds and disruption of skin integrity with needle pricks and use of intravenous fluids.
- 3. High risk pregnancy includes all except:
- a. Twins
- b. 25 years old primi
- c. Hydramnios
- d. Previous LSCS
- Ans: b
- Ref: Dutta Obstetrics 8th Edition, Page No: 717
- According to WHO, risk approach for MCH care is to identify the high-risk cases from a large group of antenatal mothers. These cases are:
- During pregnancy:
- • Elderly primi (>30 years)
- • Short statured primi (<140 cm)
- • Threatened abortion and APH
- • Malpresentation
- • Preeclampsia and eclampsia
- • Anemia
- • Elderly grand multipara
- • Twins and hydramnios
- • Previous stillbirth, IUD, manual removal of placenta
- • Prolonged pregnancy
- • History of previous cesarean section and instrumental delivery
- • Pregnancy associated with medical diseases.
- During labor:
- • PROM
- • Prolonged labor
- • Hand, feet or cord prolapse
- • Placenta retained more than half an hour
- • PPH
- • Puerperal fever and sepsis
- 4. Which of the following nerve does not supply the pinna?
- a. Greater auricular nerve
- b. Greater occipital nerve
- c. Lesser occipital nerve
- d. Vagus nerve
- Answer: B
- Ref: PL Dhingra and Shruti Dhingra ENT and HNS 6th Edition, Page No: 4
- Pinna
- • Greater auricular nerve (C2,3) supplies most of the medial surface of pinna and only posterior part of the lateral surface
- • Lesser occipital (C2) supplies upper part of medial surface.
- • Auriculotemporal (V3) supplies tragus, crus of helix and the adjacent part of the helix.
- • Auricular branch of vagus (CN X), also called Arnold’s nerve, supplies the concha and corresponding eminence on the medial surface.
- • Facial nerve, which is distributed with fibres of auricular branch of vagus, supplies the concha and retroauricular groove.
- External auditory canal
- • Anterior wall and roof: auriculotemporal (V3).
- • Posterior wall and floor: auricular branch of vagus (CN X).
- • Posterior wall of the auditory canal also receives sensory fibres of CN VII through auricular branch of vagus
- Winners:
- Fastest correct answer respondents:
- - Dr Kismat Gurung
- - Dr Ashish (MDMS Discussion group)
- Lucky draw winners:
- - Dr Prabina Aryal
- - Dr Reena Lamichhane
Winners are requested to send an email to onlinepgnepal@gmail.com to claim Free Access to Today's CEE MD/MS Based Model test (11:00 AM, Asar 13, Saturday)
-
174
- Most common site of peripheral arterial disease
- a. Femoral and popliteal arteries
- b. Abdominal and iliac arteries
- c. Tibial artery
- d. Peroneal artery
- Ans: a
- Ref: Harrison’s Principles of Internal Medicine; 20th Edition, Page No: 1923
- The primary sites of involvement are the abdominal aorta and iliac arteries (30% of symptomatic patients), the femoral and popliteal arteries (80–90% of patients), and the more distal vessels, including the tibial and peroneal arteries (40–50% of patients). Atherosclerotic lesions occur preferentially at arterial branch points, which are sites of increased turbulence, altered shear stress, and intimal injury. Involvement of the distal vasculature is most common in elderly individuals and patients with diabetes mellitus.
-
175
- Answers and Winners of
- Revision QUIZ Based on Question, Answer, and Explanation (QAE) file of Asar 13:
1. Cobb angle in scoliosis is greater than: 10 degrees
2. Kangaroo mother care started in which country: Colombia
3. Myringostapediopexy is which type of Tympanoplasty: Type III
4. Chopart’s dislocation involves which joint: talo-navicular/calcaneocuboid joints (midtarsal)
5. Mesencephalic nucleus is which type of neuron (According to the number of their processes ) : Unipolar
[Please refer QAE File for Explanations]
- Winners:
- Fastest Correct answer respondents:
- - Dr Sandeep Sapkota
- - Dr Prabhat Rijal
- Lucky draw winners:
- - Dr Pawan Rastrabadi
- - Dr Sarik Shrestha
Winners are requested to send an email to onlinepgnepal@gmail.com to claim Free Access to SUBJECTWISE (PEDIATRICS & GYNAE/OBS) BASED MODEL TEST [Asar 15, Monday, 07:00 PM].
-
176
- Pediatric Quiz
- Answers and Winners:
- 1. A 6-year-old girl presents to hospital with a large right-sided abdominal mass. It does not cross the midline. On further questioning she has had macroscopic haematuria and weight loss of 4 kg over the last 4 months. She has reduced appetite and lethargy. Her blood pressure is 125/73 mmHg, heart rate 120 bpm. Which of the following is not a complication of this malignancy?
- a. Malnutrition
- b. Hypertension
- c. Renal impairment
- d. Urinary catecholamines
- Ans: d
- Ref: Nelson Textbook of Pediatrics, Elsevier, 20th Edition, Page No: 2464, 2465
- Wilms’ tumour can result in hypertension and renal impairment, dependent on the functioning of the contralateral kidney. General complications of malignant disease include metastases and malnutrition due to poor appetite, vomiting and increased metabolic demands.
- Sympathetic nervous system stimulation does not occur with Wilms’ tumours but with neuroblastoma, an important differential diagnosis, due to catecholamine production from the tumour which originates from the adrenal medulla. Catecholamines can be detected in urine samples.
- 2. COACH syndrome does not include:
- a. Cerebellar vermis hypoplasia
- b. Congenital ataxia
- c. Choledochocele
- d. Coloboma
- Ans: c
- Ref: Nelson Textbook of Pediatrics, Elsevier, 20th Edition, Page No: 1969
- COACH syndrome: Cerebellar vermis hypoplasia, oligophrenia, congenital ataxia, ocular coloboma, and hepatic fibrosis (MKS3, CC2D2A, RPGRIP1L)
- 3. Most common spontaneous thromboembolism in neonates is:
- a. Mesenteric thrombosis
- b. Renal vein thrombosis
- c. Middle cerebral artery thrombosis
- d. Portal vein thrombosis
- Ans: b
- Ref: Nelson Textbook of Pediatrics, Elsevier, 20th Edition, Page No: 2395
- • Renal vein thrombosis is the most common spontaneous thromboembolism in neonates.
- • Affected infants may present with hematuria, an abdominal mass, and/or thrombocytopenia.
- • Infants of diabetic mothers are at increased risk, although the mechanism for this increased risk is unknown. Approximately 25% of cases are bilateral.
- 4. Which mechanism in phototherapy is chiefly responsible for reduction in serum bilirubin?
- a. Photo-oxidation
- b. Configurational isomerization
- c. Structural isomerization
- d. Conjugation
- Ans: c
- Ref: OP Ghai Pediatrics 8th Edition, Page No: 175
- Phototherapy -Phototherapy remains the mainstay of treating hyperbilirubinemia in neonates. Phototherapy is highly effective and carries an excellent safety track record of over 50 yr. It acts by converting insoluble bilirubin (unconjugated) into soluble isomers that can be excreted in urine and feces. Many review articles have provided detailed discussion on phototherapy related issues. The bilirubin molecule isomerizes to harmless forms under blue-green light (460-490 nm); and the light sources having high irradiance in this particular wavelength range are more effective than the others.
- For phototherapy to be effective, bilirubin needs to be present in skin so there is no role for prophylactic phototherapy. Phototherapy acts by several ways:
- • Configurational isomerization: Here the Z-isomers of bilirubin are converted into E-isomers. The reaction is instantaneous upon exposure to light but reversible as bilirubin reaches into the bile duct. After exposure of 8-12 hr of phototherapy, this constitutes about 25% ofTSB, which is nontoxic. Since this is excreted slowly from body this is not a major mechanism for decrease in TSB.
- • Structural isomerization: This is an irreversible reaction where the bilirubin is converted into lumirubin. The reaction is directly proportional to dose of phototherapy. This product forms 2-6% of TSB which is rapidly excreted from body thus is mainly responsible for phototherapy induced decline in TSB.
- • Photo oxidation: This is a minor reaction, where photo-products are excreted in urine
- Winners:
- Fastest Correct answer respondents:
- - Dr Rajeev Bastola
- - Dr Diwas Sunar
- Lucky draw winners:
- - Dr Manisha Sah
- - Dr Sabin Rajbhandari
Winners are requested to send an email to onlinepgnepal@gmail.com to claim Free Access to SUBJECTWISE (PEDIATRICS & GYNAE/OBS) BASED MODEL TEST [Asar 15, Monday, 07:00 PM].
-
177
- Obstetrics and Gynecology Quiz
- Answers and Winners:
- 1. National guidelines on cervical cancer screening and prevention (2010) Nepal call for screening of:
- a. At least 50 percent of women aged 30–60 years
- b. At least 50 percent of women aged 18–60 years
- c. At least 100 percent of women aged 30–60 years
- d. At least 100 percent of women aged 18–60 years
- Ans: a
- Cervical cancer screening and prevention training:
- Cervical cancer is the most common cancer of women in Nepal, accounting for 21.4 percent of all cancer among 34–64 year old women.
- The national guidelines on cervical cancer screening and prevention (2010) call for screening at least 50 percent of women aged 30–60 years and for reducing the mortality due to cervical cancer by 10 percent with recommended screening among this group every five years.
- Cervical cancer screening is done by visual inspection of the cervix by trained nurses or doctors using acetic acid. If any signs of a pre-cancerous lesion are seen, women are referred for cryotherapy to cure the lesion. This approach is cost-effective as the early detection of lesions and early management by cryotherapy will usually prevent progression to cervical cancer, and the cost of scaling up this activity is relatively low.
- 2. A pelvis characterized by an anteroposterior diameter of the inlet greater than the transverse diameter is classified as:
- a. Gynecoid
- b. Android
- c. Anthropoid
- d. Platypelloid
- Ans: c
- Ref: Dutta Obstetrics 8th Edition, Page No: 403
- • Normal female pelvis – Gynaecoid pelvis.
- • Male type pelvis – Android pelvis.
- • Most common type of pelvis – Gynaecoid pelvis.
- • Least common type pelvis – Platypelloid pelvis.
- • The only pelvis with AP diameter more than transverse diameter – Anthropoid pelvis.
- • Face to pubes delivery is most common in Anthropoid pelvis.
- • Direct occipitoposterior position is most common in Anthropoid pelvis.
- • Persistent occipitoposterior position is most common in Android pelvis.
- • Deep transverse arrest/ Nonrotation/dystocia is most common in Android pelvis.
- • Broad flat pelvis – Platypelloid pelvis.
- • Transverse diameter is much more than AP diameter – Platypelloid pelvis.
- • Engagement by exaggerated posterior asynclitism occurs in Platypelloid pelvis.
- • Super subparietal instead of biparietal diameter engages in Platypelloid pelvis.
- 3. Lochia in correct order during puerperium:
- a. Rubra, serosa, alba
- b. Serosa, rubra, alba
- c. Alba, serosa, rubra
- d. Alba, mucosa, serosa
- Ans: a
- Ref: Dutta Obstetrics 8th Edition, Page No: 170
- Lochia
- It is the vaginal discharge for the first fortnight during Puerperium
- • The discharge originates from the uterine body, cervix and vagina.
- • It has got a peculiar offensive fishy smell.
- • Its reaction is alkaline tending to become acid towards the end.
- • Depending upon the variation of the color of the discharge, it is named as:
- – Lochia rubra (red) 1-4 days.
- – Lochia serosa (5-9 days) — the color is yellowish or pink or pale brownish.
- – Lochia alba — (pale white) — 10-15 days.
- • Composition:
- • Lochia rubra consists of blood, shreds of fetal membranes and decidua, vernix caseosa, lanugo and meconium.
- • Lochia serosa consists of less RBC but more leukocytes, wound exudate, mucus from the cervix and microorganisms (anaerobic streptococci and staphylococci).
- • Lochia alba contains plenty of decidual cells, leukocytes, mucus, cholestrin crystals, fatty and granular epithelial cells and microorganisms.
- • The average amount of discharge for the first 5–6 days is estimated to be 250 mL.
- • The normal duration may extend up to 24–36 days. The red lochia may persist for longer duration especially in women who get up from the bed for the first time in later period.
- 4. A 27-year-old woman comes to the emergency department at 14 weeks gestation with abdominal pain. She states she has passed a large amount of blood from her vagina in the past 2 hours. Her vitals are within normal limits and she describes her abdominal pain as a 4/10. Physical exam is notable for a dilated cervix. The patient states she does not want any invasive procedures in her workup but is accepting to necessary medical interventions. Which of the following is the best next step in management?
- a. Admission and external fetal monitoring
- b. Dilation and curettage
- c. Expectant management
- d. Mifepristone
- Ans: c
- • This patient is presenting with a dilated cervix with abdominal pain and passage of blood from her vagina suggesting a diagnosis of an inevitable abortion. She does not desire any invasive procedures and is stable, thus management is expectant only.
- • An inevitable abortion presents with abdominal pain, a dilated cervix, and vaginal bleeding. In contrast, a completed abortion presents with a closed (no cervical dilation) cervix in which all of the products of conception have passed. The management of an inevitable abortion depends on the patient’s vital signs and their preferences. In an unstable patient, optimal management is dilation and curettage. If the patient is stable, expectant management, misoprostol, or dilation and curettage are all appropriate and the optimal course of care is dependent on patient preferences. Stable patients who do not want invasive management can be discharged and managed expectantly but should be followed for eventual passage of products of conception with a repeat cervical examination.
- • Admission and external fetal monitoring is appropriate management after premature rupture of membranes in a viable fetus or in a pregnant patient whose fetus demonstrates concerning heart rate variability. An inevitable or missed abortion does not need admission nor does it need external fetal monitoring.
- • Dilation and curettage could be appropriate management in an inevitable abortion to expedite the process; however, this is at the discretion of the mother. This mother does not want any invasive procedures and this procedure is not medically necessary.
- • Mifepristone helps soften the cervix, increases sensitivity to prostaglandins, and necrotizes uterine decidua, and may help expedite the passage of an inevitable abortion; however, it is not necessary in an inevitable abortion in a patient who does not want any interventions that are not medically necessary.
- Winners:
- Fastest Correct answer respondents:
- - Dr Naresh Gharti
- - Dr Subash Lamichhane
- Lucky draw winners:
- - Dr Subechhya Basnet
- - Dr Priya Parajuli
Winners are requested to send an email to onlinepgnepal@gmail.com to claim Free Access to SUBJECTWISE (PEDIATRICS & GYNAE/OBS) BASED MODEL TEST [Asar 15, Monday, 07:00 PM].
-
178
- Congratulations to all the winners.
- First two fastest correct respondents from Viber discussion group:
- 1. Dr. Ajwani Rimal
- 2. Dr. Dinesh Neupane
- Two lucky winners among all correct answers respondents:
- 1. Dr. Roshan Karki
- 2. Dr. Shristi Thapa
- We will contact you soon for your free tokens or you can contact us through messenger or viber for the free tokens (For CEE based MDMS exam on 14th Ashadh, Sunday, 7pm)
- Explanations to yesterday’s quiz:
- Ans: ABBC
- 1. Which among the following is the more common tumor of testes?
- a. Embryonal cell carcinoma
- b. Interstitial tumor
- c. Lymphoma
- d. None
- Ans: A, Embryonal cell carcinoma
- Ref: Short Practice of Surgery, Bailey and Love, 27th edition, page no.1506
- Tumours of the testis are classified according to their predominant cellular type:
- 1. Germ cell tumours (90–95%) (these include seminoma, embryonal cell carcinoma, yolk sac tumour, teratoma, and choriocarcinoma);
- 2. Interstitial tumours (1–2%) (these include Leydig cell tumours);
- 3. Lymphoma (3–7%);
- 4. Other tumours (1–2%).
- 2. You are assisting a case of inguinal hernia repair. The operating surgeon accidentally cut the bladder wall. The bladder rupture in such conditions is more likely to be?
- a. Intraperitoneal
- b. Extraperitoneal
- c. Equal chance of either
- d. None
- Ans, B, Extraperitoneal
- Ref: Short Practice of Surgery, Bailey and Love, 27th edition, page no.1425
- Bladder rupture
- • This can be intraperitoneal (20%) or extraperitoneal (80%).
- • Intraperitoneal rupture is usually secondary to a blow or fall on a distended bladder, and more rarely to surgical damage.
- • Extraperitoneal rupture is caused by blunt trauma or surgical damage.
- • Gross haematuria can be absent.
- • It may be difficult to distinguish extraperitoneal rupture from rupture of the membranous urethra.
- • Intraperitoneal rupture is associated with sudden severe pain in the hypogastrium, often accompanied by syncope. The shock subsides and the abdomen distends and there is no desire to micturate.
- Peritonitis does not follow immediately if the urine is sterile; varying degrees of rigidity are present on examination. The bladder may be injured in (1) inguinal or femoral herniotomy, (2) hysterectomy and (3) excision of the rectum.
- 3. Skin is the heaviest/largest organ of human body which comprises of ____ % of normal adult body weight?
- a. 6%
- b. 16%
- c. 26%
- d. 36%
- Ans: B. 16%
- Ref, BRS Cell biology and histology, 7th edition, page no. 241
- Skin overview
- a) The skin is the heaviest organ, about 16% of the total body weight.
- b) It is composed of two layers, the epidermis and the dermis, which interdigitate to form an irregular contour.
- c) A deeper superficial fascial layer, the hypodermis, lies under the skin. This layer, which is not considered part of the skin, consists of loose connective tissue that binds skin loosely to the subjacent tissue.
- d) The skin contains several epidermal derivatives (sweat glands, hair follicles, sebaceous glands, nails, and the mammary glands).
- e) The skin along with its derivatives is called the integument.
- Function
- a) Protects the body against injury, desiccation, and infection;
- b) Regulates body temperature; absorbs ultraviolet (UV) radiation, which is necessary for synthesis of vitamin D;
- c) Contains receptors for touch, temperature, and pain stimuli from the external environment.
- d) Additionally, skin acts as an excretory organ via sebaceous, sweat, and apocrine glands.
- 4. A patient is diagnosed as a case of mania. All of the followings are the basis for the diagnosis, except one. Please choose the right option.
- a. Grandiosity
- b. Impulsivity
- c. Increased need for sleep
- d. Distractibility
- Ans: C, Increased need for sleep
- Ref. FA, USMLE step 1, 2020, Page no. 560-561
- Manic episode
- • Distinct period of abnormally and persistently elevated, expansive, or irritable mood and activity or energy lasting ≥ 1 week.
- • Diagnosis requires hospitalization or marked functional impairment with ≥ 3 of the following (DIG FAST):
- 1) Distractibility
- 2) Impulsivity/Indiscretion - seeks pleasure without regard to consequences (hedonistic)
- 3) Grandiosity - inflated self-esteem
- 4) Flight of ideas - racing thoughts goal-directed
- 5) Activity/psychomotor
- 6) Agitation
- 7) Decreased need for Sleep
- ? Talkativeness or pressured speech
-
179
Answers to yesterday's Questions (BCB)
- 1. Quantity of water available per capita considered adequate to meet the needs
- of all urban domestic purposes?
- a. 100-150 L
- b. 150-200 L
- c. 200-250 L
- d. 250-300 L
B
- 2 L is the amount of water required for meeting the physiological requirements of
- the body everyday
- But water is also required for many other activities of daily life such as cooking,
- washing, bathing etc:
- Taking this into consideration 150 200 L of water per capita is the amount
- considered adequate to meet the needs of an urban domestic purposes
Ref: Park's Textbook of Preventive and Social Medicine, 20th edition p617
2."Strings of pearls" appearance is seen in?
- a. SAIO
- b.Gallstone ileus
- c. Chronic pancreatitis
- d. Duodenal perforation
C
- Explanation
- Chronic pancreatitis is characterized by irregularities of of the pancreatic ducts, ductal
- strictures, and area of duct dilation. The major as well as the side branch ducts may be
- involved. For unexplained reason, some patients with chronic pancreatitis develop dilated
- main pancreatic ducts (large ductal disease), whereas others retain ducts of normal or
- even smaller than normal calibers (small ductal disease). Some patients with chronic
- pancreatitis can be shown to have major ducts that have the appearance of a "chain of
- lakes" or a "strings of pearls" that is the result of segment of dilated duct separated by
- areas of ductal stricture.
Ref: Sutton's Radiology 7/e, Volume 1, Page 798
3. Which of the following condition is most likely associated with the DNA topoisomerase I antibody test is positive. ?
- a. Ankylosing spondylitis
- b. Diffuse systemic sclerosis
- c. Discoid lupus erythematosus
- d. Limited scleroderma
B
Ref, Robbins & Cotran, Pathologic basis of disease, 7e, p 237-239
- Explanation
- Raynaud phenomenon, skin changes, and esophageal dysmotility also can occur in limited scleroderma (CREST syndrome), but lung and renal involvement typically do not. In diffuse systemic sclerosis, the anti–DNA topoisomerase I antibody is often present, and patients can develop interstitial lung disease and renal disease with hyperplastic arteriolosclerosis. A feature of discoid lupus erythematosus is skin rashes, but usually there is no internal organ
- involvement. Ankylosing spondylitis is one of the spondyloarthropathies; it is characterized by low back pain from sacroiliitis and positive serology for HLA-B27. Reiter syndrome is characterized by conjunctivitis, arthritis, and nongonococcal urethritis, with a positive serology for HLA-B27. In rheumatoid arthritis, there is often progressive joint deformity; the serologic tests likely to be positive include rheumatoid factor and antibodies to cyclic citrullinated peptide (anti-CCP). The anti-Sm or anti–double-stranded DNA antibodies are more specific for systemic lupus erythematosus, and renal disease in these patients is most likely due to glomerulonephritis.
-
180
- Answer to daily question of yesterday (Asar 13):
- Answer sequence(ABA)
- Q1.There are important structural and functional changes in the isthmus during pregnancy.
- During the first trimester, isthmus hypertrophies and elongates to about:
- a.Three times its original length
- b. Five times its original length
- c. Seven times its original length
- d. One and half times its original length
- Ans: a.Three times its original length
Ref: Dc Dutta Obstetrics 8th edition page no 53,54
- UTERUS
- • There is enormous growth of the uterus during pregnancy.
- • The uterus which in nonpregnant state weighs about 60 g, with a cavity of 5–10 mL and measures about 7.5 cm in length, at term, weighs 900–1,000 g and measures 35 cm in length.
- • The capacity is increased by 500–1,000 times. Changes occur in all the parts of the uterus—body, isthmus and cervix
- ISTHMUS
- • There are important structural and functional changes in the isthmus during pregnancy.
- • During the first trimester, isthmus hypertrophies and elongates to about 3 times its original length. It becomes softer.
- • With advancing pregnancy beyond 12 weeks, it progressively unfolds from above, downward until it is incorporated into the uterine cavity.
- • The circularly arranged muscle fibers in the region function as a sphincter in early pregnancy and thus help to retain the fetus within the uterus.
- • Incompetency of the sphincteric action leads to mid-trimester abortion and the encirclage operation done to rectify the defect is based on the principle of restoration of the retentive function of the isthmus.
- Q2. Small intestine avulsions, intestinal, omental and mesenteric lacerations and intestinal contusion and perforations may occur in seat belt users during traafic accident called seat belt syndrome. This is due to:
- a. Acute extension over a lap strap
- b. Acute flexion over a lap strap
- c. Acute Extension followed by flexion
- d. Rotational force
- Ans: b. Acute flexion over a lap strap
- Ref: Reddy, The essential of forensic medicine and toxicology 33rd edition, page number 283
- Seat Belt Injuries:
- • Seat belts are of the lap strap and shoulder diagonal type, which are of the "inertia-reel" type, which allow slow movement but jam at a sudden tug.
- • They are less effective than three point belts.
- • A seat-belt acts in the following ways:
- (1) It restrains the body against severe deceleration, keeping it away from windscreen, steering wheel and other obstructions.
- (2) It spreads the decelration force over a considerable area of broad strap against the body surface.
- (3) The strap stretches during deceleration and reduces G-forces by slightly extending the time of deceleration.
- ( 4) It prevents ejection from the vehicle into the road.
- It reduces risk of death by about 40%.
- • Small intestine avulsions, intestinal, omental and mesenteric lacerations and intestinal contusion and perforations occur, known as "seat belt syndrome", due to acute flexion over a lap strap.
- • Rupture of the spleen, liver, pancreas, caecum and bladder occur due to compression between belt and vettebrae.
- • Abrasions, contusions and haematomas of the lower abdomen and chest wall can occur as a direct result of seat belt trauma.
- • The abdominal aorta can be crushed.
- • Transverse fractures of lumbar vertebrae, usually at the second or third segment of interspace can occur due to acute trunk hyperflexion over the belt fulcrum.
- • The posterior arch, pedicle or transverse processes may be fractured.
- Q3. Disinfection is the antimicrobial reduction of the number of viable microorganisms and are often subdivided into high level, intermediate, and low level. Low-level disinfectants are generally effective against:
- a. Enveloped viruses
- b. Mycobacteria
- c. Bacterial spores
- d. All of the above
- Ans: a. Enveloped viruses
- Ref: Gerald E. McDonnell Antisepsis, Disinfection, and Sterilization page number 4
- Disinfection:
- • The antimicrobial reduction of the number of viable microorganisms, or bioburden, on or in a product or surface to a level previously specified as appropriate for its intended further handling or use.
- • In general, disinfection is used to describe a product (a disinfectant) or process that is effective against most pathogens, with the exception of bacterial spores, which are considered the organisms most resistant todisinfection and sterilization.
- • Disinfectants are often subdivided into high level, intermediate, and low level (depending on the product claims and registrations).
- • High-level disinfectants are considered effective against most microbial pathogens, with the exception of large numbers of bacterial spores.
- • These products are usually sporicidal over longer exposure effective against most microbial pathogens, with the exception of large numbers of bacterial spores times.
- • Intermediate-level disinfectants are effective against mycobacteria, vegetative bacteria, most viruses, and fungi, but not necessarily bacterial or some fungal spores.
- • Low-level disinfectants are generally effective against most bacteria, some (in particular, enveloped) viruses, and some fungi, but not mycobacteria and bacterial spores.
-
181
- Dear doctors,
- Congratulations to the winners of yesterday’s quiz. You will be contacted with the free tokens for the upcoming CEE based MDMS exam (going to be held on 21st Ashadh, Sunday, 7pm) very soon.
- First two correct responders:
- 1. Dr. Dinesh Neupane
- 2. Dr. Subhas Lamichhane
- Luckiest two correct responders:
- 1. Dr. Manish (from Viber group)
- 2. Dr. Adhikari Trayush (from FB page)
- Yo can contact us in any way to claim ta free tokens.
- Correct Ans: CDBD
- 1. Out of all platelets extruded from the bone marrow, 60-70% of the platelets are circulated in the blood. What is the location of the remainder platelets?
- a. Liver
- b. Peri-vascular areas
- c. Spleen
- d. Lymph nodes
- Ans: C, Spleen
- Ref, Ganong’s Review of Medical Physiology, 26th edition, page no. 545
- PLATELETS
- - Platelets are small, granulated bodies that aggregate at sites of vascular injury.
- - They lack nuclei and are 2–4 μm in diameter.
- - There are about 300,000/μL of circulating blood, and they normally have a half-life of about 4 days.
- - The megakaryocytes, giant cells in the bone marrow, form platelets by pinching off bits of cytoplasm and extruding them into the circulation.
- - Between 60% and 75% of the platelets that have been extruded from the bone marrow are in the circulating blood, and the remainder are mostly in the spleen.
- - Splenectomy causes an increase in the platelet count (thrombocytosis).
- 2. In uncompensated metabolic alkalosis
- a. The plasma pH, the plasma HCO3– concentration, and the arterial PCO2 are all low.
- b. The plasma pH is high and the plasma HCO3– concentration and arterial PCO2 are low.
- c. The plasma pH and the plasma HCO3– concentration are low and the arterial PCO2 is normal.
- d. The plasma pH and the plasma HCO3– concentration are high and the arterial PCO2 is normal.
- Ans: D, The plasma pH and the plasma HCO3– concentration are high and the arterial PCO2 is normal.
- Ref, Ganong’s Review of Medical Physiology, 26th edition, page no. 635
- METABOLIC ACIDOSIS & ALKALOSIS
- - Blood pH changes can also arise by non-respiratory mechanism.
- - Metabolic acidosis (or non-respiratory acidosis) occurs when strong acids are added to blood. If, for example, a large amount of acid is ingested (eg, aspirin overdose), acids in the blood are quickly increased.
- - The H2CO3 that is formed is converted to H2O and CO2, and the CO2 is rapidly excreted via the lungs. This is the situation in uncompensated metabolic acidosis.
- - Note that in contrast to respiratory acidosis, metabolic acidosis does not include a change in Pco2; the shift toward metabolic acidosis occurs along an isobar line.
- - When the free [H+] level falls as a result of addition of alkali, or more commonly, the removal of large amounts of acid (eg, following vomiting), metabolic alkalosis results.
- 3. Nepal Medical Council holds the right to erase the name of medical practitioners depending upon the gravity and frequency of the violation of code of ethics or medical negligence along with any offences involving moral turpitude. And, the medical practitioner can apply for restoration of his registration any time after 2 years. But, after analysis of the application by The Professional Conduct and Health Committee, based on the merits, can extend the erasure for extra how many months/years?
- a. 6 months
- b. 1 year
- c. 2 years
- d. 3 years
- Ans: B, 1 year
- Ref, Code of Ethics and Professional Conduct-2017, Nepal Medical Council
- Erasure of name from medical register
- - Nepal Medical Council holds the right to erase the name of medical practitioners depending upon the gravity and frequency of the violation of code of ethics or medical negligence along with any offences involving moral turpitude.
- - The council can order the erasure of name of medical practitioner from the medical register.
- - The erasure remains effective unless the medical practitioner’s application is accepted for restoration of name to the medical register.
- Application for restoration of registration
- - Application for restoration of registration can be made at any time after two years from the date of erasure.
- - The Professional Conduct and Health Committee will analyse every application on its merits and may recommend unconditional or conditional restoration or extend the period for another one year.
- - In case of unsuccessful application, a further period of at least 12 months must elapse before a further application can be made.
- Appeal against erasure
- - If a medical practitioner’s registration is erased from the medical register for whatsoever reason, s/he has the right to appeal to the court against it.
- 4. A 29-year-old man presents to the emergency department complaining of stomach cramps, agitation, severe muscle aches, and diarrhea. Physical examination reveals that the patient is sweating; has dilated pupils, a fever, and a runny nose; and shows goose bumps on his skin. What is the most effective immediate treatment or relief of this patient’s symptoms among the following provided options?
- a. Naloxone
- b. Naltrexone
- c. An antipsychotic
- d. Clonidine
- Ans: D, Clonidine
- Ref, BRS Physiology, 6th edition, page no. 88
- • The most likely cause of this patient’s symptoms of sweating, muscle aches, stomach cramps, diarrhea, ever, runny nose, goose bumps, and dilated pupils is heroin withdrawal.
- • Of the choices given, the most effective immediate treatment or heroin withdrawal is clonidine to stabilize the autonomic nervous system.
- • Psychotic symptoms are uncommon in opioid withdrawal, and this patient does not need an antipsychotic.
- • Naloxone and naltrexone as well as stimulants will worsen rather than ameliorate the patient’s withdrawal symptoms.
-
182
Answers to yesterday's Questions (AAA)
1. Earliest radiological sign of Pulmonary Venous Hypertension in Chest X-ray is:
- a. Cephalization of pulmonary vascularity
- b. Pleural effusion
- c. Kerley B lines
- d. Alveolar pulmonary edema
A
- Explanation
- The first radiographic sign of pulmoi ary venous hypertension is cephalization of the
- pulmonary vessels due to pulmonary vein and artery dilatation.
- In the normal individual, there is continuous passage of fluid from the pulmonary veins into
- adjacent interlobular lymphatics that return the fluid to the central mediastinal veins. If the
- lymphatic reserve is overcome by increased transudate as a result of elevated pulmonary
- venous pressure, the interlobular septa are thickened and become visible radiographically.
- The first radiologic sign of pulmonary venous hypertension is cephalization of the
- pulmonary vessels.
- Ref:; Textbook of radiology and Imaging By David Sutton, 5th Edition, Pages 288-89
2. Cryptorchid testis and an inguinal hernia are accompanied with which penile abnormality?
- a. Hypospadias
- b. Phimosis
- c. Epispadias
- d. Bowen disease
A
Ref, Robbins & Cotran, Pathologic basis of disease, 7e, p 1035
- Explanation
- Hypospadias is a congenital condition seen in about 1 in 300 male infants. The inguinal hernia and the cryptorchidism are abnormalities that may accompany this condition. Phimosis is a constriction preventing retraction of the prepuce. It can be congenital, but more likely is the result of inflammation of the foreskin of the penis (e.g., balanitis, a form of local inflammation of the glans penis). Epispadias is a congenital condition in which the urethra opens on the dorsal aspect of the penis. Bowen disease, which is squamous cell carcinoma in situ of the penis, occurs in adults.
3. Theory of web of causation was given by?
- a. Mc Mohan and Pugh
- b. Pettenkoffer
- c. John snow
- d. Louis Pasteur
A
Ref, Park’s Textbook of Preventive and Social Medicine 22nd edition, p32
Explanation
- • Father of (Modern) Medicine: Hippocrates
- • Father of Indian Medicine: Charaka
- • Hindu God of Medicine: Dhanvantari
- • Father of (Modern) Surgery: Ambroise Pare
- • Father of Indian Surgery: Sushruta
- ��� Father of Epidemiology/Modern Epidemiology: John Snow
-
183
- Answers to yesterday’s questions (Wednesday):
- Q1.Chocolate agar is used for isolation of;
- a.N. gonorrhoea
- b.H. influenzae
- c.B. pertusis
- d.E.coli
- Ans: b, H. influenza
- Ref: First aid for USMLE Step 1 2020 pg170
- ●H influenzae = Chocolate agar
- ●N gonorrhoeae, N meningitidis=Thayer-Martin agar.
- ●B pertussis= Bordet-Gengou agar (Bordet for Bordetella),Regan-Lowe medium
- ●C diphtheriae =Tellurite agar, Löffler medium
- ●M tuberculosis =Löwenstein-Jensen medium, Middlebrook medium
- ●M pneumoniae= Eaton agar
- ●E coli Eosin–methylene blue (EMB)#Legionella=Charcoal yeast extract agar buffered with cysteine and iron
- ● Fungi =Sabouraud agar
- Q2.Hemorrhagic conjunctivitis is caused by
- a.Adenovirus 3
- b.Entero virus 70
- c.Herpes simplex virus
- d.None of the above
- Ans: b, Entero virus 70
- Ref: Parsons diseases of eye 22nd edition pg 185
- •Hemorrhagic conjunctivitis is due to picornaviruses, namely, coxsackie virus and enterovirus 70.
- •It is also known as Apollo conjunctivitis and occurs in a pandemic form producing a violent inflammatory conjunctivitis with lacrimation and photophobia.
- •Subconjunctival haemorrhages and enlarged preauricular lymph nodes are common. The cornea is usually affected.
- Q3. Trachoma is caused by
- a.Chlamydia trachomatis A-C
- b.Chlamydia trachomatis D-K
- c.Chlamydia trachomatis L1-3
- d.All of the above.
- Ans:a, Chlamydia trachomatis A-C
- Ref: Parsons diseases of eye 22nd edition pg 186
•Trachoma is caused by Chlamydia trachomatis serotypes A, B, C, so called because it seemed to have a cloak (chylamydos) to the original observers, Halberstaedter and Prowazek.
•The organism is classified as a special type of bacterium which is a prokaryotic, obligatory intracellular parasite, and the two species of the genus are C. trachomatis and C. psittaci.
- •Predisposing factors:
- oAge: In endemic areas, children are often infected in the first few years of life Gender: Female preponderance
- oEnvironmental factors: The disease flourishes among people living in unhygienic and crowded surroundings. It is contagious in its acute stages
- oSource of infection: Spread by the transfer of conjunctival secretions through fingers or towels and, above all, by flies, which are attracted by the presence of profuse ocular discharge. On the other hand, scrupulous cleanliness prevents extension of the disease to healthy subjects.
-
184
- Dear doctors,
- Congratulations to the winners of yesterday’s quiz. You will be contacted with the free tokens for the upcoming CEE based MDMS exam (going to be held on 28th Ashadh, Sunday, 7pm) very soon.
- First two correct responders:
- 1. Dr. Ashok Dhital
- 2. Dr. Dinesh Neupane
- Luckiest two correct responders:
- 1. Dr. Shristi Devkota
- 2. Dr. Bishal Gaire
- You can contact us in any way to claim the free tokens.
- Correct Ans: DADC
- 1. All of the following can induce oxytocin secretion except?
- a. Suckling of breast
- b. Dilation of the cervix
- c. Orgasm
- d. None of the above
- Ans: D, None of the above
- Ref, BRS Physiology, 6th edition, page no. 238
- Regulation of oxytocin secretion
- Suckling
- -Is the major stimulus for oxytocin secretion.
- -Afferent fibers carry impulses from the nipple to the spinal cord. Relays in the hypothalamus trigger the release of oxytocin from the posterior pituitary.
- -The sight or sound of the infant may stimulate the hypothalamic neurons to secrete oxytocin, even in the absence of suckling.
- Dilation of the cervix and orgasm
- - Increases the secretion of oxytocin.
- 2. One of the celebrities from Nepal visits Goa for a vacation. One day after spending a few hours sunbathing in the sea beach, she notices tanning of her body. This is because of exposure to:
- a. UV rays
- b. Yellow light
- c. White light
- d. Infrared rays
- Ans: A, UV rays
- Ref, Textbook of Dermatology, Neena Khanna
- Tanning
- -Etiology: Following exposure to UVR, pigmentation occurs in two phases:
- Immediate pigmentation:
- Occurs within 5 min of exposure to UVA and is due to:
- -Photo-oxidation of already formed melanin.
- -Rearrangement of melanosomes.
- Delayed pigmentation: Begins about 24 h after exposure to both UVB as well as UVA. It is due to:
- -Proliferation of melanocytes.
- -Increased activity of enzymes in melanocytes resulting in increased production of melanosomes.
- -Increased transfer of newly formed melanosomes to adjoining keratinocytes.
- Clinical features
- Pigmentation following exposure to light occurs in two phases:
- - Immediate pigmentation: begins about 5 min after exposure and lasts for about 15 min.
- - Delayed pigmentation: begins about 24 h after exposure and lasts for several days.
Degree of pigmentation depends on the constitutional skin color. Lighter skins burn on UV exposure while darker skins tan.
- 3. The body can be envisaged as three-dimensional segments of tissue called angiosomes, each with an arterial supply and a venous drainage. Blood equilibrates and flows between neighboring angiosomes via some vessels, which tend to be situated within muscles. These vessels are known as;
- a. Bridging vessels
- b. Buddy vessels
- c. Partner vessels
- d. Choke vessels
- Ans: D, Choke vessels
- Ref: Short Practice of Surgery, Bailey and Love, 27th edition, page no. 594
- Blood supply of the skin
- • The body can be envisaged as three-dimensional segments of tissue called angiosomes, each with an arterial supply and a venous drainage. Blood equilibrates and flows between neighboring angiosomes via ‘choke’ vessels, which tend to be situated within muscles.
- • Cutaneous arteries, direct branches of segmental arteries (concentrated at the dorsoventral axes and intermuscular septae), perforate the underlying muscles or run directly within fascial layers to the skin from the deep tissues.
- • The blood supply to the skin anastomoses in subfascial, fascial, sub dermal, dermal and sub epidermal plexi. The epidermis contains no blood vessels so cells there derive nourishment by diffusion.
- • The venous drainage of the skin is via both valved and non valved veins. Unvalved veins allow oscillating flow in the subdermal plexus between cutaneous territories, equilibrating flow and pressure. The valved cutaneous veins drain via plexi to the deep veins.
- 4. A patient appears awake and is not talking. There are no voluntary movements. He can signal with vertical eye movements. The CT scan study shows infarction of ventral pons. The diagnosis is:
- a. Coma
- b. Abulia
- c. Locked in state
- d. Catatonia
- Ans, C, Locked in state
- Ref : Harrison’s 19th ed./p.1771,2578
- • Locked-in syndrome is a rare neuropsychological disorder.
- • Locked-in syndrome (LIS) is a condition in which a patient is aware but cannot move or communicate verbally due to complete paralysis of nearly all voluntary muscles in the body except for the eyes.
- • Its primary features are quadriplegia and paralysis of the cranial nerves except for those responsible for vertical eye movements.
- • Characteristic features: Conscious, alert and awake as the tegmental ascending reticular activating system (ARAS) concerned with arousal is intact, so be careful with your bedside comments. Blinking movements of the eyes are preserved; hence patient is able to communicate in a telegraphic pattern (“on or off'' movements of eyelids).
- • Vertical movements are intact as it is controlled by the interstitial nucleus of Cajal and the rostral part of the medial longitudinal fasciculus (MLF), which is situated in the tegmentum of the midbrain, which is spared in the locked in syndrome.
-
185
Answers to yesterday's Questions (BCB)
- 1. What is the kVp value of the low kVp chest x-ray film?
- a. 20- 40 kVp
- b. 60-80 kVp
- c. 120-170 kVp
- d. 200-350 kVp
Ans, B, 60-80 kVp
Ref, Textbook of radiology and imaging David Sutton, 7e, p2
- Explanation
- Using a low kVp (60-80 kV) produces a high-contrast film in which miliary shadowing and calcification being more clearly seen than on a high kV film. With high kilovoltages of 120-170 kVp the films are of lower contrast in which increased visualization of the hidden areas of the lung due to better penetration of overlying structures. The bones and pulmonary calcification are less well seen.
- 2. Stunning of myocardium without any acute coronary syndrome is:
- a. Subendocardial infarction
- b. Restrictive cardiomyopathy
- c. Takotsubo cardiomyopathy
- d. Transmural infarction
Ans, C, Takotsubo cardiomyopathy
- Ref, Robbins Basic Pathology 9th Edition, p543
- • When there is pathology involving only smaller intra mural vessels, the elevated levels of catechols exacerbate ischemia caused by the vasospasm. This may lead to sudden cardiac death or an ischemic dilated cardiomyopathy, so-called takotsubo cardiomyopathy.
- 3. P-value is the probability of
- a. Not rejecting a null hypothesis when true
- b. Rejecting a null hypothesis when true
- c. Not rejecting a null hypothesis when false
- d. Rejecting a null hypothesis when false
Ans, B, Rejecting a null hypothesis when true
- Ref, Simple Biostatistics by Indrayan & Indrayan, 1/e p156
- P-value:
- Is the ‘Probability of Type I error’ (Null hypothesis is true but rejected)
- Significance level is the maximum tolerable probability of Type I error
- P- value is calculated (on basis of data while Alpha is fixed in advance: by the choice of level of significance employed in the test
- P – value calculated can be less than, equal to or greater than alpha
- Keep Type I error to be minimum (P < a): Then results are declared statistically significant.
-
186
Answers to yesterday’s daily questions:
- 1.True statement about gliobastoma multiforme:
- a. Common in 3rd decade
- b. Most common location is supratentorial
- c. Mostly located in posterior fossa
- d. It has good prognosis
- Ans,b. Most common location is supratentorial
- Ref:Harrison 19th/p.91,17thed./p.2603
- Glioblastoma Multiforme (GBM)
- • GBM is the most common and most aggressive type of primary brain tumor in humans, involving glial cells and accounting for 52% of all parenchymal brain tumor cases and 20% of all intracranial tumors.
- • Excepting the brainstem gliomas, glioblastoma has the worst prognosis of any CNS malignancy
- • Composed of a heterogenous mixture of poorly differentiated neoplastic astrocytes, glioblastomas primarily affect adults, and they are located preferentially in the cerebral hemispheres (so supratentorial)
- • It occurs most often in the subcortical white matter of the cerebral hemispheres. The most frequently affected sites are the temporal (31%), parietal (24%), frontal (23%), and occipital (16%) lobes. Combined fronto-temporal location is particularly typical.
- • Sites for glioblastomas that are much less common are the brainstem (which often is found in affected children), the cerebellum, and the spinal cord.
- 2. A 28 years old man has anterior lenticonus and now ESRD. His maternal uncle also died of similar illness. Diagnosis is:
- a.ARPKD
- b.ADPKD
- c.Oxalosis
- d.Alport’s syndrome
- Ans,d.Alport's syndrome
- Ref: Harrison’s 19th /e.p2513
- • The given clinical picture is typical of Alport's syndrome, which is characterized by triad of hereditary nephritis (ESRD), sensorineural deafness and ocular abnormalities (anterior lenticonus).
- • Primary treatment is control of systemic hypertension and use of ACE inhibitors to slow renal disease progression.
- 3.All of the following are the causes of sudden increase in pain in osteochondroma, except:
- a. Sarcomatous change
- b. Fracture
- c. Bursitis
- d. Degenerative changes
- Ans. is ‘d’ i.e. Degenerative changes
- Ref: Maheshwari, 3rd ed./p.225
- • Osteochondroma usually presents as a painless swelling.
- • Occurrence of pain suggests some complications such as : bursitis at the tip of the swelling, fracture through stalk of the osteochondroma, impingement on or tethering of neighbouring structures such as a tendon, muscle or nerve, malignant transformation
- • A malignant transformation of osteochondroma should be suspected if the osteochondroma continues to grow after skeletal maturity or shows a rapid increase in the size or appearance of sudden pain.
- • The malignancy associated with osteochondroma is most often chondrosarcoma, although malignant fibrous histiocytoma and osteosarcoma have also been reported.
-
187
- Answer to daily question (Ashar 26, Friday)
- Q1.Anterior boundary of inguinal canal is:
- a.Fascia transversalis
- b.Fascia transversalis and internal oblique
- c.Aponeurosis of external oblique and internal oblique muscle
- fibres
- d.Aponeurosis of external oblique and fascia transversalis
- Ans: c. Aponeurosis of external oblique and internal oblique muscle fibres
- Ref: BDC Human Anatomy 5/e, p 208
- Boundaries of inguinal canal:
- The anterior wall is formed by the following.
- (a) In its whole extent: (1) Skin; (2) superficial fascia; and (3) external oblique aponeurosis.
- (b) In its lateral one-third: The fleshy fibres of the internal oblique muscle.
- The posterior wall is formed by the following.
- (a) In its whole extent: (i) The fascia transversalis, (ii) the extraperitoneal tissue, and (iii) the parietal peritoneum.
- (b) In its medial two-thirds: (i) The conjoint tendon; (ii) at its medial end by the reflected part of the inguinal ligament, (iii) over its lateral one-third by the interfoveolar ligament.
- Roof
- It is formed by the arched fibres of the internal oblique and transversusabdominis muscles.
- Floor
- It is formed by the grooved upper surface or the inguinal ligament; and at the medial end by the lacunar ligament.
- 2. Clinically femoral hernia is located at
- a.Medial and upper to pubic tubercle
- b.Lateral and below to pubic tubercle
- c.Sapheno-femoral junction
- d.Lateral and above to pubic tubercle
- Ans: b. Lateral and below to pubic tubercle
- Ref: SRB's Manual of Surgery by SriramBhat M 4/e, Pg. 822
- Clinical Features of femoral hernia
- Common in females (2:1 ratio), common in multipara.
- Rare before puberty. 20% occurs bilateral, however, more common on right side.
- Presents as a swelling in the groin below and lateral to the pubic tubercle (Inguinal hernia is above and medial to thepubic tubercle).
- Swelling, impulse on coughing, reducibility, gurgling sound during reduction, dragging pain, are the usual features.
- When obstruction and strangulation occurs which is more common, presents with features of intestinal obstruction— painful, tender, inflamed, irreducible swelling without any impulse. They also present with abdominal distension, vomiting and features of toxicity.
- Often femoral hernia can be associated with inguinal hernia also.
- 40% of femoral hernias present as emergency hernia with obstruction/strangulation.
- 3. Malignant otitis externa is caused by
- a.Haemophilus influenza
- b.Pseudomonas aeruginosa
- c.Staphylococcus aureus
- d.Proteus vulgaris
- Ans: b. Pseudomonas aeruginosa
- Ref: Dhingra Diseases of Ear, Nose and Throat, 6th Edition Pg 52
- Malignant (necrotizing) otitis externa:
- It is an inflammatory condition caused by pseudomonas infection usually in the elderly diabetics, or in those on immunosuppressive drugs.
- Its early manifestations resemble diffuse otitis externa but there is excruciating pain and appearance of granulations in the ear canal. Facial paralysis is common.
- Infection may spread to the skull base and jugular foramen causing multiple cranial nerve palsies. Anteriorly, infection spreads to temporomandibular fossa, posteriorly to the mastoid and medially into the middle ear and petrous bone.
-
188
Answers and Winners of QUIZ for Today evening CEE MD/MS Based Online Model test [07:00 PM, Wednesday, Asar 17]
- 1. A person following trauma was found to have injury to the anterior part of the spinal cord. Which among the following sensation is most likely to be intact?
- a. Pain
- b. Itch
- c. Tickle
- d. Vibration
- Ans: D, Vibration
- Ref: Guyton’s physiology 13th Ed, Page 609
- Anterolateral system carries:
- • Pain
- • Thermal sensations including both warm and cold sensations
- • Crude touch and pressure sensations capable only of crude localization
- • Tickle and itch sensation
- • Sexual sensation
- Dorsal column carries:
- • Touch sensation requiring a high degree of localization of stimulus
- • Touch sensation requiring transmission of graded intensity
- • Phasic sensation such as vibration
- • Sensations that signal movement against the skin
- • Position sensation from joints
- • Pressure sensation related to fine degrees of judgment of pressure intensity
- 2. Principles of Health Education include all except
- a. Participation
- b. Motivation
- c. Reinforcement
- d. Punishment
- Ans: d
- Ref: Park 23rd Edition, Page No: 861, 862
- Principles of Health Education:
- • Credibility
- • Interest
- • Participation
- • Motivation
- • Comprehension
- • Reinforcement
- • Learning by doing
- • Known to unknown
- • Setting an example - Good human relations
- 3. Intraparenchymal intracranial pressure monitoring is preferred over ventriculostomy in which of the following patients?
- a. A 24-year-old man admitted with traumatic brain injury following a motor vehicle accident and a Glasgow Coma Scale score of 6
- b. A 35-year-old woman with autoimmune hepatitis admitted with fulminant hepatic failure and an international normalized ratio of 3.5
- c. A 48-year-old man admitted to the ICU with subarachnoid hemorrhage following a rupture of an anterior cerebral artery aneurysm
- d. A 68-year-old woman admitted with a hemorrhagic stroke in the right temporal region with associated midline shift visualized on CT scan
- Ans: b
- Ref: Harrison’s Principles of Internal Medicine; 19th Edition, Chap. 330
- • Monitoring intracranial pressure (ICP) is an important tool in the care of selected critically ill patients due to neurologic injury or disease. ICP can be monitored by ventriculostomy, intraparenchymal transducer, epidural catheter, or subarachnoid catheters.
- • Epidural or subarachnoid catheters are less reliable and are less frequently used.
- • Ventriculostomy is considered the “gold standard” for ICP monitoring because it offers the advantage of providing a means of cerebrospinal fluid drainage in the event of elevated ICP. In addition, ventriculostomy monitoring also provides greater accuracy because it has the ability to be recalibrated, whereas the intraparenchymal monitor does not. Patients who are most likely to be considered for ICP monitoring are those with severe primary neurologic disorders, including severe traumatic brain injury (Glasgow Coma Scale ≤8), large tissue shifts from supratentorial ischemic or hemorrhagic stroke, intraventricular hemorrhage, or posterior fossa stroke.
- • Individuals with fulminant hepatic failure are also at risk for severe cerebral edema leading to herniation. ICP monitoring is recommended for these individuals, although intraparenchymal monitoring is often more appropriate than ventriculostomy due to the presence of coagulopathy.
- • These patients are at particularly high risk of bleeding. In addition, individuals with diffuse cerebral edema, such as in fulminant hepatic failure, typically have small ventricles leading to difficulty in placement.
- 4. Stridor in laryngomalacia is:
- a. Expiratory, low-pitched
- b. Inspiratory, high-pitched
- c. Expiratory, high-pitched
- d. Inspiratory, low-pitched
- Ans: d
- Ref: Nelson Textbook of Pediatrics, Elsevier, 20th Edition, Page No: 2036
- Laryngomalacia is the most common congenital laryngeal anomaly and the most common cause of stridor in infants and children.
- Sixty percent of congenital laryngeal anomalies in children with stridor are due to laryngomalacia.
- Stridor is inspiratory, low-pitched, and exacerbated by any exertion: crying, agitation, or feeding. The stridor is caused, in part, by decreased laryngeal tone leading to supraglottic collapse during inspiration.
- Symptoms usually appear within the 1st 2 wk of life and increase in severity for up to 6 mo, although gradual improvement can begin at any time.
- Gastroesophageal reflux disease and neurologic disease influence the severity of the disease and thereby the clinical course.
- Winners:
- Fastest Correct Answer respondents:
- Dr Pawan Katwal
- Dr Bikesh Suwal
- Luck draw winners:
- - Dr Tyaysuh Adhikari
- - Dr Chitra Giri
Winners are requested to send an email to onlinepgnepal@gmail.com to claim Free Access to Today evening CEE MD/MS Based Online Model test [07:00 PM, Wednesday, Asar 17]
-
189
- 1. Inferior thyroid vein drains into:
- a. External jugular vein
- b. Internal jugular vein
- c. Inferior venacava
- d. Brachiocephalic vein
- Ans: d
- Ref: Gray’s anatomy, 41st Edition; Page No: 470
- • The venous drainage of the thyroid gland is usually via superior, middle and inferior thyroid veins.The superior thyroid vein emerges from the upper part of the gland and runs with the superior thyroid artery towards the carotid sheath; it drains into the internal jugular vein.
- • The middle thyroid vein collects blood from the lower part of the gland; it emerges from the lateral surface of the gland and drains into the internal jugular vein. The inferior thyroid veins arise in a glandular venous plexus, which also connects with the middle and superior thyroid veins. These veins form a pretracheal plexus, from which the left inferior vein descends to join the left brachiocephalic vein, and the right descends obliquely across the brachiocephalic artery to join the right brachiocephalic vein at its junction with the superior vena cava.
- • The inferior thyroid veins often open via a common trunk into the superior vena cava or left brachiocephalic vein. They drain the oesophageal, tracheal and inferior laryngeal veins and have valves at their terminations.
- 2. The following are the contraindications of tubal reconstructive surgery except:
- a. Length of tube <4 cms
- b. Patients over 30 years of age
- c. Pelvic tuberculosis
- d. Reversal done after 5 years of sterilization
- Ans: b
- Ref: Dutta Gynecology 6th Edition, Page No: 248
- Tubal reconstruction surgery can be done for a number of reasons including for reversal of sterilization procedure. The most favorable outcome is seen when it is done for reversal of sterilization procedures.
- Factors for Poor Outcome Following Tuboplasty
- • Dense pelvic adhesions.
- • Loss of fimbriae.
- • Bilateral hydrosalpinx > 3 cm.
- • Length of the reconstructed tube < 4 cm.
- • Reversal done after 5 years of sterilization operation.
- • Presence of other factors for infertility.
- 3. All of the following drugs can cause gynaecomastia except:
- a. Digoxin
- b. Amiloride
- c. Cimetidine
- d. Spironolactone
- Ans: b
- Ref: KD Tripathi Essentials of Medical Pharmacology; 7th Edition, Page No: 589
- Amiloride:
- • It is 10 times more potent than triamterene (dose 5–10 mg OD–BD).
- • At higher doses it also inhibits Na+ reabsorption in PT, but this is clinically insignificant. It decreases Ca2+ and Mg2+ excretion but increases urate excretion.
- • Usual side effects are nausea, diarrhoea and headache.
- • Amiloride blocks entry of Li+ through Na+ channels in the CD cells and mitigates diabetes insipidus induced by lithium.
- • Given as an aerosol it affords symptomatic improvement in cystic fibrosis by increasing fluidity of respiratory secretions.
- 4. Learned behaviour which is socially acquired is known as:
- a. Customs
- b. Acculturation
- c. Standard of living
- d. Culture
- Ans: d
- Ref: Park's Textbook of Preventive and Social Medicine; 23rd Edition; Page No: 672
- Culture: Is the learned behaviour which is socially acquired.
- Acculturation: Is ‘cultural contact’ or mixing of two cultures. It can occur through
- – Trade and commerce
- – Industrialization
- – Propagation of religion
- – Education
- – Conquest
- Custom: The established patterns of behavior that can be objectively verified within a particular social setting
- – Folkways: Right ways of doing things in less vital areas of human conduct
- – Mores: More stringent customs
- Standard of Living: Refers to the usual scale of our expenditure, goods we consume and services we enjoy. Standard of living (WHO) includes
- – Income and Occupation,
- – Standards of housing, sanitation and nutrition,
- – Level of provision of health, educational, recreational and other service
-
190
Answers and Winners of QUIZ for Today's CEE MD/MS Based Online Model test [11:00 AM, Asar 20, Saturday]
- 1. What is transfer of tissues from one organism to another part in the same organism known as?
- a. Autotransplant
- b. Allotransplant
- c. Isotransplant
- d. Xenotransplant
- Ans: a
- Ref: Schwartz’s Principles of Surgery, Eleventh Edition, Page No: 356
- According to the degree of immunologic similarity between the donor and recipient, transplants are divided into three main categories:
- an autotransplant is the transfer of cells, tissue, or an organ from one part of the body to another part in the same person, so no immunosuppression is required; this type of transplant includes skin, artery or vein, bone, cartilage, nerve, and islet cell transplants;
- an allotransplant is the transfer of cells, tissue, or an organ from one person to another of the same species; with the exception of identical twins, the immune system of the recipient recognizes the donated organ as a foreign body, so immunosuppression is required in order to avoid rejection; and
- a xenotransplant is the transfer of cells, tissue, or an organ from one organism to another of a different species. To date, animal-to-human transplants are still experimental procedures, given the very complex immunologic and infectious issues that have yet to be solved.
- 2. Which of the following criteria does not fall under Severe Acute Malnutrition?
- a. Weight-for-height below -3 standard deviation (<-3SD) on the WHO Growth Standard
- b. Presence of bipedal edema
- c. Mid-upper arm circumference (MUAC) below 12.5 cm (age 6 mo or more)
- d. All the above criteria meet for Severe Acute Malnutrition
- Ans: c
- Ref: OP Ghai Pediatrics 9th Edition, Page No: 98
- Severe Acute Malnutrition In children 6 to 59 months of age
- • Presence of 1. Weight-for-height below -3 standard deviation (<-3SD) on the WHO Growth Standard;
- • OR 2. Presence of bipedal edema;
- • OR 3. Mid-upper arm circumference (MUAC) below 11.5 cm (age 6 mo or more)
- 3. Subacute Sclerosing Panencephalitis is the late complication of
- a. Tetanus
- b. Measles
- c. Polio
- d. Diphtheria
- Ans: b
- Ref: Harrison’s Principles of Internal Medicine; 20th Edition, Page No: 900
- Subacute Sclerosing Panencephalitis (SSPE)
- • SSPE is a rare chronic, progressive demyelinating disease of the CNS associated with a chronic nonpermissive infection of brain tissue with measles virus.
- • The incidence has declined dramatically since the introduction of a measles vaccine.
- • Most patients give a history of primary measles infection at an early age (2 years), which is followed after a latent interval of 6–8 years by the development of a progressive neurologic disorder. Some 85% of patients are between 5 and 15 years old at diagnosis.
- • Initial manifestations include poor school performance and mood and personality changes.
- • Typical signs of a CNS viral infection, including fever and headache, do not occur. As the disease progresses, patients develop progressive intellectual deterioration, focal and/or generalized seizures, myoclonus, ataxia, and visual disturbances. In the late stage of the illness, patients are unresponsive, quadriparetic, and spastic, with hyperactive tendon reflexes and extensor plantar responses.
- 4. Prominent Occiput is seen in?
- a. Edward Syndrome
- b. Cat Eye Syndrome
- c. Patau Syndrome
- d. Down’s Syndrome
- Ans: a
- Ref: Nelson Textbook of Pediatrics, Elsevier, 20th Edition, Chapter 81 Table 81.2
- Trisomy 18 (Edward Syndrome)
- • Low birth Weight
- • Closed fists with index finger overlapping the 3rd
- • Digit and the 5th digit overlapping the 4th
- • Narrow hips with limited abduction
- • Short sternum
- • Rocker bottom feet
- • Microcephaly
- • Prominent occiput
- • Micrognathia
- • Cardiac and renal malformations
- • 70 year Mental retardation
- • 15% cases are lethal in 1st year
- Winners:
- Fastest correct answer respondents:
- - Dr Prabin Paudyal
- - Dr Niraj Acharya
- Lucky draw winners:
- - Dr Suraj Tiwari
- - Dr Deepak Kharel
Winners are requested to send an email to onlinepgnepal@gmail.com to claim Free Access to Today's CEE MD/MS Based Online Model test [11:00 AM, Asar 20, Saturday].
-
191
- 1. Unilateral, very severe, short lasting headache(2-45 min), retro orbital and associated with lacrimation and nasal congestion, more than 5 attacks per day, rapid course(<72hour) and excellent response to indomethacin is seen in:
- a. Cluster headache
- b. Migraine
- c. Paroxysmal hemicrania
- d. SUNCT
- Ans: c
- Ref: Harrison’s Principles of Internal Medicine; 20th Edition, Page No: 3105
- Paroxysmal hemicrania:
- • PH is characterized by frequent unilateral, severe, short-lasting episodes of headache. Like cluster headache, the pain tends to be retroorbital but may be experienced all over the head and is associated with autonomic phenomena such as lacrimation and nasal congestion.
- • Patients with remissions are said to have episodic PH, whereas those with the nonremitting form are said to have chronic PH. The essential features of PH are unilateral; very severe pain; short-lasting attacks (2–45 min); very frequent attacks (usually >5 a day); marked autonomic features ipsilateral to the pain; rapid course (<72 h); and excellent response to indomethacin.
- • In contrast to cluster headache, which predominantly affects males, the male-to-female ratio in PH is close to 1:1
- 2. Mutation in COL4A5 seen in:
- a. Good Pasteur syndrome
- b. Alport syndrome
- c. Xeroderma pigmentosum
- d. Osteogenesis imperfecta
- Ans: B
- Ref: Robbins Pathology; 9th edition; Pg562,701,1185
- • Hereditary nephritis refers to a group of glomerular diseases caused by mutations in genes encoding GBM proteins. The most common of these rare diseases are Alport syndrome and thin basement membrane disease. In Alport syndrome, nephritis is accompanied by sensorineural deafness and various eye disorders, including lens dislocation, posterior cataracts, and corneal dystrophy. Thin basement membrane disease is the most common cause of benign familial hematuria with no systemic manifestations.
- Pathogenesis
- • The GBM is composed largely of type IV collagen, which is made up of heterotrimers of α3, α4, and α5 type IV collagen. This form of type IV collagen is crucial for normal function of the lens, cochlea, and glomerulus. Mutation of any one of the α chains results in defective heterotrimer assembly and, consequently, the manifestations of Alport syndrome.
- Clinical Features
- • The majority of Alport syndrome patients have X-linked disease as a result of mutation of the gene on the X chromosome encoding the α5 chain of type IV collagen. Males therefore are affected more frequently and more severely than females and are more likely to develop end-stage renal disease and deafness. Rare autosomal recessive or dominant cases are linked to defects in the genes that encode α3 or α4 type IV collagen. Individuals with hereditary nephritis present at 5 to 20 years of age with gross or microscopic hematuria and proteinuria, and progress to overt renal failure by the ages of 20 to 50 years. Female carriers of X-linked Alport syndrome or carriers of either gender of the autosomal forms usually present with persistent asymptomatic hematuria and follow a benign nonprogressive clinical course. Heterozygous mutations in the genes encoding the α3 and α4 chains of type IV collagen are found in 40% of patients with thin basement membrane disease.
- Goodpasture Syndrome
- • Goodpasture syndrome is an uncommon autoimmune disease in which kidney and lung injury are caused by circulating autoantibodies against the noncollagenous domain of the α3 chain of collagen IV.
Osteogenesis imperfecta (OI), or brittle bone disease, is a phenotypically diverse disorder caused by deficiencies in the synthesis of type I collagen. It is the most common inherited disorder of connective tissue. OI principally affects bone, but also impacts other tissues rich in type I collagen (joints, eyes, ears, skin, and teeth). It usually results from autosomal dominant mutations (more than 800 have been identified) in the genes that encode the α1 and α2 chains of type I collagen.
- 3. A 44-year-old man is admitted to the emergency department with excessive vomiting and dehydration. Radiographic images demonstrate that part of the bowel is being compressed between the abdominal aorta and the superior mesenteric artery. Which of the following intestinal structures is most likely being compressed?
- a. Second part of duodenum
- b. Transverse colon
- c. Third part of duodenum
- d. First part of duodenum
- Ans: c
- Ref: Gray’s Anatomy for Students, 331
- • The third part of the duodenum takes a pathology situated anterior to the abdominal aorta and inferior to the superior mesenteric artery (a major ventral branch of the abdominal aorta). Because the third part of the duodenum lies in the angle between (“sandwiched”) these two structures, constrictions of this portion of the duodenum can occur readily.
- • The second part of the duodenum lies parallel with, and to the right of, the abdominal aorta and is not normally in close proximity to the superior mesenteric artery.
- • The transverse colon takes a horizontal pathology through the anterior abdominal cavity but travels superior or anterior to the superior mesenteric artery.
- • The first part of the duodenum continues from the pylorus, flexing to lead to the second part of the duodenum; thus, it is not located near the superior mesenteric artery or abdominal aorta.
- 4. Out of the following, which one is pathognomonic of miliary tuberculosis?
- a. Hepatomegaly
- b. Splenomegaly
- c. Lymphadenopathy
- d. Choroidal tubercles
- Ans: d
- Ref: Harrison’s Principles of Internal Medicine; 19th Edition, Page No: 1112
- Clinical manifestations of miliary tuberculosis are protean. Hepatomegaly, splenomegaly, and lymphadenopathy are frequent physical findings. Choroidal tubercles in eye examination is pathognomonic of miliary tuberculosis.
-
192
- 1. Subacute necrotizing encephalomyelopathy is:
- a. Rabies
- b. Leigh disease
- c. Wernicke’s encephalopathy
- d. MELAS
- Ans: b
- Ref: Nelson Textbook of Pediatrics, Elsevier, 20th Edition, Page No: 732
- Leigh Disease (Subacute Necrotizing Encephalomyelopathy)
- Leigh disease is a heterogenous neurologic disease that remains a neuropathologic description characterized by demyelination, gliosis, necrosis, relative neuronal sparing, and capillary proliferation in specific brain regions.
- In decreasing order of severity, the affected areas are the basal ganglia, brainstem cerebellum, and cerebral cortex
- 2. Drug of choice for renal colic
- a. Methadone
- b. Pentazocine
- c. Diclofenac
- d. Piroxicam
- Ans: c
- Ref: Bailey and Love’s Short Practice of Surgery 27th Edition, Page No: 1407
- Urinary stones
- • The commonest urinary tract stones are calcium oxalate
- • Stones are best diagnosed on a non-contrast computed tomography kidney–ureter–bladder (KUB)
- • Most stones <5 mm will pass spontaneously
- • Medical expulsive therapy remains controversial
- • Indications for surgical intervention are persistent pain, obstruction and infection
- Emergency setting
- • A CT scan confirms the diagnosis and, as indicated above, small stones (<5 mm), especially in the distal ureter, are treated expectantly as they are likely to pass spontaneously.
- • Medical expulsive therapy may be considered.
- • The patient is usually given a non-steroidal anti-inflammatory drug such as diclofenac for pain relief and observed for further episodes of pain, but also temperature, pulse, blood pressure and white blood count are monitored for signs of developing infection and the estimated glomerular filtration rate (eGFR) is monitored for signs of a decline in renal function.
- 3. The order of appearance of elbow ossification centers is:
- a. capitellum, radial head, internal epicondyle, trochlea
- b. capitellum, radial head, trochlea, internal epicondyle
- c. radial head, trochlea, internal epicondyle, capitellum
- d. trochlea, internal epicondyle, capitellum, radial head
- Ans: a
- The order of "I" and "T" are most important to remember.
- The order of appearances of the elbow ossification centers is highly reliable and in most individuals, is consistent: capitellum, radial head, internal epicondyle, trochlea, olecranon and external epicondyle.
- Two counting methods are taught to help remember the ages at which the ossification centers appear: 1-3-5-7-9-11 (simple) and 1-5-7-10-10-11 (more accurate).
- • Capitellum: 1 year
- • Radial head: 3 years
- • Internal epicondyle: 5 years
- • Trochlea: 7 years
- • Olecranon: 9 years
- • External epicondyle: 11 years
- The order of "I" and "T" are most important to remember; the trochlea ossification center should not appear before the internal (medial) epicondyle ossification centre. If you can see a trochlea but no internal epicondyle, then you need to look very hard for the avulsed ossification centre.
- 4. A patient is able to recognize a person by a name but not face. The lesion, in this case is in:
- a. Frontal lobe
- b. Occipital lobe
- c. Post parietal lobe
- d. Temporal lobe
- Ans: d
- REF: Ganong, 25thedition, page 293
- Recognition of faces:
- • An important part of the visual input goes to the inferior temporal lobe, where representations of objects, particularly faces, are stored.
- • Faces are particularly important in distinguishing friends from foes and the emotional state of those seen. In humans, storage and recognition of faces is more strongly represented in the right inferior temporal lobe in right-handed individuals, though the left lobe is also active. Damage to this area can cause prosopagnosia, the inability to recognize faces. Patients with this abnormality can recognize forms and reproduce them.
- • They can recognize people by their voices, and many of them show autonomic responses when they see familiar as opposed to unfamiliar faces. However, they cannot identify the familiar faces they see. The left hemisphere is also involved, but the role of the right hemisphere is primary.
- • The presence of an autonomic response to a familiar face in the absence of recognition has been explained by postulating the existence of a separate dorsal pathway for processing information about faces that leads to recognition at only a subconscious level.
-
193
Answers and Winners of COMMUNITY MEDICINE QUIZ
- 1. Early expanding stage is denoted by:
- a. Decreased birth rate and Decreased death rate
- b. Increased birth rate and Increased death rate
- c. Decreased birth rate and Increased death rate
- d. Unchanged birth rate and Decreased death rate
- Ans: d
- Ref: Park's Textbook of Preventive and Social Medicine; 23rd Edition; Page No: 479
- Demographic cycle
- The history of world population since 1650 suggests that there is a demographic cycle of 5 stages through which a nation passes:
- (1) FIRST STAGE (High stationary)
- This stage is characterized by a high birth rate and a high death rate which cancel each other and the population remains stationary.
- (2) SECOND STAGE (Early expanding)
- The death rate begins to decline, while the birth rate remains unchanged. Many countries in South Asia, and Africa are in this phase. Birth rates have increased in some of these countries possibly as a result of improved health conditions, and shortening periods of breast-feeding.
- (3) THIRD STAGE (Late expanding)
- The death rate declines still further, and the birth rate tends to fall. The population continues to grow because births exceed deaths. India has entered this phase. In a number of developing countries (e.g., China, Singapore) birth rates have declined rapidly.
- (4) FOURTH STAGE (Low stationary)
- This stage is characterized by a low birth and low death rate with the result that the population becomes stationary.
- Zero population growth has already been recorded in Austria during 1980-85. Growth rates as little as 0.1 were recorded in UK, Denmark, Sweden and Belgium during 1980-85. In short, most industrialized countries have undergone a demographic transition shifting from a high birth and high death rates to low birth and low death rates.
- (5) FIFTH STAGE: (Declining)
- The population begins to decline because birth rate is lower than the death rate. Some East European countries, notably Germany and Hungary are experiencing this stage.
- 2. Maternal mortality of Nepal as per NDHS 2016 is (per Lakh live birth)
- a. 254
- b. 239
- c. 208
- d. 164
- Ans: b
- Nepal Demographic and Health Survey 2016
- • The 2016 Nepal Demographic and Health Survey (2016 NDHS) was implemented by New ERA under the aegis of the Ministry of Health of Nepal. Funding for the survey was provided by the United States Agency for International Development (USAID). ICF provided technical assistance through The DHS Program, which assists countries in the collection of data to monitor and evaluate population, health, and nutrition programs.
- Maternal mortality rate: 239 per 1 lakh live birth
- Neonatal mortality rate: 21 deaths per 1,000 live births
- Infant mortality rate: 32 per 1,000 live births
- Under-5 mortality rate: 39 per 1,000 live births.
- 84% of women received antenatal care from a skilled provider at least once for their last birth.
- 78% of children have received all basic vaccinations
- Total Fertility Rate: 2.3 births per woman
- General Fertility Rate: 88 per 1000
- CBR: 22/4 per 1000
- Exclusive breastfeeding: 66%
- Vitamin A coverage (children age 6-59 months): 86%
- 95% of children live in households that consume iodized salt
- 3. A researcher selected all possible samples form and plotted their means on a line graph .This distribution is called as:
- a. Sample distribution
- b. Sampling distribution
- c. Population distribution
- d. Parametric distribution
- Ans: b
- Ref: Kothari Research methodology, Page 155
- • In statistics, a sampling distribution or finite -sample distribution is the probability distribution of a given statistic based on a random sample.
- • They allow analytical considerations to be based on the sampling distribution of a statistic, rather than on the joint probability distribution of all the individual sample values.
- 4. Validity of a test is based upon all except:
- a. Sensitivity
- b. Specificity
- c. Precision
- d. Accuracy
- Ans: c
- Ref: Park's Textbook of Preventive and Social Medicine; 23rd Edition; Page No: 138-139
- Validity: Refers to what extent the test measures which it purports to measure (adequacy/accuracy of measurement)
- Validity has 2 components:
- • Sensitivity
- • Specificity
- Inherent properties of a screening test:
- • Sensitivity
- • Specificity
- • Predictive accuracy
- Precision: The degree to which further measurements or calculations show the same or similar results. Precision is also known as: Reliability, Repeatability, Consistency or Reproducibility.
- 5. True about triage is:
- a. Yellow-least priority
- b. Red-morbidity
- c. Green-ambulatory
- d. Blue-ambulatory
- Ans: c
- • Triage: Consists of rapidly classifying the injured 'on the basis of severity of their injuries and likelihood of their sur¬vival' with prompt medical intervention
- • First come first serve is NOT followed in emergencies
- • Triage sieve: Quick survey to separate the dead and the walking from the injured
- • Triage sort: Remaining casualties are assessed and allocated to categories
- • Triage system: Most commonly uses FOUR color code system:
- o Red (Highest Priority ) : Immediate resuscitation or limb/life saving surgery in next 6 hours
- o Yellow (High Priority) : Possible resuscitation or limb/life saving surgery in next 24 hours
- o Green (Low Priority) :Minor illness/ ambulatory patients
- o Black (Least Priority ) : Dead and moribund patients
- • Tagging: Is the procedure where identification, age, place of origin, triage category, diagnosis and initial treatment are tagged on to every victim of disaster through a colour coding
- • Mitigation: Measures designed to either prevent hazards from causing emergency or to lessen the effects of emer¬gency.
- Winners:
- Fastest correct answer respondents:
- - Dr Subekshya Joshi
- - Dr Ratna Mani Sapkota
- Lucky draw winners:
- - Dr Tarique Siddique
- - Dr Pragya Jha
Winners are requested send an email to onlinepgnepal@gmail.com to claim Free Access to Tomorrow evening Subjectwise (FORENSIC & COMMUNITY MEDICINE) Based Model test of Asar 22 (Monday 07:00 PM).
-
194
Answers and Winners of FORENSIC MEDICINE QUIZ
- 1. Rifling and Land means:
- a. Grooves and Ridges
- b. Ridges and Grooves
- c. Loaded and Unloaded
- d. Loaded and Unloaded
- Ans: a
- Ref: Reddy Forensic Medicine, 33rd Edition, Page No: 209
- Rifled Arms:
- • The bore is scored internally with a number of shallow spiral "grooves", varying from two to more than 20, the most common being six, which run parallel to each other but twisted spirally, from breech to muzzle.
- • These grooves are called "rifling" and the projecting ridges between these grooves are called "lands".
- • Rifling is made by a broach.
- 2. Fatal concentration of chloroform is:
- a. 5 %
- b. 10 %
- c. 20 %
- d. 40 %
- Ans: d
- Ref: Reddy Forensic Medicine, 33rd Edition, Page No: 380
- The fatal concentration of some of the anaesthetic agents in blood is:
- • Chloroform: 40 to 60 mg%
- • Ethyl chloride: 40%
- • Diethyl ether: 180 mg%
- • Trichloroethylene: 50 mg%
- • Divinyl ether: 50mg%
- • Halothane: 20 mg%
- 3. Rabbit punch causes injury to:
- a. Spinal cord
- b. Testicle
- c. Cerebellum
- d. Teeth
- Ans: a
- Ref: Reddy Forensic Medicine, 33rd Edition, Page No: 266
- Fractures of the spine need not injure the cord, but the cord is rarely injured without associated fractures of vertebral column.
- "Whiplash injury" is an exception to this general rule. This is the injury sustained commonly by the occupants of the front seat. When the vehicle comes to a sudden stop, the forward thrust produces a state of acute hyperflexion, but this is converted into acute hyperextension as the forehead strikes the windscreen which causes injury to the cervical column.
- In such cases, and also due to a sharp blow against the spinous process of an upper cervical vertebra (rabbit punch), fatal contusion or laceration of the spinal cord may occur without fracture of spine.
- 4. In Nepal, newly married couple should register about the event in Office of the Local Registrar by:
- 10 days
- 35 days
- 60 days
- 120 days
- Ans: b
- Birth, Death and Other Personal Events (Registration) Act, 2033 (1976)
- It shall be duty of the following persons to give information to the Office of the Local Registrar in the prescribed form for the registration of the personal event within Thirty-Five days of the occurring of such event in the following circumstance:
- Notice of the birth or death, by the head of the family, and in his/her absence, from amongst one of the major member of the family
- Notice of marriage by both the husband and wife
- Notice of divorce by husband or wife
- Notice of migration by the head of the family in the case migration of a family and by the person in the case for migration of an individual.
- If any personal event occurs in abroad, a person who has a duty to forward such information shall forward it within Sixty days upon his/her arrival in Nepal.
- Winners:
- Fastest correct answer respondents:
- - Dr Bijay Ranabhat
- - Dr Aakanshya Shrestha
- Lucky draw winners:
- - Dr Ashish Sapkota
- - Dr Yasodha Poudel
Winners are requested send an email to onlinepgnepal@gmail.com to claim Free Access to Today evening Subjectwise (FORENSIC & COMMUNITY MEDICINE) Based Model test of Asar 22 (Monday 07:00 PM).
-
195
- 1. If the total cholesterol values for a certain population are approximately normally distributed with a mean of 210 mg/100 ml and a standard deviation of 22 mg/100 ml, find the probability that an individual picked at random from this population will have a cholesterol value between 188 and 210 mg/100 ml?
- a. 0.3413
- b. 0.1587
- c. 0.0062
- d. 0.3830
188 to 210 means: – 1 SD to Mean Corresponds to 34.1 % area
- 2. pH-gated urea channel in H. pylori bacterium is called:
- a. UreI
- b. UreJ
- c. UreK
- d. UreL
- Ans: a
- Ref: The Helicobacter pylori UreI protein: role in adaptation to acidity and identification of residues essential for its activity and for acid activation; by Bury-Moné S1; Mol Microbiol. 2001 Nov;42(4):1021-34.
- • Helicobacter pylori is a human gastric pathogen that survives the strong acidity of the stomach by virtue of its urease activity.
- • This activity produces ammonia, which neutralizes the bacterial microenvironment.
- • UreI, an inner membrane protein, is essential for resistance to low pH and for the gastric colonization of mice by H. pylori.
- • In the heterologous Xenopus oocytes expression system, UreI behaves like an H+-gated urea channel, and His-123 was found to be important for low pH activation.
- 3. A young pregnant woman presents with fulminant hepatic failure. The most likely etiological agent is
- a. Hepatitis A virus
- b. Hepatitis B virus
- c. Hepatitis C virus
- d. Hepatitis E virus
- Ans: d
- Ref: Harrison’s Principles of Internal Medicine; 20th Edition, Page No: 2353-2363
- Hepatitis E:
- - Previously labeled epidemic or enterically transmitted non-A, non-B hepatitis,
- - In outbreaks of waterborne hepatitis E in India and Asia, the case fatality rate is 1–2%
- - Up to 10–20% fatality rate in pregnant women
- Complications of infective hepatitis:
- - Relapsing hepatitis
- - Cholestatic hepatitis, characterized by protracted cholestatic jaundice and pruritus
- - Fulminant hepatitis (massive hepatic necrosis)
- - Chronic hepatitis
- - Cirrhosis and
- - Hepatocellular carcinoma
- 4. Autoinfection is seen in:
- a. Enterobius
- b. Ankylostoma
- c. Ascariasis
- d. Echinococcus
- Ans is a
- Ref: Jawetz’s Medical Microbiology; Lange; 27th Edition, Page No: 724,725,726
- The main symptom associated with Enterobius (pinworm) infections is perianal pruritus, especially at night, caused by a hypersensitivity reaction to the eggs that are laid around the perianal region by female worms, which migrate down from the colon at night. Scratching the anal region promotes transmission, as the eggs are highly infectious within hours of being laid (hand-to-mouth transmission).
- Other organism associated with autoinfection:
- • Strongyloides
- • H. Nana
-
196
Answers & Winners of QUIZ for Today evening CEE MD/MS BASED ONLINE MODEL TEST [Asar 24, Wednesday, 07:00 PM].
- 1. Adductor canal contains all except:
- a. Femoral artery
- b. Femoral vein
- c. Saphenous nerve
- d. Nerve to vastus lateralis
- Ans: d
- Ref: Snell Clinical Anatomy; 9th edition; Page No: 457
- Adductor (Subsartorial) Canal
- The adductor canal is an intermuscular cleft situated on the medial aspect of the middle third of the thigh beneath the Sartorius muscle. It commences above at the apex of the femoral triangle and ends below at the opening in the adductor magnus.
- • The anteromedial wall is formed by the sartorius muscle and fascia.
- • The posterior wall is formed by the adductor longus and magnus.
- • The lateral wall is formed by the vastus medialis.
- The adductor canal contains the terminal part of the femoral artery, the femoral vein, the deep lymph vessels, the saphenous nerve, the nerve to the vastus medialis, and the terminal part of the obturator nerve.
- 2. Otolith organs are concerned with function of:
- a. Hearing
- b. Rotatory nystagmus
- c. Linear acceleration
- d. Angular acceleration
- Ans: c
- Ref: PL Dhingra and Shruti Dhingra ENT and HNS 6th Edition, Page No: 16
- Otolith organs (present in maculae) are concerned with linear acceleration, gravity and head tilt movements and they also help to maintain static equilibrium.
- 3. In which part of fallopian tube, ectopic pregnancy will have earliest interruption?
- a. Ampulla
- b. Isthmus
- c. Infundibulum
- d. Interstitium
- Ans: b
- Ref: Dutta Obstetrics 8th Edition, Page No: 209
- Because of the unfavourable environment, early interruption of pregnancy is inevitable within 6–8 weeks. Earliest interruption occurs in the isthmial implantation whereas pregnancy may continue up to 3–4 months in interstitial implantation.
- Isthmic rupture usually occurs at 6–8 weeks, the ampullary one at 8–12 weeks and the interstitial one at about 4 months.
- 4. In cardiac assessment, calcium score can be evaluated by:
- a. Echocardiography
- b. Stress testing
- c. Thallium radionuclide study
- d. CT scan
- Ans: d
- Ref: Bailey and Love’s Short Practice of Surgery 27th Edition, Page No: 888
- With the development of the latest computed tomography (CT) scanners, which have the ability to correct for respiratory and cardiac movements, multislice high-resolution CT scanning may become an alternative to coronary angiography.
- It allows assessment of coronary disease, particularly proximal CAD, and gives some information about the degree of coronary artery calcification (calcium score) that is very helpful when stratifying patients to determine which ones will benefit from the more invasive coronary angiography.
- Winners:
- Fastest correct answer respondents:
- - Dr Gagan Sharma
- - Dr Dinesh Neupane
- Luck draw winners:
- - Dr Satish Shah
- - Dr Deepak Kharel
Winners are requested to send an email to onlinepgnepal@gmail.com to claim Free Access to Today evening CEE MD/MS BASED ONLINE MODEL TEST [Asar 24, Wednesday, 07:00 PM].
-
197
- 1. Wilson disease is caused by mutation of which gene?
- a. ATP5B
- b. ATP6B
- c. ATP7B
- d. ATP8B
- Ans: c
- Ref: Robbin’s Basic Pathology, 10th edition, Page 657
- Wilson disease is an autosomal recessive disorder caused by mutation of the ATP7B gene, which results in impaired copper excretion into bile and a failure to incorporate copper into ceruloplasmin.
- 2. Which of the following is not associated with blunt trauma to eye?
- a. Commotion retinae
- b. Concussion cataract
- c. Globe rupture
- d. Sympathetic ophthalmitis
- Ans: d
- Ref: Khurana Ophthalmology, 6th edition, page 427 – 438
- • Sympathetic ophthalmitis is a serious bilateral granulomatous panuveitis which follows a penetrating ocular trauma.
- • The injured eye is called exciting eye and the fellow eye which also develops uveitis is called sympathizing eye. Very rarely, sympathetic ophthalmitis can also occur following an intraocular surgery.
- 3. Neck holding comes at what age:
- a. 2 months
- b. 3 months
- c. 4 months
- d. 5 months
- Ans: B
- Ref: OP Ghai Pediatrics 8th Edition, Page No: 49
- Gross motor developmental milestones
- 3 month Neck holding
- 15 month Roll over
- 6 month Sits in tripod position
- 8 month Sitting without support
- 9 month Stand with support
- 4. The final stage of chronic gastritis is?
- a. Superficial gastritis
- b. Atrophic gastritis
- c. Gastric atrophy
- d. Intestinal metaplasia
- Ans: c
- Ref: Harrison’s Principles of Internal Medicine; 19th Edition, Page No: 1930
- Early phase of chronic gastritis is superficial gastritis in which inflammatory changes are limited to lamina propria and intact gastric glands. Next stage is atrophic gastritis in which inflammatory infiltrate extends deeper into mucosa, with destruction of gastric glands. Final stage of chronic gastritis is gastric atrophy in which glandular structures are lost, and there is a paucity of inflammatory infiltrates. Endoscopically, mucosa is thin and underlying blood vessels can be visualized. Intestinal metaplasia refers to the conversion of gastric glands to small-bowel mucosal glands containing goblet cells. Intestinal metaplasia is an important predisposing factor for gastric cancer.
-
198
Answers and Winners of QUIZ for CEE MD/MS BASED ONLINE MODEL TEST [Asar 27, Saturday, 11:00 AM].
- 1. A 63-year-old woman develops exertional angina and has had two episodes of syncope. Examination shows a systolic ejection murmur with radiation to the carotids and a soft S2. Which of the following is the most likely diagnosis?
- a. Mitral stenosis
- b. Mitral insufficiency
- c. Aortic stenosis
- d. Aortic insufficiency
- Ans: c
- Ref: Hurst's The Heart, 11th Edition 11th Edition, Page No: 1645–1647
- Aortic stenosis is most likely to be associated with angina pectoris, syncope, and exertional dyspnea. Exertional syncope is caused by either systemic vasodilation in the presence of fixed or inadequate cardiac output, an arrhythmia, or both. Syncope at rest is most frequently a result of a transient ventricular tachyarrhythmia.
- 2. Fall in systolic pressure >10 mmHg with inspiration is:
- a. Pulsus alternans
- b. Pulsus paradoxus
- c. Pulsus bigeminy
- d. Water hammer pulse
- Ans: b
- Ref: Harrison 19th Edition, Page No: 1445, 1446
- The character of the pulse is best appreciated at the carotid level.
- • A weak and delayed pulse (pulsus parvus et tardus) defines severe aortic stenosis (AS). Some patients with AS may also have a slow, notched, or interrupted upstroke (anacrotic pulse) with a thrill or shudder.
- • With chronic severe AR, by contrast, the carotid upstroke has a sharp rise and rapid fall-off (Corrigan’s or water-hammer pulse).
- • Some patients with advanced AR may have a bifid or bisferiens pulse, in which two systolic peaks can be appreciated.
- • A bifid pulse is also described in patients with hypertrophic obstructive cardiomyopathy (HOCM), with inscription of percussion and tidal waves. A bifid pulse is easily appreciated in patients on intraaortic balloon counterpulsation (IABP), in whom the second pulse is diastolic in timing.
- • Pulsus paradoxus refers to a fall in systolic pressure >10 mmHg with inspiration that is seen in patients with pericardial tamponade but also is described in those with massive pulmonary embolism, hemorrhagic shock, severe obstructive lung disease, and tension pneumothorax.
- • Pulsus paradoxus is measured by noting the difference between the systolic pressure at which the Korotkoff sounds are first heard (during expiration) and the systolic pressure at which the Korotkoff sounds are heard with each heartbeat, independent of the respiratory phase. A pulsus paradoxus may be palpable at the brachial artery or femoral artery level when the pressure difference exceeds 15 mmHg. This inspiratory fall in systolic pressure is an exaggerated consequence of interventricular dependence.
- • Pulsus alternans, in contrast, is defined by beat-to-beat variability of pulse amplitude. It is present only when every other phase I Korotkoff sound is audible as the cuff pressure is lowered slowly, typically in a patient with a regular heart rhythm and independent of the respiratory cycle.
- • Pulsus alternans is seen in patients with severe left ventricular systolic dysfunction and is thought to be due to cyclic changes in intracellular calcium and action potential duration. When pulsus alternans is associated with electrocardiographic T-wave alternans, the risk for an arrhythmic event appears to be increased.
- 3. According to Frank starling law, force of contraction is directly proportional to:
- a. Muscle tension
- b. End systolic length of muscle fibres
- c. End diastolic length of muscle fibres
- d. Contractility of heart
- Ans: c
- Ref: Ganong Review of Medical Physiology, Lange; 25th Edition, Page No: 546
- • The length–tension relationship in cardiac muscle is similar to that in skeletal muscle ,when the muscle is stretched, the developed tension increases to a maximum and then declines as stretch becomes more extreme. Starling pointed this out when he stated that the “energy of contraction is proportional to the initial length of the cardiac muscle fiber” (Starling’s law of the heart or the Frank-Starling law). For the heart, the length of the muscle fibers (ie, the extent of the preload) is proportional to the end-diastolic volume. The relation between ventricular stroke volume and end-diastolic volume is called the Frank-Starling curve.
- When cardiac output is regulated by changes in cardiac muscle fiber length, this is referred to as heterometric regulation. Conversely, regulation due to changes in contractility independent of length is sometimes called homometric regulation.
- 4. Craniotabes is seen in all except:
- a. Rickets
- b. Syphilis
- c. Thalassemia
- d. Osteogenesis imperfect
- Ans: c
- Ref: Nelson Textbook of Pediatrics, Elsevier, 20th Edition, Page No: 331
- Craniotabes is a softening of the cranial bones and can be detected by applying pressure at the occiput or over the parietal bones. The sensation is similar to the feel of pressing into a ping-pong ball and then releasing. It is a normal finding in many newborns, especially near the suture lines, but it typically disappears within a few months of birth.
- Craniotabes is seen in
- • Rickets
- • Syphilis
- • Osteogenesis imperfecta
- • Hydrocephalus
- Winners:
- Fastest correct answer respondents:
- - Dr Kaushal Kafle
- - Dr Pragya Sharma
- Lucky draw winners:
- - Dr Aslok Patel
- - Dr Ashmita Nepal
Winners are requested to send an email to onlinepgnepal@gmail.com to claim Free Access to CEE MD/MS BASED ONLINE MODEL TEST [Asar 27, Saturday, 11:00 AM].
-
199
Answers & Winners of QUIZ of Tomorrow evening Subject-wise (Orthopedics & ENT) MD/MS Based Online Model test [07:00 PM, Asar 29, Monday]
- 1. A 56-year-old woman presents to the emergency department with several episodes in which she felt "dizzy." She has had these symptoms on and off for the past year and can recall no clear exacerbating factor or time of day when her symptoms occur. She has a perpetual sensation of fullness in her ear but otherwise has no symptoms currently. Her temperature is 97.6°F (36.4°C), blood pressure is 122/77 mmHg, pulse is 85/min, respirations are 13/min, and oxygen saturation is 98% on room air. Cardiopulmonary exam is unremarkable. The patient's gait is stable. Which of the following is also likely to be found in this patient?
- a. Conductive hearing loss
- b. Gradually improving symptoms
- c. Positional vertigo
- d. Sensorineural hearing loss
- Ans: d
- Ref: PL Dhingra and Shruti Dhingra ENT and HNS 7th Edition, Page No: 111
- • This patient's chronic ear fullness and vertigo-like symptoms suggest a diagnosis of Meniere disease which is associated with sensorineural hearing loss.
- • The classic constellation of Meniere disease consists of 4 symptoms: episodic dizziness or whirling vertigo, low-frequency sensorineural hearing loss, tinnitus, and a sensation of aural fullness in the ear.
- • These symptoms may or may not develop simultaneously and present in a chronic and undulating fashion. Symptoms are generally not positional and do not resolve on their own.
- • The vertigo episodes of Meniere disease can last hours. Treatment involves salt restriction and diuretics.
- 2. Auditory brainstem implant is kept in:
- a. Fourth ventricle
- b. Pons
- c. Medulla
- d. Facial canal
- Ans: a
- Ref: PL Dhingra and Shruti Dhingra ENT and HNS 7th Edition, Page No: 141
- Auditory Brainstem Implant (ABI)
- • This implant is designed to stimulate the cochlear nuclear complex in the brainstem directly by placing the implant in the lateral recess of the fourth ventricle.
- • Such an implant is needed when CN VIII has been severed in surgery of vestibular schwannoma. In these cases, cochlear implants are obviously of no use.
- • In unilateral acoustic neuroma, ABI is not necessary as hearing is possible from the contralateral side but in bilateral acoustic neuromas as in NF2, rehabilitation is required by ABI.
- 3. Incorrectly matched is:
- a. Thumb sign: Acute epiglottitis
- b. Invasive sinusitis: Diabetics
- c. Croup: Parainfluenzae virus
- d. Steeple sign: Nasopharyngeal carcinoma
- Ans: d
- Ref: PL Dhingra and Shruti Dhingra ENT and HNS 7th Edition, Page No: 507
- • Acute epiglottitis in children is caused by Haemophilus influenzae type B. It produces a typical “Thumb sign” on lateral X-ray film. Ampicillin was considered the drug of choice but now many organisms have become resistant to it and ceftriaxone is preferred.
- • Acute fulminant fungal sinusitis is an invasive sinusitis and is commonly seen in diabetics, HIV infected patients and transplant patients receiving chemotherapy for immunosuppression. Therapy in such cases should be urgent, aggressive surgical debridement and amphotericin-B.
- • Acute laryngotracheobronchitis or croup is a viral infection caused by parainfluenzae type 1, 2, and sometimes 3. Critical area involved is subglottic larynx producing oedema with stridor and respiratory distress. X-ray (PA view) larynx shows typical “steeple sign” but X-rays are avoided as any manipulation may precipitate acute obstruction.
- 4. Flat and dome shaped graph in tympanogram is found in:
- a. Otosclerosis
- b. Ossicular discontinuity
- c. TM perforation
- d. Middle ear fluid
- Ans: d
- Ref: PL Dhingra and Shruti Dhingra ENT and HNS 7th Edition, Page No: 27
- Types of tympanograms
- I. Type A: Normal tympanogram.
- II. Type As: Compliance is lower at or near ambient air pressure. Seen in fixation of ossicles, e.g. otosclerosis or malleus fixation.
- III. Type Ad: High compliance at or near ambient pressure. Seen in ossicular discontinuity or thin and lax tympanic membrane.
- IV. Type B: A flat or dome-shaped graph. No change in compliance with pressure changes. Seen in middle ear fluid or thick tympanic membrane.
- V. Type C: Maximum compliance occurs with negative pressure in excess of 100 mm H2O. Seen in retracted tympanic membrane and may show some fluid in middle ear.
- Winners:
- Fastest correct answer respondents:
- - Dr Suvu Bhandari
- - Dr Bibek Chhetri
- Luck draw winners:
- - Dr Anandita Gupta
- - Dr Ajay Shrestha
Winners are requested to send an email to onlinepgnepal@gmail.com to claim Free Access to Tomorrow evening Subject-wise (Orthopedics & ENT) MD/MS Based Online Model test [07:00 PM, Asar 29, Monday]
-
200
Answers & Winners of QUIZ of Today evening Subject-wise (Orthopedics & ENT) MD/MS Based Online Model test [07:00 PM, Asar 29, Monday]
- 1. Upper age limit of patient in Juvenile Idiopathic Arthritis is:
- a. 12 years
- b. 14 years
- c. 16 years
- d. 18 years
- Ans: c
- Clinical definition of Juvenile Idiopathic Arthritis
- Broad term that encompasses different types of pediatric chronic arthritis that is characterized by joint pain and inflammation
- 1 or more joints are involved for at least 6 weeks in patients < 16 years of age
- Subtypes
- • Polyarticular juvenile idiopathic arthritis (JIA): ≥ 5 joints involved
- • Pauciarticular JIA: 1-4 joints involved
- • Systemic JIA: Visceral involvement
- 2. In a 10-year-old male with transcervical fracture neck femur, best treatment is:
- a. Spica
- b. Austin Moore pins
- c. K-Wires
- d. Cannulated Cancellous screw
- Ans: d
- Ref: Femoral Neck Fractures in Children: A Review; Thomas Palocaren; Indian J Orthop. 2018 Sep-Oct; 52(5): 501–506.
- Paediatric femoral neck fractures are uncommon injuries and are usually caused by high-energy trauma.
- Low-energy trauma can result in pathologic neck fractures and stress fractures of the neck, due to repetitive activity.
- Surgical options can vary based on age, Delbet classification and displacement of the fracture.
- Treatment for displaced fractures is by closed or open reduction and smooth/cancellous screw fixation.
- Compression screw and side plate fixation is indicated for basal fractures. Fixation should be supplemented by spica cast immobilization in younger children. The high rate of complications occurs due to the vascular anatomy of the hip and proximal femur.
- Avascular necrosis, coxa vara, premature physeal closure, and nonunion are the most common and these often result in poor outcome.
- 3. What is a common source of autogenous cortical bone graft?
- a. Fibula
- b. Iliac crest
- c. Proximal tibia
- d. Ribs
- Ans: a
- Ref: Essential Orthopedics by Maheshwari & Mhaskar, 5th Edition, Page No: 83 – 85
- There are three types of bone grafts; autograft (from the same person); allograft (from another person of the same species); xenograft (from a different species).
- • Autografts are used most commonly.This is the ‘gold standard’ bone grafting technique. Human body has a lot of ‘spare’ bone for such use. Iliac crests are the commonest site for taking bone grafts. When the graft is required for osteogenic purpose (as in non-union), cancellousbone grafts are preferred. It is available in plenty from iliac crests and upper end of tibia. When the graft is used for providing stability (as for filling bone gaps), cortical graft is used. Fibulae are the common source of cortical bone grafts.
- • Allogenous grafting: Allogenous bone grafts (Allografts) are usually required when enough bone is not available from the host e.g., where a big defect is created following a tumour resection. Such bone grafts could be obtained from another human being, living or dead. The latter is called cadaveric graft.
- • Xenografting: Bone grafts from other species, usually bovine are now available off the shelf.
- • Artificial bone: This is a material derived from corals. It is hydroxyapetite with porous structure. It is supposed to have osteo-conductive potential, and is being used in some countries.
- 4. What chromosomal translocation is present in Ewing’s sarcoma?
- a. 7 and 11
- b. 11 and 21
- c. 14 and 7
- d. 22 and 11
- Ans: d
- Ref: Apley & Solomon’s System of Orthopaedics & Trauma, 10th edition, page 179
- • In the case of Ewing’s sarcoma, more than 95% of tumours demonstrate EWS-ETS fusion gene rearrangements, most commonly the result of translocations between chromosome 22 and chromosome 11 (t22;11(q24;q12) translocation) or 21 (t21;22(q22;q12)).
- • The ETS transcription factor family is responsible for regulating cellular differentiation, the cell cycle, cell migration and apoptosis. Therefore mutations will result in deregulation of programmed cell death and disruption of cell-cycle control.
- Winners:
- Fastest correct answer respondents:
- - Dr Mohsin Hasan
- - Dr Anuradha Pradhan
- Luck draw winners:
- - Dr Sarik Shrestha
- - Dr Yasodha Paudel
Winners are requested to send an email to onlinepgnepal@gmail.com to claim Free Access to Today evening Subject-wise (Orthopedics & ENT) MD/MS Based Online Model test [07:00 PM, Asar 29, Monday]
-
201
Answers and Winners of QUIZ for Today evening CEE MD/MS Based Online Model test [07:00 PM, Asar 31, Wednesday]
- 1. Faint opacity which results due to superficial scars involving Bowman’s layer and superficial stroma is
- a. Nebular corneal opacity
- b. Macular corneal opacity
- c. Leukomatous corneal opacity
- d. Adherent leucoma
- Ans: a
- Ref: AK Khurana 6th edition page 134
- • Nebular corneal opacity: It is a faint opacity which results due to superficial scars involving Bowman’s layer and superficial stroma. A thin, diffuse nebula covering the pupillary area interferes more with vision than the localised leucoma away from pupillary area. Further, the nebula produces more discomfort to patient due to blurred image owing to irregular astigmatism than the leucoma which completely cuts off the light rays.
- • Macular corneal opacity: It is a semi-dense opacity produced when scarring involves about half the corneal stroma.
- • Leucomatous corneal opacity (leucoma simplex): It is a dense white opacity which results due to scarring of more than half of the stroma.
- • Adherent leucoma: It results when healing occurs after perforation of cornea with incarceration of iris
- 2. Movements of pronation and supination occurs in all the following joints EXCEPT:
- a. Superior radioulnar joint
- b. Middle radioulnar joint
- c. Inferior radioulnar joint
- d. Radiocarpal joint
- Ans: b
- Snell’s Anatomy by Regions, 9th Edition, Page No: 409, 410
- The movements of pronation and supination of the forearm involve a rotary movement around a vertical axis at the proximal and distal radioulnar joints. The axis passes through the head of the radius above and the attachment of the apex of the triangular articular disc below.
- • Pronation is performed by the pronator teres and the pronator quadratus.
- • Supination is performed by the biceps brachii and the supinator.
- • Supination is the more powerful of the two movements because of the strength of the biceps muscle. Because supination is the more powerful movement, screw threads and the spiral of corkscrews are made so that the screw and corkscrews are driven inward by the movement of supination in right-handed people.
- Middle radio-ulnar joint is a syndesmosis (fibrous) joint mainly helps in binding radius and ulna together. The other function of this joint is weight transmission from radius towards ulna.
- 3. Le Fort classification is for fracture of:
- a. Orbit
- b. Frontal bone
- c. Maxilla
- d. Mandible
- Ans: c
- Ref: PL Dhingra and Shruti Dhingra ENT and HNS 7th Edition, Page No: 207
- Fractures of maxilla
- They are classified into three types:
- Le Fort I (transverse): Fracture runs above and parallel to the palate. It crosses lower part of nasal septum, maxillary antra and the pterygoid plates.
- Le Fort II (pyramidal): Fracture passes through the root of nose, lacrimal bone, floor of orbit, upper part of maxillary sinus and pterygoid plates. This fracture has some features common with the zygomatic fractures.
- Le Fort III (craniofacial dysjunction): There is complete separation of facial bones from the cranial bones. The fracture line passes through root of nose, ethmofrontal junction, superior orbital fissure, lateral wall of orbit, frontozygomatic and temporozygomatic sutures and the upper part of pterygoid plates.
- 4. What is the pH of propofol?
- a. 6.8
- b. 7
- c. 7.4
- d. 7.6
- Ans: b
- Ref: Miller’s Anesthesia, 8th edition, page 823
- • Numerous formulations of propofol are marketed today.
- • Propofol has a pH of 7 and appears as a slightly viscous, milky white substance, a result of small lipid droplets in solution.
- • The formulation most commonly used is that of 1% propofol, 10% soybean oil, and 1.2% purified egg phospholipid added as emulsifier, with 2.25% of glycerol as a tonicity-adjusting agent, and sodium hydroxide to change the pH.
- • Following concerns regarding microbial growth in the emulsion, ethylenediaminetetraacetic acid (EDTA) was added for its bacteriostatic activities.
- Winners:
- Fastest correct answer respondents:
- - Dr Bijay Rajbhanshi
- - Dr Sitaram Neupane
- Lucky draw winners among respondents:
- - Dr Binay Sah
- - Dr Rabina KC
Winners are requested to send an email to onlinepgnepgnepal@gmail.com to claim Free Access to Today evening CEE MD/MS Based Online Model test [07:00 PM, Asar 31, Wednesday.
-
202
Answers and Winners of QUIZ for Today's CEE MD/MS Based Online Model test [11:00 AM, Shrawan 3, Saturday]
- 1. Asthma is diagnosed with the post bronchodilator reversibility in FEV1 more than:
- a. 10%
- b. 12%
- c. 14%
- d. 20%
- Ans: b
- Ref: GINA 2020 Guideline
- Ref: Harrison’s Principles of Internal Medicine; 20th Edition, Page No: 1963
- Simple spirometry confirms airflow limitation with a reduced FEV1, FEV1/FVC ratio, and PEF. Reversibility is demonstrated by a >12% and 200-mL increase in FEV1 15 min after an inhaled short-acting β2-agonist (SABA; such as inhaled albuterol 400 μg) or in some patients by a 2-4 week trial of oral corticosteroids (OCS) (prednisone or prednisolone 30-40 mg daily).
- 2. What is the most common site of ectopic testes?
- a. Superficial inguinal pouch
- b. Deep inguinal pouch
- c. Perineum
- d. Femoral triangle
- Ans: a
- Ref: Bailey and Love’s Short Practice of Surgery 27th Edition, Page No: 1498
- • Incomplete descent of the testis occurs when the testis is arrested in some part of its normal path to the scrotum.
- • An ectopic testis is a testis that is abnormally placed outside this path.
- • An ectopic testis has taken a non-standard path through the body and has ended up in an unusual location, the commonest site being the superficial inguinal pouch, which lies just inferior and medial to the superficial inguinal ring. Other rarer ectopic sites include the femoral triangle, the root of the penis and perineum.
- 3. Recurrence rate of febrile seizure is:
- a. 1%
- b. 5%
- c. 30%
- d. 95%
- Ans: c
- Ref: Nelson Textbook of Pediatrics, Elsevier, 20th Edition, Page No: 2829
- Febrile seizures recur in approximately 30% of those experiencing a first episode, in 50% after 2 or more episodes, and in 50% of infants younger than 1 yr old at febrile seizure onset. Several factors affect recurrence risk.
- • Febrile seizures are seizures that occur between the age of 6 and 60 mo with a temperature of 38°C (100.4°F) or higher, that are not the result of central nervous system infection or any metabolic imbalance, and that occur in the absence of a history of prior afebrile seizures.
- • A simple febrile seizure is a primary generalized, usually tonic–clonic, attack associated with fever, lasting for a maximum of 15 min, and not recurrent within a 24-hr period.
- • A complex febrile seizure is more prolonged (>15 min), is focal, and/or reoccurs within 24 hr. Febrile
- • status epilepticus is a febrile seizure lasting longer than 30 min. Some use the term simple febrile seizure plus for those with recurrent febrile seizures within 24 hr.
- • Most patients with simple febrile seizures have a very short postictal state and usually return to their baseline normal behavior and consciousness within minutes of the seizure.
- 4. Accumulation of sphingomyelin in phagocytic cells is feature of:
- a. Gaucher Disease
- b. Niemann Pick Disease
- c. Down Syndrome
- d. Tay Sachs Disease
- Ans: b
- Ref: Robbins and Cotran Pathologic basis of disease 9/e p 152
- Niemann-Pick Disease Types A and B
- Niemann-Pick disease types A and B are two related disorders that are characterized by lysosomal accumulation of sphingomyelin due to an inherited deficiency of sphingomyelinase.
- Type A is a severe infantile form with extensive neurologic involvement, marked visceral accumulations of sphingomyelin, and progressive wasting and early death within the first 3 years of life.
- In contrast, type B disease patients have organomegaly but generally no central nervous system involvement. They usually survive into adulthood.
- 5. Most sensitive index of recent transmission of malaria is:
- a. Infant parasite rate
- b. Parasite density
- c. proportional case rate
- d. Spleen rate
- Ans: a
- Ref: Park's Textbook of Preventive and Social Medicine; 23rd Edition; Page No: 261
- o SPLEEN RATE: It is defined as the percentage of children between 2 and 10 years of age showing enlargement of spleen. Adults are excluded from spleen surveys, because causes other than malaria frequently operate in causing splenic enlargement in them. The spleen rate is widely used for measuring the endemicity of malaria in a community.
- o AVERAGE ENLARGED SPLEEN: This is a further refinement of spleen rate, denoting the average size of the enlarged spleen. It is a useful malariometric index.
- o PARASITE RATE: It is defined as the percentage of children between the ages 2 and 10 years showing malaria parasites in their blood films.
- o PARASITE DENSITY INDEX: It indicates the average degree of parasitaemia in a sample of well-defined group of the population. Only the positive slides are included in the denominator
- o INFANT PARASITE RATE: It is defined as the percentage of infants below the age of one year showing malaria parasites in their blood films. It is regarded as the most sensitive index of recent transmission of malaria in a locality. If the infant parasite rate is zero for 3 consecutive years in a locality, it is regarded as absence of malaria transmission even though; the Anopheline vectors responsible for previous transmission may remain.
- o PROPORTIONAL CASE RATE : Since the morbidity rate is difficult to determine, except in conditions when the diagnosis and reporting of each case is carried to perfection, proportional case rate is used . It is defined as the number of cases diagnosed as clinical malaria for every 100 patients attending the hospitals and dispensaries. This is a crude index because the cases are not related to their time/ space distribution.
- Winners:
- fastest correct answer respondents:
- - Dr Ashma Shrestha
- - Dr Sanjeet Jaiswal
- Lucky draw winners:
- - Dr Suman Acharya
- - Dr Tripti Devkota
Winners are requested to send an email to onlinepgnepal@gmail.com to claim Free Access to Today's CEE MD/MS Based Online Model test [11:00 AM, Shrawan 3, Saturday]
-
203
- Answer to daily question (Ashar 26, Friday)
- Q1.Anterior boundary of inguinal canal is:
- a.Fascia transversalis
- b.Fascia transversalis and internal oblique
- c.Aponeurosis of external oblique and internal oblique muscle
- fibres
- d.Aponeurosis of external oblique and fascia transversalis
- Ans: c. Aponeurosis of external oblique and internal oblique muscle fibres
- Ref: BDC Human Anatomy 5/e, p 208
- Boundaries of inguinal canal:
- The anterior wall is formed by the following.
- (a) In its whole extent: (1) Skin; (2) superficial fascia; and (3) external oblique aponeurosis.
- (b) In its lateral one-third: The fleshy fibres of the internal oblique muscle.
- The posterior wall is formed by the following.
- (a) In its whole extent: (i) The fascia transversalis, (ii) the extraperitoneal tissue, and (iii) the parietal peritoneum.
- (b) In its medial two-thirds: (i) The conjoint tendon; (ii) at its medial end by the reflected part of the inguinal ligament, (iii) over its lateral one-third by the interfoveolar ligament.
- Roof
- It is formed by the arched fibres of the internal oblique and transversusabdominis muscles.
- Floor
- It is formed by the grooved upper surface or the inguinal ligament; and at the medial end by the lacunar ligament.
- 2. Clinically femoral hernia is located at
- a.Medial and upper to pubic tubercle
- b.Lateral and below to pubic tubercle
- c.Sapheno-femoral junction
- d.Lateral and above to pubic tubercle
- Ans: b. Lateral and below to pubic tubercle
- Ref: SRB's Manual of Surgery by SriramBhat M 4/e, Pg. 822
- Clinical Features of femoral hernia
- Common in females (2:1 ratio), common in multipara.
- Rare before puberty. 20% occurs bilateral, however, more common on right side.
- Presents as a swelling in the groin below and lateral to the pubic tubercle (Inguinal hernia is above and medial to thepubic tubercle).
- Swelling, impulse on coughing, reducibility, gurgling sound during reduction, dragging pain, are the usual features.
- When obstruction and strangulation occurs which is more common, presents with features of intestinal obstruction— painful, tender, inflamed, irreducible swelling without any impulse. They also present with abdominal distension, vomiting and features of toxicity.
- Often femoral hernia can be associated with inguinal hernia also.
- 40% of femoral hernias present as emergency hernia with obstruction/strangulation.
- 3. Malignant otitis externa is caused by
- a.Haemophilus influenza
- b.Pseudomonas aeruginosa
- c.Staphylococcus aureus
- d.Proteus vulgaris
- Ans: b. Pseudomonas aeruginosa
- Ref: Dhingra Diseases of Ear, Nose and Throat, 6th Edition Pg 52
- Malignant (necrotizing) otitis externa:
- It is an inflammatory condition caused by pseudomonas infection usually in the elderly diabetics, or in those on immunosuppressive drugs.
- Its early manifestations resemble diffuse otitis externa but there is excruciating pain and appearance of granulations in the ear canal. Facial paralysis is common.
- Infection may spread to the skull base and jugular foramen causing multiple cranial nerve palsies. Anteriorly, infection spreads to temporomandibular fossa, posteriorly to the mastoid and medially into the middle ear and petrous bone.
-
204
Medical Exams Nepal: MEN’s MD/MS quiz for CEE: MD/MS mock test (Shrawan 04)
Correct answer sequence: BDDC
- Congratulations to all the winners.
- First two fastest correct respondents:
- 1. Dr. Ayasha Shrestha
- 2. Davidson (From Viber group, name to be confirmed)
- Two lucky winners among all correct respondents:
- 1. Dr. Nira
- 2. Dr. Prayash Poudel
- We will contact you soon for your free tokens or you can contact us via viber, messenger or email.
- 1. What is the main (primary) function of oblique muscles over eyeball?
- a. Adduction and abduction
- b. Rotation
- c. Upward and downward movements
- d. Popping in and out of eyeball
- Ans: B, Rotation
- Ref, Textbook of Medical Physiology, Guyton and Hall, 13th edition, page no. 666
- Muscular Control of Eye Movements:
- The eye movements are controlled by three pairs of muscles:
- (1) The medial and lateral recti
- (2) The superior and inferior recti, and
- (3) The superior and inferior obliques
- -The medial and lateral recti contract to move the eyes from side to side.
- -The superior and inferior recti contract to move the eyes upward or downward.
- -The oblique muscles function mainly to rotate the eyeballs to keep the visual fields in the upright position.
- 2. A medical officer works in a PHC near one of the cities of Nepal. One day, he receives an envelope with Rs. 2000 kept inside. There is also a letter inside the envelope which says, “Thank you doctor for sending the patients to my office for CT-scans and USG.” Receiving this amount is:
- a. Ethical
- b. Ethical given the circumstances of Nepal
- c. Allowed as salary of an MO is not satisfactory
- d. Unethical and not allowed
- Ans: D, Unethical and not allowed
- Ref, Code of Ethics, NMC 2017
- Gratuity and commissions
- It is unethical to receive or offer any gratuity, commissions or bonuses in consideration of or in return for referring or recommending patient for prescribing medical and surgical treatments, investigations or consultations.
- 3. A 56 years old man who worked 25 years as a guard in various companies comes to you with complains of dragging sensation over bilateral thighs and multiple lumps in the same region. After examination, you diagnose the case as Varicose veins, clinically. Venous drainage of legs depend on efficient functioning of all of the following except?
- a. Superficial veins
- b. Deep veins
- c. Perforators
- d. None of the above
- Ans: D, None of the above
- Ref, Illustrated Synopsis of Dermatology and Sexually Transmitted Diseases, Neena Khanna, 4th edition, page no. 165
- Anatomy of leg veins
- Venous drainage of legs depends on efficient functioning of three components
- - Superficial veins.
- - Deep veins.
- - Perforators.
- Blood in superficial veins drains into deep veins through the perforators (located in the lower part of the leg) with the help of gravity when the calf muscles relax.
- Deep veins in the calf pump the blood to the heart when the calf muscles contract (calf muscle pump). Reflux is prevented by valves.
- 4. Non organic vaginismus is characterised by an involuntary spasm of:
- a. Upper 1/3rd of vagina
- b. Middle 1/3rd of vagina
- c. Lower 1/3rd of vagina
- d. Cervix
- Ans: C, Lower 1/3rd of vagina
- Ref, A Short Textbook of Psychiatry, Niraj Ahuja, 7th edition, page no. 130
- Sexual Pain Disorders
- - Nonorganic vaginismus
- This disorder is characterised by an involuntary spasm of the lower 1/3rd of vagina, interfering with coitus. Penile entry is either painful or impossible. Before a presumption of nonorganic vaginismus, it is important to rule out organic factors (e.g. local pathology causing pain).
- - Nonorganic dyspareunia
- This disorder is characterised by pain in the genital area of either male or female, during coitus. Before a presumption of nonorganic dyspareunia, it is particularly important to rule out organic factors (e.g. local pathology causing pain) in both males and females.
-
205
- Answers todaily questions (31st Ashar)
- Q1. Munson sign is seen in:
- a.Pterygium
- b.Keratoconus
- c.Sclerocornea
- d.Trachoma
- Ans: b, Keratoconus
- Ref:Nelson textbook of pediatrics 21st edition pg13016
- Keratoconus
- • This is a disease of unclear pathogenesis characterized by progressive thinning and bulging of the central cornea, which becomes cone shaped. Although familial cases are known, most cases are sporadic.
- • Eye rubbing and contact lens wear have been implicated as pathogenic, but the evidence to support this is equivocal. The incidence is increased in individuals with atopy, Down syndrome, Marfan syndrome, and retinitis pigmentosa.
- • Most cases are bilateral, but involvement may be asymmetric. The disorder usually presents and progresses rapidly during adolescence; progression slows and stabilizes when patients reach full growth. Descemet membrane may occasionally be stretched beyond its elastic breaking point, causing an acute rupture in the membrane with resultant sudden and marked corneal edema (acute hydrops ) and decrease in vision.
- • The corneal edema resolves as endothelial cells cover the defective area. Some degree of corneal scarring occurs, but the visual acuity is often better than before the initial incident.
- • Signs of keratoconus include Munson sign (bulging of the lower eyelid on looking downward) and the presence of a Fleischer ring (a deposit of iron in the epithelium at the base of the cone).
- • Glasses and contact lenses are the first step in treating the visual distortion caused by keratoconus. Corneal cross linking is a relatively new procedure using riboflavin and UV light, and may arrest the progression of keratoconus.
- • If the cornea vaults too severely for the vision to be corrected with contact lenses then a corneal transplant must be performed to restore vision.
- Q2. Drug induced lupus is characterized by:
- a.Anti nuclear antibodies
- b.Antihistone antibodies
- c.Antibodies against dsDNA
- d.Anti Smith antibodies
- Ans:b, antihistone antibodies
- Ref: Nelson textbook of pediatrics 21st edition pg5272
- • Drug-induced lupus refers to the presence of SLE manifestations triggered by exposure to specific medications, including hydralazine, minocycline, many anticonvulsants, sulfonamides, and antiarrhythmic agents .
- • In individuals prone to SLE, these agents may act as a trigger for true SLE, but more often these agents provoke a reversible lupus-like syndrome.
- • Unlike SLE, drug-induced lupus affects males and females equally.
- • A genetic predisposition toward slow drug acetylation may increase the risk of drug-induced lupus. Circulating antihistone antibodies are often present in drug-induced SLE; these antibodies are only detected in up to 20% of individuals with SLE.
- • Hepatitis, which is rare in SLE, is more common in drug-induced lupus.
- • Individuals with drug-induced lupus are less likely to demonstrate antibodies to dsDNA, hypocomplementemia, and significant renal or neurologic disease.
- • In contrast to SLE, manifestations of drug-induced lupus typically resolve after withdrawal of the offending medication; however, complete recovery may take several months to years, requiring treatment with hydroxychloroquine, NSAIDs, and/or corticosteroids.
- Q3. Pseudomembranous colitis is caused by
- a. Metronidazole
- b. Oral vancomycin
- c. Clindamycin
- d. All of the above
- Ans: c, Clindamycin
- Ref:First aid for USMLE Step 1 2020 pg182
- Clostridium difficile
- • Produces toxins A and B, which damage enterocytes. Both toxins lead to watery diarrhea= pseudomembranous colitis.Often 2° to antibiotic use, especially clindamycin or ampicillin; associated with PPIs.
- • Diagnosed by PCR or antigen detection of one or both toxins in stool.
- • Complications: toxic megacolon.
- • Treatment: oral vancomycin, metronidazole, or fidaxomicin. For recurrent cases, consider repeating prior regimen or fecal microbiota transplant.
-
206
- Answers to daily questions, (Correct sequence : CAC)
- 1.In hypokalemia with serum K+ levels between 2 to 2.5 mEq/ L, all the following can be seen except:
- a. Short QT interval
- b. Tall spiky T waves
- c. Both (a) and (b)
- d. U wave
- Ans,c. Both (a) and (b)
- Ref: Harrison 17thedition/p.282
- ECG changes in hypokalemia:
- The electrocardiographic changes of hypokalemia are due to delayed ventricular repolarization and do not correlate well with the plasma K+ concentration.
- Early changes include:
- • Flattening or inversion of T wave
- • Prominent U wave
- • ST-segment depression and
- • Prolonged QT interval.
- Severe K+ depletion may result in:
- • Prolonged PR interval
- • Widening of the QRS complex and
- • An increased risk of ventricular arrhythmias, especially in patients with myocardial ischemia or left ventricular hypertrophy.
- 2.A 30-year-old man has excessive sleep at work attributed to sleep discomfort at night. He also has recent history of falling while partying with friends. What are the other features that can be seen associated with his condition:
- a. Paralysis during sleep-wake transition with hallucinations
- b. Snoring with witnessed sleep apnea
- c. Pain in the legs before going to sleep
- d. Generalized seizures in the wake state
- Ans. a. Paralysis during sleep-wake transition with hallucinations
- Ref: Harrison 19/e p189. 18/e p265
- • History of fall is highly suggestive of atonia/paralysis and may be due to Narcolepsy. In obstructive sleep apnea,patient usually falls asleep during daytime with no loss of postural tone.
- • Narcolepsy is both a disorder of the ability to sustain wakefulness voluntarily and a disorder of REM sleep regulation. Paralysis during sleep-wake transition with hallucinations is seen in narcolepsy.
- Narcolepsy
- • Narcolepsy is a common sleep disorder characterized by excessive day time sleepiness, cataplexy (episodes of muscle weakness triggered by emotions), hypnagogic hallucinations and sleep paralysis.
- Narcolepsy (Classic tetrad)-
- • Excessive day time somnolence.
- • Sudden weakness or loss of muscle tone without loss of consciousness often elicited by emotions (cataplexy).
- • Hallucination at sleep onset (Hypnagogic hallucination).
- • Muscle paralysis upon awakening.
- 3. A 55 year old female came with iliotibial band contracture. What is the most likely test to be done in this case:
- a. Allis test
- b. Thomas test
- c. Ober test
- d. Trendelenburg test
- Ans, c. Ober test
- Ref: Apley’s system of orthopaedics and fracture 9th ed., pg. 495
- • Ober test is used to determine Iliotibial band contracture.
- • Thomas test is used to find out fixed flexion deformity of hip joint.
- • Trendelenberg test is to find out abductor weakness of hip joint.
- • Allis test or Galeazzi sign is used to diagnose DDH in children.
-
207
- Answers to daily questions:
- Q1. Late postoperative complications are those arising;
- a. After 24 hours of surgery
- b. After 72 hours of surgery
- c. After one week of surgery only
- d. Only after 15 days of surgerys
- Ans: b, After 72 hours of surgery
- Ref: Short Practice of Surgery, Bailey and Love, 27th edition, page no.292
- Classification of postoperative complications
- There are three common approaches for the classification of postoperative complications of surgery:
- •Linked to time after surgery:
- oimmediate (within 6 h of procedure);
- oearly (6–72 h);
- olate (>72 h).
- •Generic and surgery specific.
- •Clavian-Dindo: this system relates to surgical complications only and is used to objectively and reproducibly measure the impact of the surgical complication on the outcome of the procedure. It is included here for completeness and will be discussed no further.
- Q2. During reconstruction of an amputed limb, which of the following is done first?
- a. Arterial repair
- b. Venous repair
- c. Nerve anastomoses
- d. Fixation of the bone
- Ans: d, Fixation of the bone
- Ref: Campbell’s Operativ3 Orthopaedics 11th edition, page no.3718-24
- During reconstruction of an amputated limb, the order of repair is as follows;
- 1. Shorten and internally fix bone
- 2. Repair extensor tendons
- 3. Repair flexor tendons (2 or 3 may be reversed, or flexor tendon repair may be delayed)
- 4. Repair arteries
- 5. Repair nerves
- 6. Repair veins
- Q3. At a distance of 1 meter, intensity of normal conversation is:
- a. 30 dB
- b. 50 dB
- c. 90 dB
- d. 60 dB
- Ans: d, 60 dB
- Ref: Diseases of Ear Nose and Throat, PL Dhingra, 6th edition, page no.19
- Explanation:
- At a distance of 1 m, intensity of
- •Whisper = 30 dB
- •Normal conversation = 60 dB
- •Shout = 90 dB
- •Discomfort of the ear = 120 dB
- •Pain in the ear = 130 dB
-
|
|